Sunteți pe pagina 1din 336

Seales y Sistemas

Collection Editor:
Richard Baraniuk
Seales y Sistemas

Collection Editor:
Richard Baraniuk
Authors:

Thanos Antoulas Ricardo Radaelli-Sanchez


Richard Baraniuk Justin Romberg
Steven J. Cox Phil Schniter
Benjamin Fite Melissa Selik
Roy Ha JP Slavinsky
Michael Haag Ricardo von Borries
Don Johnson

Translated By:
Fara Meza
Erika Jackson

Online:
< http://cnx.org/content/col10373/1.2/ >

CONNEXIONS

Rice University, Houston, Texas


This selection and arrangement of content as a collection is copyrighted by Richard Baraniuk. It is licensed under
the Creative Commons Attribution License 2.0 (http://creativecommons.org/licenses/by/2.0/).
Collection structure revised: September 28, 2006
PDF generated: December 19, 2013
For copyright and attribution information for the modules contained in this collection, see p. 310.
Table of Contents
1 Seales
1.1 Clasicacin y Propiedades de las Seales . . . . . . . . . . . . . . . . . . . . . . . . . . . . . . . . . . . . . . . . . . . . . . . . . . . 1
1.2 Operaciones para Seales . . . . . . . . . . . . . . . . . . . . . . . . . . . . . . . . . . . . . . . . . . . . . . . . . . . . . . . . . . . . . . . . . . . . 9
1.3 Seales tiles . . . . . . . . . . . . . . . . . . . . . . . . . . . . . . . . . . . . . . . . . . . . . . . . . . . . . . . . . . . . . . . . . . . . . . . . . . . . . . 12
1.4 Funcin de Impulso . . . . . . . . . . . . . . . . . . . . . . . . . . . . . . . . . . . . . . . . . . . . . . . . . . . . . . . . . . . . . . . . . . . . . . . . 16
1.5 El Exponencial Complejo . . . . . . . . . . . . . . . . . . . . . . . . . . . . . . . . . . . . . . . . . . . . . . . . . . . . . . . . . . . . . . . . . . . 19
1.6 Seales en Tiempo-Discreto . . . . . . . . . . . . . . . . . . . . . . . . . . . . . . . . . . . . . . . . . . . . . . . . . . . . . . . . . . . . . . . . 23
Solutions . . . . . . . . . . . . . . . . . . . . . . . . . . . . . . . . . . . . . . . . . . . . . . . . . . . . . . . . . . . . . . . . . . . . . . . . . . . . . . . . . . . . . . . . 27

2 Sistemas
2.1 Clasicacin y Propiedades de los Sistemas . . . . . . . . . . . . . . . . . . . . . . . . . . . . . . . . . . . . . . . . . . . . . . . . . 29
2.2 Propiedades de los Sistemas . . . . . . . . . . . . . . . . . . . . . . . . . . . . . . . . . . . . . . . . . . . . . . . . . . . . . . . . . . . . . . . . 33
3 Anlisis de Sistemas de Tiempo Continuo en el Dominio del Tiempo
3.1 Sistemas Lineales CT y Ecuaciones Diferenciales . . . . . . . . . . . . . . . . . . . . . . . . . . . . . . . . . . . . . . . . . . . . 39
3.2 Convolucin de Tiempo-Continuo . . . . . . . . . . . . . . . . . . . . . . . . . . . . . . . . . . . . . . . . . . . . . . . . . . . . . . . . . . . 42
3.3 Propiedades de la Convolucin . . . . . . . . . . . . . . . . . . . . . . . . . . . . . . . . . . . . . . . . . . . . . . . . . . . . . . . . . . . . . 48
3.4 Estabilidad BIBO . . . . . . . . . . . . . . . . . . . . . . . . . . . . . . . . . . . . . . . . . . . . . . . . . . . . . . . . . . . . . . . . . . . . . . . . . . 55

4 Anlisis de Sistemas Discretos en el Dominio del Tiempo


4.1 Anlisis en el Dominio del Tiempo para Sistemas Discretos . . . . . . . . . . . . . . . . . . . . . . . . . . . . . . . . . 59
4.2 Convolucin Discreta . . . . . . . . . . . . . . . . . . . . . . . . . . . . . . . . . . . . . . . . . . . . . . . . . . . . . . . . . . . . . . . . . . . . . . . 63
4.3 Convolucin Circular y el DFT . . . . . . . . . . . . . . . . . . . . . . . . . . . . . . . . . . . . . . . . . . . . . . . . . . . . . . . . . . . . . 68
Solutions . . . . . . . . . . . . . . . . . . . . . . . . . . . . . . . . . . . . . . . . . . . . . . . . . . . . . . . . . . . . . . . . . . . . . . . . . . . . . . . . . . . . . . . . 73

5 Repaso de Algebra Lineal


5.1 Algebra Lineal: Conceptos Bsicos . . . . . . . . . . . . . . . . . . . . . . . . . . . . . . . . . . . . . . . . . . . . . . . . . . . . . . . . . 75
5.2 Conceptos Bsicos de Vectores . . . . . . . . . . . . . . . . . . . . . . . . . . . . . . . . . . . . . . . . . . . . . . . . . . . . . . . . . . . . . . 79
5.3 Eigenvectores y Eigenvalores . . . . . . . . . . . . . . . . . . . . . . . . . . . . . . . . . . . . . . . . . . . . . . . . . . . . . . . . . . . . . . . . 79
5.4 Diagonalizacin de Matrices . . . . . . . . . . . . . . . . . . . . . . . . . . . . . . . . . . . . . . . . . . . . . . . . . . . . . . . . . . . . . . . . 85
5.5 Generalidades de Eigenvectores y Eigenvalores . . . . . . . . . . . . . . . . . . . . . . . . . . . . . . . . . . . . . . . . . . . . . . 88
5.6 Eigenfunciones de los Sistemas LTI . . . . . . . . . . . . . . . . . . . . . . . . . . . . . . . . . . . . . . . . . . . . . . . . . . . . . . . . . 89
5.7 Propiedades de la Transformada de Fourier . . . . . . . . . . . . . . . . . . . . . . . . . . . . . . . . . . . . . . . . . . . . . . . . . 92
Solutions . . . . . . . . . . . . . . . . . . . . . . . . . . . . . . . . . . . . . . . . . . . . . . . . . . . . . . . . . . . . . . . . . . . . . . . . . . . . . . . . . . . . . . . . 93

6 Series Fourier del Tiempo Continuo


6.1 Seales Peridicas . . . . . . . . . . . . . . . . . . . . . . . . . . . . . . . . . . . . . . . . . . . . . . . . . . . . . . . . . . . . . . . . . . . . . . . . . . 95
6.2 Series de Fourier: El Mtodo de Eigenfunciones . . . . . . . . . . . . . . . . . . . . . . . . . . . . . . . . . . . . . . . . . . . . . 96
6.3 Derivacin de la Ecuacin de Coecientes de Fourier . . . . . . . . . . . . . . . . . . . . . . . . . . . . . . . . . . . . . . . . 99
6.4 Generalidades de las Series de Fourier . . . . . . . . . . . . . . . . . . . . . . . . . . . . . . . . . . . . . . . . . . . . . . . . . . . . . 100
6.5 Propiedades de la Serie de Fourier . . . . . . . . . . . . . . . . . . . . . . . . . . . . . . . . . . . . . . . . . . . . . . . . . . . . . . . . . 103
6.6 Propiedades de Simetra de las Series de Fourier . . . . . . . . . . . . . . . . . . . . . . . . . . . . . . . . . . . . . . . . . . . 106
6.7 Propiedad de Convolucin Circular de las Series de Fourier . . . . . . . . . . . . . . . . . . . . . . . . . . . . . . . . 110
6.8 Series de Fourier y los Sistemas LTI . . . . . . . . . . . . . . . . . . . . . . . . . . . . . . . . . . . . . . . . . . . . . . . . . . . . . . . 111
6.9 Convergencia de las Series de Fourier . . . . . . . . . . . . . . . . . . . . . . . . . . . . . . . . . . . . . . . . . . . . . . . . . . . . . . 114
6.10 Condiciones de Dirichlet . . . . . . . . . . . . . . . . . . . . . . . . . . . . . . . . . . . . . . . . . . . . . . . . . . . . . . . . . . . . . . . . . 116
6.11 El Fenmeno de Gibbs . . . . . . . . . . . . . . . . . . . . . . . . . . . . . . . . . . . . . . . . . . . . . . . . . . . . . . . . . . . . . . . . . . . 118
6.12 Resumen de las Series de Fourier . . . . . . . . . . . . . . . . . . . . . . . . . . . . . . . . . . . . . . . . . . . . . . . . . . . . . . . . . 121
Solutions . . . . . . . . . . . . . . . . . . . . . . . . . . . . . . . . . . . . . . . . . . . . . . . . . . . . . . . . . . . . . . . . . . . . . . . . . . . . . . . . . . . . . . . 123

7 Espacios de Hilbert y Expansiones Ortogonales


iv

7.1 Espacios Vectoriales . . . . . . . . . . . . . . . . . . . . . . . . . . . . . . . . . . . . . . . . . . . . . . . . . . . . . . . . . . . . . . . . . . . . . . . 125


7.2 Normas . . . . . . . . . . . . . . . . . . . . . . . . . . . . . . . . . . . . . . . . . . . . . . . . . . . . . . . . . . . . . . . . . . . . . . . . . . . . . . . . . . . 127
7.3 Producto Interno . . . . . . . . . . . . . . . . . . . . . . . . . . . . . . . . . . . . . . . . . . . . . . . . . . . . . . . . . . . . . . . . . . . . . . . . . . 129
7.4 Espacios de Hilbert . . . . . . . . . . . . . . . . . . . . . . . . . . . . . . . . . . . . . . . . . . . . . . . . . . . . . . . . . . . . . . . . . . . . . . . . 131
7.5 Desigualdad de Cauchy-Schwarz . . . . . . . . . . . . . . . . . . . . . . . . . . . . . . . . . . . . . . . . . . . . . . . . . . . . . . . . . . . 132
7.6 Espacios de Hilbert comunes . . . . . . . . . . . . . . . . . . . . . . . . . . . . . . . . . . . . . . . . . . . . . . . . . . . . . . . . . . . . . . 139
7.7 Tipos de Bases . . . . . . . . . . . . . . . . . . . . . . . . . . . . . . . . . . . . . . . . . . . . . . . . . . . . . . . . . . . . . . . . . . . . . . . . . . . . 142
7.8 Expansin de Bases Ortonormales . . . . . . . . . . . . . . . . . . . . . . . . . . . . . . . . . . . . . . . . . . . . . . . . . . . . . . . . . 146
7.9 Espacio de Funciones . . . . . . . . . . . . . . . . . . . . . . . . . . . . . . . . . . . . . . . . . . . . . . . . . . . . . . . . . . . . . . . . . . . . . . 150
7.10 Base de la Ondoleta de Haar . . . . . . . . . . . . . . . . . . . . . . . . . . . . . . . . . . . . . . . . . . . . . . . . . . . . . . . . . . . . . 151
7.11 Bases Ortonormales en Espacios Reales y Complejos . . . . . . . . . . . . . . . . . . . . . . . . . . . . . . . . . . . . . 158
7.12 Teoremas de Plancharel y Parseval . . . . . . . . . . . . . . . . . . . . . . . . . . . . . . . . . . . . . . . . . . . . . . . . . . . . . . . 160
7.13 Approximacin y Proyeccin en el Espacio de Hilbert . . . . . . . . . . . . . . . . . . . . . . . . .. . . . . . . . . . . . 161
Solutions . . . . . . . . . . . . . . . . . . . . . . . . . . . . . . . . . . . . . . . . . . . . . . . . . . . . . . . . . . . . . . . . . . . . . . . . . . . . . . . . . . . . . . . 165

8 Transformada Discreta de Fourier


8.1 Anlisis de Fourier . . . . . . . . . . . . . . . . . . . . . . . . . . . . . . . . . . . . . . . . . . . . . . . . . . . . . . . . . . . . . . . . . . . . . . . . 167
8.2 Anlisis de Fourier en Espacios Complejos . . . . . . . . . . . . . . . . . . . . . . . . . . . . . . . . . . . . . . . . . . . . . . . . . 168
8.3 Ecuacin de Matriz para la DTFS . . . . . . . . . . . . . . . . . . . . . . . . . . . . . . . . . . . . . . . . . . . . . . . . . . . . . . . . . 175
8.4 Extensin Peridica de las DTFS . . . . . . . . . . . . . . . . . . . . . . . . . . . . . . . . . . . . . . . . . . . . . . . . . . . . . . . . . . 176
8.5 Desplazamientos Circulares . . . . . . . . . . . . . . . . . . . . . . . . . . . . . . . . . . . . . . . . . . . . . . . . . . . . . . . . . . . . . . . . 184
Solutions . . . . . . . . . . . . . . . . . . . . . . . . . . . . . . . . . . . . . . . . . . . . . . . . . . . . . . . . . . . . . . . . . . . . . . . . . . . . . . . . . . . . . . . 190

9 Transformada Rpida de Fourier (FFT)


9.1 DFT: Transformada Rpida de Fourier . . . . . . . . . . . . . . . . . . . . . . . . . . . . . . . . . . . . . . . . . . . . . . . . . . . . 191
9.2 La Transformada Rpida de Fourier (FFT) . . . . . . . . . . . . . . . . . . . . . . . . . . . . . . . . . . . . . . . . . . . . . . . . 192
9.3 Derivando la Transformada Rpida de Fourier . . . . . . . . . . . . . . . . . . . . . . . . . . . . . . . . . . . . . . . . . . . . . 193
Solutions . . . . . . . . . . . . . . . . . . . . . . . . . . . . . . . . . . . . . . . . . . . . . . . . . . . . . . . . . . . . . . . . . . . . . . . . . . . . . . . . . . . . . . . 197

10 Convergencia
10.1 Convergencia de Secuencias . . . . . . . . . . . . . . . . . . . . . . . . . . . . . . . . . . . . . . . . . . . . . . . . . . . . . . . . . . . . . . 199
10.2 Convergencia de Vectores . . . . . . . . . . . . . . . . . . . . . . . . . . . . . . . . . . . . . . . . . . . . . . . . . . . . . . . . . . . . . . . . 201
10.3 Convergencia Uniforme de Secuencias de Funciones. . . . . . . . . . . . . . . . . . . . . . . . . . . . . . . . . . . . . . . 204

11 Transformada de Fourier Discreta en el Tiempo (DTFT)


11.1 Tabla de Transformadas de Fourier Comunes . . . . . . . . . . . . . . . . . . . . . . . . . . . . . . . . . . . . . . . . . . . . . 207
11.2 Transformacin Discreta de Fourier . . . . . . . . . . . . . . . . . . . . . . . . . . . . . . . . . . . . . . . . . . . . . . . . . . . . . . 208
11.3 Transformada Discreta de Fourier (DFT) . . . . . . . . . . . . . . . . . . . . . . . . . . . . . . . . . . . . . . . . . . . . . . . . . 210
11.4 Transformada de Fourier Discreta en el Tiempo (DTFT) . . . . . . . . . . . . . . . . . . . . . . . . . . . . . . . . . 212
11.5 Propiedades de la Transformada de Fourier Discreta en el Tiempo . . . . . . . . . . . . . . . . . . . . . . . . 214
11.6 Par de la Transformada de Fourier Discreta en el Tiempo . . . . . . . . . . . . . . . . . . . . . . . . . . . . . . . . 214
11.7 Ejemplos de DTFT . . . . . . . . . . . . . . . . . . . . . . . . . . . . . . . . . . . . . . . . . . . . . . . . . . . . . . . . . . . . . . . . . . . . . . 215
Solutions . . . . . . . . . . . . . . . . . . . . . . . . . . . . . . . . . . . . . . . . . . . . . . . . . . . . . . . . . . . . . . . . . . . . . . . . . . . . . . . . . . . . . . . 219

12 Transformada de Fourier de Tiempo Continuo (CTFT)


12.1 Transformada de Fourier de Tiempo Continuo (CTFT) . . . . . . . . . . . . . . . . . . . . . . . . . . . . . . . . . . . 221
12.2 Propiedades de la Transformada de Fourier de Tiempo-Continuo . . . . . . . . . . . . . . . . . . . . . . . . . 223
Solutions . . . . . . . . . . . . . . . . . . . . . . . . . . . . . . . . . . . . . . . . . . . . . . . . . . . . . . . . . . . . . . . . . . . . . . . . . . . . . . . . . . . . . . . 226

13 Teorema de Muestreo
13.1 Muestreo . . . . . . . . . . . . . . . . . . . . . . . . . . . . . . . . . . . . . . . . . . . . . . . . . . . . . . . . . . . . . . . . . . . . .. . . . . . . . . . . . 227
13.2 Reconstruccin . . . . . . . . . . . . . . . . . . . . . . . . . . . . . . . . . . . . . . . . . . . . . . . . . . . . . . . . . . . . . . . . . . . . . . . . . . . 231
13.3 Ms sobre Reconstruccin Perfecta . . . . . . . . . . . . . . . . . . . . . . . . . . . . . . . . . . . . . . . . . . . . . . . . . . . . . . . 236
13.4 Teorema de Nyquist . . . . . . . . . . . . . . . . . . . . . . . . . . . . . . . . . . . . . . . . . . . . . . . . . . . . . . . . . . . . . . . . . . . . . . 238

Available for free at Connexions <http://cnx.org/content/col10373/1.2>


v

13.5 Aliasing . . . . . . . . . . . . . . . . . . . . . . . . . . . . . . . . . . . . . . . . . . . . . . . . . . . . . . . . . . . . . . . . . . . . . . . . . . . . . . . . . . 240


13.6 Filtros Anti-Aliasing . . . . . . . . . . . . . . . . . . . . . . . . . . . . . . . . . . . . . . . . . . . . . . . . . . . . . . . . . . . . . . . . . . . . . 243
13.7 Procesamiento de Tiempo Discreto de Seales de Tiempo Continuo . . . . . . . . . . . . . . . . . . . . . . . 245
Solutions . . . . . . . . . . . . . . . . . . . . . . . . . . . . . . . . . . . . . . . . . . . . . . . . . . . . . . . . . . . . . . . . . . . . . . . . . . . . . . . . . . . . . . . 248

14 Transformada de Laplace y Diseo de Sistemas


14.1 La Transformada de Laplace . . . . . . . . . . . . . . . . . . . . . . . . . . . . . . . . . . . . . . . . . . . . . . . . . . . . . . . . . . . . . 251
14.2 Propiedades de la Transformada de Laplace . . . . . . . . . . . . . . . . . . . . . . . . . . . . . . . . . . . . . . . . . . . . . . 253
14.3 Tabla de Transformadas de Laplace Comunes . . . . . . . . . . . . . . . . . . . . . . . . . . . . . . . . . . . . . . . . . . . . 255
14.4 Regin de Convergencia para la Transformada de Laplace . . . . . . . . . . . . . . . . . . . . . . . . . . . . . . . . 255
14.5 La Transformada Inversa de Laplace . . . . . . . . . . . . . . . . . . . . . . . . . . . . . . . . . . . . . . . . . . . . . . . . . . . . . 257
14.6 Polos y Ceros . . . . . . . . . . . . . . . . . . . . . . . . . . . . . . . . . . . . . . . . . . . . . . . . . . . . . . . . . . . . . . . . . . . . . . . . . . . . 259

15 La Transformada Z y Filtros Digitales


15.1 La Transformada Z: Denicin . . . . . . . . . . . . . . . . . . . . . . . . . . . . . . . . . . . . . . . . . . . . . . . . . . . . . . . . . . . 263
15.2 Tabla de Transformadas-Z Comunes . . . . . . . . . . . . . . . . . . . . . . . . . . . . . . . . . . . . . . . . . . . . . . . . . . . . . . 268
15.3 Regin de Convergencia para la Transformada- Z . . . . . . . . . . . . . . . . . . . . . . . . . . . . . . . . . . . . . . . . . 269
15.4 La Transformada Inversa de Z . . . . . . . . . . . . . . . . . . . . . . . . . . . . . . . . . . . . . . . . . . . . . . . . . . . . . . . . . . . . 278
15.5 Funciones Racionales . . . . . . . . . . . . . . . . . . . . . . . . . . . . . . . . . . . . . . . . . . . . . . . . . . . . . . . . . . . . . . . . . . . . . 281
15.6 La Ecuacin de Diferencia . . . . . . . . . . . . . . . . . . . . . . . . . . . . . . . . . . . . . . . . . . . . . . . . . . . . . . . . . . . . . . . . 283
15.7 Entendiendo las Gracas de Polos y Ceros en el Plano-Z . . . . . . . . . . . . . . . . . . . . . .. . . . . . . . . . . . 286
15.8 Diseo de Filtros usando la Graca de Polos y Ceros de la Transformada-Z . . . . . . . . . . . . . . . 290

16 Tareas
16.1 Homework 1 . . . . . . . . . . . . . . . . . . . . . . . . . . . . . . . . . . . . . . . . . . . . . . . . . . . . . . . . . . . . . . . . . . . . . . . . . . . . . 296
Glossary . . . . . . . . . . . . . . . . . . . . . . . . . . . . . . . . . . . . . . . . . . . . . . . . . . . . . . . . . . . . . . . . . . . . . . . . . . . . . . . . . . . . . . . . . . . . 300
Index . . . . . . . . . . . . . . . . . . . . . . . . . . . . . . . . . . . . . . . . . . . . . . . . . . . . . . . . . . . . . . . . . . . . . . . . . . . . . . . . . . . . . . . . . . . . . . . 304
Attributions . . . . . . . . . . . . . . . . . . . . . . . . . . . . . . . . . . . . . . . . . . . . . . . . . . . . . . . . . . . . . . . . . . . . . . . . . . . . . . . . . . . . . . . . 310

Available for free at Connexions <http://cnx.org/content/col10373/1.2>


vi

Available for free at Connexions <http://cnx.org/content/col10373/1.2>


Chapter 1

Seales

1.1 Clasicacin y Propiedades de las Seales1


1.1.1 Introduccin
Este mdulo explicar algunos fundamentos para la clasicacin de seales. Es bsicamente una lista de
deniciones y propiedades que son fundamentales para la discusin de seales y sistemas. Deber notar que
2
en algunas discusiones como la de seales de energa vs. seales de potencia han sido asignadas con su
propio mdulo para una discusin mas completa, y no van a ser incluidas.

1.1.2 Clasicacin de Seales


Junto con las clasicaciones de seales mostradas a continuacin, es importante entender la Clasicacin de
Sistemas (Section 2.1).

1.1.2.1 Tiempo Continuo vs. Tiempo Discreto

Como el nombre lo sugiere, esta clasicacin se puede establecer, despus de saber si el eje del tiempo (eje
de las abscisas) es discreto o continuo (Figure 1.1). Una seal continua en el tiempo tendr un valor
para todos los nmeros reales que existen en el eje del tiempo. En contraste a esto, una seal discreta
3
(Section 1.6)en el tiempo es comnmente creada utilizando el Teorema de Muestreo para discretizar una
seal continua, de esta manera la seal nada mas tendr valores en los espacios que tienen una separacin
igual y son creados en el eje del tiempo.

1 This content is available online at <http://cnx.org/content/m12818/1.8/>.


2 "Signal Energy vs. Signal Power" <http://cnx.org/content/m10055/latest/>
3 "The Sampling Theorem" <http://cnx.org/content/m0050/latest/>

Available for free at Connexions <http://cnx.org/content/col10373/1.2>

1
2 CHAPTER 1. SEALES

Figure 1.1

1.1.2.2 Anlogo vs. Digital

La diferencia entre lo anlogo y lo digital es muy similar a la diferencia entre el tiempo continuo y el tiempo
discreto. Sin embargo, en este caso, la diferencia es con respecto al valor de la funcin (eje de las ordenadas)
(Figure 1.2). Anlogo corresponde al eje y continuo, mientras lo digital corresponde al eje y discreto. Un
ejemplo de una seal digital es una secuencia binaria, donde la funcin solo tiene valores de cero o uno.

Figure 1.2

1.1.2.3 Peridico vs. Aperidico

Seales peridicas (Section 6.1) se repiten con un periodo T , mientras las seales aperidicas o no peridicas
no se repiten(Figure 1.3). Podemos denir una funcin peridica mediante la siguiente expresin matemtica,
donde t puede ser cualquier nmero y T es una constante positiva:

f (t) = f (T + t) (1.1)

El periodo fundamental de esta funcin, f (t), es el valor ms pequeo de T que permita la validacin
de la (1.1).

Available for free at Connexions <http://cnx.org/content/col10373/1.2>


3

(a)

(b)

Figure 1.3: (a) Una seal peridica con periodo T0 (b) Una seal Aperidica

1.1.2.4 Causal vs. Anticausal vs. Nocausal

Las seales causales son seales que tienen valor de cero en el tiempo negativo, y las seales anticausales
tienen valor cero en el tiempo positivo. Las seales nocausales son seales con valor de cero en el tiempo
positivo y negativo(Figure 1.4).

Available for free at Connexions <http://cnx.org/content/col10373/1.2>


4 CHAPTER 1. SEALES

(a)

(b)

(c)

Figure 1.4: (a) Una seal causal (b) Una seal anticausal (c) Una seal nocausal

1.1.2.5 Par vs. Impar

Una seal par es cualquier seal f (t) que satisface f (t) = f (t). las seales pares se pueden detectar
fcilmente por que son simtricas en el eje vertical. Una seal impar, es una seal f que satisface
f (t) = f (t) (Figure 1.5).

Available for free at Connexions <http://cnx.org/content/col10373/1.2>


5

(a)

(b)

Figure 1.5: (a) Una seal par (b) Una seal impar

Usando las deniciones de par e impar, podemos demostrar que cualquier seal se puede escribir como
una combinacin de una seal par e impar. Cada seal tiene una descomposicin par-impar. Para demostrar
esto, no tenemos ms que examinar una ecuacin.

1 1
f (t) = (f (t) + f (t)) + (f (t) f (t)) (1.2)
2 2
Al multiplicar y sumar esta expresin, demostramos que lo explicado anteriormente es cierto. Tambin se
puede observar que f (t) + f (t) satisface a una funcin par, y que f (t) f (t) satisface a una funcin
impar (Figure 1.6).

Example 1.1

Available for free at Connexions <http://cnx.org/content/col10373/1.2>


6 CHAPTER 1. SEALES

(a)

(b)

(c)

(d)

Figure 1.6: (a)Available


Esta seal seratdescompuesta
for free usando la descomposicin Par-Impar (b) Parte Par:
Connexions <http://cnx.org/content/col10373/1.2>
e (t) = 12 (f (t) + f (t)) (c) Parte Impar: o (t) = 21 (f (t) f (t)) (d) Revisa: e (t) + o (t) = f (t)
7

1.1.2.6 Determinstico vs. Aleatorio

Una seal determinstica es una seal en la cual cada valor est jo y puede ser determinado por una
expresin matemtica, regla, o tabla. Los valores futuros de esta seal pueden ser calculados usando sus
valores anteriores teniendo una conanza completa en los resultados. Una seal aleatoria4 , tiene mucha
uctuacin respecto a su comportamiento. Los valores futuros de una seal aleatoria no se pueden predecir
5
con exactitud, solo se pueden basar en los promedios de conjuntos de seales con caractersticas similares
(Figure 1.7).

(a)

(b)

Figure 1.7: (a) Seal Determinstica (b) Seal Aleatoria

1.1.2.7 Hemisferio Derecho vs. Hemisferio Izquierdo

Este tipo de seales son aquellas cuyo valor es cero entre una variable denida y la innidad positiva o
negativa. Matemticamente hablando, una seal de hemisferio-derecho es denida como cualquier seal
donde f (t) = 0 para t < t1 < , y una seal de hemisferio-izquierdo es denida como cualquier seal donde
f (t) = 0 para t > t1 > . Las siguientes guras son un ejemplo de esto (Figure 1.8). Las dos guras
empiezan en t1 y luego se extienden a innidad positiva o negativa con casi todos los valores siendo cero.

4 "Introduction to Random Signals and Processes" <http://cnx.org/content/m10649/latest/>


5 "Random Processes: Mean and Variance" <http://cnx.org/content/m10656/latest/>

Available for free at Connexions <http://cnx.org/content/col10373/1.2>


8 CHAPTER 1. SEALES

(a)

(b)

Figure 1.8: (a) Seal de Hemisferio-Derecho (b) Seal de Hemisferio-Izquierdo

1.1.2.8 Tamao nito vs. Tamao innito

Como el nombre lo implica, las seales se pueden caracterizar dependiendo de su tamao el cual puede ser
innito o nito. Casi todas las seales nitas se utilizan cuando se tiene una seal discreta o se tiene una
secuencia de valores. En trminos matemticos, f (t) es una seal de tamao nito si tiene un valor que
no sea cero en un intervalo nito
t1 < f (t) < t2
donde t1 > y t2 < . Se puede ver un ejemplo en Figure 1.9. De igual manera, una seal de tamao
innito f (t), es denida con valores no-cero para todos los nmeros reales:

f (t)

Available for free at Connexions <http://cnx.org/content/col10373/1.2>


9

Figure 1.9: Seal de tamao nito. Note que solo tiene valores que no son cero en un conjunto, intervalo
nito.

1.2 Operaciones para Seales6


Este mdulo muestra dos operaciones para seales, cambio en el tiempo y escala en el tiempo. Operacin de
seales son operaciones realizadas sobre la variable tiempo de la seal. Estas operaciones son componentes
comunes en el mundo real y como tales las debemos entender a fondo cuando se est aprendiendo sobre
sistemas y seales.

1.2.1 Desplazamiento en el eje del Tiempo


El desplazamiento en el tiempo, como su nombre lo sugiere, es trasladar la seal en el eje del tiempo. Esto
se hace sumando o restando la cantidad del desplazamiento de tiempo a la funcin. Restando una cantidad
ja en la variable de el tiempo tendr un cambio en la seal hacia la derecha (retrasa) por esa cantidad,
por el contrario al sumar una cantidad a la variable de el tiempo la seal se desplazar hacia la izquierda
(avanza).

6 This content is available online at <http://cnx.org/content/m12823/1.7/>.

Available for free at Connexions <http://cnx.org/content/col10373/1.2>


10 CHAPTER 1. SEALES

Figure 1.10: f (t T ) mueve (retrasa) f a la derecha T .

1.2.2 Escala en el eje del Tiempo


Escalar el tiempo es comprimir y/o expandir una seal al multiplicar las variables del tiempo por alguna
cantidad. Si esa cantidad es mayor que uno, la seal se vuelve angosta, esto es conocido como compresin,
cuando la cantidad es menor que uno, la seal se vuelve ancha y a esto lo conoceremos como expansin.
Normalmente, estas operaciones les toman a las personas un tiempo en comprender, debido a que la intuicin
de las personas es que al multiplicar por una cantidad ms grande que uno la seal ser expandida y menor
que uno ser comprimida.

Figure 1.11: f (at) comprime f por a.

Example 1.2
Las seales cambiadas y escaladas en el tiempo pueden ser contrarias unas de las otras. Este
ejemplo muestra una manera de practicar estas operaciones hasta que desarrolle un sentido de
como se debera ver la seal despus de ciertas operaciones.

Available for free at Connexions <http://cnx.org/content/col10373/1.2>


11

Dado f (t) , graque f ( (at)).

(a) (b)

(c)

Figure 1.12: (a) Empieze con f (t)


` (b)
` Luego remplace t con at para obtener f (at) (c) Finalmente,
remplace t con t ab para obtener f a t ab = f (at b)

1.2.3 Reexin en el eje del Tiempo


Una pregunta muy natural que se considera cuando se est aprendiendo a escalar el tiempo es: qu pasara
si la variable del tiempo es multiplicada por un nmero negativo? La respuesta para esto es la inversin en
el tiempo. Esta operacin invierte el eje del tiempo, en otras palabras, cambia la seal respecto al eje de las
ordenadas.

Available for free at Connexions <http://cnx.org/content/col10373/1.2>


12 CHAPTER 1. SEALES

Figure 1.13: Reexin en el eje del Tiempo

1.3 Seales tiles7


Antes de ver este mdulo, usted tendr que tener una idea bsica sobre lo que es una seal, sus clasicaciones
y operaciones (Section 1.1). Como un repaso, una seal es una funcin denida con respecto a una variable
independiente. Regularmente, esta variable es el tiempo pero podra representar un ndice para una secuencia,
o un ndice para cualquier nmero de cosas, o cualquier nmero de dimensiones. La mayor parte, si es que
no todas, las seales que usted ver en sus estudios y en el mundo real podrn ser creadas de las seales
bsicas que aqu va a estudiar. Por esta razn, estas seales elementales son comnmente conocidas como
los fundamentos para cualquier otra seal.

1.3.1 Senosoidales
Probablemente la seal elemental ms importante que usted usar es el senosoidal evaluado en su parte real.
En su forma de tiempo-continuo, la forma general de la funcin se expresa as

x (t) = Acos (t + ) (1.3)

donde A es la amplitud, es la frecuencia, y representa el desplazamiento. Note que es comn ver que
t es remplazado con 2f t. Las seales senosoidales son peridicas, esto hace que su periodo, o cualquier
seal peridica puedan ser expresada de la siguiente manera

2
T = (1.4)

7 This content is available online at <http://cnx.org/content/m12819/1.11/>.

Available for free at Connexions <http://cnx.org/content/col10373/1.2>


13

Figure 1.14: Senosoidal con A = 2, w = 2, y = 0.

1.3.2 Funciones de Exponenciales Complejos


Tal vez esta seal es tan importante como la senosoidal, la funcin de exponencial complejo se convertir
en una parte crtica para el estudio de seales y sistemas. La expresin general se escribe de la siguiente
manera
f (t) = Best (1.5)

donde s, mostrado abajo, es un nmero complejo en trminos de , con una fase constante, y con siendo
la frecuencia:
s = + j
Por favor vea el mdulo de Exponencial Complejo (Section 1.5) o los mdulos de las otras seales elemen-
8
tales .

1.3.3 Exponenciales reales


Como el nombre lo implica, los exponenciales reales contienen nmeros no imaginarios y son simplemente
expresados de la siguiente manera
f (t) = Bet (1.6)

donde B y son parmetros reales. Las funciones de exponencial complejo oscilan, sin embargo, esta seal
nada mas crece o decae dependiendo del valor de .

Exponencial que decae , cuando <0


Exponencial que Crece, cuando >0
8 "Elemental Signals": Section Complex Exponentials <http://cnx.org/content/m0004/latest/#sec2>

Available for free at Connexions <http://cnx.org/content/col10373/1.2>


14 CHAPTER 1. SEALES

(a) (b)

Figure 1.15: Ejemplos de Exponenciales Reales (a) Exponencial que decae (b) Exponencial que Crece

1.3.4 Funcin de impulso unitario


La funcin de impulso unitario (Section 1.4) (o la funcin delta de Dirac) es una seal que tiene
una altura innita y un ancho casi inexistente. Sin embargo, por la manera que es denida, al ser integrada
da un valor de uno. Mientras en el mundo de ingeniera esta seal es til y ayuda a entender muchos
conceptos, algunos matemticos tienen problemas con esta al ser llamada funcin, porque no est denida
en t = 0. Los ingenieros se evitan este problema al mantenerla denida con una integral. El impulso unitario
es comnmente conocido como
(t)
La propiedad ms importante de esta funcin es demostrada con la siguiente integral:

Z
(t) dt = 1 (1.7)

1.3.5 Funcin de Escaln unitario


Otra funcin bsica para este curso es la funcin de Escaln unitario que se dene como

0 if t<0
u (t) = (1.8)
1 if t0

Available for free at Connexions <http://cnx.org/content/col10373/1.2>


15

1 1

t t
(a) (b)

Figure 1.16: Funciones Bsicas del Escaln (a) Escaln unitario de Tiempo-Continuo (b) Escaln
unitario de Tiempo-Discreto

Note que esta funcin es discontinua en el origen; sin embargo no se necesita denirla en este punto ya
que no es necesario en la teora de la seal. La funcin de Escaln unitario es una seal muy til para probar
y denir otras seales. Por ejemplo, usando varias de estas seales movidas en el tiempo y multiplicadas por
otras seales, se puede obtener alguna porcin de la seal por la que fue multiplicada y eliminar el resto.

1.3.6 Funcin Rampa


Esta funcin est relacionada con la funcin descrita anteriormente. La funcin Escaln unitario va desde
cero a uno instantneamente, pero esta funcin es la que mejor se parece a una funcin en la vida real, donde
se necesita un tiempo para que la seal vaya incrementandose desde cero a su valor ajustado, en este caso
uno. La funcin rampa est denida as:


0 if t<0


r (t) = t
t0 if 0 t t0 (1.9)


1 if t > t0

Available for free at Connexions <http://cnx.org/content/col10373/1.2>


16 CHAPTER 1. SEALES

t
t0
Figure 1.17: Funcin Rampa

1.4 Funcin de Impulso9


En ingeniera usualmente se maneja la idea de una accin ocurriendo a un determinado punto. Puede ser
una fuerza en ese punto o una seal en un punto del tiempo, se convierte necesario desarrollar alguna manera
cuantitativa de denir este hecho. Esto nos lleva a la idea de un pulso unitario. Es probablemente la segunda
seal ms importante en el estudio de seales y sistemas despus del Exponencial Complejo (Section 1.5).

1.4.1 Funcin Delta de Dirac


La Funcin Delta de Dirac, conocida tambin como el impulso unitario o funcin delta es una funcin
inntamente angosta, inntamente alta, cuya integral tiene un valor unitario (Ver (1.10) abajo). Tal vez
la manera mas simple de visualizar esto es usar un pulso rectangular que va de a 2 a a + 2 con una altura
1
de . Al momento de tomar su lmite,
 limit , podemos observar que su ancho tiende a ser cero y su altura
0
tiende a innito conforme su rea total permanece constante con un valor de uno. La funcin del impulso
usualmente se escribe como (t).
Z
(t) dt = 1 (1.10)

9 This content is available online at <http://cnx.org/content/m12824/1.9/>.

Available for free at Connexions <http://cnx.org/content/col10373/1.2>


17

Figure 1.18: Esta es una manera de visualizar la Funcin Delta de Dirac.

Figure 1.19: Por que es difcil dibujar algo que es innitamente alto, nosotros representamos la Delta
de Dirac con una echa centrada en el punto donde es aplicada. Si queremos escalarla, podemos escribir
el valor de escalamiento a un lado de la echa. Este es un muestreo unitario ( no tiene escala).

1.4.1.1 La propiedad de desplazamiento del impulso

El primer paso para comprender los resultados que esta funcin nos brinda, es examinar lo que sucede cuando
esta funcin es multiplicada por alguna otra funcin.

f (t) (t) = f (0) (t) (1.11)

Esta funcin es cero en todas partes excepto en el origen, as que bsicamente estamos eliminando el valor
de la funcin de multiplicacin al evaluarla en cero.
A primera vista esto no parece tener mayor importancia, porque ya sabemos que el impulso evaluado en
cero es innito, y todo lo multiplicado por innito da un resultado innito. Pero, qu pasa si integramos el
resultado de la multiplicacin?

Available for free at Connexions <http://cnx.org/content/col10373/1.2>


18 CHAPTER 1. SEALES

Propiedad de Desplazamiento
R R

f (t) (t) dt =
f (0) (t) dt
R
= f (0) (t) dt (1.12)

= f (0)
Finalmente lo que obtuvimos es una simple funcin evaluada en cero. Si hubiramos usado (t T ) en vez
de (t), podramos haber desplazado f (T ). A esto es lo que llamaremos la propiedad de desplazamiento
de la funcin de Dirac, el cual se usa frecuentemente para denir el impulso unitario.
Esta propiedad es muy til al momento de desarrollar la idea de convolucin (Section 3.2) la cual es
una de los fundamentos principales para el procesamiento de seales. Al usar convolucin y esta propiedad
podemos representar una aproximacin a cualquier resultado de un sistema si se conoce la respuesta al
impulso del sistema y su seal de entrada. De clic en el link de convolucin que aparece arriba para mas
informacin sobre este tema.

1.4.1.2 Otras Propiedades del Impulso

En esta seccin se muestran algunas otras propiedades de el impulso sin entrar en los detalles de probar
las propiedades- esta parte la dejaremos para que usted verique las propiedades ya que son sencillas de
comprobar. Note que estas propiedades funcionan para el tiempo continuo y discreto.
Propiedades de Impulso Unitario
1
(t) = || (t)
(t) = (t)
d
(t) = dt u (t), donde u (t) es el escaln unitario.

1.4.2 Impulso de tiempo-discreto (muestreo unitario)


La extensin de la funcin impulso unitario al tiempo-discreto se convierte en una trivialidad. Todo lo que
realmente necesitamos es darnos cuenta que la integracin en tiempo-continuo equivale a una sumatoria en
tiempo-discreto. Por lo tanto buscaremos una seal que al sumarla sea cero y al mismo tiempo sea cero en
todas partes excepto en el origen.

Impulso de Tiempo-Discreto

1 if n=0
[n] = (1.13)
0 otherwise

Figure 1.20: Representacin grca del impulso discreto

Available for free at Connexions <http://cnx.org/content/col10373/1.2>


19

Al analizar una grca de tiempo-discreto de cualquier seal discreta, uno puede notar que todas las
seales discretas estn compuestas de un conjunto de muestras unitarias que estn escalados y desplazados
en el tiempo. Si dejamos que el valor de una secuencia en cada entero k sea descrita por s [k] y la muestra
unitaria retrasado que ocurre en k sea escrito como [n k], nosotros podramos escribir cualquier seal
como la suma de impulsos unitarios retrasados que son escalados por un valor de la seal, o por coecientes
de escalamiento.

X
s [n] = s [k] [n k] (1.14)
k=

Esta descomposicin es una propiedad que solo se aplica a seales de tiempo-discreto y resulta ser una
propiedad muy til para estas seales.

note: Usando el razonamiento anterior, nosotros hemos desarrollado la ecuacin(1.14), la cual es


un concepto fundamental usado en la convolucin de tiempo-discreto (Section 4.2).

1.4.3 La Respuesta de Impulso


La respuesta de impulso es exactamente lo que su nombre implica- la respuesta de un sistema LTI, como
por ejemplo un ltro, cuando la seal de entrada del sistema es un impulso unitario (o muestra unitaria). Un
sistema puede ser completamente descrito por su respuesta al impulso por las razones explicadas previamente,
ya que todas las seales pueden ser representadas por una superposicin de seales. Una respuesta al impulso
10
da una descripcin equivalente a la dada por una funcin de transferencia , ya que existen Transformadas
de Laplace (Section 14.1) para cada una.

note: Casi toda la literatura usa (t) y [n] para diferenciar entre un impulso de tiempo-continuo
y un impulso de tiempo-discreto.

1.5 El Exponencial Complejo11


1.5.1 Bases para el Exponencial
El exponencial complejo es una de las seales mas importantes y fundamentales en el anlisis de seales
y sistemas. Su importancia proviene de que sus funciones sirven como una base para las seales peridicas,
como tambin sirven para poder caracterizar seales lineales de tiempo invariante (Section 2.1). Antes de
12
continuar, usted debera familiarizarse con los nmeros complejos .

1.5.1.1 Exponential Bsico

Para todos los nmeros x, nosotros podemos derivar y denir fcilmente una funcin exponencial de una
serie de Taylor mostrada aqu:
x1 x2 x3
ex = 1 + + + + ... (1.15)
1! 2! 3!

X 1 k
ex = x (1.16)
k!
k=0

Podemos probar, usando un examen racional, que esta serie converge. De esta manera, podemos decir que
la funcin exponencial mostrada arriba es continua y se puede denir fcilmente.

10 "Transfer Functions" <http://cnx.org/content/m0028/latest/>


11 This content is available online at <http://cnx.org/content/m12825/1.6/>.
12 "Complex Numbers" <http://cnx.org/content/m0081/latest/>

Available for free at Connexions <http://cnx.org/content/col10373/1.2>


20 CHAPTER 1. SEALES

De esta denicin, podemos probar la siguiente propiedad de las exponenciales que resulta ser muy til,
especialmente para los exponenciales que se discutirn en la siguiente seccin.

ex1 +x2 = ex1 ex2 (1.17)

1.5.1.2 Exponencial Complejo de Tiempo-Continuo

Para todos los nmeros complejos s, podemos denir una seal exponencial compleja de tiempo-
continuo como:
f (t) = Aest
(1.18)
= Aejt
donde A es una constante, t es la variable independiente tiempo, y para s imaginaria, s = j . De esta
ecuacin podemos revelar una importante identidad, la identidad de Euler (para ms informacin sobre
13
Euler lea su biografa short biography ):

Aejt = Acos (t) + j (Asin (t)) (1.19)

De la identidad de Euler podemos separar la seal en su parte imaginaria y en su parte real. Tambin
podemos observar el uso de exponenciales para representar cualquier seal real. Si modicamos la frecuencia
y el ngulo, podriamos representar cualquier seal por medio de una superposicin de muchas sealestodas
deben ser representadas por un exponencial.
La expresin anterior no incluye ninguna informacin del ngulo. Debemos generalizar la expresin de
exponenciales para generalizar funciones senosoidales con cualquier valor en el ngulo, esto se logra al hacer
una sustitucin de s, s = + j , que al nal nos lleva a

f (t) = Aest
= Ae(+j)t (1.20)

t jt
= Ae e

donde S se dene como la amplitud compleja, o fasor, de los primeros dos trminos de la ecuacin de
arriba
S = Aet (1.21)

Tomando en cuenta la identidad de Euler, podemos escribir el exponencial como un senosoidal, donde el
trmino del ngulo es mas notable.

f (t) = Aet (cos (t) + jsin (t)) (1.22)

Esta frmula se puede descomponer en su parte real e imaginaria:

Re (f (t)) = Aet cos (t) (1.23)

Im (f (t)) = Aet sin (t) (1.24)

13 http://www-groups.dcs.st-and.ac.uk/history/Mathematicians/Euler.html

Available for free at Connexions <http://cnx.org/content/col10373/1.2>


21

1.5.1.3 Exponencial Complejo en Tiempo-Discreto

Finalmente, hemos llegado a la ltima forma de seales exponenciales que nos interesan estudiar, la seal
exponencial compleja en tiempo-discreto, de la cual no daremos tantos detalles como lo hicimos para su
contraparte, ya que las dos siguen las mismas propiedades y usan la misma lgica ya explicada previamente.
Por ser discreta, tiene una diferencia en la notacin usada para representar su naturaleza discreta

f [n] = BesnT
(1.25)
= BejnT

donde nT representa los instantes de tiempo-discreto de la seal.

1.5.2 La Relacin de Euler


Junto a la identidad de Euler, Euler tambin describe una manera de representar una seal exponencial
compleja en trminos de su parte real e imaginaria usando la siguiente relacin:

ejwt + e(jwt)
cos (t) = (1.26)
2

ejwt e(jwt)
sin (t) = (1.27)
2j

ejwt = cos (t) + jsin (t) (1.28)

1.5.3 Dibujando el Exponencial Complejo


Hasta este momento, nosotros hemos demostrado como un exponencial complejo se puede separar en su
parte real e imaginaria. Ahora tenemos que ver como se gracan todas estas partes. Podemos observar
que la parte real y la parte imaginaria estn compuestas por un senosoidal multiplicado por una funcin de
exponencial real. Tambin sabemos que los senosoidales oscilan entre el valor uno y negativo uno. Entonces
se puede ver que las partes reales e imaginarias del exponencial complejo oscilarn dentro de una ventana
denida por la parte real del exponencial.

Available for free at Connexions <http://cnx.org/content/col10373/1.2>


22 CHAPTER 1. SEALES

(a) (b)

(c)

Figure 1.21: Las formas posibles para la parte real de un exponencial complejo. Note que la oscilacin
es el resultado de un coseno con un mximo local en t = 0. (a) Si es negativa, tenemos el caso de una
ventana de un exponencial que desciende. (b) Si es positivo, tenemos el caso de una ventana de un
exponencial que crece. (c) Si es cero, tenemos una ventana constante.

Mientras el determina el ndice de decrecimiento/ crecimiento, determina el ndice de las oscilaciones.


Esto se puede notar al observar que es parte del argumento usado en la parte que corresponde al senosoidal.
Exercise 1.5.1 (Solution on p. 27.)
Cmo se ven las partes imaginarias del exponencial complejo en el dibujo previo?

Example 1.3
La siguiente demostracin le permite ver como el argumento cambia la forma del exponencial
14
complejo. Por favor oprima aqu para ver las instrucciones de como se usa este demo.

This is an unsupported media type. To view, please see


http://cnx.org/content/m12825/latest/Complex_Exponential.vi

1.5.4 El Plano Complejo


15
Se convierte de extrema importancia el ver la variable compleja s como un punto en el plano complejo (el
plano-s).
14 "How to use the LabVIEW demos" <http://cnx.org/content/m11550/latest/>
15 "The Complex Plane" <http://cnx.org/content/m10596/latest/>

Available for free at Connexions <http://cnx.org/content/col10373/1.2>


23

Figure 1.22: Este es el plano-s. Note que en cualquier momento en que s se encuentre en el lado
derecho del plano, el exponencial complejo crecer con el tiempo, mientras que si por el contrario s se
encuentra en el lado izquierdo, el exponencial complejo disminuir.

1.6 Seales en Tiempo-Discreto16


Hasta este punto, hemos tratado solo con seales y sistemas anlogos. En trminos matemticos, seales
anlogas son funciones que constan de cantidades continuas como sus variables independientes, por ejemplo,
17
espacio y tiempo. Seales de tiempo-discreto son funciones denidas en nmeros enteros; son secuencias.
18
Uno de los resultados fundamentales en la teora de seales detalla las condiciones en las cuales las seales
anlogas pueden ser trasformadas en una seal de tiempo-discreto y ser recuperada sin ningn tipo de error.
Este resultado es importante por que las seales de tiempo-discreto pueden ser manipuladas por sistemas
de respuesta instantnea como los son los programas de computadoras. En los mdulos subsecuentes se
describen como todos los sistemas anlogos se pueden implementar virtualmente con el uso de software.
Sin darle importancia a estos resultados, las seales de tiempo-discreto tienen una forma ms general,
abarcando seales derivadas de seales anlogas y de otro tipo de seales. Por ejemplo, los caracteres que
forman un archivo de escritura proveniente de una secuencia, que tambin son una seal de tiempo-discreto.
19
Tambin tenemos que tratar con seales y sistemas de valor simblico .
Como en seales anlogas, buscamos distintas maneras de descomponer seales discretas con valor real
en sus componentes ms simples. Con este mtodo que nos lleva a un mayor entendimiento de estructura de
seales, podemos usar esta estructura para representar informacin (crear maneras de representar informa-
cin con seales) y de extraer informacin (extraer la informacin que es representada). Para seales de valor
simblico este mtodo es diferente: desarrollamos una representacin comn para todas las seales de valor
simblico para as representar la informacin que ellas contienen de una manera unicada. Desde el punto
de vista de la representacin de informacin, la cuestin ms importante es la eciencia para las seales de

16 This content is available online at <http://cnx.org/content/m12820/1.9/>.


17 "Discrete-Time Signals and Systems" <http://cnx.org/content/m10342/latest/>
18 "The Sampling Theorem" <http://cnx.org/content/m0050/latest/>
19 "Discrete-Time Signals and Systems" <http://cnx.org/content/m10342/latest/#para11>

Available for free at Connexions <http://cnx.org/content/col10373/1.2>


24 CHAPTER 1. SEALES

valor simblico y reales; la eciencia es la manera ms compacta y rpida de representar informacin para
que pueda ser despus extrada.

1.6.1 Seales de Valores Reales y Complejos


Una seal discreta es representada simblicamente como s (n), donde n = {. . . , 1, 0, 1, . . . }. Usualmente
dibujamos seales discretas por medio de diagramas de lnea (Stem Plots) para enfatizar el hecho que son
funciones denidas en nmeros enteros. Podemos retrasar la seal discreta por un nmero, tal como se hace
en las seales anlogas. El retraso de un muestreo unitario es expresado por (n m), y es igual a uno
cuando n = m.

Seal del Coseno en Tiempo-Discreto

sn
1

n

Figure 1.23: Seal del Coseno en Tiempo-Discreto es gracada con una "stem plot". Puede usted
encontrar la frmula para esta seal?

1.6.2 Exponenciales Complejos


La seal ms importante es la secuencia del exponencial complejo .

s (n) = ej2f n (1.29)

1.6.3 Senosoidales
Los senosoidales discretos tienen la forma de s (n) = Acos (2f n + ). Al contrario de exponenciales com-
plejos y senosoidales anlogos que pueden tener frecuencias con cualquier valor real, frecuencias para seales
21 , 12

discretas dan formas nicas solo cuando f est en el intervalo . Esta propiedad se puede tener al
notar que cuando se suma un valor a la frecuencia del exponencial complejo discreto no afecta en ninguna
manera a los valores de la seal.
ej2(f +m)n = ej2f n ej2mn
(1.30)
= ej2f n
La derivacin es porque el exponencial complejo evaluado en un mltiplo de 2 es igual a uno.

Available for free at Connexions <http://cnx.org/content/col10373/1.2>


25

1.6.4 Muestreo Unitario


La segunda seal importante en el tiempo discreto, est denida por:

1 if n=0
(n) = (1.31)
0 otherwise

Muestreo Unitario

n
1

Figure 1.24: Muestreo Unitario.

Al examinar la grca de seales discretas, como el coseno mostrado en la gura Figure 1.23 ( Seal del
Coseno en Tiempo-Discreto ), se puede observar que todas la seales consisten en muestreos unitarios que
son desplazados y escalados por un valor real. El valor de una secuencia a cualquier nmero m es escrito por
s (m) y el desplazamiento que ocurre en m es escrito por (n m), por esta razn podemos descomponer
cualquier seal en una suma de muestras unitarias desplazadas a una localizacin apropiada y escalada
por el valor de una seal.

X
s (n) = s (m) (n m) (1.32)
m=

Este tipo de descomposicin es nica para seales discreta.


Sistemas discretos pueden actuar sobre seales en tiempo discreto en forma similar a las vistas en seales y
sistemas anlogos. Debido al rol que juega el software sobre sistemas discretos, una gran variedad de sistemas
pueden ser desarrolladas y construidas a diferencia de las que se pueden lograr usando seales anlogas. De
hecho, una clase especial de seales anlogas pueden ser convertidas en seales discretas, procesadas por
software, y convertidas despus en seales anlogas, todo esto sin errores. Para estas seales, varios sistemas
pueden ser producidos en software, con realizaciones anlogas equivalentes siendo difciles de formar, si no
es que imposibles de disear.

1.6.5 Seales de Valores Simblicos


Otro aspecto interesante de seales discretas es que sus valores no tienen que ser nmeros reales. Nosotros
si tenemos seales discretas con valores reales como el sinusoidal, pero tambin tenemos seales que indican
una secuencia de nmeros usados en el teclado de computadoras. Esos caracteres no son nmeros reales,
y como posible coleccin de valores, tienen muy poca estructura matemtica y nada ms constante con el
hecho que son miembros de un conjunto. Cada elemento de una seal de valores simblicos s (n) toma valores
{a1 , . . . , aK } que forman parte de un alfabeto A. Esta terminologa tcnica no restringe los smbolos a
ser miembros de un alfabeto del idioma ingles o griego. Ellos pueden representar caracteres en un teclado,
byte (secuencias de 8-bits), nmeros que pudieran signicar una temperatura. Los sistemas digitales son
construidos de circuitos digitales, que consisten completamente de circuitos con elementos anlogos. La
retransmisin y recepcin de seales discretas, como el correo electrnico, son posibles gracias al uso de

Available for free at Connexions <http://cnx.org/content/col10373/1.2>


26 CHAPTER 1. SEALES

sistemas y seales anlogas. Entender como las seales discretas y anlogas se interelacionan una con otra
es el objetivo principal de este curso.

Available for free at Connexions <http://cnx.org/content/col10373/1.2>


27

Solutions to Exercises in Chapter 1


Solution to Exercise 1.5.1 (p. 22)
Se ve igual excepto que la oscilacin es senosoidal y no cosenoidal (pasa por el origen y no tiene ningn
mximo local en t = 0).

Available for free at Connexions <http://cnx.org/content/col10373/1.2>


28 CHAPTER 1. SEALES

Available for free at Connexions <http://cnx.org/content/col10373/1.2>


Chapter 2

Sistemas

2.1 Clasicacin y Propiedades de los Sistemas1


2.1.1 Introduccin
En este mdulo algunas de las clasicaciones bsicas de sistemas sern temporalmente introducidas mientras
que las propiedades ms importantes de sistemas sern explicadas. Como puede ser visto, las propiedades de
los sistemas proveen una manera sencilla de separar un sistema de otro. Entender la diferencia bsica entre
sistemas, y sus propiedades, ser un concepto fundamental utilizado en todos los cursos de seales y sistemas,
as como de procesamiento digital de seales (Digital Signal Processing) DSP. Una vez que el conjunto de
seales puede ser identicado por compartir propiedades particulares, uno ya no tiene que proveer ciertas
caractersticas del sistema cada vez, pero pueden ser aceptadas debido a la clasicacin de los sistemas.
Tambin cabe recordar que las clasicaciones presentadas aqu pueden no ser exclusivas (los sistemas pueden
2
pertenecer a diferentes clasicaciones) ni nicas (hay otros mtodos de clasicacin ). Algunos ejemplos de
3
sistemas simples se podrn encontrar aqui .

2.1.2 Classicacin de los Sistemas


4
A travs de la siguiente clasicacin, tambin es importante entender otras Clasicaciones de Seales .

2.1.2.1 Contino vs. Discreto

Esta tal vez sea la clasicacin ms sencilla de entender como la idea de tiempo-discreto y tiempo continuo
que es una de las propiedades ms fundamentales de todas las seales y sistemas. Un sistema en donde las
seales de entrada y de salida son continuas es unsistema continuo, y uno en donde las seales de entrada
y de salida son discretas es un sistema discreto .

2.1.2.2 Lineal vs. No-lineal

Un sistema lineal es un sistema que obedece las propiedades de escalado (homogeneidad) y de superposicin
(aditiva), mientras que un sistema no-lineal es cualquier sistema que no obedece al menos una de estas
propiedades.
Para demostrar que un sistema H obedece la propiedad de escalado se debe mostrar que:

H (kf (t)) = kH (f (t)) (2.1)

1 This content is available online at <http://cnx.org/content/m12822/1.4/>.


2 "Introduction to Systems" <http://cnx.org/content/m0005/latest/>
3 "Simple Systems" <http://cnx.org/content/m0006/latest/>
4 "Signal Classications and Properties" <http://cnx.org/content/m10057/latest/>

Available for free at Connexions <http://cnx.org/content/col10373/1.2>

29
30 CHAPTER 2. SISTEMAS

Figure 2.1: Un diagrama de bloque demostrando la propiedad de escalado de linealidad

Para demostrar que un sistema H obedece la propiedad de superposicin de linealidad se debe mostrar
que:
H (f1 (t) + f2 (t)) = H (f1 (t)) + H (f2 (t)) (2.2)

Figure 2.2: Un diagrama de bloque demostrando la propiedad de superposicin de linealidad

Es posible vericar la linealidad de un sistema en un paso sencillo. Para hace esto, simplemente combi-
namos los dos primero pasos para obtener

H (k1 f1 (t) + k2 f2 (t)) = k2 H (f1 (t)) + k2 H (f2 (t)) (2.3)

2.1.2.3 Invariante en el Tiempo vs. Variante en el Tiempo

Un sistema invariante en el tiempo es aquel que no depende de cuando ocurre: la forma de la salida no
cambia con el retraso de la entrada. Es decir que para un sistema H donde H (f (t)) = y (t), H es invariante
en el tiempo si para toda T
H (f (t T )) = y (t T ) (2.4)

Available for free at Connexions <http://cnx.org/content/col10373/1.2>


31

Figure 2.3: Este diagrama de bloque muestra la condicin de la invariante en el tiempo. La Salida es
la misma si el retraso es colocado en la entrada o en la salida.

Cuando esta propiedad no aplica para un sistema, entonces decimos que el sistema es variante en el
tiempo o que vara en el tiempo.

2.1.2.4 Causal vs. No-Causal

Un sistema causal es aquel que es no-anticipativo; esto es, que las salidas dependen de entradas presentes
y pasadas, pero no de entradas futuras. Todos los sistemas en tiempo real deben ser causales, ya que no
pueden tener salidas futuras disponibles para ellos.
Uno puede pensar que la idea de salidas futuras no tiene mucho sentido fsico; sin embargo, hasta ahora
nos hemos estado ocupando solamente del tiempo como nuestra variable dependiente, el cual no siempre es
el caso. Imaginmonos que quisiramos hacer procesamiento de seales; Entonces la variable dependiente
representada por los pxeles de la derecha y de la izquierda (el futuro) de la posicin actual de la imagen.
Entonces tendramos un sistema no-causal.

Available for free at Connexions <http://cnx.org/content/col10373/1.2>


32 CHAPTER 2. SISTEMAS

(a)

(b)

Figure 2.4: (a) Para que un sistema tpico sea causal... (b) ...la salida en tiempo t0 , y (t0 ), puede
solamente depender de la porcin de la seal de entrada antes t0 .

2.1.2.5 Estable vs. Inestable

Un sistema estable es uno donde las salidas no divergen as como las entradas tampoco divergen. Hay
muchas maneras de decir que una seal diverge; por ejemplo puede tener energa innita. Una denicin
particularmente til de divergencia es relacionar si la seal esta acotada o no. Entonces se reere al sistema
como entrada acotada-salida acotada (BIBO) (Bounded input-bounded output) establece que toda
posible entrada acotada produce una salida acotada.
Representado esto de una manera matemtica, un sistema estable debe tener las siguientes
propiedades,donde x (t) es la entrada y y (t) es la salida; la cual debe satisfacer la condicin

|y (t) | My < (2.5)

cuando tenemos una entrada del sistema esta puede ser descrita como

|x (t) | Mx < (2.6)

Mx y My ambas representan un conjunto de nmeros enteros positivos y esta relacin se mantiene para
toda t.
Si estas condiciones no son satisfechas, es decir, las salidas del sistema con entrada acotada crecen sin
limite (divergen), entonces el sistema es inestable. Notemos que la estabilidad BIBO de un sistema lineal
5
invariante en el tiempo (LTI) es descrito cuidadosamente en trminos de si es o no completamente integrable
la respuesta al impulso.

5 "BIBO Stability of Continuous Time Systems" <http://cnx.org/content/m10113/latest/>

Available for free at Connexions <http://cnx.org/content/col10373/1.2>


33

2.2 Propiedades de los Sistemas6


2.2.1 Sistemas Lineales
Si un sistema es lineal, quiere decir que cuando la entrada de un sistema dado es escaldado por un valor, la
salida del sistema es escalado por la misma cantidad.

Escalado Lineal

(a) (b)

Figure 2.5

En la Figure 2.5(a) de arriba, la entrada x del sistema lineal L da la salida y. Si x es escalada por un
valor y es pasada a travs del mismo sistema, como en la Figure 2.5(b), la salida tambin ser escalada
por .
Un sistema lineal tambin obedece el principio de superposicin. Esto signica que si dos entradas son
sumadas juntas y pasadas a travs del sistema lineal, la salida ser equivalente a la suma de las dos entradas
evaluadas individualmente.

(a) (b)

Figure 2.6

6 This content is available online at <http://cnx.org/content/m12826/1.2/>.

Available for free at Connexions <http://cnx.org/content/col10373/1.2>


34 CHAPTER 2. SISTEMAS

Principio de Superposicin

Figure 2.7: Si Figure 2.6 es cierto, entonces el principio de superposiccin dice que Figure 2.7 (Principio
de Superposicin) tambin es cierto. Esto es vlido para un sistema lineal.

Esto es, si Figure 2.6 es cierta, entonces Figure 2.7 (Principio de Superposicin) tambin es cierta para
un sistema lineal. La propiedad de escalado mencionada anteriormente tambin es vlida para el principio de
superposicin. Por lo tanto, si las entradas x y y son escaladas por factores y , respectivamente, entonces
la suma de estas entradas escaladas dar la suma de las salidas escaladas individualmente.

(a) (b)

Figure 2.8

Principio de Superposicin con Escaldo Lineal

Figure 2.9: Dado Figure 2.8 para un sistema lineal, Figure 2.9 (Principio de Superposicin con Escaldo
Lineal) tambin es vlido.

2.2.2 Time-Invariant Systems


Un sistema invariante en el tiempo TI (Time-Invariant) tiene la propiedad de que cierta entrada siempre
dar la misma salida, sin consideracin alguna a cuando la entrada fue aplicada al sistema.

Available for free at Connexions <http://cnx.org/content/col10373/1.2>


35

Sistema Invariante en el Tiempo

(a) (b)

Figure 2.10: Figure 2.10(a) muestra una entrada en tiempo t mientras que Figure 2.10(b) muestra la
misma entrada t0 segundos despus. En un sitema invariante en el tiempo ambas salidas sern identicas
excepto la de la Figure 2.10(b) estar retrasada por t0 .

En esta gura, x (t) x (t t0 ) son pasadas a travs del sistema TI. Ya que el sistema TI es invariante
y
en el tiempo, las entradas x (t) y x (t t0 ) producen la misma salida. La nica diferencia es que la salida
debida a x (t t0 ) es cambiada por el tiempo t0 .
Si un sistema es invariante en el tiempo o de tiempo variado puede ser visto en la ecuacin diferencial
(o ecuacin en diferencia) descrita. Los sistemas invariantes en el tiempo son modelados con ecua-
ciones de coecientes constantes. Una ecuacin diferencial(o en diferencia) de coecientes constantes
signica que los parmetros del sistema no van cambiando a travs del tiempo y que la entrada nos dar el
mismo resultado ahora, as como despus.

2.2.3 3 Sistemas Lineales Invariantes en el Tiempo (LTI)


A los sistemas que son lineales y al mismo tiempo invariantes en el tiempo nos referiremos a ellos como
sistemas LTI (Linear Time-Invariant).

Sistemas Lineales Invariantes en el Tiempo

(a) (b)

Figure 2.11: Esto es una combinacin de los dos casos de arriba. Dado que la entrada Figure 2.11(b)
es una versin escalada y desplazada en el tiempo de la entrada de Figure 2.11(a), tambin es la salida.

Como los sistemas LTI son subconjuntos de los sistemas lineales, estos obedecen al principio de super-
posicin. En la gura de abajo, podemos ver el efecto de aplicar el tiempo invariante a la denicin de
sistema lineal de la seccin anterior.

Available for free at Connexions <http://cnx.org/content/col10373/1.2>


36 CHAPTER 2. SISTEMAS

(a) (b)

Figure 2.12

Superposicin en Sistemas Lineales Invariantes en el Tiempo

Figure 2.13: El principio de superposicin aplicadado a un sistema LTI

2.2.3.1 Sistemas LTI en Series

Si dos o mas sistemas estn en serie uno con otro, el orden puede ser intercambiado sin que se vea afectada
la salida del sistema. Los sistemas en series tambin son llamados como sistemas en cascada.

Available for free at Connexions <http://cnx.org/content/col10373/1.2>


37

Sistema LTI en Cascada

(a)

(b)

Figure 2.14: El orden de los sistemas LTI en cascada pueden ser intercambiado sin verse afectado el
resultado.

2.2.3.2 Sistemas LTI en Paralelo

Si dos o mas sistemas LTI estn en paralelo con otro, un sistema equivalente es aquel que esta denido como
la suma de estos sistemas individuales.

Sistemas LTI en Paralelo

(a) (b)

Figure 2.15: Los sistemas de paralelo pueden ser resumidos en la suma de los sistemas.

Available for free at Connexions <http://cnx.org/content/col10373/1.2>


38 CHAPTER 2. SISTEMAS

2.2.4 Causalidad
Un sistema es causal si este no depende de valores futuros de las entradas para determinar la
salida. Lo que signica que si la primer entrada es recibida en tiempo t0 , el sistema no deber dar ninguna
salida hasta ese tiempo. Un ejemplo de un sistema no-causal puede ser aquel que al detectar que viene un
entrada da la salida antes de que la entrada llegue.

Sistema no-Causal

Figure 2.16: En este sistema no-causal, la salida es producida dado a una entradad que ocurri despus
en el tiempo.

Un sistema causal tambin se caracteriza por una respuesta al impulso h (t) que es cero para t < 0.

Available for free at Connexions <http://cnx.org/content/col10373/1.2>


Chapter 3

Anlisis de Sistemas de Tiempo


Continuo en el Dominio del Tiempo

3.1 Sistemas Lineales CT y Ecuaciones Diferenciales1


3.1.1 Sistemas Lineales de Tiempo-Continuo
Fsicamente realizable, los sistemas lineales invariantes en el tiempo pueden ser descritos por un conjuto de
ecuaciones lineales diferenciales:

Figure 3.1: Describcin grca de un sistema bsico lineal invariante en el tiempo con una entrada,
f (t) y una salida, y (t).

dn y (t) dn1 y (t) dy (t) dm f (t) df (t)


n
+ an1 n1
+ + a1 + a0 y (t) = bm + + b1 + b0 f (t)
dt dt dt dtm dt
Equivalentemente,
n m
X di y (t) X di f (t)
ai = bi (3.1)
i=0
dti i=0
dti
con an = 1.
Es fcil mostra que estas ecuaciones denen un sistema que es lineal e invariante en el tiempo. Entonces
una pregunta natural es cmo encontrar la respuesta al impulso del sistema y (t) para una entrada f (t)?.
Recordando que tal solucin se puede escribir como:

y (t) = yi (t) + ys (t)


Nos referimos a yi (t) como la respuesta de salida-cero  la solucin homogenea debido a las condiciones
iniciales del sistema. Nos referimos a ys (t) como la respuesta de estado-cero  la solucin particular en
respuesta a la entrada f (t). Ahora veremos como resolver cada una de estos componentes de la respuesta
del sistema.
1 This content is available online at <http://cnx.org/content/m12985/1.2/>.
Available for free at Connexions <http://cnx.org/content/col10373/1.2>

39
CHAPTER 3. ANLISIS DE SISTEMAS DE TIEMPO CONTINUO EN EL
40
DOMINIO DEL TIEMPO

3.1.1.1 Encontrando la Respuesta de Entrada-Cero

La respuesta de la entrada-cero, yi (t), es la respuesta del sistema debido solo a las condiciones iniciales .

Example 3.1: Respuesta de la Entrada-Cero


Poner un voltaje a travs de una capacitor en un circuito dibujado a continuacin y deje todo lo
dems solo.

Figure 3.2

Example 3.2: Respuesta de la Entrada-Cero


Imagine una masa unidia a un resorte como se muestra a continuacin. Cuando jala la masa hacia
abajo y la suelte, usted tiene un ejemplo de la respuesta de la entrada-cero.

Figure 3.3

No hay entrada, as que resolvemos para: y0 (t) tal que

n
X di y0 (t)
ai = 0 , an = 1 (3.2)
i=0
dti

Si D es el operador derivado, podemos escribir la ecuacin anterior como:

Dn + an1 Dn1 + + a0 y0 (t) = 0



(3.3)

nPuesto que necesitamos o una suma de varios y0 (t)'s dervados para ser 0 para todo t, entonces
dy0 (t) d2 y0 (t)
y0 (t) , dt , dt2 , . . . deben de ser todos de la misma forma.
Solo el exponencial, est donde s C, tiene esta propiedad (vase un libro de texto de Ecuaciones
Diferenciales para ms detalles). As que asumimos que,

y0 (t) = cest , c 6= 0 (3.4)

para algun c y s.
d d2
Since
dt y0 (t) = csest , dt2 y0 (t) = cs2 est , . . . tenemos

Dn + an1 Dn1 + + a0 y0 (t) = 0




c sn + an1 sn1 + + a1 s + a0 est = 0



(3.5)

(3.5) se mantiene para todo t solo cuando

sn + an1 sn1 + + a1 s + a0 = 0 (3.6)

Available for free at Connexions <http://cnx.org/content/col10373/1.2>


41

Donde esta ecuacin es conocida como la ecuacin caracterstica del sistema. Los posibles valores de s
son las raices o ceros de este polinomio {s1 , s2 , . . . , sn }

(s s1 ) (s s2 ) (s s3 ) . . . (s sn ) = 0

es decir las posibles soluciones son: c1 es1 t , c2 es2 t , . . ., cn esn t . Ya que el sistema es lineal, la solucin general
es de la forma:
y0 (t) = c1 es1 t + c2 es2 t + + cn esn t (3.7)

Entonces, resolver para {c1 , . . . , cn } usando las condiciones iniciales.

Example 3.3
Vase Lathi p.108 para un buen ejemplo.

Generalmente asumimos que las IC's de un sistema son cero, lo que implica que yi (t) = 0. Sin embargo,
elmtodo de resolver para yi (t) se probar ms adelante.

3.1.1.2 Encontrando la Respuesta del Estado-Cero

Resolviendo una ecuacin lienal diferencial

n m
X di y (t) X di f (t)
ai = bi (3.8)
i=0
dti i=0
dti

dada una entrada especca f (t) es una tarea difcil en general. Ms importantemente, el mtodo depende
completamente en la naturaleza de f (t); si cambiamos la seal de entrada, debemos de re-resolver el sistema
de ecuaciones para encontrar la respuesta del sistema.
La Convolucin (Section 3.2) nos ayuda a pasar estas dicultades. En la seccin explicamos como la
convolucin nos ayuda a determinar la salida del sistema, dada solo la entrada f (t) y la respuesta al impulso
2
del sistema, h (t).
Antes de derivar el procedimiento de convolucin, mostramos que la respuesta al impulso es derivada
facilmente de su ecuacin lineal diferencial (LDE). Mostraremos la derivacin del siguiente LDE, donde
m < n:
dn y (t) dn1 y (t) dy (t) dm f (t) df (t)
n
+ a n1 n1
+ + a1 + a0 y (t) = b m m
+ + b1 + b0 f (t) (3.9)
dt dt dt dt dt
Podemos reescribir (3.9) como
QD [y (t)] = PD [f (t)] (3.10)

donde QD [] es una operador que mapea y (t) al lado izquierdo de la (3.9)

dn y (t) dn1 y (t) dy (t)


QD [y (t)] = n
+ an1 n1
+ + a1 + a0 y (t) (3.11)
dt dt dt
y PD [] mapea f (t) al lado derecho de la (3.9). Lathi muestra (en el Apendice 2.1) que la respuesta al
impulso del sistema descrito por (3.9) es dado por:

h (t) = bn (t) + PD [yn (t)] (t) (3.12)

donde para m < n we have bn = 0. Tambin, yn igual a la respuesta salida-cero con las condiciones
iniciales.
y n1 (0) = 1, y n2 (0) = 1, . . . , y (0) = 0


2 "Photo Story 3 Tech Module" <http://cnx.org/content/m18124/latest/>

Available for free at Connexions <http://cnx.org/content/col10373/1.2>


CHAPTER 3. ANLISIS DE SISTEMAS DE TIEMPO CONTINUO EN EL
42
DOMINIO DEL TIEMPO

3.2 Convolucin de Tiempo-Continuo 3

3.2.1 Motivacin
La convolucin nos ayuda a determinar el efecto que tiene el sistema en la seal de entrada. Puede ser visto
que el sistema lineal de tiempo invariante (Section 2.1) es completamente caracterizado por su respuesta
al impulso. A primera vista, esto puede parecer de pequeo uso, ya que las funciones de impulso no estn
bien denidas en aplicaciones reales. Sin embargo la propiedad de desplazamiento del impulso nos dice
que una seal puede ser descompuesta en una suma innita (integral) de impulsos escalados y desplazados.
Conociendo como un sistema afecta un impulso simple, y entendiendo la manera en que una seal es abarcada
por impulsos escaldos y sumados, suena razonable que sea posible escalar y sumar la respuesta al impulso
a un sistema en para poder determinar que seal de salida resultara de una entrada en particular. Esto es
la convolucin determina la salida del sistema por medio
precisamente lo que la convolucin hace -
conocimiento de la entrada y la respuesta al impulso del sistema.
En el resto de este modulo, vamos a examinar exactamente como la convolucin es denida a partir
del razonamiento anterior. Esto resultara en la integral de convolucin (vase la siguiente seccin) y sus
propiedades (Section 3.3). Estos conceptos son muy importantes en la Ingeniera Elctrica y harn la vida
de los ingenieros mas sencilla si se invierte el tiempo en entender que es lo que esta pasando.
Para poder entender completamente la convolucin, ser de utilidad tambin ver la convolucin de tiempo
4 5
discreto ). Tambin ser de gran ayuda experimentar con los applets disponibles en internet. Este recurso
nos ofrecer una aproximacin mas crucial del concepto.

3.2.2 Integral de Convolucin


Como mencionamos anteriormente, la integral de convolucin nos da una manera matemtica fcil de expresar
la salida de un sistema LTI basado en una seal arbitraria, x (t),y la respuesta al impulso, h (t). La integral
de convolucin es expresada como
Z
y (t) = x ( ) h (t ) d (3.13)

La convolucin es una herramienta muy importante que es representada por el smbolo , y puede ser escrita
como
y (t) = x (t) h (t) (3.14)

Haciendo unos cambios simples en las variables de la integral de convolucin, = t , podemos ver que
la convolucin es conmutativa:
x (t) h (t) = h (t) x (t) (3.15)

Para ms informacin de las caractersticas de la integral de convolucin, lase sobre la Propiedades de la


Convolucin (Section 3.3).
Ahora presentaremos dos aproximaciones distintas que se derivan de la integral de convolucin. Estos
procesos, junto con un ejemplo bsico, nos ayudaran para construir una intuicin sobre la convolucin.

3.2.3 Proceso I: El mtodo corto


Este proceso sigue de cerca el mencionado en la seccin anterior en la Motivacin (Section 3.2.1: Motivacin).
Para iniciar esto, es necesario establecer las asunciones que haremos. En este momento, la nica obligada
en nuestro sistema es que este sea lineal e invariante en el tiempo.

Breve descripcin de los pasos de este Proceso:


3 This content is available online at <http://cnx.org/content/m12828/1.4/>.
4 "Discrete Time Convolution" <http://cnx.org/content/m10087/latest/>
5 http://www.jhu.edu/signals

Available for free at Connexions <http://cnx.org/content/col10373/1.2>


43

1. Un impulso de entrada, nos da como salida una respuesta al impulso.


2. Un impulso desplazado nos da como salida una respuesta al impulso desplazada. Esto es debido a la
invariante en el tiempo del sistema
3. Podemos escalar el impulso de entrada para obtener como salida un impulso escaldo. Esto es usando
la propiedad de linealidad de la multiplicacin escalar.
4. Podemos sumar un nmero innito de estos impulsos escalados para obtener un nmero innito de
sumas de respuestas al impulso escaladas. Esto es usando la cualidad de la aditividad de linealidad.
5. Ahora vemos que esta suma innita no es mas que una integral, as que podemos convertir ambos lados
en integrales.
6. Reconociendo que la entrada es la funcin f (t), tambin reconocemos que la salida es exactamente la
integral de convolucin .

Figure 3.4: Empezamos con un sistema denido por su respuesta al impulso, h (t).

Figure 3.5: Despus consideramos una versin desplazada del impulso de entrada. Debido al tiempo
invariante del sistema, obtenemos una versin de una salida desplazada de la respuesta al impulso.

Available for free at Connexions <http://cnx.org/content/col10373/1.2>


CHAPTER 3. ANLISIS DE SISTEMAS DE TIEMPO CONTINUO EN EL
44
DOMINIO DEL TIEMPO

R1 R1
1f ( )(t  ) d h 1f ( )h(t  ) d

Figure 3.6: Ahora usamos la parte de escalado de linealidad, escalando el sistemas por un valor, f ( ),
que es constante con respecto al sistema variable, t.

Figure 3.7: Ahora podemos usar el aspecto de aditividad de linealidad para sumar un nmero innito
de estos, para cada posible . Como una suma innita es exactamente una integral, terminamos con
la integral conocida como integral de convolucin. Usando la propiedad de desplazamiento, podemos
reconocer el lado izquierdo como la entrada f (t).

3.2.4 Proceso II: El mtodo largo


Este mtodo realmente no es muy diferente del anterior, sin embargo es un poco mas riguroso y mas largo.
Esperanzadamente si no se comprendi bien el mtodo de arriba, esto te ayudara para terminar de entender
la convolucin.
6
El primer paso en este mtodo
es denir una realizacin particular de la funcin de impulso unitario .
1

if 2 <t< 2
Para esto usaremos (t) =
0 otherwise

6 "Photo Story 3 Tech Module" <http://cnx.org/content/m18124/latest/>

Available for free at Connexions <http://cnx.org/content/col10373/1.2>


45

Figure 3.8: La realizacin de la funcin de impulso unitario que usaremos para esta derivacin.

Despus de denir la realizacin de la unidad de respuesta al impulso, podemos obtener nuestra integral
de convolucin de los siguientes pasos que se encuentran en la siguiente tabla. Notemos que la columna de
la izquierda representa la entrada y la columna de la derecha es la salida del sistema dada esa entrada.

Proceso II de la Integral de Convolucin

Entrada Salida

limit (t) h limit h (t)


0 0
limit (t n) h limit h (t n)
0 0
limit f (n) (t n) h limit f (n) h (t n)
0 0
P P
limit n f (n) (t n) h limit n f (n) h (t n)
R0
R0


f ( ) (t ) d h
f ( ) h (t ) d
R
f (t) h y (t) = f ( ) h (t ) d

Table 3.1

3.2.5 Implementacin de la Convolucin


Tomando una visin mas cercana de la integral de convolucin, encontramos que estamos multiplicando la
seal de entrada por una respuesta al impulso invertida en tiempo e integrndola. Esto nos dar el valor de la
salida de un valor dado de t. Si despus nosotros cambiamos la respuesta al impulso invertida en tiempo por
una pequea cantidad, obtenemos la salida para otro valor de t. Repitiendo esto para cada posible valor de t,
nos da la funcin total de salida. Nosotros nunca haremos este mtodo a mano, nos proporciona informacin
con algunas entradas de que es lo que esta pasando. Encontramos que esencialmente estamos revirtiendo
la funcin de la respuesta al impulso y desplazndola a travs de la funcin de entrada, integrndola como
vamos. Este mtodo, descrito como el metodo grco, nos proporciona una manera sencilla de resolver

Available for free at Connexions <http://cnx.org/content/col10373/1.2>


CHAPTER 3. ANLISIS DE SISTEMAS DE TIEMPO CONTINUO EN EL
46
DOMINIO DEL TIEMPO

las salidas para seales sencillas, mientras tanto mejoraremos nuestra intuicin para los casos mas complejos
7
donde conaremos en las computadoras. De hecho las Texas Instruments se convierten en Procesamiento
8
Digital de Seales las cuales tienen un conjunto especial para cmputos as como para la convolucin.

Example 3.4
9
Esta demostracin ilustrara el mtodo grco para la convolucin. Vase aqu para las instruc-
ciones de cmo se usa este demo.
This media object is a LabVIEW VI. Please view or download it at
<CT_Convolution.llb>

3.2.6 Ejemplos Bsicos


Veamos ejemplos de la convolucin bsica de tiempo-continuo para poder expresar algunas ideas mencionadas
anteriormente en el ejemplo corto. Vamos a convolver dos pulsos unitarios, x (t) y h (t).

(a) (b)

Figure 3.9: Aqui hay dos seales bsicas que usaremos para convolucionar juntas.

3.2.6.1 Reejada y Desplazada

Ahora tomaremos una de las funciones y la reejaremos a travs del eje de las y. Despus debemos desplazar
la funcin, as como el origen, el punto de la funcin que originalmente estaba en el origen, esta marcada como
el punto . Este paso se muestra en la siguiente gura, h (t ). Como la convolucin es conmutativa no
importa que funcin es reejada y desplazada, sin embargo conforme las funciones se vuelven mas complicadas
reejar y desplazar la correcta nos har el problema mas sencillo.

7 http://www.ti.com
8 http://dspvillage.ti.com/docs/toolssoftwarehome.jhtml
9 "How to use the LabVIEW demos" <http://cnx.org/content/m11550/latest/>

Available for free at Connexions <http://cnx.org/content/col10373/1.2>


47

Figure 3.10: El pulso unitario reejado y desplazado.

3.2.6.2 Regiones de Integracin

Ahora veremos la funcin y dividiremos su dominio en dos lmites diferentes de integracin. Estas dos
regiones diferentes pueden ser entendidas pensando en como nos desplazamos h (t ) sobre la otra funcin.
Si la funcin fuera mas complicada necesitaremos tener mas limites para que las partes sobrepuestas de
las dos funciones puedan ser expresadas en una integral lineal sencilla. Para este problema tendremos las
siguientes cuatro regiones. Comprense estos limites de integracin con los de los bosquejos de h (t )
y x (t) para ver si se puede entender el por que tenemos cuatro regiones. Ntese que t en los limites de
integracin se reere al lado derecho de la funcin h (t ), marcada como t entre cero y uno en la grca.

Los cuatro limites de integracin

1. t<0
2. 0t<1
3. 1t<2
4. t2

3.2.6.3 Usando la Integral de Convolucin

Finalmente estamos listos para unas pequeas matemticas. Usando la integral de convolucin, integremos
el producto de x (t) h (t ). Para la primer y cuarta regin es trivial pues siempre ser, 0. La segunda
regin, 0 t < 1, requiere las siguientes matemticas:

Rt
y (t) = 0
1d
(3.16)
= t

La tercer regin, 1 t < 2, Tomemos nota de los cambios en nuestra integracin. Conforme movemos
h (t ) a travs de la otra funcin, la parte izquierda de la funcin, t 1, se convierte es nuestro limite

Available for free at Connexions <http://cnx.org/content/col10373/1.2>


CHAPTER 3. ANLISIS DE SISTEMAS DE TIEMPO CONTINUO EN EL
48
DOMINIO DEL TIEMPO

inferior para la integral. Esto se muestra a travs de la integral de convolucin como


R1
y (t) = t1
1d
= 1 (t 1) (3.17)

= 2t
Las formulas anteriores muestran el mtodo para calcular la convolucin; sin embargo no deje que la
simplicidad de este ejemplo lo confunda cuando trabaje con otros problemas. El mtodo ser el mismo, solo
se tendr que tratar con mas matemticas en integrales mas complicadas.

3.2.6.4 Resultados de la Convolucin

As, obtenemos los siguientes resultados para nuestras cuatro regiones:





0 if t < 0

t if 0 t < 1

y (t) = (3.18)
2 t if 1 t < 2




0 if t 2

Ahora que encontramos el resultado para cada una de las cuatro regiones, podemos combinarlas y gracar
la convolucin de x (t) h (t).

Figure 3.11: Muestra la respuesta al impulso de la entrada, x (t).

3.3 Propiedades de la Convolucin10


En este modulo veremos varias de las propiedades de convolucin que mas prevalecen. Ntese que estas
propiedades se aplican a ambas convoluciones de tiempo continuo (Section 3.2) y de tiempo discreto (Sec-
tion 4.2). (Vase los dos mdulos anteriores si necesita un repaso de convolucin). Tambin para algunas

10 This content is available online at <http://cnx.org/content/m12829/1.2/>.

Available for free at Connexions <http://cnx.org/content/col10373/1.2>


49

demostraciones de las propiedades, usaremos las integrales de tiempo-continuo, pero podemos probarlas de
la misma manera usando las sumatorias de tiempo-discreto.

3.3.1 Asociatividad
Theorem 3.1: Ley Asociativa

f1 (t) (f2 (t) f3 (t)) = (f1 (t) f2 (t)) f3 (t) (3.19)

Figure 3.12: Implicacin grca de la propiedad de asociatividad de la convolucin.

3.3.2 Conmutatividad
Theorem 3.2: : Ley Conmutativa

y (t) = f (t) h (t)


(3.20)
= h (t) f (t)

Proof:
Para probar la (3.20), lo nico que tenemos que hacer es un pequeo cambio de variable en nuestra
integral de convolucin (o suma),

Z
y (t) = f ( ) h (t ) d (3.21)

Dejando = t , podemos mostrar fcilmente que la convolucin es conmutativa:


R
y (t) =
f (t ) h ( ) d
R (3.22)
=
h ( ) f (t ) d

f (t) h (t) = h (t) f (t) (3.23)

Available for free at Connexions <http://cnx.org/content/col10373/1.2>


CHAPTER 3. ANLISIS DE SISTEMAS DE TIEMPO CONTINUO EN EL
50
DOMINIO DEL TIEMPO

Figure 3.13: La gura muestra que ambas funciones pueden ser vistas como entradas del sistema
mientras lo otro es la respuesta al impulso.

3.3.3 Distribucin
Theorem 3.3: Ley Distributiva

f1 (t) (f2 (t) + f3 (t)) = f1 (t) f2 (t) + f1 (t) f3 (t) (3.24)

Proof:
La demostracin de este teorema puede ser tomada directamente de la denicin de convolucin y
usando la linealidad de la integral.

Available for free at Connexions <http://cnx.org/content/col10373/1.2>


51

Figure 3.14

3.3.4 Desplazamiento en el Tiempo


Theorem 3.4: Propiedad de Desplazamiento
Para c (t) = f (t) h (t), entonces

c (t T ) = f (t T ) h (t) (3.25)

y
c (t T ) = f (t) h (t T ) (3.26)

Available for free at Connexions <http://cnx.org/content/col10373/1.2>


CHAPTER 3. ANLISIS DE SISTEMAS DE TIEMPO CONTINUO EN EL
52
DOMINIO DEL TIEMPO

(a)

(b)

(c)

Figure 3.15: Demostracin Grca de la propiedad de desplazamiento.

3.3.5 Convolucin con un Impulso


Theorem 3.5: Convolucin con Impulso Unitario

f (t) (t) = f (t) (3.27)

Proof:
Para este demostracin, dejaremos que (t) sea el impulso unitario localizado en el origen. Usando
la denicin de convolucin empezamos con la integral de convolucin
Z
f (t) (t) = ( ) f (t ) d (3.28)

De la denicin del impulso unitario, conocemos que ( ) = 0 siempre que 6= 0. Usamos este
hecho para reducir la ecuacin anterior y obtener lo siguiente:

R
f (t) (t) = ( ) f (t) d

R (3.29)
= f (t) ( ) d

La integral de ( ) solo tendr un valor cuando =0 (de la denicin del impulso unitario), por
lo tanto esa integral ser igual a uno. Donde podemos simplicar la ecuacin de nuestro teorema:

f (t) (t) = f (t) (3.30)

Available for free at Connexions <http://cnx.org/content/col10373/1.2>


53

(a)

(b)

Figure 3.16: Las guras y ecuaciones anteriores, revelan la funcin identidad del impulso unitario.

3.3.6 Ancho
En tiempo continuo, si la Duracin (f1 ) = T1 y la Duracin (f2 ) = T2 , entonces

Duracin (f1 f2 ) = T1 + T2 (3.31)

Available for free at Connexions <http://cnx.org/content/col10373/1.2>


CHAPTER 3. ANLISIS DE SISTEMAS DE TIEMPO CONTINUO EN EL
54
DOMINIO DEL TIEMPO

(a)

(b)

(c)

Figure 3.17: En tiempo continuo, la duracin de la convolucin resulta igual a la suma de las longitudes
de cada una de las dos seales convolucionadas.

En tiempo discreto si la Duracin (f1 ) = N1 y la Duracin (f2 ) = N2 , entonces

Duracin (f1 f2 ) = N1 + N2 1 (3.32)

Available for free at Connexions <http://cnx.org/content/col10373/1.2>


55

3.3.7 Causalidad
Si f y h son ambas causales, entonces f h tambin es causal.

3.4 Estabilidad BIBO11


BIBO es para entrada acotada, salida acotada (bounded input, bounded output). BIBO estable es una
condicin, tal que cualquier entrada acotada nos da una salida acotada. Esto es que conforme nosotros
pongamos una entrada estable, nos garantiza obtener una salida estable.
Para entender este concepto, primero debemos ver que signica exactamente acotado. Una seal acotada
es cualquier seal tal que existe un valor donde el valor absoluto de la seal nunca es mayor para algn valor.
Ya que el valor es arbitrario, lo que queremos decir es que en ningn punto la seal puede tender a innito.

Figure 3.18: Una seal acotada es una seal para la cual existe un constante A tal que |f (t) | < A

Una vez que ya tenemos identicado lo que signica una seal acotada, debemos mover nuestra atencin
a la condicin que un sistema debe poseer para garantizar que si una seal pasa a travs del sistema, una
seal acotada se presentara en la salida. Resulta que un sistema LTI (Section 2.1) continuo en el tiempo con
una respuesta al impulso h (t) es estabilidad BIBO si y solo si
Condicin de Tiempo-Continuo para Estabilidad BIBO
Z
|h (t) |dt < (3.33)

Esto es decir que la funcin de transferencia es absolutamente integrable.
Extendiendo este concepto a tiempo- discreto tomamos la transicin estndar de integracin a sumatoria
y obtenemos la funcin de transferencia h (n), que debe ser absolutamente sumable. Esto es
Condicin de Tiempo-Discreto para Estabilidad BIBO

X
|h (n) | < (3.34)
n=
11 This content is available online at <http://cnx.org/content/m12834/1.3/>.

Available for free at Connexions <http://cnx.org/content/col10373/1.2>


CHAPTER 3. ANLISIS DE SISTEMAS DE TIEMPO CONTINUO EN EL
56
DOMINIO DEL TIEMPO

3.4.1 Estabilidad y Laplace


La estabilidad es muy fcil de deducir desde la grca de polos y ceros (Section 14.6) de una funcin de
transferencia. La nica condicin necesaria para demostrar la estabilidad es demostrar que el eje-j es en
la regin de convergencia

(a) (b)

Figure 3.19: (a) Ejemplo de una grca de polos y ceros para un sistema estable de tiempo-continuo.
(b) Ejemplo de una grca de polos y ceros para un sistema inestable de tiempo continuo.

3.4.2 Estabilidad y la Transformada Z


La estabilidad para las seales de tiempo-discreto (Section 1.1) en el dominio-z (Section 15.1) es tan fcil
de demostrar como lo es para las seales de tiempo-continuo en el domino Laplace. Sin embargo en lugar
de la regin de convergencia ROC (region of convergence) necesaria para contener el eje-j , la ROC debe
contener el crculo unitario.

Available for free at Connexions <http://cnx.org/content/col10373/1.2>


57

(a) (b)

Figure 3.20: (a) Un sistema estable de tiempo-discreto. (b) Un sistema inestable de tiempo-discreto.

Available for free at Connexions <http://cnx.org/content/col10373/1.2>


CHAPTER 3. ANLISIS DE SISTEMAS DE TIEMPO CONTINUO EN EL
58
DOMINIO DEL TIEMPO

Available for free at Connexions <http://cnx.org/content/col10373/1.2>


Chapter 4

Anlisis de Sistemas Discretos en el


Dominio del Tiempo

4.1 Anlisis en el Dominio del Tiempo para Sistemas Discretos1


Una seal discreta s (n) es retrasada por n0 muestras, cuando se escribe s (n n0 ), con n0 > 0. Optamos
2
por que n0 tenga avances negativos sobre los nmeros enteros. Opuesto a los retrasos anlogos , los retrasos
discretos en el tiempo solo pueden tener el valor de nmeros enteros. En el dominio de la frecuencia, el
retraso de la seal corresponde a un desplazamiento linear en el ngulo de la seal discreta de la Transformada
s (n n0 ) e(j2f n0 ) S ej2f

de Fourier .
Un sistema lineal discreto tiene propiedades de superposicin:

S (a1 x1 (n) + a2 x2 (n)) = a1 S (x1 (n)) + a2 S (x2 (n)) (4.1)

Un sistema discreto es llamado invariante al desplazamiento ( anlogo a los sistemas de tiempo


3
invariante ) si el retraso en la entrada ocurre en la salida.
Si S (x (n)) = y (n), entonces
S (x (n n0 )) = y (n n0 ) (4.2)

Nosotros usamos el trmino invariante al desplazamiento para enfatizar que el retraso puede ocurrir solo
en los nmeros enteros del sistema discreto, mientras tanto en las seales anlogas, los retrasos pueden ser
valores arbitrarios.
Nosotros queremos concentrarnos en sistemas que son lineales e invariantes al desplazamiento. Esto ser
lo que nos permitir tener todo el control en el anlisis del dominio de la frecuencia y el control de su
implementacin. Por que no tenemos una conexin fsica en la construccindel sistema, necesitamos sola-
mente tener especicaciones matemticas. En los sistemas anlogos, las ecuaciones diferenciales especican
la entrada y la salida del dominio del tiempo. La correspondiente especicacin discreta esta dada en una
ecuacin diferencial:

y (n) = a1 y (n 1) + + ap y (n p) + b0 x (n) + b1 x (n 1) + + bq x (n q) (4.3)

La salida de la seal y (n) es relacionada a sus valores pasados por medio de y (n l), l = {1, . . . , p}, a
los valores actuales y a los valores pasados de la entrada de la seal se representan por medio de x (n). Las
caractersticas del sistema son determinadas por la eleccin de cuantos nmeros tendran los coecientes p y
q el valor de los coecientes {a1 , . . . , ap } y {b0 , b1 , . . . , bq }.
1 This content is available online at <http://cnx.org/content/m12830/1.6/>.
2 "Simple Systems": Section Delay <http://cnx.org/content/m0006/latest/#delay>
3 "Simple Systems" <http://cnx.org/content/m0006/latest/#para4wra>

Available for free at Connexions <http://cnx.org/content/col10373/1.2>

59
CHAPTER 4. ANLISIS DE SISTEMAS DISCRETOS EN EL DOMINIO DEL
60
TIEMPO

Ademas: Hay una asimetra en los coecientes: Cundo es a0 ? Estos coecientes multiplicaran
el trmino y (n) en (4.3). Nosotros dividiremos la ecuacin por ella, lo cual no cambiara la relacin
de salida-entrada. Hemos creado la siguiente convencin: a0 es siempre uno.

Al contrario de una ecuacin diferencial que solo provee una descripcin implcita de un sistema
(nosotros debemos resolver la ecuacin diferencial), las ecuaciones diferenciales proveen una manera ex-
plicita de resolverlas; calculando las salidas para cada entrada. Nosotros simplemente expresaremos las
ecuaciones diferenciales con un programa que calcula cada salida usando valores previos, las corrientes del
sistema y las entradas previas.
Las ecuaciones diferenciales son usualmente expresadas en el software con iteraciones de "for" . El
programa de MATLAB da informacin de los primeros 1000 valores formados por medio de las salidas.

for n=1:1000
y(n) = sum(a.*y(n-1:-1:n-p)) + sum(b.*x(n:-1:n-q));
end

Un detalle importante emerge cuando nosotros consideramos hacer que este programa funcione; de hecho,
como esta escrito tiene (por lo menos) dos errores. Qu valores de entrada y salida se pueden usar para
calcular y (1)? Nosotros necesitamos valores para y (0), y (1), ..., valores que no tenemos aun. Para
calcular estos valores necesitaremos valores previos. La manera de salir de este problema es especicar las
condiciones iniciales : debemos proveer p los valores de la salida que ocurren antes que las entradas
iniciales. Estos valores pueden ser arbitrarios, la decisin impacta el como responde el sistema con las
entradas. Una decisin ocasiona un sistema lineal: Hacer la condicin inicial cero. La razn se encuentra
4
en la denicin de un sistema lineal : La nica manera que las salidas de las suma pueda ser la suma de la
salida individual ocurre cuando la condicin inicial en cada caso es cero.

Exercise 4.1.1 (Solution on p. 73.)


La cuestin de la condicin inicial se resuelve al entender la ecuacin diferencial para cada entrada
que empieza en algn ndice. No obstante, el programa no trabaja por causa de la programacin,
no es conceptual y contiene errores. Qu es esto? , Com se puede "arreglar"?

Example 4.1
Vamos a considerar un sistema simple con: p=1 y q = 0.

y (n) = ay (n 1) + bx (n) (4.4)

Para calcular la salida de algn ndice, esta ecuacin diferencial indica que nosotros necesitamos
la salida previa y (n 1) y que la seal de la entrada occura en ese momento del tiempo. Sin
entrar a detalle, vamos a calcular la salida del sistema para un muestreo unitario como entrada:
x (n) = (n). Ya que la entrada es cero para un ndice negativo, nosotros debemos empezar por
tratar de calcular la salida en n = 0.

y (0) = ay (1) + b (4.5)

Cul es el valor de y (1)? Ya que hemos usado la entrada de valor cero para todos los ndices
negativos, es razonable asumir que la salida tambin tiene un valor de cero. Seguramente, la
5
ecuacin diferencial no describir el sistema lineal si la entrada, la cual es cero todo el tiempo
no produjo cero en la salida. Con esto podemos asumir: y (1) = 0, dejando y (0) = b. Para
n > 0, la entrada de un muestrario unitario es cero, lo cual nos deja con la ecuacin diferencial

4 "Simple Systems": Section Linear Systems <http://cnx.org/content/m0006/latest/#linearsys>


5 "Simple Systems": Section Linear Systems <http://cnx.org/content/m0006/latest/#linearsys>

Available for free at Connexions <http://cnx.org/content/col10373/1.2>


61

y (n) = ay (n 1) , n > 0 . Con esto nosotros podemos preveer como el ltro responde a esta
entrada para hacer una tabla:
y (n) = ay (n 1) + b (n) (4.6)

n x (n) y (n)
1 0 0
0 1 b
1 0 ba
2 0 ba2
: 0 :

n 0 ban

Table 4.1

Los valores del coeciente determinan el comportamiento de la salida. El parmetro b puede ser
cualquier valor, y sirve como ganancia. El efecto del parmetro a es ms complicado (Table 4.1).
Si es igual a cero, la salida simplemente es igual a la entrada por la ganancia b. Para todos los
valores que no son cero de a, la salida perdura por siempre; IRR
tales sistemas son conocidos como
(Respuesta al Impulso Innito). La razn para esta terminologa es que el muestrario unitario
tambin conocido como un impulso(especialmente en una situacin analoga) y el sistema responden
a un impulso que perdura por siempre. Si a es positivo y menor que uno, la salida es una
descomposicin exponencial. Cuando a = 1, la salida es un escaln unitario. Si a es negativa y ms
grande que 1, la salida oscila mientras occurre una descomposicin exponencial. Cuando a = 1,
la salida cambia su signo para siempre, alternando entre b y b. Hay efectos ms dramticos
cuando |a| > 1; si es positivo o negativo, la salida de la seal se hace ms y ms grande creciendo
exponencialmente.

x(n)

n
y(n) y(n)
a = 0.5, b = 1 y(n)
a = 0.5, b = 1 4 a = 1.1, b = 1
1 1

n 2
n

n -1 0
n

Figure 4.1: La entrada para el ejemplo de sistema simple, un muestreo unitario es mostrada arriba,
con las salidas para varios valores de parametros de sistemas mostradas abajo.

Available for free at Connexions <http://cnx.org/content/col10373/1.2>


CHAPTER 4. ANLISIS DE SISTEMAS DISCRETOS EN EL DOMINIO DEL
62
TIEMPO

Valores positivos de a son usados en modelos de poblacin para describir como el tamao de
una poblacin crece a travs del tiempo. Aqu, n puede corresponder a generacin. La ecuacin
diferencial indica que el nmero en la siguiente generacin es algn mltiplo de un valor previo.
Si este mltiplo es menor que uno, la poblacin se extingue; si es mayor que uno, la poblacin se
incrementa. La misma ecuacin diferencial tambin describe el efectos del los interes compuesto.
Aqu n marca el tiempo en el cual el interes compuesto ocurre ( diario, mensual, etc.), a es igual
a la taza de inters compuesto, y b=1 ( el banco no da ninguna ganancia). En la aplicacin para
procesar seales, nosotros tpicamente requerimos que la salida continu acotada para cualquier
entrada. Para nuestro ejemplo, eso signica que podemos restrigir |a| = 1 y escoger valores para
esto y la ganancia segn su aplicacin.

Exercise 4.1.2 (Solution on p. 73.)


Note que en la ecuacin diferencial (4.3),

y (n) = a1 y (n 1) + + ap y (n p) + b0 x (n) + b1 x (n 1) + + bq x (n q)

no tiene trminos como y (n + 1) o x (n + 1) en el lado derecho de la ecuacin. Pueden estos


trminos ser incluidos? Por qu o por qu no?

y(n)
1
5

Figure 4.2: La graca muestra la respuesta de un muestreo unitario con un ltro boxcar de longitud 5.

Example 4.2
Un sistema un poco diferente que no contiene los coecientes "a". Considere la ecuacin diferencial:

1
y (n) = (x (n) + + x (n q + 1)) (4.7)
q
Ya que la salida del sistema nada mas depende de los valores actuales y previos de los valores de
entrada, nosotros no necesitamos preocuparnos por las condiciones iniciales. Cuando la entrada es
1
un muestreo unitario, el resultado es igual a
q para n = {0, . . . , q 1}, entonces es igual a cero
despus de eso. Estos sistemas son conocidos como FIR (Respuesta de Impulso Finito o en ingles
Finite Impulse Response) por que su respuesta de muestreo unitario tiene una duracin nita. Al
gracar esta respuesta se ve como (Figure 4.2)la respuesta de muestreo unitario es un pulso de
1
ancho q q . Esta seal es tambin conocida como boxcar , por eso a este sistema se le le da
y altura
el nombre de ltro de boxcar. (En la proxima seccin derivaremos su respuesta en frecuencia y
desarrollaremos su interpretacin al ltrar.) Por ahora, observe que la ecuacin diferencial dice que
cada valor en la salida es igual al promedio de la corriente de las entradas de los valores anteriores.
As, el valor de la salida es igual al promedio actual de los valores de las entradas precias q . Por lo
tanto este sistema se puede usar para producir el promedio de las temperaturas semanales (q = 7)
que se pueden actualizar diariamente.

Available for free at Connexions <http://cnx.org/content/col10373/1.2>


63

4.2 Convolucin Discreta6


4.2.1 Ideas Generales
Convolucin es un valor que se extiende a todos los sistemas que son invariantes linear del tiempo
(Section 2.1) (LTI - Linear Time Invariant). La idea de convolucin discreta es la misma que la de
convolucin continua (Section 3.2). Por esta razn, puede ser de gran ayuda el ver las dos versiones para
que usted entienda la extrema importancia del concepto. Recuerde que la convolucin es un instrumento
poderoso al determinar el resultado de un sistema despus de saber la una entrada arbitraria y la respuesta
al impulso del sistema. Puede ser tambin til al ver la convolucin grcamente con sus propios ojos y jugar
7
con este concepto un poco, as que experimente con las aplicaciones que estn disponibles en la Internet.
Estos recursos ofrecern mtodos diferentes para aprender este concepto crucial.

4.2.2 Suma de Convolucin


Como ya ha sido mencionado, la suma de convolucin provee una manera matemticamante concisa para
expresar el resultado de un sistema LTI, basado en una entrada arbitraria para una seal discreta y tambin
el saber la respuesta del sistema. La suma de convolucin es expresada como


X
y [n] = x [k] h [n k] (4.8)
k=

As como en tiempo continuo la convolucin es representado por el smbolo *, y puede ser escrita como

y [n] = x [n] h [n] (4.9)

Al hacer un simple cambio de variables en la suma de convolucin, k = n k , podemos demostrar fcilmente


que la convolucin es conmutativa:
x [n] h [n] = h [n] x [n] (4.10)

Para mas informacin sobre las caractersticas de convolucin, lea las propiedades de convolucin (Sec-
tion 3.3).

4.2.3 Derivacin
Sabemos que las seales discretas pueden ser representadas por la suma de impulses discretos que estn
desplazados y escalados. Ya que estamos asumiendo que el sistema es linear e invariante con el tiempo,
se ve razonable decir que la entrada de la seal esta formada por impulses que tambin estn escalados y
desplazados, esto en turno dara como resultado del sistema una suma de respuesta de impulse que tambin
estn escaladas y desplazadas. Esto es exactamente lo que ocurre en convolucin. Abajo presentamos una
manera matemtica rigurosa de ver esta derivacin:
Al dejar H ser un sistema DT LTI, empezaremos con la siguiente ecuacin y trabajaremos hasta llegar a
la suma de convolucion !
y [n] = H [x [n]]
P 
= H k= x [k] [n k]
P
k= H [x [k] [n k]]
= (4.11)
P
= k= x [k] H [ [n k]]
P
= k= x [k] h [n k]

6 This content is available online at <http://cnx.org/content/m12833/1.4/>.


7 http://www.jhu.edu/signals

Available for free at Connexions <http://cnx.org/content/col10373/1.2>


CHAPTER 4. ANLISIS DE SISTEMAS DISCRETOS EN EL DOMINIO DEL
64
TIEMPO

Demos un vistazo rpido a los pasos tomados en la derivacin anterior. Despus de nuestra ecuacin
inicial, nosotros usamos la propiedad de desplazamiento (Section 1.4.1.1: La propiedad de desplazamiento
del impulso)del DT para re-escribir la funcin, x [n], como una suma de funciones multiplicada por una
suma unitaria. Despus, movemos el operador H y la sumatoria H [] es linear, en el sistema DT. Por esta
linealidad y por el hecho que, x [k] es constante podemos extraer la constantes ya mencionadas y nada mas
multiplicar la ecuacin por H []. Final mente , usamos el dato que H [] es invariante con el tiempo para
llegar a nuestra ecuacin deseada- la suma de convolucion !
Un ejemplo graco puede ayudar en demostrar por que la convolucin funciona

Figure 4.3: Una simple entrada con impulso da como resultado la respuesta de impulso del sistema.

Figure 4.4: Un impulso escalado como entrada da como resultado una respuesta escalada, ya que tiene
la propiedad de escalmiento para un sistema lineal.

Available for free at Connexions <http://cnx.org/content/col10373/1.2>


65

Figure 4.5: Ahora usaremos la propiedad de tiempo invariante del sistema para demostrar que una
entrada que esta dasplazada da como resultado una salida con la misma forma, solo que esta desplazada
por la misma cantidad que la entrada.

Figure 4.6: Ahora usaremos la propiedad de adicion del sistema lineal para completar la gura.ya que
cualquier seal discreta es nada mas la suma de impulsos discretos que estan desplazados y escalados,
podemos encontrar la salida con tan solo saber la seal de entrada y su respuesta al impulso.

4.2.4 Convolucion a travs del Eje del Tiempo ( Un mtodo graco)


En esta seccin desarrollaremos una interpretacin graca de la convolucin discreta. Empezaremos por
escribir la suma de convolucin dejando x ser causal, de tamao -m y h ser causal, del tamao-k , en un

Available for free at Connexions <http://cnx.org/content/col10373/1.2>


CHAPTER 4. ANLISIS DE SISTEMAS DISCRETOS EN EL DOMINIO DEL
66
TIEMPO

sistema LTI. Esto nos da una sumatoria nita,

m1
X
y [n] = x [l] h [n l] (4.12)
l=0

Note que para cualquier n tenemos la suma de productos de xl desplazados en el tiempo por hl . Esto es
una manera de decir que multiplicamos los trminos de x por los trminos reexionados en el tiempo de h
y los sumamos despus.
Regresamos a los ejemplos anteriores:

Figure 4.7: Esto es lo que esperamos encontrar.

Figure 4.8: Esto es el reejo de la respuesta del impulso, h , y se empieza a mover atravez del tiempo
0.

Available for free at Connexions <http://cnx.org/content/col10373/1.2>


67

Figure 4.9: Continuamos con el moviimiento. Vea que al tiempo 1 , estamos multiplicando dos
elementos de la sealde entraday dos elementos de la respuesta del impulso.

Figure 4.10

Available for free at Connexions <http://cnx.org/content/col10373/1.2>


CHAPTER 4. ANLISIS DE SISTEMAS DISCRETOS EN EL DOMINIO DEL
68
TIEMPO

Figure 4.11: Si seguimos esto un paso mas, n = 4, pordiamos ver que producimos la misma salida que
vimos en el ejemplo inicial.

Los que estamos haciendo en la demostracin de arriba es reejar la respuesta del impulso en el tiempo
y  hacerlo caminar a travs de la entrada en la seal. Claramente, esto da el mismo resultado que escalar,
desplazar, y sumar respuestas de impulsos.
Este mtodo de reexin en el tiempo, y de mover a travs de la seal es una manera comn de presentar
la convolucin, ya que demuestra como la convolucin construye el resultado a travs del eje del tiempo.

4.3 Convolucin Circular y el DFT8


4.3.1 Introduccin
Usted debera familiarizarse con la convolucin discreta (Section 4.2), que nos explica como dos seales
discretas x [n], la entrada del sistema, y h [n], la respuesta del sistema, se puede denir el resultado del
sistema como
y [n] = x [n] h [n]
P (4.13)
= k= x [k] h [n k]

Cuando dos DFT son dadas (secuencias de tamao nito usualmente del tamao N ), nosotros no podemos
multiplicar esas dos seales as como as, como lo sugiere la formula de arriba usualmente conocida como
convolucin linear. Ya que las DFT son peridicas, tienen valores no cero para nN as la multiplicacin
de estas dos seales ser no cero para n N. Necesitamos denir otro tipo de convolucion que dar como
resultado nuestra seal convuelta teniendo el valor de cero fuera del rango n = {0, 1, . . . , N 1}. Esto nos
ayuda a desarrollar la idea de convolucin circular, tambin conocida como convolucin cclica o peridica.
8 This content is available online at <http://cnx.org/content/m12831/1.3/>.

Available for free at Connexions <http://cnx.org/content/col10373/1.2>


69

4.3.2 Formula de la Convolucin Circular


Qu pasa cuando multiplicamos dos DFT una con la otra, donde Y [k] es la DFT de y [n]?

Y [k] = F [k] H [k] (4.14)

cuando 0k N 1
Usando la formula sintetizada de DFT para y [n]
N 1
1 X 2
y [n] = F [k] H [k] ej N kn (4.15)
N
k=0

PN 1 2
Y aplicando anlisis a la formula F [k] = m=0 f [m] e(j) N kn
PN 1 PN 1
1 2 2
y [n] = Nk=0 m=0 f [m] e(j) N kn H [k] ej N kn
PN 1  PN 1  (4.16)
1 j 2
N k(nm)
= m=0 f [m] N k=0 H [k] e

donde podemos reducir la segunda sumatoria de la ecuacin de arriba en h [((n m))N ] =


1
PN 1 2
N k=0 H [k] ej N k(nm)
N
X 1
y [n] = f [m] h [((n m))N ]
m=0

Igual a la convolucin circular! cuando tenemos 0nN 1 arriba , para obtener dos:

y [n] f [n] ~ h [n] (4.17)

note: Que la notacin ~ representa la convolucion circular "mod N".

4.3.2.1 Pasos para la Convolucin Circular

Los pasos a seguir para la convolucion cclica son los mismos que se usan en la convolucin linear, excepto
que todos los clculos para todos los ndices estn hecho"mod N" = "en la rueda"

Pasos para la Convolucin Cclica

Paso 1: "Graque" f [m] y h [(( (m () ()N ]

(a) (b)

Figure 4.12: Step 1

Available for free at Connexions <http://cnx.org/content/col10373/1.2>


CHAPTER 4. ANLISIS DE SISTEMAS DISCRETOS EN EL DOMINIO DEL
70
TIEMPO

Paso 2: "Rote" h [(( (m () ()N ] n en la direccin ACW ( direccin opuesta al reloj) para obtener
h [((n ( (m () ()N ] (por ejemplo rote la secuencia, h [n], en direccin del reloj por n pasos).

Figure 4.13: Step 2

Paso 3: Multiplique punto por punto la rueda f [m] y la rueda h [((n ( (m () ()N ] wheel.

sum = y [n]

Paso 4: Repite para 0nN 1


Example 4.3: Convolve (n = 4)

(a) (b)

Figure 4.14: Dos seales discretas que seran convolucionadas.

h [(( (m () ()N ]

Figure 4.15

Multiplique f [m] y sume para dar: y [0] = 3

Available for free at Connexions <http://cnx.org/content/col10373/1.2>


71

h [((1 ( (m () ()N ]

Figure 4.16

Multiplique f [m] y sume para dar: y [1] = 5

h [((2 ( (m () ()N ]

Figure 4.17

Multiplique f [m] y sume para dar: y [2] = 3

h [((3 ( (m () ()N ]

Figure 4.18

Multiplique f [m] y sume para dar: y [3] = 1


Example 4.4
9
La Siguiente Demostracin le permite este algoritmo. Vea aqu para instrucciones de como se usa
este demo.
9 "How to use the LabVIEW demos" <http://cnx.org/content/m11550/latest/>

Available for free at Connexions <http://cnx.org/content/col10373/1.2>


CHAPTER 4. ANLISIS DE SISTEMAS DISCRETOS EN EL DOMINIO DEL
72
TIEMPO

This is an unsupported media type. To view, please see


http://cnx.org/content/m12831/latest/DT_Circular_Convolution.llb

4.3.2.2 Algoritmo Alterno

Algoritmo de Convolucin Circular Alterno

Paso 1: Calcule el DFT de f [n] que da F [k] y calcule el DFT de h [n] que da H [k].
Paso 2: Multiplique punto por punto Y [k] = F [k] H [k]
Paso 3: Invierta el DFT Y [k] que da y [n]
Parece una manera repetitiva de hacer las cosas, pero existen maneras rpidas de calcular una secuencia
DFT.
Para convolucionar circularmente dos secuencias de 2 N -puntos:
N
X 1
y [n] = f [m] h [((n m))N ]
m=0

Para cualquier n : N mltiplos, N 1 sumas



N puntos implica N2 multiplicaciones, N (N 1) sumas implica una complejidad de O N2 .

Available for free at Connexions <http://cnx.org/content/col10373/1.2>


73

Solutions to Exercises in Chapter 4


Solution to Exercise 4.1.1 (p. 60)
Los ndices pueden ser negativos, y esta condicin no es permitida en MATLAB. Para arreglar esto, debemos
empezar la seal despus de esta condicin.
Solution to Exercise 4.1.2 (p. 62)
Estos trminos requieren que el sistema conozca el valor futuro de las entradas o de las salidas antes de que
el valor actual haya sido calculado. As que estos trminos pueden causar problemas.

Available for free at Connexions <http://cnx.org/content/col10373/1.2>


CHAPTER 4. ANLISIS DE SISTEMAS DISCRETOS EN EL DOMINIO DEL
74
TIEMPO

Available for free at Connexions <http://cnx.org/content/col10373/1.2>


Chapter 5

Repaso de Algebra Lineal

5.1 Algebra Lineal: Conceptos Bsicos1


Este pequeo tutorial da algunos trminos clave de lgebra lineal, no pretende remplazar o ser muy provechoso
como en aquellos que usted pretende ganar una profundidad en lgebra lineal. En cambio esto es una pequea
introduccin a algunos trminos e ideas de lgebra lineal para darnos un pequeo repaso para aquellos que
tratan de tener un mejor entendimiento o de aprender sobre eigenvectores (vectores propios) y eigenfunciones
(funciones propias), que juegan un papel muy importante en la obtencin de ideas importantes en Seales
y Sistemas. La meta de estos conceptos es de proveer un respaldo para la descomposicin de seales y para
conducirnos a la derivacin de las Series de Fourier (Section 6.2).

5.1.1 Independencia Lineal


Un conjunto de vectores {x1 , x2 , . . . , xk } , xi Cn es linealmente independiente si ninguno de los
vectores puede escribirse como una combinacin lineal de los otros.

Denition 5.1: 1 Linealmente Independiente


Un conjunto dado de vectores {x1 , x2 , . . . , xn }, es linealmente independiente si

c1 x1 + c2 x2 + + cn xn = 0
solo cuando c1 = c2 = = cn = 0
Example
Dados los siguientes dos vectores:

3
x1 =
2

6
x2 =
4
Estos son no linealmente independientes por el siguiente argumento, el cual por inspeccin, se
puede ver que no se apega a la denicin anterior de independencia lineal:

(x2 = 2x1 ) (2x1 + x2 = 0)


. Otro mtodo para ver la independencia de los vectores es gracando los vectores. Observando
estos dos vectores geomtricamente (como en la siguiente Figure 5.1), uno puede otra vez probar
que estos vectores son no linealmente independientes.
1 This content is available online at <http://cnx.org/content/m12862/1.3/>.
Available for free at Connexions <http://cnx.org/content/col10373/1.2>

75
76 CHAPTER 5. REPASO DE ALGEBRA LINEAL

Figure 5.1: Representacin grca de dos vectores que no son linealmente independientes.

Example 5.1
Dados los siguientes dos vectores:

3
x1 =
2

1
x2 =
2

Estos son linealmente independientes ya que

c1 x1 = (c2 x2 )

solo si c1 = c2 = 0. Basados en la denicin, esta demostracin muestra que estos vectores son
linealmente independientes. Tambin podemos gracar estos dos vectores (vase Figure 5.2) para
checar la independencia lineal.

Figure 5.2: Representacin grca de dos vectores que son linealmente independientes.

Exercise 5.1.1 (Solution on p. 93.)


Son {x1 , x2 , x3 } linealmente independientes?


3
x1 =
2

Available for free at Connexions <http://cnx.org/content/col10373/1.2>


77


1
x2 =
2

1
x3 =
0

Como podemos ver en los dos ejemplos anteriores, a veces la independencia de vectores puede ser vista
fcilmente a travs de una grca. Sin embargo esto no es tan sencillo, cuando se nos dan tres o ms
vectores. Puede decir fcilmente cuando o no estos vectores son independientes Figure 5.3. Probablemente
no, esto es, por lo cual el mtodo usado en la solucin anterior se vuelve importante.

Figure 5.3: Grca de tres vectores. Puede ser mostrado que la combinacin lineal existe entre los
tres, y por lo tanto estos son no linealmente independientes.

note: Un conjunto de m vectores en Cn no puede ser linealmente independiente si m > n.

5.1.2 Subespacio Generado


Denition 5.2: Subespacio Generado
2
El subespacio generado o span del conjuto de vectores {x1 , x2 , . . . , xk } es el conjunto de vectores
que pueden ser escritos como una combinacin lineal de {x1 , x2 , . . . , xk }

subespacio generado ({x1 , . . . , xk }) = {1 x1 + 2 x2 + + k xk , i Cn }

Example
Dado el vector
3
x1 =
2

el subespacio generado de x1 es una linea.

Example
Dado los vectores
3
x1 =
2
2 "Subspaces", Denition 2: "Span" <http://cnx.org/content/m10297/latest/#defn2>

Available for free at Connexions <http://cnx.org/content/col10373/1.2>


78 CHAPTER 5. REPASO DE ALGEBRA LINEAL


1
x2 =
2

El subespacio generado por estos vectores es C2 .

5.1.3 Bases
Denition 5.3: Base
Una base para Cn es un conjunto de vectores que: (1) generan Cn y (2) es linealmente independi-
ente.

Claramente, un conjunto de n vectores linealmente independientes es una base para Cn .


Example 5.2
Dado el siguiente vector

0
.
.
.

0


ei =
1


0

.
..

0
donde el 1 esta siempre en la i-esima posicin y los valores restantes son ceros. Entonces la base
para Cn es
{ei , i = [1, 2, . . . , n] }
note: {ei , i = [1, 2, . . . , n] } es llamada la base cannica.

Example 5.3

1
h1 =
1

1
h2 =
1

{h1 , h2 } es una base para C2 .

Available for free at Connexions <http://cnx.org/content/col10373/1.2>


79

Figure 5.4: Grca de bases para C2

Si {b1 , . . . , b2 } es una base para Cn , entonces podemos expresar cualquier x Cn como una combinacin
lineal de bi 's:
x = 1 b1 + 2 b2 + + n bn , i C

Example 5.4
Dado el siguiente vector,

1
x=
2

escribiendo x en trminos de {e1 , e2 } nos da

x = e1 + 2e2

Exercise 5.1.2 (Solution on p. 93.)


Trate de escribir x en trminos de {h1 , h2 } (denidos en el ejemplo anterior).

En los dos ejemplos de bases anteriores, x es el mismo vector en ambos casos, pero podemos expresarlo de
varias diferentes maneras (dimos solo dos de las muchas posibilidades). Se puede extender aun ms la idea
de bases para espacio de funciones.

note: : Como se menciono en la introduccin, estos conceptos de lgebra lineal nos ayudaran para
entender las Series de Fourier (Section 6.2), las que nos dicen que podemos expresar las funciones
peridicas f (t), en trminos de sus funciones de bases ej0 nt .

5.2 Conceptos Bsicos de Vectores3


5.3 Eigenvectores y Eigenvalores4
En esta seccin, nuestro sistema lineal ser una matriz de nn de nmeros complejos. Algunos conceptos
de este modulo estn basado en los conceptos bsicos de lgebra lineal (Section 5.1).

3 This content is available online at <http://cnx.org/content/m12863/1.2/>.


4 This content is available online at <http://cnx.org/content/m12870/1.2/>.

Available for free at Connexions <http://cnx.org/content/col10373/1.2>


80 CHAPTER 5. REPASO DE ALGEBRA LINEAL

5.3.1 Eigenvectores y Eigenvalores


Sea A una matriz de nn donde A es un operador lineal en los vectores de Cn .

Ax = b (5.1)

donde x y b son vectores de n1 (Figure 5.5).

(a)

(b)

Figure 5.5: Ilustracin de un sistema lineal y vectores.

Denition 5.4: Eigenvector


Un eigenvector de A es un vector v Cn tal que

Av = v (5.2)

donde es llamado el eigenvalor correspondiente. A solo cambia la longitud de v , no su direccin.

5.3.1.1 Modelo Grco

A travs de las siguientes Figure 5.6 y Figure 5.7, veamos las diferencias de la (5.1) y de la (5.2).

Available for free at Connexions <http://cnx.org/content/col10373/1.2>


81

Figure 5.6: Representa la (5.1), Ax = b.

Si v es un eigenvector de A, entonces solo su longitud cambia. Vase Figure 5.7 y note que la longitud
de nuestro vector esta simplemente escalada por una variable , llamada eigenvalor:

Figure 5.7: Representa la (5.2), Av = v .

note: Cuando tratamos con una matriz A, los eigenvectores son los vectores posibles ms simples
para trabajar.

5.3.1.2 Ejemplos

Exercise 5.3.1 (Solution on p. 93.)


Por inspeccin y entendimiento de eigenvectores, encuentre los dos eigenvectores v1 y v2 , de

3 0
A=
0 1

Tambin cules son los eigenvalores correspondientes, 1 y 2 ? No se preocupe si tiene problemas


viendo estos valores de la informacin dada hasta ahora, veremos otras maneras mas rigurosas de
encontrar estos valores.

Available for free at Connexions <http://cnx.org/content/col10373/1.2>


82 CHAPTER 5. REPASO DE ALGEBRA LINEAL

Exercise 5.3.2 (Solution on p. 93.)


Muestre que estos dos vectores,

1
v1 =
1

1
v2 =
1

3 1
son eigenvectores de A, donde A= . Tambin encuentre los eigenvalores correspon-
1 3
dientes.

5.3.2 Calculando Eigenvalores y Eigenvectores


En los ejemplos anteriores, conamos en su entendimiento de la denicin y de algunas observaciones para
encontrar y probar los valores de los eigenvectores y eigenvalores. Sin embrago como se puede dar cuenta,
encontrar estos valores no siempre es fcil. A continuacin veremos un mtodo matemtico para calcular
eigenvalores y eigenvectores de una matriz.

5.3.2.1 Encontrando Eigenvalores

Encontrar C tal que v 6= 0, donde 0 es el vector cero. Empezaremos con la (5.2), trabajemos de la
siguiente manera mientras encontramos una manera explicita de calcular .

Av = v

Av v = 0

(A I) v = 0
En el paso previo, usamos el hecho de que
v = Iv
donde I es la matriz identidad.

1 0 ... 0

0 1 ... 0
I=

.. .
0 . .
0 .

0 ... ... 1
Por lo tanto, A I es justo una matriz nueva.

Example 5.5
Dada la siguiente matriz, A, entonces podemos encontrar nuestra nueva matriz, A I .

a1,1 a1,2
A=
a2,1 a2,2

Available for free at Connexions <http://cnx.org/content/col10373/1.2>


83


a1,1 a1,2
A I =
a2,1 a2,2

Si (A I) v = 0 para algn v 6= 0, entonces A I es no invertible. Esto quiere decir:

det (A I) = 0

este determinante (el mostrado arriba) se vuelve una expresin polinomial (de grado n). Vase el siguiente
ejemplo para entender mejor.

Example 5.6
Empezando con la matriz A (mostrada a continuacin), encontremos la expresin polinomial, donde
nuestros eigenvalores sern variables dependientes.


3 1
A=
1 3

3 1
A I =
1 3

2 2
det (A I) = (3 ) (1) = 2 6 + 8

= {2, 4}

Example 5.7
Empezando con la matriz A (mostrada a continuacin),encontremos la expresin polinomial, donde
nuestros eigenvalores sern variables dependientes.


a1,1 a1,2
A=
a2,1 a2,2

a1,1 a1,2
A I =
a2,1 a2,2

det (A I) = 2 (a1,1 + a2,2 ) a2,1 a1,2 + a1,1 a2,2

Si no lo han notado, calcular los eigenvalores es equivalente a calcular las races de

det (A I) = cn n + cn1 n1 + + c1 + c0 = 0

note: Por lo tanto usando unos pequeos clculos para resolver las races de nuestro polinomio,
podemos encontrar los eigenvalores de la matriz.

Available for free at Connexions <http://cnx.org/content/col10373/1.2>


84 CHAPTER 5. REPASO DE ALGEBRA LINEAL

5.3.2.2 Encontrando Eigenvectores

Dado un eigenvalor, i , el eigenvector asociado esta dado por

Av = i v


v1 1 v1

. .
A . = .

. .

vn n vn
conjunto de n ecuaciones con n incognitas. Simplemente se resuelven las solve the n ecuaciones para
encontrar los eigenvectores.

5.3.3 Punto Principal


El decir que los eigenvectores de A, {v1 , v2 , . . . , vn }, generan el subespacio 5 Cn , signica que {v1 , v2 , . . . , vn }
6 n
son linealmente independientes y que podemos escribir cualquier x C como

x = 1 v1 + 2 v2 + + n vn (5.3)

donde {1 , 2 , . . . , n } C Todo lo que estamos haciendo es reescribir x en trminos de los eigenvectores


de A. Entonces,
Ax = A (1 v1 + 2 v2 + + n vn )

Ax = 1 Av1 + 2 Av2 + + n Avn

Ax = 1 1 v1 + 2 2 v2 + + n n vn = b
por lo tanto podemos escribir,
X
x= i vi
i
Y esto nos lleva a la siguiente representacin del sistema:

Figure 5.8: Ilustracin del sistema donde descomponemos nuestro vector, x, en la suma de sus
eigenvectores.

donde en la Figure 5.8 tenemos,


X
b= i i vi
i

note: Descomponiendo nuestro vector, x, en una combinacin de eigenvectores, la solucin de


Ax esta dada en piezas fciles de digerir".

5 "Linear Algebra: The Basics": Section Span <http://cnx.org/content/m10734/latest/#span_sec>


6 "Linear Algebra: The Basics": Section Linear Independence <http://cnx.org/content/m10734/latest/#lin_ind>

Available for free at Connexions <http://cnx.org/content/col10373/1.2>


85

5.3.4 Problema de Prctica


Exercise 5.3.3 (Solution on p. 93.)
Para la siguiente matriz, A y vector x, resulvase por sus productos. Trate de resolverlos por los
dos diferentes mtodos: directamente y usando eigenvectores.


3 1
A=
1 3

5
x=
3

5.4 Diagonalizacin de Matrices7


De nuestro entendimiento de eigenvalores y eigenvectores (Section 5.3) hemos descubierto ciertas cosas sobre
nuestro operador, la matriz A. Sabemos que los eigenvectores de A generan el espacio Cn y sabemos como
expresar cualquier vector x en trminos de {v1 , v2 , . . . , vn }, entonces tenemos el operador A calculado. Si
tenemos A actuando en x, despus esto es igual a A actuando en la combinacin de los eigenvectores.
Todava tenemos dos preguntas pendientes:

1. Cundo los eigenvectores {v1 , v2 , . . . , vn } de A generan el espacio Cn (asumiendo que {v1 , v2 , . . . , vn }


linealmente independientes)?
2. Cmo expresamos un vector dado x en trminos de {v1 , v2 , . . . , vn }?

5.4.1 1 Respuesta a la Pregunta #1


note: Cundo los eigenvectores {v1 , v2 , . . . , vn } de A generan el espacio Cn ?

Si A tiene n diferentes eigenvalores


i 6= j , i 6= j
donde i y j son enteros, entonces A tiene n eigenvectores linealmente independientes. {v1 , v2 , . . . , vn } que
generan el espacio Cn .
note: La demostracin de esta proposicin no es muy difcil, pero no es interesante para incluirla
aqu. Si desea investigar esta idea, lase Strang G., Algebra Lineal y sus aplicaciones para la
demostracin.

Adems, n diferentes eigenvalores signica que

det (A I) = cn n + cn1 n1 + + c1 + c0 = 0

tiene n races diferentes.


7 This content is available online at <http://cnx.org/content/m12871/1.2/>.

Available for free at Connexions <http://cnx.org/content/col10373/1.2>


86 CHAPTER 5. REPASO DE ALGEBRA LINEAL

5.4.2 Respuesta a la Pregunta #2


note: Cmo expresamos un vector dado x en trminos de {v1 , v2 , . . . , vn }?
Queremos encontrar {1 , 2 , . . . , n } C tal que

x = 1 v1 + 2 v2 + + n vn (5.4)

Para poder encontrar el conjunto de variables, empezaremos poniendo los vectores {v1 , v2 , . . . , vn } como
culumnas en una matriz V de nn.

. . .

. . .
. . .

V = v1 v2 ... vn


. . .
. . .
. . .

Ahora la (5.4) se convierte en


. . .

. . . 1
. . .

.
x = v1 v2 ... vn .

.

. . .
. . . n
. . .

x=V
Lo que nos da una forma sencilla de resolver para la variable de nuestra pregunta :
1
=V x
Notese que V es invertible ya que tiene n columnas linealmnete independientes.

5.4.2.1 Comentarios Adicionales

Recordemos el conocimiento de funciones y sus bases y examinemos el papel de V.


x=V

x1 1

. .
. =V .

. .

xn n
donde es solo x expresada en una base (Section 5.1.3: Bases) diferente:

1 0 0

0 1 0
x = x1 . + x2 . + + xn .

.. .. ..

0 0 1

. . .

. . .
. . .

x = 1 v1 + 2 v2 + + n vn



. . .
. . .
. . .

V transforma x de la base cannica a la base {v1 , v2 , . . . , vn }

Available for free at Connexions <http://cnx.org/content/col10373/1.2>


87

5.4.3 Diagonalizacin de Matrices y Salidas


Tambin podemos usar los vectores {v1 , v2 , . . . , vn } para representar la salida b, del sistema:

b = Ax = A (1 v1 + 2 v2 + + n vn )

Ax = 1 1 v1 + 2 2 v2 + + n n vn = b

. . .

. . . 1 1
. . .

.
Ax = v1 v2 ... vn .

.

. . .
. . . 1 n
. . .

Ax = V

Ax = V V 1 x
donde es la matriz con eigenvalores en la diagonal:


1 0 ... 0

0 2 ... 0
= .

. .. .
.. .
. . .
.

0 0 ... n

Finalmente, podemos cancelar las x y quedarnos con una ecuacin nal para A:

A = V V 1

5.4.3.1 1 Interpretacin

Para nuestra interpretacin, recordemos nuestra formulas:

= V 1 x

X
b= i i vi
i

podemos interpretar el funcionamiento de x con A como:



x 1 1 b1
1 1
.. .. .
.
.
.
. .

. .

xn n 1 n bn

Donde los tres pasos (las echas) en la ilustracin anterior representan las siguientes tres operaciones:

1. Transformar x usando V 1 , nos da


2. Multiplicar por
3. Transformada Inversa usando V, lo que nos da b

Available for free at Connexions <http://cnx.org/content/col10373/1.2>


88 CHAPTER 5. REPASO DE ALGEBRA LINEAL

Este es el paradigma que usaremos para los sistemas LTI!

Figure 5.9: Ilustracin simple del sistema LTI.

5.5 Generalidades de Eigenvectores y Eigenvalores8


5.5.1 La Matriz y sus Eigenvectores
La razn por la cual estamos recalcando la importancia de los eigenvectores (Section 5.3) es por que la accin
de una matriz A en uno de sus eigenvectores v es

1. Extremadamente fcil (y rpido) de calcular

Av = v (5.5)

solo multiplicar v por .


2. fcil de interpretar: A solo escala v, manteniendo su direccin constante y solo altera la longitud del
vector.

Si solo cada vector fuera un eigenvector de A....

5.5.2 Usando el Espacio Generado por los Eigenvectores


Claro que no todos los vectores pero para ciertas matrices (incluidas aquellas con eigenvalores 's), cuyos
eigenvectores generan el subespacio (Section 5.1.2: Subespacio Generado) Cn , lo que signica que para cada
x Cn , podemos encontrar {1 , 2 , n } C tal que:

x = 1 v1 + 2 v2 + + n vn (5.6)

Dada la (5.6), podemos reescribir Ax = b. Esta ecuacin esta modelada en nuestro sistema LTI ilustrado
posteriormente:

Figure 5.10: Sistema LTI.

8 This content is available online at <http://cnx.org/content/m12872/1.3/>.

Available for free at Connexions <http://cnx.org/content/col10373/1.2>


89

X
x= i vi
i
X
b= i i vi
i

El sistema LTI representado anteriormente representa nuestra (5.5). La siguiente es una ilustracin de los
paso para ir de x a b.
x = V 1 x V 1 x V V 1 x = b
  

Donde los tres pasos (las echas) de la ilustracin anterior representan las siguientes tres operaciones:

1. Transformar x usando V 1 - nos da


2. Accin de A en una nueva base- una multiplicacin por
3. Regresar a la antigua base- transformada inversa usando la multiplicacin por V, lo que nos da b

5.6 Eigenfunciones de los Sistemas LTI9


5.6.1 Introduccin
Ahora que ya esta familiarizado con la nocin de eigenvector de una matriz de sistema, si no lo esta de un
pequeo repaso a lasgeneralidades de eigenvectores y eigenvalores (Section 5.5). Tambin podemos convertir
10
las mismas ideas para sistemas LTI actuando en seales. Un sistema lineal invariante en el tiempo (LTI)
H operando en una salida continua f (t) para producir una salida continua en el tiempo y (t)

H [f (t)] = y (t) (5.7)

Figure 5.11: H [f (t)] = y (t). f y t son seales de tiempo continuo(CT) y H es un operador LTI.

La matemtica es anloga a una matriz A de N xN operando en un vector x CN para producir otro


N
vector bC (vase matrices y sistemas LTI para una descripcin).

Ax = b (5.8)

9 This content is available online at <http://cnx.org/content/m12874/1.3/>.


10 "Introduction to Systems" <http://cnx.org/content/m0005/latest/>

Available for free at Connexions <http://cnx.org/content/col10373/1.2>


90 CHAPTER 5. REPASO DE ALGEBRA LINEAL

Figure 5.12: Ax = b donde x y b estan en CN y A es una matriz de N x N .

Solo como un eigenvector (Section 5.3) de A es v CN tal que Av = v , C,

Figure 5.13: Av = v donde v CN es un eigenvector de A.

podemos denir una eigenfuncin (o eigenseal) de un sistema LTI H para ser una seal f (t) tal que

H [f (t)] = f (t) , C (5.9)

Figure 5.14: H [f (t)] = f (t) donde f es una eigenfuncin de H.

Las Eiegenfunciones son las seales mas simples possibles para H para operar en ellas: para calcular
la salida, simplemente multiplicamos la entrada por un nmero complejo .

Available for free at Connexions <http://cnx.org/content/col10373/1.2>


91

5.6.2 Eigenfunciones para cualquier sistema LTI


La clase de sistemas LTI tiene un conjunto de eigenfunciones en comn: el exponencial complejo (Section 1.5)
est , s C son eigenfunciones para todo sistema LTI.

H est = s est
 
(5.10)

H est = s est donde H es un sistema LTI.



Figure 5.15:

note: Mientras que {est , s C } siempre son eigenfunciones para todo sistema LTI, estas no
son necesariamente las nicas eigenfunciones.

Podemos probar la (5.10) expresando la salida como una convolucin (Section 3.2) de la entrada est y de
la respuesta al impulso (Section 1.4) h (t) de H:
R
H [est ] =
h ( ) es(t ) d
R
=
h ( ) est e(s ) d (5.11)

est h ( ) e(s ) d
R
=

Ya que la expresin de la derecha no depende de t, es una constante s ; Por lo tanto

H est = s est
 
(5.12)

El eigenvalor s es un nmero complejo que depende del exponente s y por supuesto, el sistema H. Para
hacer esta dependencia explicita, vamos a usar la notacin H (s) s .

Figure 5.16: est es la eigenfuncin y H (s) son eigenvalores.

Available for free at Connexions <http://cnx.org/content/col10373/1.2>


92 CHAPTER 5. REPASO DE ALGEBRA LINEAL

Ya que la accin del operador LTI en esta eigenfuncin est es fcil de calcular y de interpretar, es
conveniente representar una seal arbitraria f (t) como una combinacin lineal de exponentes complejos.
Las Series de Fourier (Section 6.2) nos dan la representacin para una seal peridica continua en el tiempo,
11
mientras que (poco ms complicada) transformada de Fourier nos deja expandir seales arbitrarias de
tiempo continuo.

5.7 Propiedades de la Transformada de Fourier12


Pequea Tabla de los Pares de la Transformada de Fourier

s(t) S(f )

e(at) u (t) 1
j2f +a
2a
e(a)|t| 4 2 f 2 +a2


1 if |t| < 2 sin(f )
p (t) = f

0 if |t| > 2
sin(2W t)
1 if |f | < W
t S (f ) =
0 if |f | > W

Table 5.1

Propiedades de la Transformada de Fourier

Dominio del Tiempo Dominio de la Frecuencia

Linealidad a1 s1 (t) + a2 s2 (t) a1 S1 (f ) + a2 S2 (f )



Simetria Conjugada s (t) R S (f ) = S (f )
Simetria Par s (t) = s (t) S (f ) = S (f )
Simetria Impar s (t) = s (t) S (f ) = S (f )
 
1 f
Cambio de Escala s (at) |a| S a

Retraso en el Tiempo s (t ) e(j2f ) S (f )


Modulacin Compleja ej2f0 t s (t) S (f f0 )
S(f f0 )+S(f +f0 )
Amplitud Modulada por Coseno s (t) cos (2f0 t) 2
S(f f0 )S(f +f0 )
Amplitud Modulada por Seno s (t) sin (2f0 t) 2j
d
Derivacin
dt s (t) j2f S (f )
Rt 1
Integracin

s () d j2f S (f ) if S (0) = 0
1 dS(f )
Multiplicacin por t ts (t) (j2) df
R
rea

s (t) dt S (0)
R
Valor en el Origen s (0)
S (f ) df
R 2 R 2
Teorema de Parseval

(|s (t) |) dt
(|S (f ) |) df
Table 5.2

11 "Derivation of the Fourier Transform" <http://cnx.org/content/m0046/latest/>


12 This content is available online at <http://cnx.org/content/m12875/1.2/>.

Available for free at Connexions <http://cnx.org/content/col10373/1.2>


93

Solutions to Exercises in Chapter 5


Solution to Exercise 5.1.1 (p. 76)
Jugando un poco con los vectores y haciendo intentos de prueba y error, descubrimos la siguiente relacin:

x1 x2 + 2x3 = 0

donde encontramos una combinacin lineal de estos tres vectores igual a cero sin utilizar los coecientes igual
a cero. Por lo tanto, estos vectores son no linealmente independientes!
Solution to Exercise 5.1.2 (p. 79)

3 1
x= h1 + h2
2 2
Solution to Exercise 5.3.1 (p. 81)
Los eigenvectores que debi encontrar son:

1
v1 =
0

0
v2 =
1
Y los eigenvalores correspondientes son:
1 = 3

2 = 1
Solution to Exercise 5.3.2 (p. 82)
Para poder probar que estos dos vectores son eigenvectores, mostraremos que estas armaciones cumplen
con los requisitos que indica la denicin (Denition: "Eigenvector", p. 80).

3 1 1 2
Av1 = =
1 3 1 2

3 1 1 4
Av2 = =
1 3 1 4
Este resultado nos muestra que A solo escala los dos vectores (es decir cambia sus longitudes) y esto prueba
que la (5.2) es cierta para los siguientes dos eigenvalores que se le pidi que encontrara:

1 = 2

2 = 4
. Si quiere convencerse ms, entonces tambin se pueden gracar los vectores y su producto correspondiente
con A para ver los resultados como una versin escalada de los vectores originales v1 y v2 .
Solution to Exercise 5.3.3 (p. 85)
Mtodo Directo (usese la multiplicacin bsica de matrices)

3 1 5 12
Ax = =
1 3 3 4

Available for free at Connexions <http://cnx.org/content/col10373/1.2>


94 CHAPTER 5. REPASO DE ALGEBRA LINEAL

Eigenvectores (use los eigenvectores y eigenvalores que se encotraron anteriormente para esta misma ma-
triz)

1
v1 =
1

1
v2 =
1

1 = 2

2 = 4
Como se muestra en la (5.3), queremos representar x como la suma de sus eigenvectores escalados. Para este
caso tenemos:
x = 4v1 + v2

5 1 1
x= = 4 +
3 1 1

Ax = A (4v1 + v2 ) = i (4v1 + v2 )
Por lo tanto, tenemos

1 1 12
Ax = 4 2 + 4 =
1 1 4

Ntese que el mtodo usando eigenvectores no requiere multiplicacin de matrices. . Esto puede parecer
mas complicado hasta ahora, pero, imagine que A es de dimensiones muy grandes.

Available for free at Connexions <http://cnx.org/content/col10373/1.2>


Chapter 6

Series Fourier del Tiempo Continuo

6.1 Seales Peridicas1


Recordemos que las funciones peridicas son funciones en las cuales su forma se repite exactamente despus
de un periodo o ciclo. Nosotros representaremos la denicin de una funcin peridica matemticamente
como:
f (t) = f (t + mT ) m Z (6.1)

donde T > 0 representa el periodo. Por esta razn, usted podr ver esta seal ser llamada la seal
peridica-T. Cualquier funcin que satisfaga esta ecuacin es peridica.
Podemos pensar en funciones peridicas (con periodo-T ) de dos diferentes maneras:
#1) Como una funcin en todos R

Figure 6.1: Funcin en todosR donde f (t0 ) = f (t0 + T )

#2) O, podemos podemos recortar todas las redundancias, y pensar en ellas como funciones en un
intervalo [0, T ] (O, en trminos generales, [a, a + T ]). Si sabemos que la seal es peridica-t entonces toda
la informacin de la seal se encuentra en este intervalo.
1 This content is available online at <http://cnx.org/content/m12933/1.3/>.

Available for free at Connexions <http://cnx.org/content/col10373/1.2>

95
96 CHAPTER 6. SERIES FOURIER DEL TIEMPO CONTINUO

Figure 6.2: Remueva la redundancia de la funcion periodica para que f (t) no esta denido afuera
[0, T ].

Una funcion aperiodica CT f (t) no se repite para cualquier T R; i.e. no existe ninguna T s.t. esta
ecuacion (6.1) es verdadera.
Pregunta: La denicin de DT ?

6.1.1 Tiempo Continuo


6.1.2 Tiempo Discreto
Nota: Circular vs. Linear

6.2 Series de Fourier: El Mtodo de Eigenfunciones2


6.2.1 Introduccin
Ya que los exponenciales complejos (Section 1.5) son eigenfunciones para los sistemas lineares invariantes
en el tiempo (Section 5.6) (LTI), calcular los resultados de un sistema LTI H dado est como una entrada
nos lleva a una simple multiplicacin, donde H (s) C es una constante (que depende de S). En la gura
(Figure 6.3) mostrada abajo tenemos un simple exponencial como entrada que da el siguiente resultado:

y (t) = H (s) est (6.2)

Figure 6.3: Un simple sistema LTI.

Usando esto y el hecho que H es linear, calcular y (t) para las combinaciones de exponentes complejos
se vuelve fcil de hacer. Esta propiedad de linealidad es descrita por las dos ecuaciones mostradas abajo 

2 This content is available online at <http://cnx.org/content/m12893/1.3/>.

Available for free at Connexions <http://cnx.org/content/col10373/1.2>


97

donde se muestra la entrada del sistema linear H en el lado izquierdo y la salida (resultado), y (t), en el lado
derecho:

1.
c1 es1 t + c2 es2 t c1 H (s1 ) es1 t + c2 H (s2 ) es2 t
2.
X X
cn esn t cn H (sn ) esn t
n n

La accin que H ejerce en entradas como las que se muestran en estas dos ecuaciones son fciles de explicar:
H escala independientemente del exponencial esn t con un numero complejo diferente H (sn ) C. De
esta manera, si podemos describir la funcin f (t) como la combinacin de exponentes complejos esto nos
permitira:

Calcular el resultado de H fcilmente dado f (t) como una entrada (tomando en cuenta que conocemos
los Eigenvalores H (s))
Interpreta como H manipula f (t)

6.2.2 Series de Fourier


3
Joseph Fourier demostr que para una funcin peridica-T (Section 6.1) f (t) puede ser escrita como una
combinacin linear de senosoidales complejos armnicos.


X
f (t) = cn ej0 nt (6.3)
n=

2
Donde
T es la frecuencia fundamental. Para casi todas f (t) de inters practico, existe cn que hace la
0 =
2
(6.3) verdadera. Si f (t) es de energa nita (f (t) L [0, T ]), entonces la igualdad de (6.3) sostienen la idea
de convergencia de energa; si f (t) es continua, entonces (6.3) sostiene la idea punto por punto. Tambin
si f (t) tiene algunas condiciones intermedias (las condiciones de DIRICHLET), la ecuacin (6.3) se sostiene
punto por punto en todas partes excepto en los puntos de descontinuidad.
Los cn -son conocidos como los coecientes de Fourier- que nos dicen que tanto del sinusoidal ej0 nt
esta presente en f (t). (6.3) esencialmente descompone f (t) en pedazos, los cuales son procesados fcilmente
por una sistema LTI (ya que existe una Eigenfuncion para cada sistema
 LTI). En trminos matemticos,

(6.3) nos dice de un conjunto de exponenciales complejos armnicos ej0 nt , n Z forman una base
para el espacio de funciones T-peridicas continuas. Aqu se muestran algunos ejemplos que les ayudaran a
pensar en una seal o funcin, f (t), en trminos de sus funciones exponenciales bases.

6.2.2.1 Ejemplos

Para cada una de las funciones de abajo, descomponlas en sus partes ms simples y encuentra sus coe-
cientes de Fourier. Oprima para ver la solucin.

Exercise 6.2.1 (Solution on p. 123.)

f (t) = cos (0 t)

Exercise 6.2.2 (Solution on p. 123.)

f (t) = sin (20 t)

3 http://www-groups.dcs.st-and.ac.uk/history/Mathematicians/Fourier.html

Available for free at Connexions <http://cnx.org/content/col10373/1.2>


98 CHAPTER 6. SERIES FOURIER DEL TIEMPO CONTINUO

Exercise 6.2.3 (Solution on p. 123.)

f (t) = 3 + 4cos (0 t) + 2cos (20 t)

6.2.3 Coecientes de Fourier


En general f (t), los coecientes se pueden calcular por medio de (6.3) al despejar por cn , lo cual requiere
una pequea manipulacin algebraica (para una derivacin completa de estos coeciente por favor vea la
seccin titulada derivacin para los coeciente de Fourier (Section 6.3). El resultado nal da la siguiente
ecuacin general para estos coecientes:

Z T
1
cn = f (t) e(j0 nt) dt (6.4)
T 0

La secuencia de nmeros complejos {cn , n Z } es una representacin compleja alterna de la funcin


f (t). Conocer los coecientes Fourier cn es lo mismo que conocer f (t) y viceversa. Dada a una funcin
peridica, la podemostrasformar en su representacin de series de Fourier usando (6.4). As mismo,
podemos sacar la transformada inversa a una secuencia de nmeros complejos, cn , usando (6.3) para
reconstruir la funcin f (t).
As como es una manera natural para representar las seales que son manipuladas por los sistemas LTI,
las series de Fourier proveen una descripcin par alas seales peridicas que son convenientes de muchas
maneras. Al ver las series de Fourier f (t), podemos inferir las propiedades matemticas de f (t) como la
propiedad de suavidad, la existencia de una simetra, as como el signicado fsico de las frecuencias.

6.2.3.1 Ejemplo: Usando la Ecuacin del Coeciente de Fourier

Aqu veremos un simple ejemplo que alo mas requiere el uso de (6.4) para resolver los coecientes de Fourier.
Una vez que usted entienda la formula, la solucin se convierte en un problema comn de calculo. Encontr
los coecientes de Fourier de la siguiente ecuacin:

Exercise 6.2.4 (Solution on p. 123.)



1 if |t| T
f (t) =
0 otherwise

6.2.4 Resumen: Ecuaciones de las Series de Fourier


Nuestra primera ecuacin (6.3) es la ecuacin de sntesis, la cual construye nuestra funcin, f (t), al combinar
senosoldales.

Synthesis

X
f (t) = cn ej0 nt (6.5)
n=

Nuestra segunda ecuacin (6.4), llamada la ecuacin de anlisis, revela que tanto de cada sinusoidal existe
en f (t).
Analisis Z T
1
cn = f (t) e(j0 nt) dt (6.6)
T 0
2
Donde hemos dicho que 0 = T .

Available for free at Connexions <http://cnx.org/content/col10373/1.2>


99

note: Entienda que nuestro intervalo de integracin no tiene que ser [0, T ] nuestra ecuacin de
anlisis. Podemos usar cualquier intervalo [a, a + T ] de tamao T.
Example 6.1
Esta demostracin le ayuda a sintetizar una seal al combinar los senosoidales, muy similar a la
4
ecuacin de sntesis para las series de Fourier. Vea aqu para instrucciones de como usar este demo.

This is an unsupported media type. To view, please see


http://cnx.org/content/m12893/latest/Fourier_Component_Manipulation.vi

6.3 Derivacin de la Ecuacin de Coecientes de Fourier5


6.3.1 Introduccin
6
Usted debera estar familiarizado con la existencia de la ecuacin general de las series de Fourier que es la
siguiente:

X
f (t) = cn ej0 nt (6.7)
n=

De lo que estamos interesados aqu es el como combinar los coecientes de Fourier, cn , dado a una funcin
f (t). En la siguiente explicacin lo llevaremos paso por paso por la derivacin de la ecuacin generar para
los coecientes de Fourier dado a una funcin.

6.3.2 Derivacin
Para resolver la ecuacin (6.7) para cn , tenemos que hacer una pequea manipulacin algebraica. Primero
que todo, tenemos que multiplicar los dos lados de (6.7) por e(j0 kt) , donde k Z.

X
f (t) e(j0 kt) = cn ej0 nt e(j0 kt) (6.8)
n=

Ahora integraremos los dos lados sobre el periodo, T:


Z T Z T
X
f (t) e(j0 kt) dt = cn ej0 nt e(j0 kt) dt (6.9)
0 0 n=

En el lado derecho podemos intercambiar la sumatoria y el integral y sacar la constante fuera del integral.

Z T
X Z T
f (t) e(j0 kt) dt = cn ej0 (nk)t dt (6.10)
0 n= 0

Ahora que hemos hecho esto lo que al parecer es mas complicado, nos enfocaremos en tan solo el integral,
RT
0
ej0 (nk)t dt, que se encuentra en el lado derecho de la ecuacin. Para este integral debemos considerar
solo dos casos: n = k y n 6= k . Para n = k tenemos:
Z T
ej0 (nk)t dt = T , n = k (6.11)
0
4 "How to use the LabVIEW demos" <http://cnx.org/content/m11550/latest/>
5 This content is available online at <http://cnx.org/content/m12894/1.1/>.
6 "Fourier Series: Eigenfunction Approach": Section Fourier Series <http://cnx.org/content/m10496/latest/#fs>

Available for free at Connexions <http://cnx.org/content/col10373/1.2>


100 CHAPTER 6. SERIES FOURIER DEL TIEMPO CONTINUO

Para n 6= k , tenemos:
Z T Z T Z T
j0 (nk)t
e dt = cos (0 (n k) t) dt + j sin (0 (n k) t) dt , n 6= k (6.12)
0 0 0

Pero cos (0 (n k) t) tiene periodos con nmeros enteros para, n k, entre 0 y T. Imagine la graca de
un coseno; por que tiene periodos con nmeros enteros, hay reas de igual valor debajo y arriba del eje de
las ordenadas en la graca. Este hecho es verdadero para sin (0 (n k) t) tambin. Lo que signica
Z T
cos (0 (n k) t) dt = 0 (6.13)
0
Tambin para el integral de una funcin de seno. Por eso, podemos concluir lo siguiente sobre nuestro
integral:
T T n=k
Z
if
ej0 (nk)t dt = (6.14)
0 0 otherwise

Regresemos a nuestra complicada ecuacin, (6.10), para ver si podemos encontrar una ecuacin para nuestros
coecientes de Fourier. Usando los hechos que ya hemos probado, podemos ver que nica vez que la ecuacin
(6.10)tiene valores de no cero como resultado es cuando k y n son iguales:
Z T
f (t) e(j0 nt) dt = T cn , n = k (6.15)
0
Finalmente nuestra ecuacin general para los coecientes de Fourier es:
Z T
1
cn = f (t) e(j0 nt) dt (6.16)
T 0

6.3.2.1 Pasos para Encontrar los Coecientes de Fourier

Para encontrar los coecientes de Fourier de una f (t)peridica:


(j0 kt)
1. Por alguna k, multiplique f (t) por e , saque el rea por debajo de la curva (dividiendo por T ).
2. Repita el paso (1) para todo k Z.

6.4 Generalidades de las Series de Fourier7


6.4.1 Introduccin
El integral de convolucion (Section 3.2) es una expresin fundamental que relacin la entrada y la salida de
un sistema LTI. Sin embargo, tiene tres problemas:

1. Puede ser tediosa para calcular.


2. Ofrece una interpretacin fsica limitada de lo que el sistema esta realmente hacienda.
3. Da muy poca informacin de como disear sistemas para lograr ciertas funciones.

Las series de Fourier (Section 6.2), junto la transformada de Fourier y la transformada de La Place, provee
8 9
una manera de resolver estos tres puntos. El concepto de eigenfuncion (o eigenvector ) es esencial para todos
estos mtodos. Ahora veremos como podemos re-escribir cualquier seal f (t), en trminos de exponenciales
complejos (Section 1.5).
De hecho, al hacer nuestras anotaciones de seales y sistemas lineares menos matemticas, podemos
extraer paralelos entre seales y sistemas con y algebra linear (Section 5.1).

7 This content is available online at <http://cnx.org/content/m12896/1.2/>.


8 "Eigenfunctions of LTI Systems" <http://cnx.org/content/m10500/latest/>
9 "Matrix Diagonalization" <http://cnx.org/content/m10738/latest/>

Available for free at Connexions <http://cnx.org/content/col10373/1.2>


101

6.4.2 Eigenfunciones en Sistemas LTI


La accin que ejerce un sistema LTI H [. . . ] en una de sus eigenfunciones est es

1. Extremadamente fcil (y rpida) de calcular

H [st] = H [s] est (6.17)

2. Fcil de interpretar: H [. . . ] nada mas escala est , manteniendo una frecuencia constante.
Si tan solo todas las funciones fueran funciones de H [. . . ] ...

6.4.2.1 Sistemas LTI

... claro, no todas las funciones pueden ser esto pero para sistemas LTI, sus eigenfunciones expanden
(Section 5.1.2: Subespacio Generado) el espacio de funciones peridicas (Section 6.1), lo que signica que
para, (casi) todas las funciones peridicas podemos encontrar f (t) we can nd {cn } where nZ and ci C
such that:

X
f (t) = cn ej0 nt (6.18)
n=
Dada (6.18), podemos re-escribir H [t] = y (t) como el siguiente sistema

Figure 6.4: Funciones de Transferencia modeladas como un sistema LTI.

Donde tenemos:
X
f (t) = cn ej0 nt
n
X
y (t) = cn H (j0 n) ej0 nt
n
Esta transformacin de f (t) en y (t) tambin se puede ilustrar a travs del proceso mostrado abajo.

f (t) {cn } {cn H (j0 n)} y (t) (6.19)

Donde los tres pasos (echa) en nuestra ilustracin de arriba y representa a las siguientes tres operaciones:

1. Transformacin con anlisis(ecuacin de coecientes de Fourier (Section 6.3)):


Z T
1
cn = f (t) e(j0 nt) dt
T 0
10
2. La accin de H en las series de Fourier  iguala a una multiplicacin por H (j0 n)
3. Regrese a las antiguas bases- transforme inversamente usando nuestra ecuacin de sntesis que viene
de las series de Fourier:

X
y (t) = cn ej0 nt
n=

10 "Fourier Series: Eigenfunction Approach" <http://cnx.org/content/m10496/latest/>

Available for free at Connexions <http://cnx.org/content/col10373/1.2>


102 CHAPTER 6. SERIES FOURIER DEL TIEMPO CONTINUO

6.4.3 Interpretacin Fsica de las Series de Fourier


Las series de Fourier {cn } de una seal f (t), denida en (6.18), tiene una interpretacin fsica muy impor-
tante. El coeciente cn nos dice que tanto de la frecuencia 0 n existe en la seal.
Seales que cambien lentamente en el tiempo- seales suaves- tienen un gran cn para pequeas n.

(a) (b)

Figure 6.5: Empezaremos con nuestra seal suave f (t) en la izquierda, y despues usaremos las series
de Fourier para encontrar los coecientes de Fourier- lo cual se muestra en la gura de la derecha.

Seales que cambian rpidamente con el tiempo- seales ruidosas-tienen una gran cn para grandes n.

(a) (b)

Figure 6.6: Empezaremos con nuestra seal ruidosa f (t) en el lado izquierdo, y usaremos las series
de Fourier pada encontrar los coecientes de Fourier- lo cual se muestra en la gura de la derecha.

Example 6.2: Pulso Peridico  T T


Tenemos la siguiente funcin de pulso, f (t), en el intervalo 2, 2 :

Figure 6.7: Seal Periodica f (t)

Available for free at Connexions <http://cnx.org/content/col10373/1.2>


103

Usando nuestra formula para los coecientes de Fourier,

Z T
1
cn = f (t) e(j0 nt) dt (6.20)
T 0

Podemos calcular fcilmente nuestra cn . Dejaremos este clculo como ejercicio para usted! Despus
de resolver la ecuacin para nuestra f (t), obtenemos el siguiente resultado:


2T1 if n = 0
T
cn = 2sin(0 nT1 )
(6.21)

n if n 6= 0

T
Para T1 = 8 , vea la siguiente gura para observar los siguientes resultados:

Figure 6.8: Nuestros coecientes de Fourier cuando T1 = T


8

Nuestra seal f (t) es plana excepto por dos orilla ( discontinuidades). Por esta razn, cn
alrededor de n = 0 son grandes y cn se vuelve pequea cuando n se acerca al innito.

note: Por qu cn = 0 para n = {. . . , 4, 4, 8, 16, . . . }? ( qu parte de e(j0 nt) se encuentra


sobre el pulso de estos valores de n?)

6.5 Propiedades de la Serie de Fourier11


Empezaremos por refrescar su memoria sobre las ecuaciones bsicas de las series de Fourier (Section 6.2):


X
f (t) = cn ej0 nt (6.22)
n=

Z T
1
cn = f (t) e(j0 nt) dt (6.23)
T 0

Deje F () describen la transformacin de f (t) a los coecientes de Fourier

F (f (t)) = cn , n Z
12
F () graca funciones con valores complejos a secuencias de nmeros complejos .

11 This content is available online at <http://cnx.org/content/m12897/1.2/>.


12 "Complex Numbers" <http://cnx.org/content/m0081/latest/>

Available for free at Connexions <http://cnx.org/content/col10373/1.2>


104 CHAPTER 6. SERIES FOURIER DEL TIEMPO CONTINUO

6.5.1 Linealidad
F () es una transformacin linear.
Theorem 6.1:
Si F (f (t)) = cn y F (g (t)) = dn . Entonces

F (f (t)) = cn , C

y
F (f (t) + g (t)) = cn + dn
Proof:
Muy fcil, nada mas es la linealidad del integral.

RT
F (f (t) + g (t)) = 0
(f (t) + g (t)) e(j0 nt) dt , n Z
1 T
RT
f (t) e(j0 nt) dt + T1 0 g (t) e(j0 nt) dt , n Z
R
= T 0
(6.24)
= cn + dn , n Z
= cn + dn

6.5.2 Desplazamiento
Desplazamiento en el tiempo es igual a un desplazamiento angular de los coecientes de Fourier (Section 6.3)
Theorem 6.2:
F (f (t t0 )) = e(j0 nt0 ) cn si cn = |cn |ej(cn ) , entonces

|e(j0 nt0 ) cn | = |e(j0 nt0 ) ||cn | = |cn |

 
e(j0 t0 n) = (cn ) 0 t0 n

Proof:

RT
F (f (t t0 )) = 1
T 0
f (t t0 ) e(j0 nt) dt , n Z
R T t0
= 1
T t0
f (t t0 ) e(j0 n(tt0 )) e(j0 nt0 ) dt , nZ

j0 n t (6.25)
1 T t0
e(j0 nt0 ) dt , n Z
R
= T t0 f t e

j0 n t
= e cn , n Z

6.5.3 La Relacin de Parseval


Z T
X
2 2
(|f (t) |) dt = T (|cn |) (6.26)
0 n=

La relacin de Parseval nos permite calcular la engra de la seal de sus series de Fourier.

note: Parseval nos dice que las series de Fourier gracan maps L2 ([0, T ]) a l2 (Z).

Available for free at Connexions <http://cnx.org/content/col10373/1.2>


105

Figure 6.9

Exercise 6.5.1 (Solution on p. 124.)


Par f (t) poder tener  energa nita, que es lo que cn hace cuando n ?
Exercise 6.5.2 (Solution on p. 124.)
1
S cn = n , |n| > 0 , es f L2 ([0, T ])?
Exercise 6.5.3 (Solution on p. 124.)
Ahora , s cn = 1 , |n| > 0 , es f L2 ([0, T ])?
n
El radio de descomposicin de una serie de fourier determina si f (t) tiene energa nita.

6.5.4 Diferenciacin en el Dominio de Fourier


   
df (t)
(F (f (t)) = cn ) F = jn0 cn (6.27)
dt

Ya que

X
f (t) = cn ej0 nt (6.28)
n=

entonces
dej0 nt
d
P
dt f (t) = n= cn dt
P (6.29)
j0 nt
= c
n= n j 0 ne

Un diferenciador atena las frecuencias bajas f (t) y acenta las frecuencias altas. Remueve rasgos
generales y acenta reas con variaciones bsicas.

note: Una manera comn para medir matemticamente que la suavidad de la funcin f (t) es el
ver cuantas derivadas tienen energa nita.

Esto se hace al observar los coecientes de fourier de una seal, especcamente el que tan rpido se
  de-
dm f (t) m
scomponen cuando n . Si F (f (t)) = cn y |cn | tiene la forma n1k , entonces F dtm = (jn0 ) cn
nm th
tiene la forma . Entonces para que la m derivada tenga energa nita, necesitamos
nk

X  nm 2
| k| <
n
n

Available for free at Connexions <http://cnx.org/content/col10373/1.2>


106 CHAPTER 6. SERIES FOURIER DEL TIEMPO CONTINUO

nm 1
por lo tanto se descompone mas rpido que
nk n lo cual implica que

2k 2m > 1

o
2m + 1
k>
2
El radio de descomposicin de las series de fourier determina la suavidad.

6.5.5 Integracin en el Dominio de Fourier


Si
F (f (t)) = cn (6.30)

entonces Z t 
1
F f ( ) d = cn (6.31)
j0 n
note: Si c0 6= 0, esta expresin no tiene ningn sentido.

Integracin acenta frecuencias bajas y atena frecuencias altas. Integradores muestran las cosas gen-
erales de las seales y suprimen variaciones de corto plazo (lo cual es ruido en muchos casos). Integradores
son mejores que diferenciadores.

6.5.6 Multiplicacin de Seales


Dado a una seal f (t)
con coecientes de Fourier cn y una seal g (t) con coecientes dn , podemos denir
una nueva seal como, y (t),Pdonde y (t) = f (t) g (t). Descubrimos que la representacin de series de Fourier

de y (t), en , es tal que en = k= ck dnk . Esto es para decir que la multiplicacin de seales en el dominio
del tiempo es equivalente a la convolucin discreta (Section 4.2) en el dominio de la frecuencia. La prueba
es la siguiente
RT
en = 1
T 0
f (t) g (t) e(j0 nt) dt
R T P
= 1
c ej0 kt g (t) e(j0 nt) dt
T 0 k= Rk  (6.32)
1 T
P (j0 (nk)t)
= c
k= k T 0 g (t) e dt
P
= k= ck dnk

6.6 Propiedades de Simetra de las Series de Fourier13


6.6.1 Propiedades de Simetra
6.6.1.1 Seales Reales

Seales reales tienen una serie de fourier con un conjugado simtrico.

Theorem 6.3:

Si f (t) es real eso implica que f (t) = f (t) (f (t) es el complejo conjugado de f (t)), entonces

cn = cn lo cual implica que Re (cn ) = Re (cn ), Por ejemplo, la parte real de cn es par, y
Im (cn ) = Im (cn ), Por ejemplo, tla parte imaginaria de cn es impar. Vea Figure 6.10. Lo que
tambien implica que |cn | = |cn |, Por ejemplo, que la magnitud es par, y que la (cn ) = ( cn ),
Por ejemplo, el ngulo es impar.
13 This content is available online at <http://cnx.org/content/m12898/1.1/>.

Available for free at Connexions <http://cnx.org/content/col10373/1.2>


107

Proof:

1 T
f (t) ej0 nt dt
R
cn =
T 0R 
1 T (j0 nt)
= f (t) e dt , f (t) = f (t)
 T R0  (6.33)
1 T (j0 nt)
= T 0 f (t) e dt
= cn

(a)

(b)

Figure 6.10: Re (cn ) = Re (cn ), y Im (cn ) = Im (cn ).

Available for free at Connexions <http://cnx.org/content/col10373/1.2>


108 CHAPTER 6. SERIES FOURIER DEL TIEMPO CONTINUO

(a)

(b)

Figure 6.11: |cn | = |cn |, y (cn ) = ( cn ).

6.6.1.2 Seales Reales y Pares

Las seales reales y pares tienen series de fourier que son pares y reales.

Theorem 6.4:

If f (t) = f (t) y f (t) = f (t), Por ejemplo, las seal es real y par, entonces entonces cn = cn
y cn = cn .
Proof:
T
1
f (t) e(j0 nt) dt
R
cn = T
2
T
R02 T
1
f (t) e(j0 nt) dt + 1
f (t) e(j0 nt) dt
R
= T T2 T
2
0
T T (6.34)
1
f (t) ej0 nt dt + 1
f (t) e(j0 nt) dt
R R
= T 0
2
T 0
2

T
2
R
= T 0
2
f (t) cos (0 nt) dt
f (t) ycos (0 nt) son reales lo cual implica que cn es real.PTambin cos (0 nt) = cos ( (0 nt))

entonces cn = cn . Es tn fcil demostrar que f (t) = 2 n=0 cn cos (0 nt) ya que f (t), cn , y
cos (0 nt) son reales y pares.

6.6.1.3 Seales Reales e Impares

Seales reales e impares tienen series de fourier que son impares y completamente imaginarias.

Theorem 6.5:

Si f (t) = f (t) y f (t) = f (t) , Por ejemplo, la seal es real y impar, entonces cn = cn y

cn = cn , Por ejemplo, cn es impar y completamente imaginaria.

Available for free at Connexions <http://cnx.org/content/col10373/1.2>


109

Proof:
Hgalo usted en casa.

Si f (t) es impar, podemos expenderlos en trminos de sin (0 nt):



X
f (t) = 2cn sin (0 nt)
n=1

6.6.2 Resumen
Podemos encontrar fe (t), una funcin par, y fo (t), una funcin impar, por que

f (t) = fe (t) + fo (t) (6.35)

lo cual implica, que para cualquier f (t), podemos encontrar {an } y {bn } que da


X
X
f (t) = an cos (0 nt) + bn sin (0 nt) (6.36)
n=0 n=1

Example 6.3: La Funcin Triangular

Figure 6.12: T = 1 y 0 = 2 .

f (t) es real e impar.



4A
j 2 n2 if n = {. . . , 11, 7, 3, 1, 5, 9, . . . }



cn = j4A2 n2 if n = {. . . , 9, 5, 1, 3, 7, 11, . . . }


0 if n = {. . . , 4, 2, 0, 2, 4, . . . }

Available for free at Connexions <http://cnx.org/content/col10373/1.2>


110 CHAPTER 6. SERIES FOURIER DEL TIEMPO CONTINUO

Es cn = cn ?

Figure 6.13: Series de Fourier para una funcion triangular.

note: Usualmente podemos juntar informacin sobre la suavidad de una seal al examinar los
coecientes de Fourier.

Hecha un vistazo a los ejemplos anteriores. Las funciones del pulso y sawtooth no son continuas y sus series
1
de Fourier disminuyen como
n . La funcin triangular es continua, pero no es diferenciable, y sus series de
1
Fourier disminuyen como 2 .
n
Las siguientes 3 propiedades nos darn una mejor idea de esto.

6.7 Propiedad de Convolucin Circular de las Series de Fourier14


6.7.1 Convolucin Singular de una Seal
Dada a una seal f (t) con coecientes de Fourier cn y una seal g (t) con coecientes de Fourier dn , podemos
denir una seal, v (t), donde v (t) = f (t)~g (t) Encontramos que la representacin de las series de Fourier15
16
para y (t), an , esta que an = cn dn . f (t) ~ g (t) es una convolucin circular de dos seales peridica y es
RT RT
equivalente a una convolucin en el intervalo, f (t) ~ g (t) =
0 0
f ( ) g (t ) d dt.
note: Convulucin circular en el dominio del tiempo es equivalente a la multiplicacin de coe-
cientes de fourier.
14 This content is available online at <http://cnx.org/content/m12899/1.1/>.
15 "Fourier Series: Eigenfunction Approach" <http://cnx.org/content/m10496/latest/>
16 "Discrete Time Circular Convolution and the DTFS" <http://cnx.org/content/m10786/latest/>

Available for free at Connexions <http://cnx.org/content/col10373/1.2>


111

La prueba es la siguiente

1 T (j0 nt)
R
an = T 0 v (t) e dt
T T
1
f ( ) g (t ) d e(j0 nt) dt
R R
= T2 0 0  
1 T 1 T (j0 nt)
R R
= T 0 f ( ) g (t ) e dt d
 T R0 
T T
1 1
g () e(j0 (+ )) d d , =
R
= T 0 f ( ) T t
 R  (6.37)
1 T 1 T (j0 n)
e(j0 n ) d
R
= T 0 f ( ) T g () e d
1 T
f ( ) dn e(j0 n ) d
R
= T  0
RT 
= dn T1 0 f ( ) e(j0 n ) d
= cn dn

Example 6.4
T
Vea el pulso cuadrado con periodo, T1 = 4:

Figure 6.14

Para esta seal


1

T if n=0
cn =
1 sin( 2 n)

otherwise
2 2n

Exercise 6.7.1 (Solution on p. 124.)


1 sin2
Que seal tiene los coecientes de Fourier an = cn 2 = 4 ( n) ?
2

6.8 Series de Fourier y los Sistemas LTI17


6.8.1 Introduciendo las Series de Fourier a los Sistemas LTI
Antes de ver este modulo, usted debera familiarizarse con los conceptos de Eigenfunciones de los sistemas
LTI (Section 5.6). Recuerde, para H sistema LTI tenemos la siguiente relacin

17 This content is available online at <http://cnx.org/content/m12900/1.2/>.

Available for free at Connexions <http://cnx.org/content/col10373/1.2>


112 CHAPTER 6. SERIES FOURIER DEL TIEMPO CONTINUO

Figure 6.15: Seales de entrada y salida para nuestro sitema LTI.

donde est es una eigenfuncin de H. Su eigenvalor (Section 5.3) correspondiente H (s) pueden ser calcu-
18
lado usando la respuesta de impulso h (t)
Z
H (s) = h ( ) e(s ) d

As, usando la expansin de las series de Fourier (Section 6.2) para f (t) peridica (Section 6.1) donde
usamos la entrada
X
f (t) = cn ej0 nt
n
en el sistema,

Figure 6.16: Sistema LTI

nuestra salida y (t) ser


X
y (t) = H (j0 n) cn ej0 nt
n

Podemos ver que al aplicar las ecuaciones de expansin de series de fourier, podemos ir de f (t) a cn y
viceversa, y es lo mismo para la salida, y (t)

6.8.2 Efectos de las Series de Fourier


Podemos pensar de un sistema LTI como el ir moldeando el contenido de la frecuencia de la entrada.
Mantenga en mente el sistema bsico LTI que presentamos en Figure 6.16. El sistema LTI, H, multiplica
todos los coecientes de Fourier y los escala.
Dado los coecientes de Fourier de la entrada {cn } y los eigen valores del sistema {H (jw0 n)}, las series
de Fourier de la salida, es {H (jw0 n) cn } (una simple multiplicacin de termino por termino).

note: los eigenvalores, H (jw0 n) describen completamente lo que un sistema LTI le hace a una
seal peridica con periodo T = 2w0
18 "Continuous Time Impulse Function" <http://cnx.org/content/m10059/latest/>

Available for free at Connexions <http://cnx.org/content/col10373/1.2>


113

Example 6.5
Qu hace este sistema?

Figure 6.17

Example 6.6
Y, est sistema?

(a) (b)

Figure 6.18

6.8.3 Examples
Example 6.7: El circuito RC

1 t
h (t) = e RC u (t)
RC
Qu es lo que este sistema hace a las series de fourier de la f (t)?
Calcula los eigenvalores de este sistema

R
H (s) =
h ( ) e(s ) d
R
= 0 RC
1
e RC e(s ) d
1
R ( )( 1 +s)
= RC 0 e
RC d (6.38)
1 1 ( )( RC
1
+s)
= RC RC 1
+s
e | =0
1
= 1+RCs

Ahora, decimos que a este circuito RC lo alimentamos con una entrada f (t) peridica (con
periodo T = 2w0 ).

Available for free at Connexions <http://cnx.org/content/col10373/1.2>


114 CHAPTER 6. SERIES FOURIER DEL TIEMPO CONTINUO

Vea los eigen valores para s = jw0 n


1 1
|H (jw0 n) | = =
|1 + RCjw0 n| 1 + R C 2 w 0 2 n2
2

El circuito RC es un sistema pasa bajas: pasa frecuencias bajas n alrededor de 0) atena


frecuencias altas ( n grandes).

Example 6.8: Puls cuadrado a travs del Circuito RC

Seal de entrada : tomando las series de Fourier f (t)

1 sin 2 n

cn =
2 2n

1
t en n = 0
Sistema : Eigenvalores
1
H (jw0 n) =
1 + jRCw0 n
Seal de salida: tomando las series de Fourier de y (t)

1 sin 2 n

1
dn = H (jw0 n) cn =
1 + jRCw0 n 2 2n

1 sin 2 n

1
dn =
1 + jRCw0 n 2 2n
X
y (t) = dn ejw0 nt
n

Qu podemos decir sobre y (t) de {dn }?


1. Es y (t) real?
2. Es y (t) simtrico par? simtrico impar?
3. Com se y (t) es mas suave que f (t)? (el radio de descomposicin de dn vs. cn )

1 sin 2 n

1
dn =
1 + jRCw0 n 2 2n

1 sin 2 n

1
|dn | = q
1 + (RCw0 ) n
2 2 2 2n

6.9 Convergencia de las Series de Fourier19


6.9.1 Introduccin
Antes de ver este mdulo, esperamos que usted este completamente convencido que cualquier funcin
peridica f (t), se puede representar como una suma de senosoidales complejos (Section 1.3). Si usted no lo
esta, intente ver la seccin de generalidades de eigenfunciones (Section 5.5) o de eigenfunciones de los sistemas

19 This content is available online at <http://cnx.org/content/m12934/1.2/>.

Available for free at Connexions <http://cnx.org/content/col10373/1.2>


115

LTI (Section 5.6). Hemos demostrado que podemos representar la seal como una suma de exponenciales
usando las ecuaciones de las series de Fourier (Section 6.2) mostradas aqu:

X
f (t) = cn ej0 nt (6.39)
n
Z T
1
cn = f (t) e(j0 nt) dt (6.40)
T 0
20
Joseph Fourier insisti que estas ecuaciones eran verdaderas, pero nunca las pudo probar. Lagrange
ridiculiz pblicamente a Fourier y dijo que solo funciones continuas podran ser representadas como (6.39)
(as que probo que la (6.39) funciona para funciones continuas en el tiempo). Sin embargo, nosotros sabemos
que la verdad se encuentra entre las posiciones que Fourier y Lagrange tomaron.

6.9.2 Comprendiendo la Verdad


Formulando nuestra pregunta matemticamente, deje

N
X
fN 0 (t) = cn ej0 nt
n=N

donde cn es igual a los coecientes de Fourier, f (t). (vea (6.40)).


fN 0 (t) es la reconstruccin parcial de f (t) usando los primeros 2N + 1 coecientes de Fourier. fN 0 (t)
se aproxima af (t), con la aproximacin mejorando cuando N se vuelve grande. As que, podemos pensar
0

que el conjunto fN (t) , N = {0, 1, . . . } es una secuencia de funciones, cada una aproximando f (t)
mejor que la anterior.
La pregunta se convierte en , s esta secuencia converge a f (t)? fN 0 (t) f (t) asi como a N ?
Trataremos de responder estas preguntas al ver la convergencia de dos maneras diferentes:

1. Viendo la energa de la seal de error:

eN (t) = f (t) fN 0 (t)


dfN (t)
2. Viendo el limit en cada punto y comparndolo con f (t).
N d

6.9.2.1 Primer Mtodo

Sea eN (t) la diferencia (i.e. error) entre la seal f (t) y su reconstruccin parcial fN 0 (t)
eN (t) = f (t) fN 0 (t) (6.41)

Si f (t) L2 ([0, T ]) (energa nita), entonces la energa de eN (t) 0 cuando N es


Z T Z T 2
2
(|eN (t) |) dt = f (t) fN 0 (t) dt 0 (6.42)
0 0

Podemos comprobar esta ecuacin usando la relacin de Perseval al:

Z T    2
0
2 X dfN (t) X 2
limit |f (t) fN (t) | dt = limit |Fn (f (t)) Fn | = limit (|cn |) = 0
N 0 N d N
N = |n|>N

Donde la ecuacin antes del cero es la suma de la ltima parte de las series de Fourier. La cual se aproxima a
cero por que f (t) L2 ([0, T ]). Ya que el sistema fsicos responden a la energa, las series de Fourier proveen
una representacin adecuada para todo f (t) L2 ([0, T ]) igualando la energa nita sobre un punto.

20 http://www-groups.dcs.st-and.ac.uk/history/Mathematicians/Fourier.html

Available for free at Connexions <http://cnx.org/content/col10373/1.2>


116 CHAPTER 6. SERIES FOURIER DEL TIEMPO CONTINUO

6.9.2.2 Segundo Mtodo


dfN (t)
El hecho de que eN 0 no nos dice nada sobre que f (t) y limit sean iguales en cierto punto.
N d
Tomemos las siguientes funciones como un ejemplo:

(a) (b)

Figure 6.19

Dadas estas dos funciones f (t) y g (t), podemos observar que para todo t, f (t) 6= g (t), pero

Z T
2
(|f (t) g (t) |) dt = 0
0

De esto podemos derivar las siguientes relaciones:

Convergencia de energa 6= convergencia de punto por punto

Convergencia de punto por punto convergencia en L2 ([0, T ])


Sin embargo, lo contrario de la proposici no es cierto.
Resulta que si f (t) tiene una discontinuidad (como se puede observar en la gura de g (t)) en t0 ,
entonces
dfN (t0 )
f (t0 ) 6= limit
N d
Mientras f (t) tenga algunas de las condiciones, entonces

dfN (t0 )
f (t0 ) = limit
N d
si f (t) es continua en t = t0 .

6.10 Condiciones de Dirichlet21


Nombrado en honor del matemtico Alemn Peter Dirichlet, las condiciones Dirichlet son las condiciones
que garantizan la existencia y convergencia de las series de Fourier22 o de las trasformadas de Fourier23 .
21 This content is available online at <http://cnx.org/content/m12927/1.2/>.
22 "Fourier Series: Eigenfunction Approach" <http://cnx.org/content/m10496/latest/>
23 "Derivation of the Fourier Transform" <http://cnx.org/content/m0046/latest/>

Available for free at Connexions <http://cnx.org/content/col10373/1.2>


117

6.10.1 Las Condiciones Dbiles para las Series de Fourier


Rule 6.1: La Condicin Dbil de Dirichlet
Para que las series de fourier existan, los coecientes de fourier deben ser nitos, esta condicin
garantiza su existencia. Esencialmente dice que el integral del valor absoluto de la seal debe ser
nito. Los lmites de integracin son diferentes para el caso de las series de fourier y de los del caso
de las trasformada de Fourier. Este es el resultado que proviene directamente de las diferencias en
las deniciones de las dos.
Proof:
Las series de fourier existen (los coecientes son nitas) si

Las Condiciones Dbiles para las Series de Fourier


Z T
|f (t) |dt < (6.43)
0

Esto se puede probar usando la condicin inicial de los coecientes iniciales de las series de fourier
que pueden ser nitas.

Z T Z T
1 (j0 nt) 1
|cn | = | f (t) e dt| |f (t) ||e(j0 nt) |dt (6.44)
T 0 T 0

Recordando los exponenciales complejos


24
, sabemos que la ecuacin anterior |e(j0 nt) | = 1, nos
da Z T Z T
1 1
|f (t) |dt = |f (t) |dt (6.45)
T 0 T 0

(6.46)

note: Si tenemos la funcin:


1
f (t) = , 0<tT
t
Entonces usted debera observar que esta funcin no tiene la condicin necesaria.

6.10.1.1 Las Condiciones Dbiles para la Transformada de Fourier

Rule 6.2:
La transformada de fourier existe si

las condiciones dbiles para la trasformada de Fourier


Z
|f (t) |dt < (6.47)

Esto se puede derivar de la misma manera en la que se derivo las condiciones dbiles de Dirichlet
para las series de fourier, se empieza con la denicin y se demuestra que la transformada de fourier
debe de ser menor que innito en todas partes.

24 "Continuous Time Complex Exponential" <http://cnx.org/content/m10060/latest/>

Available for free at Connexions <http://cnx.org/content/col10373/1.2>


118 CHAPTER 6. SERIES FOURIER DEL TIEMPO CONTINUO

6.10.2 Las Condiciones Fuertes de Dirichlet


La transformada de Fourier existe si la seal tiene un nmero nito de discontinuidades y un nmero nito
de mximos y mnimos. Para que las series de fourier existan las siguientes dos condiciones se deben de
satisfacer (junto con la condicin dbil de Dirichlet):

1. En un periodo, f (t) tiene solo un nmero nito de mnimos y mximos.


2. En un periodo, f (t) tiene un numero nito de discontinuidades y cada una es nita.

Esto es a lo que nos referimos como las condiciones fuertes de Dirichlet. En teora podemos pensar en
seales que violan estas condiciones por ejemplo sin (logt), sin embargo, no es posible crear esta seal en un
laboratorio. Por eso, cualquier seal en el mundo real tendr una representacin de Fourier.

6.10.2.1 Ejemplo

Asumamos que tenemos la siguiente funcin e igualdad:

dfN (t)
f 0 (t) = limit (6.48)
N d
Si f (t) tiene las tres condiciones Fuertes de Dirichlet, entonces

f ( ) = f 0 ( )

en cualquier donde f (t) es continuo y donde f (t) es discontinuo, f 0 (t) es el valor promedio del lado
derecho e izquierdo. Vea las siguientes Figure 6.20 como un ejemplo:

(a) (b)

Figure 6.20: Funciones Discontinuas, f (t).

note: Las funciones que no cumplen con las condiciones de Dirichlet son patolgicas como inge-
nieros, no estamos interesados en ellos.

6.11 El Fenmeno de Gibbs25


6.11.1 Introduccin
Las series de Fourier (Section 6.2) son la representacin de las seales peridicas continuas en el tiempo
dado en trminos de exponenciales complejos. Las condiciones de Dirichlet (Section 6.10) sugieren que las

25 This content is available online at <http://cnx.org/content/m12929/1.1/>.

Available for free at Connexions <http://cnx.org/content/col10373/1.2>


119

seales discontinuas pueden tener representacin de serie de fourier siempre y cuando tengan un nmero
nito de discontinuidades. Esto parece decir lo contrario de lo que hemos explicado, ya que los exponenciales
complejos (Section 1.5) son funciones continuas. No parece que sea posible el poder reconstruir exactamente
una funcin discontinua de un conjunto de funciones continuas. De hecho, no se puede. Sin embargo,
podemos relajar la condicin de `exactamente' y remplazarla con la idea de casi en todos lados. Esto es
para decir que la reconstruccin es exactamente la misma de la seal origina excepto en un numero de puntos
nitas. Estos puntos ocurren en las discontinuidades.

6.11.1.1 Historia

En los 1800s, muchas maquinas fueron construidas para calcular los coecientes de Fourier y re-sintetizar:

N
X
fN 0 (t) = cn ej0 nt (6.49)
n=N

Albert Michelson (un fsico experimental extraordinario) construyo una maquina en 1898 que poda calcular
cn hasta n = (79), y re-sintetizo
79
X
f79 0 (t) = cn ej0 nt (6.50)
n=79
La maquina funciono muy bien en todos los exmenes excepto en esos que tenan funciones descontinas.
Cuando una funcin cuadrada como la mostrada en Figure 6.21 (Fourier series approximation of a square
wave),fue dada a la maquina aparecieron garabatos alrededor de las discontinuidades, aunque el numero
de los coecientes de fourier llegaron a ser casi innitos, los garabatos nunca desaparecieron- esto se puede
observar en las Figure 6.21 (Fourier series approximation of a square wave). J. Willard Gibbs fue el primero
en explicar este fenmeno en 1899, por eso estos puntos son conocidos como el Fenmeno de Gibbs.

6.11.2 Explicacin
Empezaremos esta discusin tomando una seal con un nmero nito de discontinuidades (como el pulso
cuadrado) y encontrando su representacin de series de fourier. Entonces trataremos de reconstruir esta
seal usando sus coecientes de fourier. Vemos que entre mas coecientes usemos, la seal reconstruida se
parece ms y ms a la seal original. Sin embargo, alrededor de las discontinuidades, observamos ondulaciones
que no desaparecen. Al considerar el uso de ms coecientes, las ondulaciones se vuelven estrechas, pero
no desaparecen. Cuando llegamos a un nmero casi innito de coecientes, estas ondulaciones continan
ah. Esto es cuando aplicamos la idea de casi en todos lados. Mientras estas ondulaciones siguen presentes
(estando siempre arriba del 9% de la altura del pulso), el rea dentro de ellas tiende a ser cero, lo que
signica que la energa de las ondulaciones llega a ser cero. Lo que demuestra que su anchura tiende a
ser cero y podemos saber que la reconstruccin de la seal es exactamente igual ala seal original excepto
en las discontinuidades. Ya que las condiciones de Dirichlet dicen que pueden haber un numero nito de
discontinuidades, podemos concluir que el principio de casi en todos lados es cumplido. Este fenmeno es
un caso especco de una convergencia no-uniforme.
Usaremos una funcin cuadrada, junto con sus series de Fourier, para ver gures donde se muestra este
fenmeno con detalles un poco mas matemticos.

6.11.2.1 Funcin cuadrada

Las series de Fourier de esta seal nos dicen que sus lados derechos e izquierdos son iguales. Para entender
el fenmeno de Gibbs, necesitaremos redenir la manera en la que vemos la igualdad.

   
X 2kt X 2kt
s (t) = a0 + ak cos + bk sin (6.51)
T T
k=1 k=1

Available for free at Connexions <http://cnx.org/content/col10373/1.2>


120 CHAPTER 6. SERIES FOURIER DEL TIEMPO CONTINUO

6.11.2.2 Ejemplo

La siguiente Figure 6.21 (Fourier series approximation of a square wave) nos muestra varias aproximaciones
26
para las series de fourier de la funcin cuadrada , usando un nmero variado de trminos escrito por K:

Fourier series approximation of a square wave

Figure 6.21: Aproximacion de las series de Fourier para sq (t). El numero de terminos en la suma
de Fourier esta indicada en cada graca, la funcion cuadrada es mostrada en dos periodos con una linea
quebradiza.

Cuando comparamos la funcin cuadrada con su representacin de la series de Fourier Figure 6.21 (Fourier
series approximation of a square wave), no se puede ver claramente que las dos son iguales. El hecho de
que las series de fourier requieren ms trminos para una representacin ms exacta no es importante. Sin
embargo, una inspeccin mas detallada Figure 6.21 (Fourier series approximation of a square wave) revela
un posible problema: la series de Fourier pueden igualar la funcin cuadrada en todos los valores de t? ?
En particular, en cualquier cambio de paso en la funcin, las series de fourier muestran un pico seguido por
oscilaciones rpidas. Cuando se adhieren ms trminos a las series, las oscilaciones se vuelven ms rpidas

26 "Fourier Series Approximation of a Square Wave" <http://cnx.org/content/m0041/latest/>

Available for free at Connexions <http://cnx.org/content/col10373/1.2>


121

y ms pequeas, pero los picos no disminuyen. Consideremos intuitivamente esta pregunta matemtica:
puede una funcin discontinua, como la funcin cuadrada, ser expresada como una suma, aun si es innita,
de funciones continuas? Uno tiene que sospechar que de hecho no se puede expresar. Esta pregunta le
27
produjo a Fourier criticas de la academia de ciencia Francesa (Lapace, Legendre, y Lagrande formaban
parte del comit de revisin) por muchos anos de su presentacin en 1907. Esto no fue resuelto por mas se
cien anos, y la solucin es algo importante e interesante de entender para un punto de vista practico.
Estos picos en la series de fourier de la funcin cuadrada nunca desaparecen; son llamados el fen-
meno de Gibb nombrado por el fsico Americano Josiah Willard Gibb. Ocurran cada vez que las seales
discontinuas, y siempre estarn presentes cuando la seal tiene brincos.

6.11.2.3 Redeniendo Igualdad

Regresemos a la pregunta de igualdad; como se puede justicar la seal de igualdad en la denicin de


la series de Fourier? La respuesta parcial es que cada punto todos los valores de tla igualdad no es
garantizada. Lo que los matemticos del siglo 19 fue que el error RMS de las series de Fourier eran cero.

limit rms (K ) = 0 (6.52)


K

Lo que signica es que la diferencia entre la seal actual y su representacin en fourier puede no ser cero, pero
el cuadrado de esta cantidad tiene una integral igual a cero! Es a travs del valor de RMS que denimos
igualdad: dos seales s1 (t), s2 (t) se dicen ser iguales en el promedio cuadrado si rms (s1 s2 ) = 0. Estas
seales se dicen ser iguales punto a punto si s1 (t) = s2 (t) para todos los valores de t. Para las series
de fourier, los picos del fenmeno de Gibbs tienen altura nita y anchura de cero: el error diere de cero
solo en punto aislado- cuando la seal peridica contiene discontinuidades- igual a 9% del tamao de la
discontinuidad. El valor de la funcin en conjunto de puntos nitos afecta el integral. Este efecto no explica
la razn del porque denimos el valor de una funcin descontina en su discontinuidad. El valor que se
escoja para esta discontinuidad no tiene una relevancia prctica para el espectro de la seal o el como el
sistema responde a esta seal. Las series de fourier valoradas en  en la discontinuidad es el promedio de
los valores en cualquier lado del salto.

6.12 Resumen de las Series de Fourier28


Abajo veremos algunos de los conceptos ms importantes de las series de Fourier (Section 6.2) y nuestro
entendimiento usando eigenfunciones y eigenvalores. Ojala este familiarizado con este material para que este
documento sirva como un repaso, pero si no, use todos los links de informacin dados en los temas.

1. Podemos representar una funcin peridica (Section 6.1) o una funcin en un intervalo f (t) como la
combinacin de exponenciales complejos (Section 1.5):


X
f (t) = cn ej0 nt (6.53)
n=
Z T
1
cn = f (t) e(j0 nt) dt (6.54)
T 0
Donde los coecientes de Fourier, cn , igualan cuanto de la frecuencia 0 n existen en la seal.
2. Ya que ej0 nt son eigenfunciones de sistema LTI (Section 6.8) podemos interpretar la accin de un
29
sistema en una seale en termino de sus eigenvalores :
Z
H (j0 n) = h (t) e(j0 nt) dt (6.55)

27 http://www-groups.dcs.st-and.ac.uk/history/Mathematicians/Fourier.html
28 This content is available online at <http://cnx.org/content/m12932/1.3/>.
29 "Eigenvectors and Eigenvalues" <http://cnx.org/content/m10736/latest/>

Available for free at Connexions <http://cnx.org/content/col10373/1.2>


122 CHAPTER 6. SERIES FOURIER DEL TIEMPO CONTINUO

|H (j0 n) | es grande el sistema acenta la frecuencia 0 n


|H (j0 n) | es pequeo el sistema atena el 0 n
3. En adicin el {cn } de una funcin peridica f (t) nos puede decir sobre:

simetras en f (t)
suavidad en f (t), where donde la suavidad se puede interpretar como el radio de decadencia |cn |.
4. Podemos aproximar una funcin a de-sintetizar usando algunos valores en el coeciente de fourier (
truncando la S.F.)
X
fN 0 (t) = cn ej0 nt (6.56)
nn|N |

Esta aproximacin funciona bien donde f (t) es continuo pero no funcin tambin cuando f (t) is
discontinuous. es descontinu esto es explicado por el fenmeno de Gibb (Section 6.11).

Available for free at Connexions <http://cnx.org/content/col10373/1.2>


123

Solutions to Exercises in Chapter 6


Solution to Exercise 6.2.1 (p. 97)
La parte mas confusa de esta problema es de encontrar la manera de representar esta funcin en trminos
de sus base, ej0 nt . Para hacer esto, nosotros usaremos nuestro conocimiento de la relacin de Euler (Sec-
tion 1.5.2: La Relacin de Euler) para representar nuestra funcin de coseno en trminos de su exponencial.

1  j0 t 
f (t) = e + e(j0 t)
2
Con esta forma y con (6.3), al inspeccionarla podemos ver que nuestros coecientes son:


1

2 if |n| = 1
cn =
0 otherwise

Solution to Exercise 6.2.2 (p. 97)


Como lo hicimos en nuestro otro ejemplo usaremos una vez ms la relacin de Euler (Section 1.5.2: La
Relacin de Euler) para representar nuestra funcin de seno en trminos de funciones exponenciales.

1  j0 t 
f (t) = e e(j0 t)
2j
Asi que nuestros coecientes son

j
if n = 1

2
j
cn = 2 if n = 1


0 otherwise

Solution to Exercise 6.2.3 (p. 97)


Una vez ms utilizamos la misma tcnica. Y al nal nuestra funcin es

1  j0 t  1  j20 t 
f (t) = 3 + 4 e + e(j0 t) + 2 e + e(j20 t)
2 2
De esto podemos encontrar nuestros coecientes:




3 if n=0

|n| = 1

2 if
cn =
1

if |n| = 2


0 otherwise

Solution to Exercise 6.2.4 (p. 98)


Nosotros comenzaremos al reemplazar nuestra funcin, f (t), en (6.4). Nuestro intervalo de integracin
cambiara para igualar el intervalo especicado por la funcin

Z T1
1
cn = 1e(j0 nt) dt
T T1

Note que debemos considerar dos casos: n = 0 y n 6= 0. Para n = 0 podemos ver que despus de inspeccionar
el integral obtenemos.
2T1
cn = , n=0
T

Available for free at Connexions <http://cnx.org/content/col10373/1.2>


124 CHAPTER 6. SERIES FOURIER DEL TIEMPO CONTINUO

Para n 6= 0, tenemos que tomar mas pasos para resolverlo. Empezaremos al observar el integral bsico del
integral que tenemos. Recordando nuestro clculo estamos listos para integrar:

1 1 (j0 nt) T1
cn = e |t=T1
T j0 n
Ahora evaluaremos las funciones exponenciales para los lmites dado s y expandiremos nuestra ecuacin a:

1 1  
cn = e(j0 nT1 ) ej0 nT1
T (j0 n)
2j
Ahora multiplicaremos el lado derecho de nuestra ecuacin por
2j y distribuiremos nuestro signo negativo
dentro de nuestro parntesis, podemos utilizar la relacin de Euler para simplicar nuestra expresin a:

1 2j
cn = sin (0 nT1 )
T j0 n
2
Ahora, recuerde que anteriormente denimos 0 = T . Podemos resolver esta ecuacin para T y sustituir
en.
2j0
cn = sin (0 nT1 )
j0 n2
Finalmente, si cancelamos algunos trminos llegaremos a nuestra respuesta nal para los coecientes de
Fourier: f (t):
sin (0 nT1 )
cn = , n 6= 0
n
Solution to Exercise 6.5.1 (p. 105)
2
(|cn |) < para f (t) tener energa nita.
Solution to Exercise 6.5.2 (p. 105)
2 1
Si, por que (|cn |) = n2 , la cual se puede sumar.
Solution to Exercise 6.5.3 (p. 105)
2 1
No, por que (|cn |) = n , la cual no se puede sumar.
Solution to Exercise 6.7.1 (p. 111)

Figure 6.22: Un pulso triangular con periodo de T


4
.

Available for free at Connexions <http://cnx.org/content/col10373/1.2>


Chapter 7

Espacios de Hilbert y Expansiones


Ortogonales

7.1 Espacios Vectoriales1


7.1.1
Introduccin

Denition 7.1: Espacio Vectorial


Un espacio vectorial S es una coleccin de vectores tal que (1) si f1 S f1 S para todo
escalar (donde R C) y (2) si f1 S , f2 S , entonces (f1 + f2 ) S
Para denir un espacio vectorial lineal abstracto, necesitamos:

Un conjunto de cosas llamadas "vectores" (X )


Un conjunto de cosas llamadas "escalares" (A)
Un operador de adicin de vectores ()
Un operador de multiplicacin escalar ()

Estos operadores necesitan tener todas las siguiente propiedades. La cerradura usualmente es la mas impor-
tante para mostrar.

7.1.2 Espacio Vectorial


Si los escalares son reales, S es llamado un espacio vectorial real.
Si los escalares son complejos, S es llamado un espacio vectorial complejo.
Si los"vectores" en S son funciones o variables continuas, muchas veces S es llamado un espacio lineal
de funciones.

7.1.2.1 Propiedades

Denimos un conjunto V para ser un espacio vectorial si

1. x + y = y + x para cada x y y en V
2. x + (y + z) = (x + y) + z para cada x, y , y z en V
3. Hay un nico "vector cero" tal que x + 0 = x para cada x en V
4. Para cada x en V Hay un vector nico x tal que x + x = 0.

1 This content is available online at <http://cnx.org/content/m12878/1.2/>.


Available for free at Connexions <http://cnx.org/content/col10373/1.2>

125
126 CHAPTER 7. ESPACIOS DE HILBERT Y EXPANSIONES ORTOGONALES

5. 1x = x
6. (c1 c2 ) x = c1 (c2 x) para cada x en V and c1 y c2 en C.
7. c (x + y) = cx + cy para cada x y y en V y c en C.
8. (c1 + c2 ) x = c1 x + c2 x para cada x en V y c1 y c2 en C.

7.1.2.2 Examples

Rn = espacio vectorial real


Cn = espacio
n vectorial complejo o
1
R
L (R) = f (t) , f (t) | |f (t) |dt < es un espacio vectorial

L (R) =n{ f (t) , f (t) | f (t) es acotado } es o
un espacio vectorial
R 2
L2 (R) = f (t) , f (t) | (|f (t) |) dt < = seales de energia finitas es un espacio vectorial
2
L ([0, T ]) = funciones de energia finita en un intervalo [0, T]
`1 (Z), `2 (Z), ` (Z) son espacios vectoriales
La coleccin de funciones por pedazos constantes entre los enteros y el espacio vectorial

Figure 7.1


x x0 1
2 0
R+ = , | (x0 > 0) and (x1 > 0) es no un espacio vectorial. R+ 2 ,
x1 x1 1

1
pero / R+ 2 , < 0
1
D = {z C , |z| 1 } es no un espacio vectorial. (z1 = 1) D, (z2 = j) D, pero (z1 + z2 )
/ D,
|z1 + z2 | = 2 > 1

note: El espacio vectorial es una coleccin de funciones, coleccin de secuencias, asi como una
coleccin de vectores tradicionales (es decir una lista nita de nmeros)

Available for free at Connexions <http://cnx.org/content/col10373/1.2>


127

7.2 Normas2
7.2.1 Introduccin
Mucho del lenguaje utilizado en esta seccin ser familiar para usted- debe de haber estado expuesto a los
conceptos de

3
producto interno
ortogonalidad
4
expansin de base

en el contexto de Rn . Vamos a tomar lo que conocemos sobre vectores y aplicarlo a funciones (seales de
tiempo continuo).

7.2.2 Normas
La norma de un vector es un nmero real que representa el "tamao" de el vector.

Example 7.1
En R2 , podemos denir la norma que sea la longitud geomtrica de los vectores.

Figure 7.2

T
x = (x0 , x1 ) , norma k x k= x0 2 + x1 2
Matemticamente, una norma k k es solo una funcin (tomando un vector y regresando un
nmero real) que satisface tres reglas

Para ser una norma, kk debe satisfacer:

1. la norma de todo vector es positiva k x k> 0 , x S


2. escalando el vector, se escala la norma por la misma cantidad k x k= || k x k para todos los vectores
x y escalares
3. Propiedad del Tringulo: k x + y kk x k + k y k para todos los vectores x, y . El tamao de la
suma de dos vectores es menor o igual a la suma de sus tamaos

5
Un espacio vectorial con una norma bien denida es llamado un espacio vectorial normado o espacio
lineal normado.
2 This content is available online at <http://cnx.org/content/m12877/1.2/>.
3 "Inner Products" <http://cnx.org/content/m10755/latest/>
4 "Orthonormal Basis Expansions" <http://cnx.org/content/m10760/latest/>
5 "Vector Spaces" <http://cnx.org/content/m10767/latest/>

Available for free at Connexions <http://cnx.org/content/col10373/1.2>


128 CHAPTER 7. ESPACIOS DE HILBERT Y EXPANSIONES ORTOGONALES

7.2.2.1 Ejemplos

Example 7.2
x0

x
1 Pn1
Rn ( Cn ), x= , k x k1 = i=0 |xi |, Rn con esta norma es llamado `1 ([0, n 1]).

...

xn1

Figure 7.3: Coleccin de todas las x R2 con k x k1 = 1

Example 7.3
P  21
n1 2
Rn ( Cn ), con norma k x k2 = i=0 (|xi |) , Rn es llamado `2 ([0, n 1]) (la usual "norma

Euclideana").

Figure 7.4: Coleccin de todas las x R2 with k x k2 = 1

Available for free at Connexions <http://cnx.org/content/col10373/1.2>


129

Example 7.4
Rn (or Cn , with norm k x k = maxi {i, |xi |} is called ` ([0, n 1])

Figure 7.5: x R2 con k x k = 1

7.2.2.2 Espacios de Secuencias y Funciones

Podemos denir normas similares para espacios de secuencias y funciones.


Seales de tiempo discreto= secuencia de nmeros

x [n] = {. . . , x2 , x1 , x0 , x1 , x2 , . . . }

P
k x (n) k1 = |x [i] |, x [n] `1 (Z) (k x k1 < )
i=
P 1
2 2 2
k x (n) k2 = i= (|x [i] |) , x [n] ` (Z) (k x k2 < )
P 1  
p p p
k x (n) kp = i= (|x [i] |) , x [n] ` (Z) k x k p <
k x (n) k = sup|x [i] |, x [n] ` (Z) (k x k < )
i

Para funciones continuas en el tiempo:


R  p1  
p
k f (t) kp =
(|f (t) |) dt , f (t) Lp (R) k f (t) kp <
R  p1  
T p
(En el intervalo) k f (t) kp = 0 (|f (t) |) dt , f (t) Lp ([0, T ]) k f (t) kp <

7.3 Producto Interno6


7.3.1 Denicin: Producto Interno
De seguro ya tiene una idea del producto interno, tambin conocido como producto punto, en Rn de
alguno de sus cursos de matemticas o de cmputo. Si no, deniremos el producto interno de la siguiente
manera, tenemos dados algunas x Rn y y Rn
6 This content is available online at <http://cnx.org/content/m12876/1.2/>.

Available for free at Connexions <http://cnx.org/content/col10373/1.2>


130 CHAPTER 7. ESPACIOS DE HILBERT Y EXPANSIONES ORTOGONALES

Denition 7.2: Producto Interno


El producto interno esta denido matemticamente de la siguiente manera:

< x, y > = yT x

x0

  x1
= y0 y1 ... yn1 (7.1)

.
.
.


xn1
Pn1
= i=0 xi yi

7.3.1.1 Producto Interno en 2-D

Si tenemos x R2 y y R2 , entonces podemos escribir el producto interno como:

< x, y >=k x kk y k cos () (7.2)

donde es el ngulo entre x y y.

Figure 7.6: Grca general de vectores y ngulos mencionados en las ecuaciones anteriores.

Geomtricamente, el producto interno nos dice sobre la fuerza de x en la direccin de y.


Example 7.5
Por ejemplo, si k x k= 1, entonces

< x, y >=k y k cos ()

Figure 7.7: Grca de los dos vectores del ejemplo anterior.

Available for free at Connexions <http://cnx.org/content/col10373/1.2>


131

Las siguientes caractersticas son dadas por el producto interno:

< x, y > mide la longitud de la proyeccin de y sobre x.


< x, y > es el mximo (dadas k x k, k y k) donde x y y estan en la misma direccin ( ( = 0)
(cos () = 1)).
< x, y > es cero cuando (cos () = 0) ( = 90 ), es decir x y y son ortogonales.

7.3.1.2 Reglas del Producto Interno

En general el producto interno en un espacio vectorial complejo es solo una funcin (tomando dos vectores
y regresando un nmero complejo) que satisface ciertas condiciones:

Simetria Conjugada:

< x, y >= < (x, y) >
Escalado:
< x, y >= < (x, y) >
Aditividad:
< x + y, z >=< x, z > + < y, z >
"Positividad":
< x, x >> 0 , x 6= 0

Denition 7.3: Ortogonal


Decimos que x y y son ortogonales si:
< x, y >= 0

7.4 Espacios de Hilbert7


7.4.1 Espacios de Hilbert
Un espacio vectorial S con un producto interno (Section 7.3) vlido denido en l es llamado espacio de
producto interno, que tmabin es espacio lineal normado. Un espacio de Hilbert es un espacio de
producto interno que es completo con respecto a la norma denida usando el producto interno. Los espacios
8
de Hilbert fueron nombrados despus de que David Hilbert , convirtiera esta idea a travs de sus estudios
de ecuaciones integrales. Denimos nuestra norma utilizando el producto interno como:


k x k= < x, x > (7.3)

Los Espacios de Hilbert sern de ayuda para estudiar y generalizar los conceptos de la expansin de Fourier,
transformada de Fourier, y adems son muy importantes para el estudio de mecnica quntica. Los espacios
de Hilbert son estudiados en anlisis funcional una rama de las matemticas.
7 This content is available online at <http://cnx.org/content/m12879/1.2/>.
8 http://www-history.mcs.st-andrews.ac.uk/history/Mathematicians/Hilbert.html

Available for free at Connexions <http://cnx.org/content/col10373/1.2>


132 CHAPTER 7. ESPACIOS DE HILBERT Y EXPANSIONES ORTOGONALES

7.4.1.1 Ejemplos de Espacios de Hilbert


9
A continuacin mostraremos una lista de algunos ejemplos de espacios de Hilbert . Usted puede vericar
que son validos para estudios de producto interno.

Para Cn ,

x0
n1
  x1 X
T
< x, y >= y x = y0
y1
... yn1 = xi yi

.
.
.

i=0
xn1

Espacio de funciones de energa nita compleja: L2 (R)


Z

< f, g >= f (t) g (t) dt

Espacio de secuencias sumables cuadradas: `2 (Z)




X
< x, y >= x [i] y [i]
i=

7.5 Desigualdad de Cauchy-Schwarz10


7.5.1 Introduccin
Recordando que en R2 , < x, y >=k x kk y k cos ()

| < x, y > | k x kk y k (7.4)

La misma relacin se mantiene para espacios de producto interno (Section 7.3).

7.5.1.1 Desigualdad de Cauchy-Schwarz

Denition 7.4: Desigualdad de Cauchy-Schwarz


Para x, y en un espacio de producto interno

| < x, y > | k x kk y k

que mantiene la igualdad si y solo si x y y son linealmente dependientes (Section 5.1.1: Indepen-
dencia Lineal), es decir x = y para un escalar .

7.5.2 Detector de Filtro Acoplado


Tambin conocido como una de las aplicaciones de Cauchy-Schwarz.

9 "Hilbert Spaces" <http://cnx.org/content/m10434/latest/>


10 This content is available online at <http://cnx.org/content/m12880/1.2/>.

Available for free at Connexions <http://cnx.org/content/col10373/1.2>


133

7.5.2.1 Conceptos detrs de los Filtros Acoplados

Dados dos vectores, f y g , entonces la desigualdad de Cauchy-Schwarz CSI (Cauchy-Schwarz Inequality) es


maximizada cuando f = g . Esto nos dice que:

f es en la misma "direccin" como g.


Si f y g son funciones, f = g signica que f y g tienen la misma forma.

Por ejemplo, suponiendo que tenemos un conjunto de seales, denidas como {g1 (t) , g2 (t) , . . . , gk (t)}, y
que queremos ser capaces de decir cual, y si alguna de estas seales se asemeja a otra seal dada f (t).
note: Para poder encontrar las seal(es) que se asemeja a f (t) tomamos el producto interno. Si
gi (t) se asemeja a f (t), entonces el valor absoluto del producto interno, | < f (t) , gi (t) > |, ser
largo.

Esta idea de ser capaz de medir el rango de dos seales semejantes nos da un Detector de Filtro
Acoplado.

7.5.2.2 Comparando Seales

El simple uso de Filtro Acoplado ser tomar un conjunto de candidatos de seales, digamos que nuestro
conjunto de {g1 (t) , g2 (t) , . . . , gk (t)}, y tratar de acoplarla a nuestra plantilla de seal, f (t). Por ejemplo
digamos que tenemos las siguientes plantillas (Figure 7.8 (Seal Platilla)) y seales de candidatos (Figure 7.9
(Seales Candidatas)):

Seal Platilla

Figure 7.8: Nuestra seal de la queremos encontrar una semejante a esta.

Available for free at Connexions <http://cnx.org/content/col10373/1.2>


134 CHAPTER 7. ESPACIOS DE HILBERT Y EXPANSIONES ORTOGONALES

Seales Candidatas

(a) (b)

Figure 7.9: Claramente podemos ver cual de las seales nos dar una mejor semejanza a nuestra seal
de platilla.

Ahora si nuestra nica pregunta fuera cual de estas funciones se acerca mas f (t) entonces fcilmente
tenemos la respuesta basada en la inspeccin g2 (t). Sin embargo, este no siempre ser el caso. Tambin
querremos obtener un mtodo o algoritmo que pueda automatizar estas comparaciones. O tal vez queramos
tener un valor cuantitativo expresando que tan similares son las seales. Para tratar estas especicaciones,
presentaremos un mtodo mas formal para comparar seales, el cual, tal como se menciono anteriormente,
esta basado en el producto interno.
Para poder ver cuales de las seales candidatas, g1 (t) g2 (t), mejor se asemeja a f (t) % necesitamos
realizar los siguientes pasos:

Normalice gi (t)
Tome el producto interno con f (t)
Encuentre el ms grande

O ponerlo matemticamente
| < f, gi > |
Mejor candidato = argmax (7.5)
i k gi k

7.5.2.3 Encontrando un Patrn

Extendiendo estos pensamientos del Filtro Acoplado para encontrar semejanzas entre seales, podemos usar
la idea de buscar un patrn en una seal larga. La idea es simplemente realizar en varias ocasiones el
mismo clculo como lo hicimos anteriormente; sin embargo, ahora en lugar de calcular en diferentes seales,
simplemente realizamos el producto interno con una versin diferente cambiada por nuestra seal patrn.
Por ejemplo, digamos que tenemos las siguientes dos seales: una seal patrn (Figure 7.10 (Seal Patrn))
y una seal larga (Figure 7.11 (Seal Larga)).

Available for free at Connexions <http://cnx.org/content/col10373/1.2>


135

Seal Patrn

Figure 7.10: El patrn que estamos buscando en una nuestra seal larga teniendo una longitud T .

Seal Larga

Figure 7.11: Este es una seal larga que contiene un pieza que ensambla en nuestra seal patrn.

Aqu vemos dos seales cambiadas por nuestra seal patrn, cambiando las seales por s1 and s2 . Estas
dos posibilidades nos dan los siguientes clculos y resultados:

Cambio de s1 : R s1 +T
s1
g (t) f (t s1 ) dt
qR = largo (7.6)
s1 +T 2
s1
(|g (t) |) dt
Cambio de s2 : R s2 +T
s2
g (t) f (t s2 ) dt
qR = pequeo (7.7)
s2 +T 2
s2
(|g (t) |) dt
Por lo tanto podemos denir una ecuacin generalizada para nuestro ltro acoplado:

m (s) = filtro acoplado (7.8)

R s+T
s
g (t) f (t s) dt
m (s) = (7.9)
k g (t) k |L2 ([s,s+T ])
donde el numerado de la (7.9) es la convolucin de g (t)f (t). Ahora para poder decidir si o no el resultado
de nuestro detector de ltro acoplado es bastante grande para indicar una aceptacin correcta entre las dos
seales, denimos nivel de referencia. Si

m (s0 ) nivel de referencia

Available for free at Connexions <http://cnx.org/content/col10373/1.2>


136 CHAPTER 7. ESPACIOS DE HILBERT Y EXPANSIONES ORTOGONALES

despus podemos tener una semejanza en la locacin s0 .

7.5.2.4 Ejemplos Prcticos

7.5.2.4.1 Deteccin de Imagen

En 2-D, este concepto es usado para acoplar imgenes juntas, tal como vericar las huellas digitales para
seguridad o para acoplar fotos de alguien. Por ejemplo esta idea puede ser usada para los libros del tan
conocido dnde esta Waldo? si se nos da la siguiente platilla (Figure 7.12(a)) y una pieza del libro dnde
esta Waldo?,(Figure 7.12(b)),

(a) (b)

Figure 7.12: Ejemplo de la imagen de Dnde esta Waldo?". Nuestro detector de Filtro Acoplado
puede ser implementado para detectar una semejanza posible para Waldo.

Entonces fcilmente podemos crear un programa para encontrar la imagen que ms se asemeje a la imagen
de la cabeza de Waldo en la gura larga. Simplemente podemos implementar nuestro algoritmo de ltro
acoplado: tomar el producto interno de cada cambio y vase que tan larga es nuestra respuesta resultante.
11
Esta idea fue implementada en esta misma imagen para un Proyecto de Seales y Sistemas (vase esta
liga para saber mas) en la Universidad de Rice.

7.5.2.4.2 Sistemas de Comunicaciones


12
Los detectores de Filtro Acoplado son usualmente usados en Sistemas de Comunicacin . De echo, estos los
detectores mas ptimos para el ruido Gaussaniano. Las seales en la vida real tambin son distorsionadas
por el medio ambiente que las rodea, as que es una lucha constante para descubrir maneras capaces de
recibir seales torcidas y despus ser capaces de ltrarlas de alguna manera para determinar cual era la seal
original. Los ltros acoplados nos proveen una manera de comparar la seal recibida con dos posibles seales
(plantilla) y determinar cual es la que ms se asemeja a la seal recibida.

11 http://www.owlnet.rice.edu/elec301/Projects99/waldo/process.html
12 "Structure of Communication Systems" <http://cnx.org/content/m0002/latest/>

Available for free at Connexions <http://cnx.org/content/col10373/1.2>


137

13
Por ejemplo a continuacin tenemos un ejemplo simplicado de la Frecuencia Desplazada de Keying
(Frequency Shift Keying FSK) donde tenemos las siguientes condiciones para '1' y '0':

Figure 7.13: Frecuencia Desplazada de Keying para '1' y '0'.

Basados en la codicacin anterior, podemos crear una seal digital basada en 0's y 1's poniendo juntos
los dos cdigos anteriores en nmero innito de maneras. Para este ejemplo transmitiremos tres nmeros
bsicos de 3-bits: 101, desplegado en la siguiente Figure 7.14:

Figure 7.14: La corriente del bit "101" codicado con el FSK anterior.

Ahora la imagen anterior representa nuestra seal original que ser transmitida por algn sistema de
comunicacin, el cual inevitablemente pasa a travs del canal de comunicacin, la parte del sistema que
distorsionara y alterara nuestra seal. Mientras que nuestro ruido no sea muy grande, nuestro ltro acoplado
nos mantendr despreocupados de estos cambios de nuestra seal transmitida. Una vez que la seal ha sido
recibida, pasamos la seal del ruido a travs de un sistema simple, similar a la versin simplicada mostrada
a continuacin en la Figure 7.15:

13 "Frequency Shift Keying" <http://cnx.org/content/m0545/latest/>

Available for free at Connexions <http://cnx.org/content/col10373/1.2>


138 CHAPTER 7. ESPACIOS DE HILBERT Y EXPANSIONES ORTOGONALES

Figure 7.15: Diagrama de bloque del detector de ltro acoplado.

Figure 7.15 El diagrama anterior bsicamente muestra que nuestra seal con ruido ser pasada (asumire-
mos que pasara un bit a la vez) y que esta seal ser separada y pasada a travs de dos detectores de ltros
acoplados diferentes. Cada uno comparara la seal con ruido para cada uno de los cdigos que denimos
para `1' y `0'. Despus este valor ser pasado y cualquier valor que sea grande (es decir cualquier seal de
cdigo FSK a la seal ruidosa que mejor se asemeje) ser el valor que el recibidor tome. Por ejemplo, el
primer bit que ser enviado a travs, ser un `1' as que el nivel de arriba del diagrama de bloque ser el
valor ms alto, donde denotando que el `1' fue enviado por la seal, aunque la seal aparezca muy ruidosa
y distorsionada.

7.5.3 Demostracin de la Desigualdad de Cauchy-Schwarz


Veremos la demostracin de la desigualdad de Cauchy-Schwarz (Cauchy-Schwarz Inequality (CSI) )para un
espacio vectorial real .
Rule 7.1: Desigualdad de Cauchy-Schwarz para un espacio vectorial real
Para f Espacio de Hilbert S y g Espacio de Hilbert S, mostrar que:

| < f, g > | k f kk g k (7.10)

con la igualdad si y solo si g = f .


Proof:

Si g = f , mostrar | < f, g > | =k f kk g k


2
| < f, g > | = | < f, f > | = ||| < f, f > | = ||(k f k)

| < f, g > | =k f k (|| k f k) =k f kk g k


Esto verica nuestro argumento anterior de la desigualdad de CS
Si g 6= f , mostrar | < f, g > | <k f kk g k donde tenemos f + g 6= 0 , R
2
0 < (k f + g k) =< f + g, f + g >= 2 < (f, f ) > +2 < (f, g) > + < g, g >

2 2
2 (k f k) + 2 < (f, g) > +(k g k)

Available for free at Connexions <http://cnx.org/content/col10373/1.2>


139

Y obtenemos una cuadrtica en . Visualmente el polinomio cuadrtico en >0 para todo


. Tambin ntese que el polinomio no tiene races reales y que el discriminante es menor
que 0. . .
a 2 + b + c
tiene discriminante 2 4ac donde tenemos:

2
a = (k f k)

b = 2 < (f, g) >

2
c = (k g k)
Por lo tanto podemos colocar estos valores en el discriminante del polinomio de arriba para
obtener:
2 2 2
4(| < f, g > |) 4(k f k) (k g k) < 0 (7.11)

| < f, g > | <k f kk g k (7.12)

Y nalmente hemos probado la formula de la desigualdad de desigualdad de Cauchy-Schwarz


para un espacio vectorial real.

note: Qu cambios tenemos que hacer para hacer la demostracin para un espacio vectorial
complejo? (la respuesta se la dejamos al lector)

7.6 Espacios de Hilbert comunes14


7.6.1 Espacios de Hilbert comunes
A continuacin veremos los cuatro Espacios de Hilbert (Section 7.3) mas comunes con los que usted tendr
que tratar para la discusin y manipulacin de seales y sistemas:

7.6.1.1

Rn (escalares reales ) y
Cn (escalares complejos), tambin llamado `2 ([0, n 1])
x0

x
1
x = Es una lista de nmeros (secuencia nita). El producto interno (Section 7.3) para


...

xn1
nuestros dos espacios son las siguientes:

Producto interno Rn :
< x, y > = yT x
Pn1 (7.13)
= i=0 xi yi
Producto interno Cn :

< x, y > = yT x
Pn1 (7.14)
= i=0 xi yi
14 This content is available online at <http://cnx.org/content/m12881/1.2/>.

Available for free at Connexions <http://cnx.org/content/col10373/1.2>


140 CHAPTER 7. ESPACIOS DE HILBERT Y EXPANSIONES ORTOGONALES

Modelo para: Seal de tiempo discreto en el intervalo


[0, n 1] o Seal Peridica (con periodo n) de

x0

x
1
tiempo discreto.


...

xn1

Figure 7.16

7.6.1.2

f L2 ([a, b]) es una funcin de energa nita en [a, b]


Inner Product Z b

< f, g >= f (t) g (t) dt (7.15)
a
Modelo para: Seal de tiempo continuo en el intervalo [a, b] o Seal Peridica (con periodo T = b a) de
tiempo continuo

7.6.1.3

x `2 (Z) es una secuencia innita de nmeros que son cuadrados sumables


Producto interno


X
< x, y >= x [i] y [i] (7.16)
i=

Modelo para: Seal no-peridica de tiempo discreto

7.6.1.4

f L2 (R) es una una funcin de energa nita en todo R.


Producto interno Z

< f, g >= f (t) g (t) dt (7.17)

Modelo para: Seal no-peridica de tiempo continuo

Available for free at Connexions <http://cnx.org/content/col10373/1.2>


141

7.6.2 Anlisis de Fourier Asociado


Cada uno de estos cuatro espacios de Hilbert tiene un anlisis de Fourier asociado con el.

L2 ([a, b]) Series de Fourier


`2 ([0, n 1]) Transformada Discreta de Fourier
L2 (R) Transformada de Fourier
`2 (Z) Transformada Discreta de Fourier en Tiempo

Pero los cuatros estn basados en el mismo principio (Espacio de Hilbert).

note: no todos los espacios normalizados son espacios de Hilbert

L1 (R), k f k1 =
R
Por ejemplo: |f (t) |dt. trate como usted pueda, de encontrar el producto interno que
induce esta norma, es decir...tal que: < , > such that
R 2
2
< f, f > = (|f (t) |) dt
(7.18)
2
= (k f k1 )

De echo, para todo el espacio Lp (R) , L2 (R) es el nico que es un espacio de Hilbert.

Available for free at Connexions <http://cnx.org/content/col10373/1.2>


142 CHAPTER 7. ESPACIOS DE HILBERT Y EXPANSIONES ORTOGONALES

Figure 7.17

Los espacios de Hilbert son en gran medida los ms agradables, si se usa o estudia la expansin de bases
ortonormales (Section 7.8) entonces usted empezara a ver por que esto es cierto.

7.7 Tipos de Bases15


7.7.1 Base Normada
Denition 7.5: Base Normada
una base (Section 5.1.3: Bases) {bi } donde cada bi tiene una norma unitaria

k bi k= 1 , i Z (7.19)

15 This content is available online at <http://cnx.org/content/m12902/1.2/>.

Available for free at Connexions <http://cnx.org/content/col10373/1.2>


143

note: El concepto de bases se aplica a todos los espacios vectoriales (Section 7.1). El concepto
de base normada se aplica solo a espacios normados (Section 7.2).

Tambin usted puede normalizar una base: solo multiplique cada vector de la base por una constante, tal
1
que
kbi k

Example 7.6
Dada la siguiente base:
1 1
{b0 , b1 } = ,
1 1

Normalizado con la norma `2 :


1 1
b0 =
2 1

1 1
b1 =
2 1

Normalizado con la norma `1 :


1 1
b0 =
2 1

1 1
b1 =
2 1

7.7.2 Base Ortogonal


Denition 7.6: Base Ortogonal
una base {bi } en donde los elementos son mutuamente ortogonales

< bi , bj >= 0 , i 6= j

note: El concepto de base ortogonal se aplica solo a los Espacios de Hilbert (Section 7.4.1:
Espacios de Hilbert).

Example 7.7
Base cannica para R2 , tambin referida como `2 ([0, 1]):

1
b0 =
0

0
b1 =
1
1
X
< b0 , b1 >= b0 [i] b1 [i] = 1 0 + 0 1 = 0
i=0

Available for free at Connexions <http://cnx.org/content/col10373/1.2>


144 CHAPTER 7. ESPACIOS DE HILBERT Y EXPANSIONES ORTOGONALES

Example 7.8
Ahora tenemos la siguiente base y relacin:

1 1
, = {h0 , h1 }
1 1

< h0 , h1 >= 1 1 + 1 1 = 0

7.7.3 Base Ortonormal


Colocando las dos secciones (deniciones) anteriores juntas, llegamos al tipo de base ms importante y til:

Denition 7.7: Base Ortonormal


Una base que es normalizada y ortogonal

k bi k= 1 , i Z

< bi , bj > , i 6= j

note: podemos acortar los dos argumentos en uno solo:

< bi , bj >= ij

donde
1 if i=j
ij =
0 if i 6= j
16
Donde ij re reere a la funcin delta Kronecker que tambin es escrita como [i j].

Example 7.9: Ejemplo de Base Ortonormal #1



1 0
{b0 , b2 } = ,
0 1

Example 7.10: Ejemplo de Base Ortonormal #2



1 1
{b0 , b2 } = ,
1 1

Example 7.11: Ejemplo de Base Ortonormal #3



1 1 1 1
{b0 , b2 } = ,
2 1 2 1

16 "Photo Story 3 Tech Module" <http://cnx.org/content/m18124/latest/>

Available for free at Connexions <http://cnx.org/content/col10373/1.2>


145

7.7.3.1 La belleza de las Bases Ortonormales

Trabajar con las bases Ortonormales es sencillo.Si {bi } es una base ortonormal, podemos escribir para
cualquier x
X
x= i bi (7.20)
i

Es fcil encontrar los i :


P
< x, bi > = < k k bk , bi >
P (7.21)
= k k < (bk , bi ) >

En donde en la ecuacin anterior podemos usar el conocimiento de la funcin delta para reducir la ecuacin:

1 if i=k
< bk , bi >= ik =
0 if i 6= k

< x, bi >= i (7.22)

Por lo tanto podemos concluir con la siguiente ecuacin importante para x:


X
x= < (x, bi ) > bi (7.23)
i

Los i 's son fciles de calcular (sin interaccin entres los bi 's)
Example 7.12
Dada la siguiente base:
1 1 1 1
{b0 , b1 } = ,
2 1 2 1

3
representa x=
2
Example 7.13: Serie de Fourier Levemente Modicada
Dada la base  
1 j0 nt
e |n=
T
2
en L2 ([0, T ]) donde T = 0 .


X 1
< f, ej0 nt > ej0 nt

f (t) =
n= T

Donde podemos calcular el producto interior de arriba en L2 como

Z T Z T
1 1
< f, ej0 nt >= f (t) ej0 nt dt = f (t) e(j0 nt) dt
T 0 T 0

Available for free at Connexions <http://cnx.org/content/col10373/1.2>


146 CHAPTER 7. ESPACIOS DE HILBERT Y EXPANSIONES ORTOGONALES

7.7.3.2 7.7.3.2 Expansin de una Base Ortonormal en un Espacio Hilbert

Sea {bi } una base ortonormal para un espacio de Hilbert H. Entonces, para cualquier x H podemos
escribir
X
x= i bi (7.24)
i

donde i =< x, bi >.


Anlisis: descomponer x en trminos de bi

i =< x, bi > (7.25)

"Sntesis": construir x de una combinacin de las bi


X
x= i bi (7.26)
i

7.8 Expansin de Bases Ortonormales17


7.8.1 Idea Principal
Cuando trabajamos con seales muchas veces es til romper la seal en pequeas, partes mas manejables.
Por suerte en este momento usted ya ha sido expuesto al concepto de eigenvectores (Section 5.3) y su uso en
la descomposicin de una seal en una de sus posibles bases. Haciendo esto, somos capaces de simplicar el
nuestro clculo de seales y sistemas a travs de las eigenfunciones de los sistemas LTI (Section 5.6).
Ahora veremos una forma alternativa de representar las seales, a travs del uso de una base ortonormal.
Podemos pensar en una base ortonormal como un conjunto de bloques construidos que utilizamos para
construir funciones. Construiremos la seal/ vector como una suma cargada de elementos base.

Example 7.14

1
La funcin senoidal compleja
T
ej0 nt para todo < n < forma una base ortonormal
2
para L ([0, T ]). P j0 nt
En nuestras series de Fourier series (Section 6.2) la ecuacin, f (t) = n= cn e ,el {cn }
es solo otra representacin de f (t).

note: Para la seal/ vector en un Espacio de Hilbert (Section 7.4.1: Espacios de Hilbert), la
expansin de coecientes es fcil de encontrar.

7.8.2 Representacin Alternativa


Recordando nuestra denicin de base: Un conjunto de vectores {bi } en un espacio vectorial S es una base
si

1. Las bi son linealmente independientes.


2. Los bi que generan (Section 5.1.2: Subespacio Generado) S . Esto es, podemos encontrar {i }, donde
i C (escalares) tal que
X
x= i bi , x S (7.27)
i

donde x es un vector en S, es un escalar en C, y b es un vector en S.


17 This content is available online at <http://cnx.org/content/m12928/1.1/>.

Available for free at Connexions <http://cnx.org/content/col10373/1.2>


147

La condicin 2 en la denicin anterior dice que podemos descomponer cualquier vector en trminos
de la {bi }. Condicin 1 asegura que la descomposicin es nica.

note: {i } provee una representacin alternativa de x.


Example 7.15
Veamos un simple ejemplo en R2 , donde tenemos el siguiente vector:

1
x=
2
n o
T T
Base Cannica: {e0 , e1 } = (1, 0) , (0, 1)

x = e0 + 2e1
n o
T T
Base Alternativa: {h0 , h1 } = (1, 1) , (1, 1)

3 1
x= h0 + h1
2 2

En general, dada una base {b0 , b1 } y un vector x R2 , como encontramos 0 y 1 tal que

x = 0 b0 + 1 b1 (7.28)

7.8.3 Encontrando los Alfas


Ahora tratemos con la pregunta que se present arriba sobre encontrar los i 's en genral para R2 . Empezamos
reescribiendo la (7.28) as que podemos apilar nuestras bi 's como columnas en una matriz de 22.
     
x = 0 b0 + 1 b1 (7.29)

. .

. .
. .

  0
x = b0 b1 (7.30)


. .
1
. .
. .

Example 7.16
Este es un ejemplo sencillo, que muestra pequeos detalles de la ecuacin anterior.

x [0] b0 [0] b1 [0]
= 0 + 1
x [1] b0 [1] b1 [1]
(7.31)
0 b0 [0] + 1 b1 [0]
=
0 b0 [1] + 1 b1 [1]

x [0] b0 [0] b1 [0] 0
= (7.32)
x [1] b0 [1] b1 [1] 1

Available for free at Connexions <http://cnx.org/content/col10373/1.2>


148 CHAPTER 7. ESPACIOS DE HILBERT Y EXPANSIONES ORTOGONALES

7.8.3.1 Simplicando nuestra Ecuacin

Para hacer una notacin simple, denimos los siguientes dos conceptos de la ecuacin anterior:

Matriz de la Base:
. .

. .
. .

B = b0 b1


. .
. .
. .

Vector de Coecientes:
0
=
1

Lo que nos da la siguiente ecuacin:


x = B (7.33)
P1
que es mas equivalente a x= i=0 i bi .
Example 7.17
1 0
Dada la base cannica, , , entonces tenemos la siguiente matriz de la base:
0 1

0 1
B=
1 0

Para obtener las i 's, resolvemos para el vector de coecientes en la (7.33)

= B 1 x (7.34)

Donde B 1 es la matriz inversa matrix


18
de B.

7.8.3.2 Ejemplos

Example 7.18
Veamos primero la base cannica y tratemos de calcular calculate de ahi.


1 0
B= =I
0 1

Donde I es la matriz identidad. Para poder resolver para enocontremos primero la inversa de
B (la cual es realmente trivial en este caso):


1 0
B 1 =
0 1

Por lo tanto obtenemos,


= B 1 x = x

18 "Matrix Inversion" <http://cnx.org/content/m2113/latest/>

Available for free at Connexions <http://cnx.org/content/col10373/1.2>


149

Example 7.19
1 1
Ahora veamos una base un poco mas complicada de , = {h0 , h1 } Enotnces
1 1
nuestra base y nuestra inversa de la matriz de la base se convierte en:

1 1
B=
1 1

1 1
B 1 = 2 2
1 1
2 2

y para este ejemplo esto nos da que



3
x=
2
Ahora resolvemos para

1 1
3 2.5
= B 1 x = 2 2 =
1 1
2 2 2 0.5

y obtenemos
x = 2.5h0 + 0.5h1

Exercise 7.8.1 (Solution on p. 165.)


Ahora dada la siguiente matriz de la base y x:

1 3
{b0 , b1 } = ,
2 0

3
x=
2
Para este probelma haga un bosquejo de las bases y despus represente x en trminos de b0 y b1 .

note: Un cambio de base simplemente se ve como x desde una "perspectiva diferente." B 1


transforms x de la base cannica a nuestra nueva base, {b0 , b1 }. Ntese que es un proceso total-
mente mecnico.

7.8.4 Extendiendo la Dimensin y el Espacio


Podemos extender estas ideas ms alla de R2 y verlas en Rn y Cn . Este procedimiento se extendiende
n
naturalmente a dimensiones mas grandes (> 2) . Dada la base {b0 , b1 , . . . , bn1 } para R , queremos encontrar
{0 , 1 , . . . , n1 } tal que
x = 0 b0 + 1 b1 + + n1 bn1 (7.35)

Otra vez, coloque la matriz de la base


 
B= b0 b1 b2 ... bn1

Available for free at Connexions <http://cnx.org/content/col10373/1.2>


150 CHAPTER 7. ESPACIOS DE HILBERT Y EXPANSIONES ORTOGONALES

donde las columnas iguales a los vectores de la base y que siempre ser una matriz de nn (mientras que la
matriz anterior no aparezca al cuadrado ya que dejamos trminos en notacin vectorial ). Podemos proceder
a reescribir la (7.33)

0

.

x= b0 b1 ... bn1 . = B

.

n1
y
= B 1 x

7.9 Espacio de Funciones19


20
Tambin podemos encontrar basis vectores base para espacios vectoriales (Section 7.1) con exepcin de
n
R .
Sea Pn un espacio vectorial de orden polinomial n-esimo en (-1, 1) con coecientes reales (vericar que
P2 es un espacio vectorial en casa).

Example 7.20
P2 = {todos los polinomios cuadrticos}. Sea b0 (t) = 1, b1 (t) = t, b2 (t) = t2 .
{b0 (t) , b1 (t) , b2 (t)} genera P2 , es decir puede escribir cualquier f (t) P2 como

f (t) = 0 b0 (t) + 1 b1 (t) + 2 b2 (t)

para algn i R.
note: P2 es de dimensin 3.

f (t) = t2 3t 4
Base Alternativa  
1
3t2 1

{b0 (t) , b1 (t) , b2 (t)} = 1, t,
2
escribir f (t) en trminos de la nueva base d0 (t) = b0 (t), d1 (t) = b1 (t), d2 (t) = 32 b2 (t) 21 b0 (t).

f (t) = t2 3t 4 = 4b0 (t) 3b1 (t) + b2 (t)

 
3 1
f (t) = 0 d0 (t) + 1 d1 (t) + 2 d2 (t) = 0 b0 (t) + 1 b1 (t) + 2 b2 (t) b0 (t)
2 2
3
f (t) = 0 b0 (t) + 1 b1 (t) + 2 b2 (t)
2
por lo tanto
1
0 =4
2

1 = 3

3
2 = 1
2
19 This content is available online at <http://cnx.org/content/m12913/1.1/>.
20 "Orthonormal Basis Expansions" <http://cnx.org/content/m10760/latest/>

Available for free at Connexions <http://cnx.org/content/col10373/1.2>


151

enotnces obtenemos
2
f (t) = 4.5d0 (t) 3d1 (t) + d2 (t)
3
Example 7.21
ej0 nt | L2 ([0, T ]), T = 2
Cn ej0 nt .
P
n= es una base para
0 , f (t) = n
Calculamos la expansin de coecientes con

la formula de "cambio de base"


Z T
1  
Cn = f (t) e(j0 nt) dt (7.36)
T 0

note: Hay un nmero innito de elementos en un conjuto de base, que signican que L2 ([0, T ]) es
de dimensin innita.

Espacios de dimensin-innita Innite-dimensional son difciles de visualizar. Podemos tomar mano


de la intuicin para reconocer que comparten varias de las propiedades con los espacios de dimensin
nita. Muchos conceptos aplicados a ambos (como"expansin de base").Otros no (cambio de base
no es una bonita formula de matriz).

7.10 Base de la Ondoleta de Haar21


7.10.1 Introduccin
Las series de Fourier (Section 6.2) es una til representacin ortonormal
22
en L2 ([0, T ]) especialmente para
entradas en sistemas LTI. Sin embargo es til para algunas aplicaciones, es decir, procesamiento de imagenes
23
(recordando el fenomeno de Gibb ).
Las ondoletas, descubiertas en los pasados 15 aos, son otro tipos de base para L2 ([0, T ]) y tiene varias
propiedades.

7.10.2 Comparacin de Base


Las series de Fourier - cn dan informacin frecuente. Las funciones de la base duran todo el intervalo entero.

21 This content is available online at <http://cnx.org/content/m12919/1.2/>.


22 "Orthonormal Basis Expansions" <http://cnx.org/content/m10760/latest/>
23 "Gibbs Phenomena" <http://cnx.org/content/m10092/latest/>

Available for free at Connexions <http://cnx.org/content/col10373/1.2>


152 CHAPTER 7. ESPACIOS DE HILBERT Y EXPANSIONES ORTOGONALES

Figure 7.18: Funciones de la base de Fourier

Ondoletas - las funciones de la base con frecuencia nos dan informacin pero es local en el tiempo.

Available for free at Connexions <http://cnx.org/content/col10373/1.2>


153

Figure 7.19: Funciones de la Base de la Ondoleta

En la base de Fourier, las funciones de la base son armnicas multiples de ej0 t

n o
Figure 7.20: base = 1
T
ej0 nt

24
En la base de la ondoleta de Haar , las funciones de la base son escaladas y trasladadas de la version
de la "ondoleta madre" (t).
24 "The Haar System as an Example of DWT" <http://cnx.org/content/m10437/latest/>

Available for free at Connexions <http://cnx.org/content/col10373/1.2>


154 CHAPTER 7. ESPACIOS DE HILBERT Y EXPANSIONES ORTOGONALES

Figure 7.21

Funciones base {j,k (t)} se les pone un ndice por un escalar j y un desplazamiento k.
n j  j  o
Sea (t) = 1 , 0 t < T Entonces (t) , 2 2 t k , (t) , 2 2 2j t k | j Z and
2 j
k = 0, 1, 2, . . . , 2j 1

Available for free at Connexions <http://cnx.org/content/col10373/1.2>


155

Figure 7.22


1 if 0 t < T
2
(t) = (7.37)
1 if 0 T < T
2

Available for free at Connexions <http://cnx.org/content/col10373/1.2>


156 CHAPTER 7. ESPACIOS DE HILBERT Y EXPANSIONES ORTOGONALES

Figure 7.23

j 
Sea j,k (t) = 2 2 2j t k

Figure 7.24

Ms grande j "delgado" la funcin de la base , j = {0, 1, 2, . . . }, 2j cambia a cada escala: k =


j
0, 1, . . . , 2 1
Checar: cada j,k (t) tiene energia unitaria

Available for free at Connexions <http://cnx.org/content/col10373/1.2>


157

Figure 7.25

Z 
j,k 2 (t) dt = 1 k j,k (t) k2 = 1

(7.38)

Cualesquiera dos funciones de la base son ortogonales.

(a) (b)

Figure 7.26: Integral del producto = 0 (a) Misma escala (b) Diferente escala

Available for free at Connexions <http://cnx.org/content/col10373/1.2>


158 CHAPTER 7. ESPACIOS DE HILBERT Y EXPANSIONES ORTOGONALES

Tambin, {j,k , } generan L2 ([0, T ])

7.10.3 Transformada de la Ondoleta de Haar


Usando lo que conocemos sobre espacios de Hilbert
25
: Para cualquier f (t) L2 ([0, T ]), podemos escribir

Sintesis
XX
f (t) = wj,k j,k (t) + c0 (t) (7.39)
j k

Anlisis Z T
wj,k = f (t) j,k (t) dt (7.40)
0
Z T
c0 = f (t) (t) dt (7.41)
0

note: los wj,k son reales

La transformacin de Haar es muy til especialemte en compresin de imagenes.

Example 7.22
Esta demostracin nos permite crear una seal por combinacin de sus funciones de la base de
Haar, ilustrando la ecuacin de sistesis de la ecuacin de la Transformada de la Ondoleta de Haar.
26
Vemos aqu para las instrucciones de como usar el demo.

This is an unsupported media type. To view, please see


http://cnx.org/content/m12919/latest/HaarSynth.llb

7.11 Bases Ortonormales en Espacios Reales y Complejos27


7.11.1 Notacin
El operador de la Transpuesta AT voltea la matriz a travs de su diagonal.


a1,1 a1,2
A=
a2,1 a2,2

a1,1 a2,1
AT =
a1,2 a2,2

La columna i de A es una la i de AT


25 "Inner Products" <http://cnx.org/content/m10755/latest/>
26 "How to use the LabVIEW demos" <http://cnx.org/content/m11550/latest/>
27 This content is available online at <http://cnx.org/content/m12920/1.2/>.

Available for free at Connexions <http://cnx.org/content/col10373/1.2>


159

Recordando que el, producto interno (Section 7.3)


x0

x1
x=

.
.
.


xn1

y0

y1
y=

.
.
.


yn1

y0

  y1 X
T
x y= x0 x1 ... xn1 = xi yi =< y, x >

.
.
.

i

yn1
en Rn
Transpuesta Hermitiana AH , transpuesta y conjugada


AH = AT

X
< y, x >= xH y = xi yi
i
n
en C
Sea {b0 , b1 , . . . , bn1 } una base ortonormal (Section 7.7.3: Base Ortonormal) para Cn

i = {0, 1, . . . , n 1} < bi , bi >= 1 ,

 
i 6= j, < bi , bj >= bj H bi = 0

Matriz de la base:
. . .

. . .
. . .

B = b0 b1 ... bn1


. . .
. . .
. . .

Ahora,


... b0 H ...
. . .
b0 H b0 b0 H b1 ... b0 H bn1
. . .
. . .
b1 H b1 H b0 b1 H b1 b1 H bn1

... ... ...
BH B = b0 b1 ... bn1 =

. .
. .
. .

. . .
. . .

. . .
... bn1 H ... bn1 H b0 bn1 H b1 ... bn1 H bn1

Available for free at Connexions <http://cnx.org/content/col10373/1.2>


160 CHAPTER 7. ESPACIOS DE HILBERT Y EXPANSIONES ORTOGONALES

Para una base ortonormal con una matriz de la base B

B H = B 1

( B T = B 1 in Rn ) BH es fcil calcular mientras que B 1 es difcil de calcular.


As que, para encontrar {0 , 1 , . . . , n1 } tal que

X
x= i bi
i

Calcular
= B 1 x = B H x
 

usando una base ortonormal nos libramos de la operacin inversa.

7.12 Teoremas de Plancharel y Parseval28


7.12.1 Teorema de Plancharel
Theorem 7.1: Teorema de Plancharel
El producto interno de dos vectores/seales es el mismo que en `2 el producto interno de su
expansin de coecientes.
Sea {bi } una base ortonormal para un Espacio de Hilbert H. x H, y H
X
x= i bi
i

X
y= i bi
i
entonces
X
< x, y >H = i i
i

Example
Aplicando las Series de Fourier, podemos ir de f (t) a {cn } y de g (t) a {dn }
Z T

X
f (t) g (t) dt = cn dn
0 n=

el producto interno en el dominio-tiempo = producto interno de los coeentes de Fourier.

Proof:

X
x= i bi
i
X
y= j bj
j

X X X X X X X
< x, y >H =< i bi , j bj >= i < bi , j bj >= i j < (bi , bj ) >= i i
i j i j i j i

usando las reglas del producto interno (p. 131).

28 This content is available online at <http://cnx.org/content/m12931/1.1/>.

Available for free at Connexions <http://cnx.org/content/col10373/1.2>


161

note: < bi , bj >= 0 cuando i 6= j y < bi , bj >= 1 cuando i=j

Si el espacio de Hillbert H tiene un ONB, los productos internos son equivalentes a los productos
internos en `2 .
Todo H con ONB son de alguna manera equivalente a `2 .
note: las secuencias de cuadrados sumables son importantes.

7.12.2 Teorema de Parseval


Theorem 7.2: Teorema de Parseval
La energa de una seal = suma de los cuadrados de su expansin de coecientes.
Sea x H , {bi } ONB

X
x= i bi
i

Entonces
2
X 2
(k x kH ) = (|i |)
i

Proof:
Directamente de Plancharel

2
X X 2
(k x kH ) = < x, x >H = i i = (|i |)
i i

Example
1
Series de Fourier
T
ejw0 nt
1 X 1
f (t) = cn ejw0 nt
T n T
Z T
X
2 2
(|f (t) |) dt = (|cn |)
0 n=

7.13 Approximacin y Proyeccin en el Espacio de Hilbert29


7.13.1 Introduccin
Dada una linea 'l' y un punto 'p' en el plano, Cul es el punto ms cercano 'm' a 'p' en 'l' ?

29 This content is available online at <http://cnx.org/content/m12935/1.2/>.

Available for free at Connexions <http://cnx.org/content/col10373/1.2>


162 CHAPTER 7. ESPACIOS DE HILBERT Y EXPANSIONES ORTOGONALES

Figure 7.27: Figura del punto 'p' y la linea 'l' mencionadas.

Mismo problema: Sea x y v vectores en R2 . Digamos k v k= 1. Para qu valor de es k x v k 2


minimizado? (qu punto en el espacio generado{v} mejor se aproxima a x?)

Figure 7.28

^
La condicin es que x v y v sean ortogonales.

7.13.2 Calculando
^
Cmo calcular ?
^
Sabemos que ( x v ) es perpendicular para todo vector en el espacio generado {v}, as que

^
< x v, v >= 0 , ()

^
< (x, v) > < (v, v) >= 0
por que < v, v >= 1, por lo tanto

   
^ ^
< (x, v) > = 0 =< x, v >

El vector ms cercano en el espacio generado{v} = < (x, v) > v , donde < (x, v) > v es la proyeccin de x
sobre v.
Punto a un plano?

Available for free at Connexions <http://cnx.org/content/col10373/1.2>


163

Figure 7.29

Podemos hacer lo mismo pero en dimensiones ms grandes.

Exercise 7.13.1 (Solution on p. 165.)


Sea V H un subespacio de un espacio de Hilbert (Section 7.4) H. Sea x H dado. Encontrar
y V que mejor se aproxime x. es decir, k x y k esta minimizada.
Example 7.23



1 0
a

x R3 , 0 , 1 , x = b .
V = espacio generado Por lo tanto,

0 0 c


2 1 0 a
X
y= < (x, bi ) > bi = a
0 + b 1 = b

i=1
0 0 0

Example 7.24
V = {espacio de las seales peridicas con frecuancia no mayor que 3w0 }. Dada f(t) peridica,
Cal es la seal en V que mejor se aproxima a f ?

1
1. {
T
ejw0 kt , k = -3, -2, ..., 2, 3} es una ONB para V
P3
g (t) = T1 k=3 < f (t) , ejw0 kt > ejw0 kt es la seal

2. ms cercana en V para f(t) recon-
struya f(t) usando solamente 7 trminos de su serie de Fourier (Section 6.2).

Example 7.25
Sea V = { funciones constantes por trozos entre los nmeros enteros}

1. ONB para V.


1 if i1t<i
bi =
0 otherwise

donde {bi } es una ONB.


La mejor aproximacin constante por trozos?


X
g (t) = < (f, bi ) > bi
i=

Z Z i
< f, bi >= f (t) bi (t) dt = f (t) dt
i1

Available for free at Connexions <http://cnx.org/content/col10373/1.2>


164 CHAPTER 7. ESPACIOS DE HILBERT Y EXPANSIONES ORTOGONALES

Example 7.26
Esta demostracin explora la aproximacin usando una base de Fourier y una base de las ondoletas
30
de Haar. Vase aqui para las instrucciones de como usar el demo.

This is an unsupported media type. To view, please see


http://cnx.org/content/m12935/latest/Approximation.llb

30 "How to use the LabVIEW demos" <http://cnx.org/content/m11550/latest/>

Available for free at Connexions <http://cnx.org/content/col10373/1.2>


165

Solutions to Exercises in Chapter 7


Solution to Exercise 7.8.1 (p. 149)
Para poder representar x en trminos de b0 y b1 seguimos los mismos pasos usados en los ejemplos anteri-
ores.
1 2
B=
3 0

1
0
B 1 = 2
1 1
3 6

1
= B 1 x =
2
3

Y ahora podemos escribir x en trminos de b0 y b1 .


2
x = b0 + b1
3
Y facilmente podemos sustituir nuestros valores conocidos de b0 y b1 para vericar nuestros resultados.
Solution to Exercise 7.13.1 (p. 163)

1. Encontrar una base ortonormal (Section 7.7.3: Base Ortonormal) {b1 , . . . , bk } para V
2. Proyectar x sobre V usando
k
X
y= < (x, bi ) > bi
i=1

despus y es el punto ms cercano en V a x y (x-y) V ( < x y, v >= 0 , (v) V

Available for free at Connexions <http://cnx.org/content/col10373/1.2>


166 CHAPTER 7. ESPACIOS DE HILBERT Y EXPANSIONES ORTOGONALES

Available for free at Connexions <http://cnx.org/content/col10373/1.2>


Chapter 8

Transformada Discreta de Fourier

8.1 Anlisis de Fourier1


El anlisis de Fourier es elemental para entender el comportamiento de las seales de sistemas. Este es el
resultado de que los senosoidales son eigenfunciones (Section 5.6) de sistemas lineales variantes en el tiempo
(LTI) (Section 2.1). Si pasamos cualquier senosoidal a travs de un sistema LTI, obtenemos la versin
escalada de cualquier sistema senosoidal como salida. Entonces, ya que el anlisis de Fourier nos permite
redenir las seales en terminos de senosoidales, todo lo que tenemos que hacer es determinar el efecto que
2
cualquier sistema tiene en todos los senosoidales posibles (su funcin de transferencia ) as tendremos un
entendimiento completo del sistema. As mismo, ya que podemos denir el paso de los senosoidales en el
sistema como la multiplicacin de ese senosoidal por la funcin de transferencia en la misma frecuencia,
puedes convertir el paso de la seal a travs de cualquier sistema de ser una convolucin (Section 3.3) (en
tiempo) a una multiplicacin (en frecuencia) estas ideas son lo que dan el poder al anlisis de Fourier.
Ahora, despus de haberle vendido el valor que tiene este mtodo de anlisis, nosotros debemos analizar
exactamente lo que signica el anlisis Fourier. Las cuatro transformadas de Fourier que forman parte
3 4
de este anlisis son: Series Fourier , Transformada de Fourier continua en el tiempo , Transformada de
Fourier en Tiempo Discreto (Section 11.4), y La Transformada de Fourier Discreta (Section 11.2). Para
este modulo, nosotros veremos la trasformada de Laplace (Section 14.1) y la transformada Z (Section 14.3).
Como extensiones de CTFT y DTFT respectivamente. Todas estas transformadas actan esencialmente de
la misma manera, al convertir una seal en tiempo en su seal equivalente en frecuencia (senosoidales). Sin
embargo, dependiendo en la naturaleza de una seal especica (por ejemplo, si es de tamao nito o innito,
o si son discretas o continuas en el tiempo) hay una transformada apropiada para convertir las seales en su
dominio de frecuencia. La siguiente tabla muestra las cuatro transformadas de Fourier y el uso de cada una.
Tambin incluye la convolucion relevante para el espacio especicado.

Tabla de Representaciones para Fourier

Transformada Dominio del Tiempo Dominio de la Fre- Convolucin


cuencia

continued on next page

1 This content is available online at <http://cnx.org/content/m12865/1.4/>.


2 "Transfer Functions" <http://cnx.org/content/m0028/latest/>
3 "Continuous-Time Fourier Series (CTFS)" <http://cnx.org/content/m10097/latest/>
4 "Continuous Time Fourier Transform (CTFT)" <http://cnx.org/content/m10098/latest/>

Available for free at Connexions <http://cnx.org/content/col10373/1.2>

167
168 CHAPTER 8. TRANSFORMADA DISCRETA DE FOURIER

Serie de Fourier Con- L2 ([0, T )) l2 (Z) Tiempo Continuo Cir-


tinua en el Tiempo cular

Transformada de L2 (R) L2 (R) Tiempo Continuo Lineal


Fourier en Tiempo
Continuo

Transformada de l2 (Z) L2 ([0, 2)) Tiempo Discreto Lineal


Fourier Discreta en el
Tiempo

Transformada de l2 ([0, N 1]) l2 ([0, N 1]) Tiempo Discreto Circu-


Fourier Discreta lar

Table 8.1

8.2 Anlisis de Fourier en Espacios Complejos5


8.2.1 Introduccin
6
Para este momento usted debera estar familiarizado con la derivacin de la series de Fourier de tiempo
7
continuo, funciones peridicas . Esta derivacin nos lleva a las siguientes ecuaciones las cuales usted debera
conocer:
X
f (t) = cn ej0 nt (8.1)
nn

1
f (t) e(j0 nt) dt
R
cn = T
(8.2)
1

= T < f, ej0 nt >
donde cn nos dice la cantidad de frecuencia en 0 n in f (t).
En este mdulo derivaremos una expansin similar para funciones peridicas y discretas en el tiempo. Al
hacerlo, nosotros derivaremos las series de Fourier discretas en el tiempo (DTFS), tambin conocidas
como trasformadas discretas de Fourier (Section 11.3)(DFT).

8.2.2 Derivacin del DTFS


As como en la funcin peridica continua en el tiempo puede ser vista como una funcin en el intervalo
[0, T ]
5 This content is available online at <http://cnx.org/content/m12848/1.5/>.
6 "Fourier Series: Eigenfunction Approach" <http://cnx.org/content/m10496/latest/>
7 "Continuous Time Periodic Signals" <http://cnx.org/content/m10744/latest/>

Available for free at Connexions <http://cnx.org/content/col10373/1.2>


169

(a) (b)

Figure 8.1: Solo consideraremos un intervalo para la funcin peridica en esta seccin. (a) Funcin
peridica (b) Funcin en el intervalo [0, T ]

Una seal peridica discreta en el tiempo (con periodo N) se puede ver como un conjunto de nmeros
nitos. Por ejemplo, digamos que tenemos el siguiente conjunto de nmeros que describe una seal discreta,
donde N = 4:
{. . . , 3, 2, 2, 1, 3, . . . }
; Podemos representar esta seal como una seal peridica o como un intervalo simple de la siguiente forma:

(a) (b)

Figure 8.2: Aqu nada mas observamos un periodo de la seal que tiene un vector de tamao cuatro
y esta contenida en C4 . (a) Funcin peridica (b) Funcion en el intervalo [0, T ]

note: El conjunto de seales de tiempo discreto con periodo N es igual a CN .

Tal como en el caso continuo, formaremos una base usando senosoidales armnicos. Antes de esto, es
necesario ver las senosoidales complejas discretas con mas detalle.

8.2.2.1 Senosoidales Complejos


8
Si usted esta familiarizado con la seal senosoidal bsica y con los exponenciales complejos (Section 1.5)
entonces usted no tendr ningn problema para entender esta seccin. En todos los libros, usted ver que
la senosoidal compleja discreta se escribe as:
ejn

8 "Elemental Signals" <http://cnx.org/content/m0004/latest/>

Available for free at Connexions <http://cnx.org/content/col10373/1.2>


170 CHAPTER 8. TRANSFORMADA DISCRETA DE FOURIER

Example 8.1

Figure 8.3: Senosoidal compleja con frecuencia = 0

Example 8.2

Figure 8.4: Senosoidal compleja con frecuencia =


4

8.2.2.1.1 En el Plano Complejo


9
Nuestra senosoidal compleja se puede gracar en nuestro plano complejo , el cual nos permite visualizar
fcilmente los cambios de la senosoidal compleja y extraer algunas propiedades. El valor absoluto de nuestra
senosoidal compleja tiene las siguientes caractersticas:

|ejn | = 1 , n R (8.3)

El cual nos dice que nuestra senosoidal compleja nicamente toma valores que se encuentran en el crculo
unitario. Con respecto al ngulo, la siguiente armacin es verdadera:

ejn = wn

(8.4)

9 "The Complex Plane" <http://cnx.org/content/m10596/latest/>

Available for free at Connexions <http://cnx.org/content/col10373/1.2>


171

Cuando n incrementa, podemos ver ejn igualando los valores que obtenemos al movernos en contra de
las manecillas del reloj alrededor del crculo unitario. Observe las siguiente guras Figure 8.5 para una mejor
ilustracin:

(a) (b) (c)

Figure 8.5: Estas imgenes muestran que cuando n incrementa, el valor de ejn se mueve en contra
de las manecillas del reloj alrededor del crculo unitario. (a) n = 0 (b) n = 1 (c) n = 2

note: Para que ejn sea peridica (Section 6.1), necesitamos que ejN = 1 para algn N.

Example 8.3
2
Nuestro primer ejemplo nos permite ver una seal peridica donde = 7 y N = 7.

(a) (b)

(a) N = 7 (b) Aqu tenemos una grafca de Re ej .
2 n
Figure 8.6: 7

Example 8.4
Ahora observemos los resultados de gracar una seal no peridica donde =1 y N = 7.

Available for free at Connexions <http://cnx.org/content/col10373/1.2>


172 CHAPTER 8. TRANSFORMADA DISCRETA DE FOURIER

(a) (b)

(a) N = 7 (b) Aqu tenemos una grca de Re ejn .


`
Figure 8.7:

8.2.2.1.2 Aliasing

Nuestra senosoidal compleja tiene la siguiente propiedad:

ejn = ej(+2)n (8.5)

Dada a esta propiedad, si tenemos una senosoidal con frecuencia + 2 , observaremos que esta seal tendr
un aliasing con una senosoidal de frecuencia .
note: Cada ejn es nica para [0, 2)

8.2.2.1.3 Frecuencias Negativas

Si nos dan una frecuencia < < 2 , entonces esta seal ser representada en nuestro plano complejo
como:

(a) (b)

Figure 8.8: Grca de nuestra senosoidal compleja con una frecuencia mayor que .

De nuestras imgenes mostradas arriba, el valor de nuestra senosoidal compleja en el plano complejo se
puede interpretar como girar hacia atrs (en direccin de las manecillas del reloj) alrededor del crculo
unitario con frecuencia 2 . Girar en sentido contrario de las manecillas del reloj w es lo mismo que girar
en sentido de las manecillas del reloj 2 .

Available for free at Connexions <http://cnx.org/content/col10373/1.2>


173

Example 8.5
5
Gracaremos nuestra senosoidal compleja, ejn , donde tenemos = 4 y n = 1.

Figure 8.9: La grca anterior de la frecuencia dada es idntica a una donde = 3


4
.

Esta grca es la misma que una senosoidal de frecuencia negativa 3


4 .

note: Tiene ms sentido elegir un intervalo entre [, ) para .

Recuerde que ejn y e(jn) son conjugados. Esto nos da la siguiente notacin y propiedad:

ejn = e(jn) (8.6)

Las partes reales de ambas exponenciales de la ecuacin de arriba son las mismas; la parte imaginaria son
los negativos de una a la otra. Esta idea es la denicin bsica de un conjugado.
Ya que hemos visto los conceptos de senosoidales complejas, retomaremos la idea de encontrar una base
para las seales peridicas en tiempo discreto. Despus de observar las senosoidales complejas, tenemos que
responder la pregunta sobre cuales senosoidales en tiempo discreto necesitamos para representar secuencias
peridicas con un periodo N.
note: Encuentre un conjunto de vectores bk = ejk n , n = {0, . . . , N 1} tal que {bk } sea una
base para Cn
Para resolver la pregunta de arriba, usaremos las senosoidales Armnicos con una frecuencia fundamental
2
de 0 = N :
Senosoidales Armnicas
2
ej N kn (8.7)

(a) (b) (c)

Figure 8.10: Ejemplos de nuestras


2armnicos
senosoidales. (a) Senosoidal armnico con k = 0 (b)

Parte imaginaria del senosoidal, Im ej N 1n , con k = 1 (c) Parte imaginaria del senosoidal, Im ej N 2n ,
2

con k = 2

Available for free at Connexions <http://cnx.org/content/col10373/1.2>


174 CHAPTER 8. TRANSFORMADA DISCRETA DE FOURIER

2
ej N kn es peridica con periodo N y tiene k ciclos entre n=0 y n = N 1.
Theorem 8.1:
Si dejamos
1 2
bk [n] = ej N kn , n = {0, . . . , N 1}
N
CN , entonces {bk } |k={0,...,N 1}
donde el trmino exponencial es un vector en es una base ortonor-
mal (Section 7.7.3: Base Ortonormal) para CN .
Proof:
Primero que todo, debemos demostrar que {bk } es ortonormal, por ejemplo < bk , bl >= kl
N 1 N 1
X 1 X j 2 kn (j 2
N ln)
< bk , bl >= bk [n] bl [n] = e N e
n=0
N n=0

N 1
1 X j 2 (lk)n
< bk , bl >= e N (8.8)
N n=0
Si l = k, entonces
1
PN 1
< bk , bl > = N n=0 1
(8.9)
= 1
Si l 6= k , entonces tenemos que usar la frmula de sumatoria parcial mostrada abajo:

N 1
X X X 1 N 1 N
n = n n = =
n=0 n=0
1 1 1
n=N

N 1
1 X j 2 (lk)n
< bk , bl >= e N
N n=0
2
en esta ecuacin podemos decir que = ej N (lk) , as podemos ver como esta expresin se encuentra
en la forma que necesitamos utilizar para nuestra frmula de sumatoria parcial.

2
1 1 ej N (lk)N 1 11
< bk , bl >= = =0
N 1 ej 2
N (lk) N 1 ej 2
N (lk)

As,
1 if k=l
< bk , bl >= (8.10)
0 if k 6= l

Por lo tanto: {bk } es un conjunto ortonormal. {bk } es tambin un base (Section 5.1.3: Bases), ya
que existen N vectores que son linealmente independientes (Section 5.1.1: Independencia Lineal)
(ortogonalidad implica independencia linear).
n 2
o
Finalmente, hemos demostrado que los senosoidales armnicos 1 ej N kn forman una base
N
n
ortonormal para C

Available for free at Connexions <http://cnx.org/content/col10373/1.2>


175

8.2.2.2 Series de Discretas de Fourier (DTFS)

Utilizando los pasos anteriores en la derivacin, usando nuestro entendimiento de espacio Hilbert (Section 7.3)
, y nalmente usando la expansin ortogonal (Section 7.8); el resto de la derivacin es automtica. Dada
una seal peridica discreta (vector Cn ) f [n], podemos escribir:

N 1
1 X 2
f [n] = ck ej N kn (8.11)
N k=0
N 1
1 X
f [n] e(j N kn)
2
ck = (8.12)
N n=0
1
Nota: Casi toda la gente juntan los trminos
N
en la expresin para ck .
note: Aqu se muestra la forma comn de las DTFS tomando en cuenta la nota previa:

N 1
2
X
f [n] = ck ej N kn
k=0

N 1
1 X
f [n] e(j N kn)
2
ck =
N n=0
Esto es lo que el comando fft de MATLAB hace.

8.3 Ecuacin de Matriz para la DTFS10


La DTFS es nada mas un cambio de bases
11
en CN . Para comenzar, tenemos f [n] en trminos de la base
estndar.
f [n] = f [0] e0 + f [1] e1 + + f [N 1] eN 1
Pn1 (8.13)
= k=0 f [k] [k n]

f [0] f [0] 0 0 0


f [1]
0 f [1] 0

0



f [2] =
0 + 0 + f [2] + +
0
(8.14)
. . . . .
. . .. .. .

. . .


f [N 1] 0 0 0 f [N 1]
Tomando la DTFS, podemos escribir f [n] en trminos de la base de Fourier senosoidal

N 1
2
X
f [n] = ck ej N kn (8.15)
k=0

f [0] 1 1 1
2 4


f [1]

1


ej N


ej N

4 8
ej N ej N


f [2] = c0 1 + c1

+ c2

+ ...
(8.16)
. . . .
. .. . .

. . .


2 4
f [N 1] 1 ej N (N 1) ej N (N 1)
10 This content is available online at <http://cnx.org/content/m12859/1.1/>.
11 "Linear Algebra: The Basics": Section Basis <http://cnx.org/content/m10734/latest/#sec_bas>

Available for free at Connexions <http://cnx.org/content/col10373/1.2>


176 CHAPTER 8. TRANSFORMADA DISCRETA DE FOURIER

Podemos formar la matriz base (llamaremos esto W envs deB ) al acomodar los vectores bases en las
columnas obtenemos
 
W = b0 [n] b1 [n] . . . bN 1 [n]

1 1 1 ... 1
2 4 2

1 ej N ej N ... ej N (N 1)
(8.17)

4 8 2
ej N ej N ej N 2(N 1)

1
= ...

. . . . .
.. . . . .

. . . .


2 2 2
1 ej N (N 1) ej N 2(N 1) ... ej N (N 1)(N 1)
2
con bk [n] = ej N kn
2
note: la entrada k-th la y n-th columna es Wj,k = ej N kn = Wn,k
As, aqu tenemos una simetra adicional

 
T 1
T
= W = W 1

W =W W
N

(ya que {bk [n]} son ortonormales)


Ahora podemos rescribir la ecuacin DTFS en forma de matriz, donde tenemos:

f = seal (vector en CN )
c = coecientes DTFS (vector en CN )

"synthesis" f = Wc f [n] =< c, bn >



"analysis" c = W T f = W f c [k] =< f, bk >

Table 8.2

Encontrar (e invertir) la DFTS es nada mas una multiplicacin de matrices.


Todo lo que se encuentra en CN esta limpio: no se utilizan lmites, no se usan preguntas de convergencia,
nada mas se utilice aritmtica de matrices.

8.4 Extensin Peridica de las DTFS12


8.4.1 Introduccin
Ya que contamos con entendimiento de lo que son las series discretas de Fourier (DTFS) (Section 8.2.2.2:
Series de Discretas de Fourier (DTFS)), podemos considerar la extensin peridica de c [k] (coecientes
discretos de Fourier). Las gures bsicas mostradas a continuacin muestran una simple ilustracin de como
nosotros podramos representar una secuencia en forma de seales peridicas gracadas sobre un numero
innito de intervalos.
12 This content is available online at <http://cnx.org/content/m12860/1.2/>.

Available for free at Connexions <http://cnx.org/content/col10373/1.2>


177

(a)

(b)

Figure 8.11: (a) vectores (b) secuencias periodicas

Exercise 8.4.1 (Solution on p. 190.)


Porqu una extensin peridica (Section 6.1) de los coecientes de la DFTS c [k] tiene sentido?

8.4.2 Ejemplos
Example 8.6: Funcin Cuadrada Discreta

Figure 8.12

Available for free at Connexions <http://cnx.org/content/col10373/1.2>


178 CHAPTER 8. TRANSFORMADA DISCRETA DE FOURIER

Calcule la DTFS c [k] usando:

N 1
1 X
f [n] e(j N kn)
2
c [k] = (8.18)
N n=0

Como en las series de Fourier continuas, podemos tomar la sumatoria en cualquier intervalo, para
tener
N1
1
e(j N kn)
X 2
ck = (8.19)
N
n=N1

Deje que m = n + N1 ,(as tenemos una serie geomtrica que empieza en 0)

1
P2N1 (j 2 (mN1 )k)
ck = N m=0 e
N
(8.20)
1 j 2 2N1 (j 2
N mk )
N k
P
= Ne m=0 e

Ahora, usando la formula parcial de sumatoria

M
X 1 aM +1
an = (8.21)
n=0
1a

P2N1  m
e(j N k)
2
1 j 2
N N1 k
ck = Ne m=0
(8.22)
1e ( N
j 2 (2N1 +1))
1 j 2
N N1 k
= Ne (jk 2 )
1e N

Manipulndola para que se vea como el sinc (distribuya)

( ) (N1 + 12 ) e(jk 2
N ( 1 2 ))

jk 2 jk 2 N +1
e 2N e N
1
ck = N ( ) ( )

jk 2 2 1 jk 2 1
e 2N ejk N 2 e N 2
!
sin
(
2k N1 + 1
2 ) (8.23)
N
1
= N sin( k
N )

= digital sinc

note: es peridica! Figure 8.13, Figure 8.14, y Figure 8.15 muestran como nuestra funcin y
coecientes para distintos valores de N1 .

(a) (b)

Figure 8.13: N1 = 1 (a) Grafca de f [n]. (b) Grafca de c [k].

Available for free at Connexions <http://cnx.org/content/col10373/1.2>


179

(a) (b)

Figure 8.14: N1 = 3 (a) Grafca de f [n]. (b) Grafca de c [k].

(a) (b)

Figure 8.15: N1 = 7 (a) Grafca de f [n]. (b) Grafca de c [k].

Example 8.7: Sonido de un Pjaro

Available for free at Connexions <http://cnx.org/content/col10373/1.2>


180 CHAPTER 8. TRANSFORMADA DISCRETA DE FOURIER

Figure 8.16

Example 8.8: Anlisis Espectral de la DFTS

Available for free at Connexions <http://cnx.org/content/col10373/1.2>


181

Figure 8.17

Example 8.9: Recuperando la Seal

Available for free at Connexions <http://cnx.org/content/col10373/1.2>


182 CHAPTER 8. TRANSFORMADA DISCRETA DE FOURIER

Figure 8.18

Example 8.10: Compresin (1-D)

Available for free at Connexions <http://cnx.org/content/col10373/1.2>


183

Figure 8.19

Example 8.11: Compresin de Imgenes

Available for free at Connexions <http://cnx.org/content/col10373/1.2>


184 CHAPTER 8. TRANSFORMADA DISCRETA DE FOURIER

(a)

(b)

Figure 8.20: We've cut down on storage space by > 90% (while incurring some loss) (a) Imagen de 256
por 256 (65,636 pixeles) (b) Imagen reconstruida usando 6000 coefcientes

8.5 Desplazamientos Circulares13


Las muchas propiedades de la DFTS (Section 8.2.2.2: Series de Discretas de Fourier (DTFS)) se convierten
14
sencillas (muy similares a las de las Series de Fourier ) cuando entendemos el concepto de: desplazamien-
tos circulares.

8.5.1 Desplazamientos Circulares


15
Podemos describir las secuencias peridicas teniendo puntos discretos en un crculo como su dominio.

13 This content is available online at <http://cnx.org/content/m12864/1.1/>.


14 "Fourier Series: Eigenfunction Approach" <http://cnx.org/content/m10496/latest/>
15 "Continuous Time Periodic Signals" <http://cnx.org/content/m10744/latest/>

Available for free at Connexions <http://cnx.org/content/col10373/1.2>


185

Figure 8.21

Desplazar m, f (n + m), corresponde a rotar el cilindro m puntos ACW (en contra del reloj). Para
m = 2, obtenemos un desplazamiento igual al que se ve en la siguiente ilustracin:

Figure 8.22: para m = 2

Available for free at Connexions <http://cnx.org/content/col10373/1.2>


186 CHAPTER 8. TRANSFORMADA DISCRETA DE FOURIER

Figure 8.23

Para ciclar los desplazamientos seguiremos los siguientes pasos:


1) Escriba f (n) en el cilindro, ACW

Figure 8.24: N =8

Available for free at Connexions <http://cnx.org/content/col10373/1.2>


187

2) Para ciclar por m, gire el cilindro m lugares ACW

f [n] f [((n + m))N ]

Figure 8.25: m = 3

Example 8.12
Si f (n) = [0, 1, 2, 3, 4, 5, 6, 7], entonces f (((n 3))N ) = [3, 4, 5, 6, 7, 0, 1, 2]
Es llamado desplazamiento circular, ya que nos estamos moviendo alrededor del crculo. El
desplazamiento comn es conocido como desplazamiento linear (un movimiento en una lnea).

8.5.1.1 Notas para el desplazamiento circular

f [((n + N ))N ] = f [n]


Girar por N lugares es lo mismo que girar por una vuelta completa, o no moverse del mismo lugar.

f [((n + N ))N ] = f [((n (N m)))N ]


Desplazar ACW m es equivalente a desplazar CW N m

Available for free at Connexions <http://cnx.org/content/col10373/1.2>


188 CHAPTER 8. TRANSFORMADA DISCRETA DE FOURIER

Figure 8.26

f [((n))N ]
La expresin anterior, escribe los valores de f [n] para el lado del reloj.

Available for free at Connexions <http://cnx.org/content/col10373/1.2>


189

(a) (b)

(a) f [n] (b) f ((n))N



Figure 8.27:

8.5.2 Desplazamientos Circulares y el DFT


Theorem 8.2: Desplazamiento Circular y el DFT
Si
DFT
f [n] F [k]
entonces
f [((n m))N ] e(j N km) F [k]
DFT 2

(Por ejemplo. Desplazamiento circular en el dominio del tiempo= desplazamiento del ngulo en el
DFT)
Proof:

N 1
1 X 2
f [n] = F [k] ej N kn (8.24)
N
k=0

as que el desplazar el ngulo en el DFT

F [k] e(j N kn) ej N kn


PN 1 2 2
1
f [n] = N k=0
1
PN 1 2
= N k=0 F [k] ej N k(nm) (8.25)

= f [((n m))N ]

Available for free at Connexions <http://cnx.org/content/col10373/1.2>


190 CHAPTER 8. TRANSFORMADA DISCRETA DE FOURIER

Solutions to Exercises in Chapter 8


Solution to Exercise 8.4.1 (p. 177)
2
Aliasing: bk = ej N kn
2
bk+N = ej N (k+N )n
2
= ej N kn ej2n
2
(8.26)
= ej N n
= bk
Coecientes DTFS tambin son peridicos con periodo N.

Available for free at Connexions <http://cnx.org/content/col10373/1.2>


Chapter 9

Transformada Rpida de Fourier (FFT)

9.1 DFT: Transformada Rpida de Fourier1


2
Ahora podemos calcular el espectro de una seal arbitraria: La Transformada de Fourier Discreta (DFT)
calcula el espectro en N frecuencias igualmente espaciadas de una longitud- N secuencias. Una edicin que
nunca se presenta en el "clculo" anlogo,como la realizada por un cirduito, es cuanto trabajo toma realizar
la operacin de procesamiento de seal como la ltracin. En computacin, esta consideracin traslada del
nmero de pasos bsicos de computacin requeridos para realizar el proceso. El nmero de pasos, conocidos
como,la complejidad, se vuelve equivalente a cuanto tiempo toma el clculo (que tanto tiempo tenemos
que esperar para una respuesta). La complejidad no esta atada a computadoras especicas o lenguajes
de programacin, pero a cuantos pasos son requeridos en cualquier clculo. As, un procedimiento con
complejidad indicada dice que el tiempo tomado ser proporcional a alguna cantidad de datos utilizados
en el clculo y en la cantidad demandada.
Por ejemplo, considerar la formula para la transformada discreta de Fourier. Para cada frecuencia que
elijamos, debemos multiplicar cada valor de la seal por un nmero complejo y sumar los resultados. Para
una seal valorada-real, cada multiplicacin real-por-complejo requiere dos multiplicaciones reales, signica
que tenemos 2N multiplicaciones para realizarse. Para sumar los resultados juntos, debemos mantener la
parte real y la imaginaria separadas. Sumando N nmeros requiere N 1 sumas. Constantemente, cada
frecuencia requiere 2N + 2 (N 1) = 4N 2 pasos bsicos de realizar. Como tenemos N frecuencias, el
nmero total de operaciones es N (4N 2).
En clculos de la complejidad, solo tenemos que preocuparnos de que sucede cuando la longitud incre-
menta, y tomar el trmino dominante aqu el trmino 4N 2 como reejo de cuanto trabajo esta involu-
crado haciendo la computacin. Como una constante multiplicativa no importa ya que estamos haciendo

una "proporcional" a la evaluacin,encontramos que la DFT es un O N2 procedimiento computacional.
Esta notacin es leda "orden N -cuadrado". Donde, si tenemos doble longitud, esperamos que el tiempo de
la realizacin sea aproximadamente el cuadruple.

Exercise 9.1.1 (Solution on p. 197.)


Haciendo la evaluacin de la complejidad para la DFT, asumimos que los datos son reales. Surgen
tres preguntas. Primero que nada, el espectro de las seales tiene simetra conjugada, lo que signica
N  3
que los componentes de las frecuencias negativas (k =
2 + 1, ..., N + 1 en la DFT ) pueden ser
calculadas de los componentes de la frecuencia positiva correspondiente. Esta simetra cambia la
complejidad de la Trasformada Directa de Fourier DFT?
En segundo lugar,supongamos que los datos son de valores complejos; Cal es la complejidad
de la DFT ahora?
1 This content is available online at <http://cnx.org/content/m12937/1.1/>.
2 "Discrete Fourier Transform", (1) : Discrete Fourier transform <http://cnx.org/content/m0502/latest/#eqn1>
3 "Discrete Fourier Transform", (1) : Discrete Fourier transform <http://cnx.org/content/m0502/latest/#eqn1>

Available for free at Connexions <http://cnx.org/content/col10373/1.2>

191
192 CHAPTER 9. TRANSFORMADA RPIDA DE FOURIER (FFT)

Finalmete, pregunta menos importante pero interesante, supongamos que queremos K valores
de fecuencias en lugar de N; Ahora cal es la complejidad?

9.2 La Transformada Rpida de Fourier (FFT)4


9.2.1 Introduccin
La Transformada Rpida de Fourier (Fast Fourier Transform) (FFT) es un algoritmo eciente O(NlogN)
para calcular la DFT

orignalmente descubierta por Gauss a primcipios de 1800


redescubierta por Cooley y Tukey en IBM durante 1960
C.S. Burrus, de la Universidad de Rice University siendo jefe del departamento de Ingenieria, literal-
mente "escribio el libro" de los algoritmos de la rpida Transformada Discreta de Fourier DFT.

5
La FFT explota las simetrias en la matriz W para aproximarse "divide y conquistaras". No hablaremos
6
del actual algoritmo de la FFT aqui, veamos estas notas si usted esta interesado en leer mas a cerca de la
idea detras de la FFT.

9.2.2 Comparacin De la Velocidad


En cunto es mejor O(NlogN) que O( N 2 )?

Figure 9.1: Esta gura muestra que tan lento crece el tiempo de solucin de un proceso de O(NlogN).

4 This content is available online at <http://cnx.org/content/m12942/1.1/>.


5 "Fast Fourier Transform (FFT)" <http://cnx.org/content/m10250/latest/>
6 "Fast Fourier Transform (FFT)" <http://cnx.org/content/m10250/latest/>

Available for free at Connexions <http://cnx.org/content/col10373/1.2>


193

N 10 100 1000 106 109


N2 100 104 106 1012 1018
N logN 1 200 3000 6 106 9 109

Table 9.1

Digamos que tiene una maquina de 1 MFLOP (un millin de "puntos otantes" de operacione spor
segundo). Sea N = 1millin = 106 .
2 12 6
Un algoritmo de O( N ) toma 10 procesos 10 segundos ' 11.5 das.
6
Un algoritmo de O( N logN ) toma 6 10 procesos 6 segundos.

note: N = 1millin es razonable.

Example 9.1
Una camara digital de 3 megapixeles arroja 3 106 nmeros por cada foto. As que para dos N
secuencias de punto f [n] y h [n]. Si resolvemos directamente f [n] ~ h [n] : O( N 2) operaciones.
Tomando la FFTs  O(NlogN)
multiplicando la FFTs  O(N)
la inversa de FFTs  O(NlogN).
el total de complejidad es O(NlogN).

note: FFT + computacin didigital fue completamente responsable de la "explosin" del Proce-
samiento Digital de Seales DSP en los aos 60's.

note: La Universidad de Rice fue (y sigue siendo) uno de los lugares de hacer investigacin en
DSP.

9.3 Derivando la Transformada Rpida de Fourier7


Para derivar la FFT,asumimos que la duracin de la seal es una potencia de dos: N = 2l . Considere que
pasa a los elementos pares numerados y a los elemenots impares numerados de la solucin de la secuencia en
la DFT.

22k 2(N 2)k


S (k) = s (0) + s (2) e(j) N + s (N 2) e(j) N
+ + (9.1)
2k 2(2+1)k 2(N 2+1)k
s (1) e(j) N + s (3) e(j) N + + s (N 1) e(j)
N =
2 ( N
2 1)k
(j) 2k
N (j) N
s (0)
+ s (2) e 2 + + s (N 2) e
2 +
2 ( 2 1)k
N
(j) 2k (j)
s (1) + s (3) e N
2 + + s (N 1) e
N
2 e (j2k)
N

Cada trmino en parntesis al cuadrado tiene la forma de una longitud N


2 de DFT. La primera es una
DFT de los elementos pares nuemrados,y la segunda es de los elementos impares numerados. La primer
(j2k)
DFT es combinada con la segunda multiplicadno por el exponencial complejo e N . La media-longitud

7 This content is available online at <http://cnx.org/content/m12941/1.1/>.

Available for free at Connexions <http://cnx.org/content/col10373/1.2>


194 CHAPTER 9. TRANSFORMADA RPIDA DE FOURIER (FFT)

de la transformada es cada evaluacin de los ndices de frecuencia k {0, . . . , N 1} . Normalmente el


nmero de indices de frecuencia del rango de una calculacin de la DFT esta entre cero y la longitud de
la transformacin menos uno. La ventaja computacional de la FFT viene de reconocer la natulareza
del perido de la transformada discreta de Fourier. La FFT simplemente reusa las soluciones hechas en la
(j2k)
media-longitud de la transformada y las combina a travs de sumas y multiplicaciones por , que e N
N
no es peridica sobre , para rescribir la longitud-N DFT. La Figure 9.2 (Descomposicin de la DFT de
2
longitud-8) ilustra la descomposicin. Como se mantiene, ahora resolvemos dos transformadas de longitudes-
 
N N2
2 (complejidad 2O 4 ), multiplicar una de ellas por el exponencial complejo (complejidad O (N ) ), y

sumar los resultados (complejidad O (N ) ). En este punto, el total de complejidad sigue dominado por la
mitad-longitud de la calculacin de DFT, pero el coecente de proporcionalidad ha sido reducido.
Ahora para la diversin. Por que N = 2l , cada una de las transformaciones de media-longitud puede ser
reducida a una de dos cuartos-longitud, cada una de estas en dos octavos-longitud, etc. Esta descomposicin
continua hasta que nos quedamos con una transformada de longitud -2 . Esta transformada es absolutamente
N
simple, involucrando solo sumas. Donde la primer etapa de la FFT tiene
2 transformadas de longitud-2
(vease la parte de abajo de la Figure 9.2 (Descomposicin de la DFT de longitud-8)). Pares de estas
transformadas son combinados suamando uno a otro multiplicado por el exponencial complejo. Cada par
requiere 4 sumas y 4 multiplicaciones, dando un nmero total de computaciones igual a
N
2 . Este 8 N4 =
nmero de computaciones no cambia de etapa a etapa. Ya que el nmero de etapas, el nmero de veces que
la longitud puede ser dividida por dos, igual a log2 N , la complejidad de la FFT es O (N logN ) .

Available for free at Connexions <http://cnx.org/content/col10373/1.2>


195

Descomposicin de la DFT de longitud-8

(a)

(b)

Figure 9.2: La descomposicin inicial de una longitud-8 DFT en trminos usando ndices pares e im-
pares entradas marca la primera fase de convertirse en el algoritmo de la FFT. Cuando esta transformada
de media-longitud esta completamente descompuesta, nos quedamos con el diagrama del panel inferior
que representa la solucin de la FFT de longitud-8.

Haciendo un ejemplo haremos ahorros de procesos ms obvios. Veamos los detalles de la DFT de longitud-
8. Como se muestra en la Figure 9.2 (Descomposicin de la DFT de longitud-8), primero descomponemos
la DFT en dos DFT de longitud-4,con las salidas sumadas y restadas en pares. Considerando la Figure 9.2
(Descomposicin de la DFT de longitud-8) como el ndice de frecuencia que va de 0 hasta 7,reciclamos valores
de la DFT de longitud-4 para los calculos nales por la peridicidad de la salida de la DFT. Examinando
como los pares de salidas se recojen juntas, creamos los alementos computacionales bsicos conocidos como
una mariposa (Figure 9.3 (Mariposa)).

Available for free at Connexions <http://cnx.org/content/col10373/1.2>


196 CHAPTER 9. TRANSFORMADA RPIDA DE FOURIER (FFT)

Mariposa

Figure 9.3: Los elementos computacionales bsicos de la transformada rpida de Fourier es la mariposa.
Toma dos nmeros complejos, representados por a y b, y forma las cantidades mostradas . Cada mariposa
requiere una multiplicacin compleja y dos sumas complejas.

Considerando los procesos involucrando la salida de frecuencias comunes de la DFT de dos media-longitud,
vemos que las dos multiplicaciones complejas son relacionadas unas con otras, y podemos reducir nuestro
proceso en un futuro. Para descomposiciones posteriores las DFT de longitud-4 en las DFT de longitud-
2 y combinando sus salidas, llegamos al resumen del diagrama de la transformada rpida de Fourier de
longitud-8 (Figure 9.2 (Descomposicin de la DFT de longitud-8)). Aunque la multiplicacin de los complejos
es absolutamente simple (multiplicando por e(j) signica partes negativas reales y partes imaginarias),
contemos sos para los propsitos de evaluar la complejidad como el complejo se multiplica por completo.
N
Tenemos
2 = 4 multiplicaciones complejas y 2N = 16 sumas para cada etapa y log2 N = 3 etapas, haciendo
3N
el nmero de procesos bsicos
2 log2 N como se predijo.
Exercise 9.3.1 (Solution on p. 197.)
Notese que el orden de la secuencia de entrada en dos partes de Figure 9.2 (Descomposicin de la
DFT de longitud-8) no estan absolutamente iguales. por que no?, cmo esta determinado este
orden?

Otros lagoritmos "rpidos" han sido descubiertos, todos de los cuales hacen uso de cuantos factores tiene
la transformada de longitud N. En teoria de nmeros, el nmero de factores primos de un entero dado
tiene medidas como compuesto es el. El nmero 16 y 81 son altamente compuestos (iguales a 24 y 34
1 2
respectivamente),el nmero 18 es menor asi que ( 2 3 ), y el 17 no del todo (es primo). a travs de
treinta aos de desarrollo del algoritmo de la transformada de Fourier, el algoritmo original de Cooley-
Tukey esta alejado de los usados con mas frecuancia. Es tan eciente computacionalmente hablando como
la transformada de longitud de potencia de dos es frecuentemente usada sin importar la longitud actual de
los datos.

Available for free at Connexions <http://cnx.org/content/col10373/1.2>


197

Solutions to Exercises in Chapter 9


Solution to Exercise 9.1.1 (p. 191)
Cuando la seal es de valor real, solo necesitamos la mitad del valor del espectro, pero la complejidad
permanece sin cambios. Si los datos son de valores complejos,lo cual requiere de la retencin de todos los
datos, la complejidad es nuevamente la misma. Cuando se requieren solamente K frecuencias, la complejidad
es O (KN).
Solution to Exercise 9.3.1 (p. 196)
El panel de arriba no usa el algoritmo de FFT para procesar la DFT de longitud-4 mientras que el abajo
si.El orden es determinado por el algoritmo.

Available for free at Connexions <http://cnx.org/content/col10373/1.2>


198 CHAPTER 9. TRANSFORMADA RPIDA DE FOURIER (FFT)

Available for free at Connexions <http://cnx.org/content/col10373/1.2>


Chapter 10

Convergencia

10.1 Convergencia de Secuencias1


10.1.1 Qu es una Secuencia?
Denition 10.1: secuencia
Una secuencia es una funcin gn denida en los enteros positivos 'n'. Tambin denotamos una
secuencia por: {gn } |
n=1
Example
Una secuencia de nmeros reales:
1
gn =
n
Example
Una secuencia vector:
n

sin 2
gn =
n

cos 2

Example
Una secuencia de una funcin:

1
1 if 0t< n
gn (t) =
0 otherwise

note: Una funcin puede ser pensada como un vector de dimensin innito donde por cada valor
de 't' tenemos una dimensin.

10.1.2 Convergencia de Secuencias Reales


Denition 10.2: lmite
Una secuencia {gn } |
n=1 converge a un lmite gR si para todo >0 existe un entero N tal que

|gi g| <  , i N
1 This content is available online at <http://cnx.org/content/m12943/1.2/>.

Available for free at Connexions <http://cnx.org/content/col10373/1.2>

199
200 CHAPTER 10. CONVERGENCIA

Usualmente denotamos un lmite escribiendo

limit gi = g
i


gi g

La denicin anterior signica que no importa que tan pequeo hagamos , excepto para un nmero nito
de gi 's, todos los puntos de la secuencia estan dentro de una distancia  de g.
Example 10.1
Dada las siguiente secuencia de convergencia :

1
gn = (10.1)
n
Intuitivamente podemos asumir el siguiente lmite:

limit gn = 0
n

. Ahora provemos esto rigurosamente, digamos que dado el nmero real  > 0. Escojamos N = d 1 e,
donde dxe denota el menor entero mayor que x. Entonces para nN tenemos

1 1
|gn 0| = <
n N
Donde,
limit gn = 0
n

Example 10.2
Ahora veamos la siguiente secuencia no-convergente

1 if n = par
gn =
1 if n = impar

Esta secuencia oscila entre 1 y -1, asi que nunca converge.

10.1.2.1 Problemas

Para practicar, diga cuales de las siguientes secuencias convergen y de el limite si este existe.

1. gn = n

1 if n = par
n
2. gn =
1 if n = impar
n
1 if n 6= potencia de 10
n
3. gn =
1 otherwise

n if n < 105
4. gn =
1 if n 105
n
gn = sin n

5.
6. gn = j n

Available for free at Connexions <http://cnx.org/content/col10373/1.2>


201

10.2 Convergencia de Vectores2


10.2.1 Convergencia de Vectores
Discutiremos la convergencia puntual y de la norma de vectores. Tambin existen otros tipos de convergencia
3
y uno en particular, la convergencia uniforme , tambin puede ser estudiada. Para esta discusin, asumiremos
que los vectores pertenecen a un espacio de vector normado (Section 7.2).

10.2.1.1 COnvergencia Puntual

Una secuencia (Section 10.1) {gn } |


n=1 converge puntualmente al lmite g si cada elemento de gn converge
al elemento correspondiente en g . A continuacin hay unos ejemplos para tratar de ilustrar esta idea.

Example 10.3

1
gn [1] 1+ n
gn = =
1
gn [2] 2 n

Primero encontramos los siguiente limites para nuestras dos gn 's:


limit gn [1] = 1
n

limit gn [2] = 2
n
Despus tenemos el siguiente,
limit gn = g
n

1
puntual, donde g= .
2
Example 10.4

t
gn (t) = , tR
n
Como se hizo anteriormente, primero examinamos el lmite

t0
limit gn (t0 ) = limit =0
n n n
donde t0 R. Por lo tanto limit gn = g puntualmente donde g (t) = 0 para toda t R.
n

10.2.1.2 Norma de Convergencia

La secuencia (Section 10.1) {gn } |


n=1 converge a g en la norma si limit k gn g k= 0.
Aqui k k es la norma
n
(Section 7.2) del espacio vectorial correspondiente de gn 's. Intuitivamente esto signica que la distancia
entre los vectores gn y g decrese a 0.
Example 10.5

1
1+ n
gn =
1
2 n

2 This content is available online at <http://cnx.org/content/m12944/1.2/>.


3 "Uniform Convergence of Function Sequences" <http://cnx.org/content/m10895/latest/>

Available for free at Connexions <http://cnx.org/content/col10373/1.2>


202 CHAPTER 10. CONVERGENCIA


1
Sea g=
2
q 2 2
1
k gn g k = n
1+ 1 + 2 n1
q
1 1 (10.2)
= n2 + n2

2
= n

Asi limit k gn g k= 0, Por lo tanto, gn g en la norma.


n
Example 10.6

t

n if 0t1
gn (t) =
0 otherwise

Sea g (t) = 0 para todo t.


R1 t2
k gn (t) g (t) k = 0 n2
dt
t3 1
3n2 |n=0
= (10.3)

1
= 3n2

Asi limit k gn (t) g (t) k= 0 Por lo tanto, gn (t) g (t) en la norma.


n

10.2.2 Puntual vs.Norma de Convergencia


Theorem 10.1:
Para Rm , la convergencia puntual y la norma de convergencia es equivalente.
Proof: Puntual Norma

gn [i] g [i]
Asumiendo lo anterior, entonces

m
X
2 2
(k gn g k) = (gn [i] g [i])
i=1

As,
2 Pm
limit (k gn g k) = limit i=1 2
n n
Pm
= (10.4)
i=1 limit
n
2
= 0

Proof: Norma Puntual

k gn g k 0

Pm Pm
limit i=1 2 = i=1 limit 2
n n (10.5)
= 0

Available for free at Connexions <http://cnx.org/content/col10373/1.2>


203

Ya que cada trmino es mayor o igual a cero, todos los trminos 'm' deben ser cero. As,

limit 2 = 0
n

para todo i. Por lo tanto,


gn g puntual

note: En un espacio de dimensin nita el teorema anterior ya no es cierto. Probaremos esto con
contraejemplos mostrados a continuacin.

10.2.2.1 Contra Ejemplos

Example 10.7: Puntual Norma


Dada la siguiente funcin:
n 1
if 0<t< n
gn (t) =
0 otherwise

Entonces limit gn (t) = 0 Esto signica que,


n

gn (t) g (t)

donde para todo t g (t) = 0.


Ahora,
2 R 2
(k gn k) =
(|gn (t) |) dt
1
n2 dt
R
= n (10.6)
0
= n
Ya que la norma de la funcin se eleva, no puede converger a cualquier funcin con norma nita.

Example 10.8: Norma Puntual


Dada la siguiente funcin:

1 1
if 0<t< n
gn (t) = si n es par
0 otherwise


1 if 0 < t < 1
n
gn (t) = si n es impar
0 otherwise

Entonces,
Z 1
n 1
k gn g k= 1dt = 0
0 n
donde g (t) = 0 para todo t. Entonces,

gn g en la norma

Sin embargo, en t = 0, gn (t) oscila entre -1 y 1, Y por lo tanto es no convergente. As, gn (t) no
tiene convergencia puntual.

Available for free at Connexions <http://cnx.org/content/col10373/1.2>


204 CHAPTER 10. CONVERGENCIA

10.2.2.2 Problemas

Pruebe si las siguientes secuencias tienen convergencia puntual, norma de convergencia, o ambas se mantienen
en sus limites.

1
if 0 < t

nt
1. gn (t) =
0 if t 0

e(nt) if t 0
2. gn (t) =
0 if t < 0

10.3 Convergencia Uniforme de Secuencias de Funciones.4


10.3.1 Convergencia Uniforme de Secuencias de Funciones
para esta discusin, solo consideraremos las funciones con gn donde

RR
Denition 10.3: Convergencia Uniforme
La secuencia (Section 10.1) {gn } |
n=1 converge uniformemente a la funci g si para cada  > 0
existe un entero N tal que n N implica que
|gn (t) g (t) |  (10.7)

para todo t R.
Obviamente toda secuencia uniformemente continua es de convergencia puntual (Section 10.2) . La
diferencia entre convergencia puntual y uniformemente continua es esta: Si {gn } converge puntualmente a g ,
entonces para todo  > 0 y para toda t R hay un entero N que depende de  y t tal que (10.7) se mantiene
si n N . Si {gn } converge uniformemente a g , es posible que para cada  > 0 enocntrar un entero N que
ser par todo t R.

Example 10.9

1
gn (t) = , tR
n
Sea >0 dado. Entonces escoja N = d 1 e. Obviamente,

|gn (t) 0|  , n N
para toda t. As, gn (t) converge uniformemente a 0.
Example 10.10

t
gn (t) = , tR
n
Obviamente para cualquier  > 0 no podemos encontrar una funcin sencilla gn (t) para la cual la
(10.7) se mantiene con g (t) = 0 para todo t. As gn no es convergente uniformemente. Sin embargo
tenemos:
gn (t) g (t) puntual

note: La convergencia uniforme siempre implica convergencia puntual, pero la cpnvergencia


puntual no necesariamente garantiza la convergencia uniforme.

4 This content is available online at <http://cnx.org/content/m12945/1.1/>.

Available for free at Connexions <http://cnx.org/content/col10373/1.2>


205

10.3.1.1 Problems

Pruebe rigurosamente si las siguientes funciones convergen puntualmente o uniformemente, o ambas.

1. gn (t) = sin(t)
n
t
2. gn (t) =
en
1 if t > 0
nt
3. gn (t) =
0 if t 0

Available for free at Connexions <http://cnx.org/content/col10373/1.2>


206 CHAPTER 10. CONVERGENCIA

Available for free at Connexions <http://cnx.org/content/col10373/1.2>


Chapter 11

Transformada de Fourier Discreta en el


Tiempo (DTFT)

11.1 Tabla de Transformadas de Fourier Comunes1


11.1.1 Common CTFT Properties

Time Domain Signal Frequency Domain Signal Condition

e(at) u (t) 1
a+j a>0
at 1
e u (t) aj a>0
(a|t|) 2a
e a2 + 2 a>0
(at) 1
te u (t) (a+j)2
a>0
tn e(at) u (t) n!
(a+j)n+1
a>0
(t) 1
1 2 ()
j0 t
e 2 ( 0 )
cos (0 t) ( ( 0 ) + ( + 0 ))
sin (0 t) j ( ( + 0 ) ( 0 ))
1
u (t) () + j
2
sgn (t) j

cos (0 t) u (t) 2 ( ( 0 ) + ( + 0 )) +
j
0 2 2
continued on next page

1 This content is available online at <http://cnx.org/content/m10099/2.12/>.


Available for free at Connexions <http://cnx.org/content/col10373/1.2>

207
CHAPTER 11. TRANSFORMADA DE FOURIER DISCRETA EN EL
208
TIEMPO (DTFT)


sin (0 t) u (t) 2j ( ( 0 ) ( + 0 )) +
0
0 2 2

0
e(at) sin (0 t) u (t) (a+j)2 +0 2
a>0
a+j
e(at) cos (0 t) u (t) (a+j)2 +0 2
a>0
u (t + ) u (t ) 2 sin(

)
= 2 sinc (t)
0 sin(0 t)
0 t = 0 sinc (0 ) u ( + 0 ) u ( 0 )
t t t
sinc2
   
+ 1 u +1 u  + 2
t + 1  u t u t 1 =
triag 2t
     
0 2 0 t
u 0 + 1 u 0 +

2 sinc 2 0 +1
     
0 + 1 u 0 u 0 1 =
 

triag 2 0
P P 2
n= (t nT ) 0 n= ( n0 ) 0 = T
t2
2
2 2
2
e 2
2e

Table 11.1

triag[n] is the triangle function for arbitrary real-valued n.


1+n if 1n0
triag[n] = { 1n if 0<n1
0 otherwise

11.2 Transformacin Discreta de Fourier2


11.2.1 N-puinto Punto Transformada Discreta de Fourier (DFT)
N 1
2
X
X [k] = x [n] e(j) n kn , k = {0, . . . , N 1} (11.1)
n=0
N 1
1 X 2
x [n] = X [k] ej n kn , n = {0, . . . , N 1} (11.2)
N
k=0

Note que:

X [k] es la DTFT evaluado en = 2


N k , k = {0, . . . , N 1}
Completar con ceros x [n] a M muestras antes de sacar el DFT, da como resultado una versin
muestreada de M -puntos uniformes del DTFT :

 2  NX 1
2
X ej M k = x [n] e(j) M k (11.3)
n=0

  N 1
j 2 2
X
X e M k = xzp [n] e(j) M k
n=0
2 This content is available online at <http://cnx.org/content/m12843/1.2/>.

Available for free at Connexions <http://cnx.org/content/col10373/1.2>


209

 2 
X ej M k = Xzp [k] , k = {0, . . . , M 1}

La N -pt DFT es suciente para reconstruir toda la DTFT de una secuencia de N -pt:
1
 NX
X ej = x [n] e(j)n (11.4)
n=0

1 N 1
 NX 1 X 2
X ej = X [k] ej N kn e(j)n
n=0
N
k=0

1 N 1
 NX 1 X (j)( 2
X ej = X [k] e N k )n
N
k=0 k=0

1
N
!
N 2k

1 sin N 1
 e(j)( N k) 2
X 2
j 2

X e = X [k] N 2k
N sin 2N
k=0

. D

0 2pi/N 4pi/N 2pi

N
1 sin( 2 )
Figure 11.1: Sinc dirichlet, N sin( )
2

2
DFT tiene una representacin en forma de matriz muy conveniente. Deniendo WN = e(j) N ,


X [0] WN0 WN0 WN0 WN0 ... x [0]

X [1] W0 WN1 WN2 WN3 ... x [1]
N
= (11.5)

. .
WN0 WN2 WN4 WN6

. ... .
. .


. . . . .
. . . . .
X [N 1] . . . . . x [N 1]

donde X = W (x) respectivamente. W tiene las siguientes propiedades:

W es Vandermonde: La nth columna de W es un polinomio en WNn


W es simetrico: W = W T   H  H  
1N W es unitaria: 1N W 1 W
N
= 1 W
N
1 W = I
N

Available for free at Connexions <http://cnx.org/content/col10373/1.2>


CHAPTER 11. TRANSFORMADA DE FOURIER DISCRETA EN EL
210
TIEMPO (DTFT)


1
NW = W 1 , es la matriz y DFT.
N
Para N un poder de 2, la FFT se puede usar para calcular la DFT usando
2 log2 N en vez de N2
operaciones.

N
N 2 log2 N N2
16 32 256

64 192 4096

256 1024 65536

1024 5120 1048576

Table 11.2

11.3 Transformada Discreta de Fourier (DFT)3


La transformada de Fourier Discreta en el tiempo (y tambin la transformada continua) pueden ser evaluadas
cuando tenemos una expresin analtica para la seal. Suponga que tengamos una seal, como es la seal
del habla usada en el capitulo anterior, para ella no existe una formula. Entonces cmo podra usted
calcular su espectro? Por ejemplo, cmo calculamos el espectrograma para el ejemplo de la seal de habla
4
? La transformada de Fourier discreta ( DFT) nos permite calcula el espectro de informacin discreta en
el tiempo. Estando en tiempo discreto podemos calcular exactamente el espectro, para seales anlogas
no existe manera similar para calcular su espectro similar. Para el espectro de seales anlogas se tienen
que construir equipo especial, que consiste en casi todos los casos de convertidores A / D y computaciones
discretas. Anlisis de el espectro discreta en el tiempo son mas exibles que los anlisis de las seales
continuas.
5
La formula del DTFT es una suma que conceptualmente es fcil de calcular excepto por unos problemas.

Duracin de la seal. La suma se extiende sobre la duracin de la seal, la cual tiene que ser nita
para calcular el espectro de la seal. Es extremadamente difcil guardar una seal innita, as que
asumimos que la seal se extiende sobre [0, N 1].
Frecuencia contina. Igual de importante que el problema de la duracin de la seal es el hecho
 1 1
que la frecuencia variable es continua: talvez solo se tenga que extender un periodo, como 2, 2 o
[0, 1], pero la formula DTFT requiere evaluar el espectro de todas las frecuencias dentro del periodo.
Calculemos el espectro de unas cuantas frecuencias; las mas obvias son las que tienen una espacio
k
similar f= K, k {k, . . . , K 1}.

As que denimos la transformada discreta de Fourier ( DFT) como

N 1
X j2nk
S (k) = s (n) e K , k {0, . . . , K 1} (11.6)
n=0
 k

Aqu, S (k) representa S ej2 K .
Podemos calcular el espectro en todas las frecuencias con espacio similar que queremos. Note que usted
puede pensar de esta motivacin computacional como muestrear el espectro; se vera mas sobre esta inter-
pretacin despus. El problema ahora es el saber cuantas frecuencias son sucientes para capturar el como

3 This content is available online at <http://cnx.org/content/m12844/1.1/>.


4 "Modeling the Speech Signal", Figure 5: spectrogram <http://cnx.org/content/m0049/latest/#spectrogram>
5 "Discrete-Time Fourier Transform (DTFT)", (1) <http://cnx.org/content/m10247/latest/#eqn1>

Available for free at Connexions <http://cnx.org/content/col10373/1.2>


211

el espectro cambia con la frecuencia. Una manera de responder esta pregunta es determinando la formula
de la transformada inversa discreta de Fourier: dado S (k), k = {0, . . . , K 1} cmo encontramos s (n),
PK1 j2nk
n = {0, . . . , N 1}? La formula estar en la siguiente manera s (n) = k=0 S (k) e
K . Substituyendo la
formula DFT en este prototipo para la transformada inversa da

K1
X NX1
s (m) e(j ) ej 2nk
2mk
s (n) = K K (11.7)
k=0 m=0

Note que la relacin de ortogonalidad que usamos tiene un carcter diferente ahora.

K1 K if (m = {n, n K, n 2K, . . . })
e(j ) ej
X 2km 2kn
K K = (11.8)
0 otherwise
k=0

Nosotros obtenemos valores de no cero cada vez que los dos ndices dieren por mltiple de
P K. Podemos
expresar estos resultados como K l (m n lK). As, nuestra formula se convierte

N
X 1
X
s (n) = s (m) K (m n lK) (11.9)
m=0 l=

Los nmeros n y m existen en el rango {0, . . . , N 1}. Para obtener una transformada inversa, necesitamos
sumar un solo muestreo unitario para m, n en este rango. Si no lo hiciramos, s (n) igualara a la suma
de valores, y no tendramos una transformada valida: una vez que regresemos al dominio de frecuencia, no
podramos obtener una ambiguosidad! claramente, el termino l=0 siempre provee un muestreo unitario
(nos haremos cargo del factor de K pronto). Si evaluamos el espectro en menos frecuencias que lo que dura
la seal, el termino correspondiente a m = n+K aparecer para algunos valores de m, n {0, . . . , N 1}.
Esta situacin signica que nuestra transformada prototipo iguala s (n) + s (n + K) para cualquier valor de
n. La nica manera de eliminar este problema es el requerir K N : tenemos que tener mas muestreos
de frecuencia que lo que dura la seal. De esta manera, podemos regresar del dominio de frecuencia al cual
entramos por la DFT.

Exercise 11.3.1 (Solution on p. 219.)


Cuando tenemos menos muestreos de frecuencia que lo que dura la seal, algunos valores de la
seal discreta igualan la suma de los valore de la seal original. Dada la interpretacin del muestreo
para el espectro, caracterice este efecto de una manera diferente.

Otra manera para entender este requerimiento es el usar de teora de ecuaciones lineares. Si escribimos la
expresin para el DFT como un conjunto de ecuaciones lineares,

s (0) + s (1) + + s (N 1) = S (0) (11.10)

2 2(N 1)
s (0) + s (1) e(j) K + + s (N 1) e(j) K = S (1)

.
.
.

2(K1) 2(N 1)(K1)


s (0) + s (1) e(j) K + + s (N 1) e(j) K = S (K 1)
tenemos K. Obtenemos ecuaciones en N desconocidos si queremos encontrar la seal de su espectro
muestreado. Este requerimiento es imposible de encontrar si K < N; tenemos que tener K N. Nuestra de
relacin de ortogonalidad esencialmente dice que si tenemos el suciente numero de ecuaciones (muestreos
de frecuencia), el conjunto de ecuaciones que resultan de esto se pueden resolver.

Available for free at Connexions <http://cnx.org/content/col10373/1.2>


CHAPTER 11. TRANSFORMADA DE FOURIER DISCRETA EN EL
212
TIEMPO (DTFT)

Por convencin, el numero de valores para las frecuencias del DFT K es elegido para igualar la duracin
de la seal N. El par para la transformada discreta de Fourier consiste de

Par de la Transformada Discreta de Fourier

N 1
s (n) e(j )
X 2nk
S (k) = N (11.11)
n=0

N 1
1 X 2nk
s (n) = S (k) ej N (11.12)
N
k=0

Example 11.1
Use esta demostracin para hacer un anlisis del DFT para la seal.

This is an unsupported media type. To view, please see


http://cnx.org/content/m12844/latest/DFT_Analysis.llb

Example 11.2
Use esta demostracin para sintetizar una seal de una secuencia DFT

This is an unsupported media type. To view, please see


http://cnx.org/content/m12844/latest/DFT_Component_Manipulation.llb

11.4 Transformada de Fourier Discreta en el Tiempo (DTFT)6


Transformada de Fourier Discreta en el Tiempo


X
X () = x (n) e(jn) (11.13)
n=

Transformada Inversa de Fourier Discreta en el Tiempo


Z 2
1
x (n) = X () ejn d (11.14)
2 0

6 This content is available online at <http://cnx.org/content/m12861/1.1/>.

Available for free at Connexions <http://cnx.org/content/col10373/1.2>


213

11.4.1 Espacios Relevantes


La transformada de Fourier en tiempo discreto
7
graca seales discretas en el tiempo innita en l2 , a seales
2
de frecuencia continua con tamao nito (o peridicas) en L .

Figure 11.2: Graca l2 (Z) en el dominio del tiempo para L2 ([0, 2)) en el dominio de la frecuencia.

7 "Discrete-Time Fourier Transform (DTFT)" <http://cnx.org/content/m10247/latest/>

Available for free at Connexions <http://cnx.org/content/col10373/1.2>


CHAPTER 11. TRANSFORMADA DE FOURIER DISCRETA EN EL
214
TIEMPO (DTFT)

11.5 Propiedades de la Transformada de Fourier Discreta en el


Tiempo8

Propiedades de la Transformada de Fourier Discreta en el Tiempo

Dominio de la Secuencia Dominio de la Frecuencia


 
Linearidad a1 s1 (n) + a2 s2 (n) a1 S1 ej2f + a2 S2 ej2f

S ej2f = S e(j2f )

Simetra del Conjugado s (n) real

S ej2f = S e(j2f )
 
Simetra Par s (n) = s (n)
S ej2f = S e(j2f )
 
Simetra Impar s (n) = s (n)
e(j2f n0 ) S ej2f

Retrazo del Tiempo s (n n0 )
S ej2(f f0 )

Modulacion Compleja ej2f0 n s (n)
S (ej2(f f0 ) )+S (ej2(f +f0 ) )
Modulacin de Amplitud s (n) cos (2f0 n) 2
S (ej2(f f0 ) )S (ej2(f +f0 ) )
s (n) sin (2f0 n) 2j
1 dS (ej2f )
Multiplicacin por n ns (n) (2j) df
P j20

Suma n= s (n) S e
R 21 
Valor en el Origen s (0) 21
S ej2f df
P 1  2
2
|S ej2f | df
R
Teorema de Parseval n= (|s (n) |) 2
21

Figure 11.3: Propiedades de la Transformada de Fourier Discreta en el Tiempo y sus relaciones.

11.6 Par de la Transformada de Fourier Discreta en el Tiempo9


Cuando obtenemos una seal discreta a muestrear una seal anloga, la frecuencia Nyquist corresponde a la
1 1
frecuencia discreta
2 . Para demostrar esto, note que un senosoidal en la frecuencia Nyquist 2Ts tiene una
forma muestreada que iguala

Senosoidal en la Frecuencia Nyquist de 1/2T


 
1
cos 2 2Ts nTs = cos (n)
(11.15)
n
= (1)
(j2n) n
1
2 igual e
El exponencial en la DTFT en la frecuencia 2 = e(jn) = (1) , lo que signica que la
correspondencia entre una frecuencia anloga y una frecuencia discreta es establecida:

Relacin para la Frecuencia Discreta en el Tiempo Anlogo

fD = fA Ts (11.16)

8 This content is available online at <http://cnx.org/content/m12846/1.1/>.


9 This content is available online at <http://cnx.org/content/m12845/1.1/>.

Available for free at Connexions <http://cnx.org/content/col10373/1.2>


215

onde fD y fA representan las variables de frecuencia anloga y frecuencia discreta, respectivamente. La


10
gura de aliasing provee otra manera para derivar este resultado. Conforme la duracin de cada punto en
la seal de muestreo peridica pTs (t) se hace mas pequea, las amplitudes de las repeticiones del espectro
11
de la seal, que son gobernadas por los coecientes de la series de Fourier de pTs (t) , se vuelve iguales.
1 1
As, el espectro muestrario de la seal se convierte peridico con periodo . As la frecuencia Nyquist
Ts 2Ts
1
corresponde a la frecuencia
2 .
La transformada inversa de Fourier discreta en el tiempo se deriva fcilmente en la siguiente relacin:

1 1 m=n
Z 2 if
e(j2f m) ejf n df = (11.17)
12 0 if m 6= n

As como encontramos que

1 1
s (m) e(j2f m) ej2f n df

S ej2f ej2f n df
R R P
2
21
= 2
12 mm
P R 21 ((j2f ))(mn)
= (11.18)
mm s (m) 1 e 2
df
= s (n)

Los pares para la transformada de Fourier discretos en el tiempo son

Pares de la Transformada de Fourier en Tiempo Discreto

 X
S ej2f = s (n) e(j2f n) (11.19)
nn

Pares de la Transformada de Fourier en Tiempo Discreto


Z 1
2
S ej2f ej2f n df

s (n) = (11.20)
21

11.7 Ejemplos de DTFT12


Example 11.3
Calculemos la transformada de Fourier en tiempo discreto para la secuencia del exponencial deca-
dente s (n) = an u (n) , donde u (n) la secuencia del Escaln unitario. Al remplazar la expresin de
la seal la formula de la transformada de Fourier.

Formula de la Transformada de Fourier


P
an u (n) e(j2f n)

S ej2f = n=
P n (11.21)
= n=0 ae(j2f )

La suma es un caso especial de series geomtricas.


10 "The Sampling Theorem", Figure 2: aliasing <http://cnx.org/content/m0050/latest/#alias>
11 Examinar la seal de pulso periodica revela que cuando se hace pequea, el valor de c , el coeciente mas grande de
0
Fourier, decae al valor cero: |c0 | = A T
. As, para mantener un teorema de muestreo viable en trminos matemticos, la
amplitud A debe incrementar a 1
, convirtindose innitamente grande conformela duracin del pulso disminuye. Sistemas
prcticos usan un valor pequeo de , digamos 0.1Ts y usan amplicadores para reescalar la seal.
12 This content is available online at <http://cnx.org/content/m12847/1.1/>.

Available for free at Connexions <http://cnx.org/content/col10373/1.2>


CHAPTER 11. TRANSFORMADA DE FOURIER DISCRETA EN EL
216
TIEMPO (DTFT)

Series Geomtricas

X 1
n = , || < 1 (11.22)
n=0
1

As, por mientras que |a| < 1 , tenemos nuestra transformada de Fourier.

1
S ej2f =

(11.23)
1 ae(j2f )

Usando la relacin de Euler, podemos expresar la magnitud y el ngulo de este espectro.

1
|S ej2f | = q

(11.24)
2
(1 acos (2f )) + a2 sin2 (2f )
 
asin (2f )
S ej2f

= arctan (11.25)
1 acos (2f )

No importa que valor de a escojamos, las formulas anteriores demuestran claramente la natu-
raleza peridica del espectro de seales discretas en el tiempo. Figure 11.4 muestra como el espectro
es un funcin peridica. Tan solo tenemos que considerar el espectro entre
1
21
2 para denirla y
unambiguosamente. Cuando a > 0 , tenemos un espectro de pasa bajas  el espectro desaparece
1
cuando la frecuencia incrementa de 0 a
2  con una a incrementa nos lleva a un contenido mayor
de frecuencias bajas; para a<0 ,tenemos un espectro de pasa altas. (Figure 11.5).

Figure 11.4: El espectro de la seal exponencial (a = 0.5) es mostrado sobre el rango de frecuencias
[2, 2], claramente demostrando la periodicidad de toda la espectra discreta en el tiempo. EL ngulo
tiene las unidades en grados.

Available for free at Connexions <http://cnx.org/content/col10373/1.2>


217

Figure 11.5: El espectro de varias seales exponenciales es mostrado aqu . Cual es la relacin
aparente entre el espectro de a = 0.5 y a = 0.5 ?

Example 11.4
Anlogo a una seal de pulso anlogo encontramos el espectro de la secuencia de pulso de tamao-
N pulse sequence.

1 if 0nN 1
s (n) = (11.26)
0 otherwise

La transformada de Fourier de esta secuencia tiene la forma de una serie geomtrica truncada.

N
X 1
j2f
e(j2f n)

S e = (11.27)
n=0

Para las llamadas series geomtricas nitas, sabemos que

Series Geomtricas Finitas


N +n 0 1
X 1 N
n = n0 (11.28)
n=n0
1

para todos los valores de .

Exercise 11.7.1 (Solution on p. 219.)


Derive esta formula para la formula de series geomtrica nitas. El truco es el considerar la
diferencia entre la suma de las series y la suma de las series multiplicada por .

Aplicando este resultado da (Figure 11.6.)

1e(j2f N )

S ej2f = 1e(j2f )
(11.29)
= e((jf ))(N 1) sin(f N)
sin(f )

Available for free at Connexions <http://cnx.org/content/col10373/1.2>


CHAPTER 11. TRANSFORMADA DE FOURIER DISCRETA EN EL
218
TIEMPO (DTFT)

sin(N x)
El radio de las funciones de seno tiene la forma genrica de
sin(x) , que es mejor conocida como la
funcin de sinc discreta , dsinc (x) . Por lo tanto, nuestra transformada puede ser expresada como
S ej2f = e((jf ))(N 1) dsinc (f ) . El espectro del pulso discreto contiene muchas ondulaciones, las
cuales el numero incrementa con N , la duracin del pulso.

Figure 11.6: El espectro para un pulse de tamao-diez es mostrado aqu. Pued usted explicar la
apariencia complicada que el ngulo tiene?

Available for free at Connexions <http://cnx.org/content/col10373/1.2>


219

Solutions to Exercises in Chapter 11


Solution to Exercise 11.3.1 (p. 211)
Esta situacin nos lleva a aliasing en el dominio del tiempo.
Solution to Exercise 11.7.1 (p. 217)

N +n
X 0 1 N +n
X 0 1

n n = N +n0 n0 (11.30)
n=n0 n=n0

la cual, despus de algunas manipulaciones, da la formula de la suma geomtrica.

Available for free at Connexions <http://cnx.org/content/col10373/1.2>


CHAPTER 11. TRANSFORMADA DE FOURIER DISCRETA EN EL
220
TIEMPO (DTFT)

Available for free at Connexions <http://cnx.org/content/col10373/1.2>


Chapter 12

Transformada de Fourier de Tiempo


Continuo (CTFT)

12.1 Transformada de Fourier de Tiempo Continuo (CTFT)1


12.1.1 Introduccin
2
Debido al gran nmero de seales de tiempo-continuo que estan presentes en las series de Fourier nos da una
primera ojeada de cuantas maneras podemos representar algunas de estas seales de manera general: como
una superposicin de un nmero de seales senosoidales. Ahora podemos ver la manera de representar seales
noperiodicas de tiempo continuo usando la misma idea de superposicin. A continuacin presentaremos la
Transformada de Fourier de Tiempo-Continuo (CTFT), tambin conocida solo como Transformada
de Fourier (FT). Por que la CTFT ahora trataremos con seales no periodicas, encontraremos una manera
de incluir todaslas frecuencias en ecuaciones en general.

12.1.1.1 Ecuaciones

Transformada de Fourier de Tiempo-Continuo


Z
F () = f (t) e(jt) dt (12.1)

Inversa de la CTFT Z
1
f (t) = F () ejt d (12.2)
2

note: No se confunda con la notacin - es comn ver la formula anterior escrita un poco diferente.
Una de las diferencias ms comunes echa por los profesores es la forma de escribir el exponente.
Arriba escribimos la variable de la frecuancia readial en el exponencial, donde = 2f , pero
tambin vemos que los profesores incluyen la expresin ms explicicta, j2f t, en el exponencial.
3
Vase aqui para una descripcin de la notacin utilizada en los modulos de Procesamiento Digital
de Seales DSP.
1 This content is available online at <http://cnx.org/content/m12955/1.1/>.
2 "Classic Fourier Series" <http://cnx.org/content/m0039/latest/>
3 "DSP notation" <http://cnx.org/content/m10161/latest/>

Available for free at Connexions <http://cnx.org/content/col10373/1.2>

221
CHAPTER 12. TRANSFORMADA DE FOURIER DE TIEMPO CONTINUO
222
(CTFT)

La ecuacuin anterior para las CFT y su inversa vienen directamente de las series de Fourier y de nuesro
entendimiento de sus coecientes. Para la CTFT simplemente utilizamos la intergracin en lugar de la
simulacin para ser capaces de expresar las seales peridicas. Esto debera tener sentido ya que simlemente
estamos extendiendo las ideas de las series de Fourier para las CTFT para incluir las seales no-peridicas,y
4
as todo el espectro de la frecuencia. Vase la Derivacin de la Transformada de Fourier para una mirada
ms profunda del tema.

12.1.2 Espacios Relevantes


El mapeo de la Transformada de Fourier de Tiempo-Continuo de longitud-innita, en seales de tiempo-
continuo L2 a longitud-innita,seales de frecuancia-continua en L2 . Revisando el Anlisis de Fourier (Sec-
tion 8.1) para una descripcin de todos los espacios usados en el anlisis de Fourier.

Figure 12.1: Mapeando L2 (R) en el dominio del tiempo a L2 (R) en el dominio de frecuencia.

Para ms informacin de las caracteristicas de la CTFT, por favor vase el modulo de las Propiedades
5
de la Transformada de Fourier .

12.1.3 Problemas de Ejemplo


Exercise 12.1.1 (Solution on p. 226.)
Encontrar la Transformada de Fourier(CTFT) de la funcin


e(t) if t 0
f (t) = (12.3)
0 otherwise

Exercise 12.1.2 (Solution on p. 226.)


Encontrar la inversa de la Transformada de Fourier de la onda cuadrada denda como:

1 if || M
X () = (12.4)
0 otherwise

4 "Derivation of the Fourier Transform" <http://cnx.org/content/m0046/latest/>


5 "Properties of the CTFT" <http://cnx.org/content/m10100/latest/>

Available for free at Connexions <http://cnx.org/content/col10373/1.2>


223

12.2 Propiedades de la Transformada de Fourier de Tiempo-


Continuo6
En este modulo veremos algunas de las propiedades bsicas de la Transformada de Fourier de Tiempo-
7
Continuo (CTFT). La primera seccin contiene una tabla que ilustra las propiedades, y la siguiente seccin
discute unas de las propiedades mas interesantes ms a fondo. En la tabl, oprima en el nombre de la
operacin para ver la explicacin se se encuentra ms adelante. Vase este modulo (Section 5.7) para una
tabla expandida de las propiedades de Fourier.

note: Discutiremos estas propiedades para seales aperiodicas de tiempo-continuo pero entender-
emos que propiedades similares se matienen para seales de tiempo-continuo y seales peridicas.

12.2.1 Tabla de Propiedades de CTFT

Nombre de la Operacin Seal ( f (t) ) Transformada ( F () )


Adicin (Section 12.2.2.1: Lineal- f1 (t) + f2 (t) F1 () + F2 ()
idad)

Multiplicacin Escalar (Sec- f (t) F (t)


tion 12.2.2.1: Linealidad)

Simetra (Section 12.2.2.2: F (t) 2f ()


Simetra)
1

Escalamiento en el Tiempo (Sec- f (t) || F
tion 12.2.2.3: Escalamiento en el
Tiempo)

Desplazamiento en el Tiempo f (t ) F () e(j )


(Section 12.2.2.4: Desplaza-
miento en el tiempo)

Modulacin (Desplazamiento de f (t) ejt F ( )


Frecuencias) (Section 12.2.2.5:
Modulaci(Desplazo de la Fre-
cuencia))

Convolucin en el Tiempo (Sec- (f1 (t) , f2 (t)) F1 (t) F2 (t)


tion 12.2.2.6: Convolucin)
1
Convolucin en la Frecuencia f1 (t) f2 (t) 2 (F1 (t) , F2 (t))
(Section 12.2.2.6: Convolucin)
dn n
Diferenciacin (Section 12.2.2.7:
dtn f (t) (j) F ()
Time Diferenciacin)

Table 12.1

12.2.2 Discusin de las Propiedades de la Transformada de Fourier


Despus de haber visto la tabla anterior y tener un sentimiento de las proiedades de la CTFT, ahora nos
tomaremos un poc ms de tiempo para discutir de las propiedades ms importantes y tiles.

6 This content is available online at <http://cnx.org/content/m12957/1.2/>.


7 "Continuous Time Fourier Transform (CTFT)" <http://cnx.org/content/m10098/latest/>

Available for free at Connexions <http://cnx.org/content/col10373/1.2>


CHAPTER 12. TRANSFORMADA DE FOURIER DE TIEMPO CONTINUO
224
(CTFT)

12.2.2.1 Linealidad

La combinacin de las propiedades de la adicin y de la multiplicacin escalar de la tabla anterior demuestran


la propiedad bsica de linealidada. Lo que debe de ver es que si uno toma la Transformada de Fourier de una
combinacin lineal de seales entonces esta ser la misma que la combinacin lineal de la transformada de
Fourier de cada seal individual. Esto es crucial cuando usamos la tabla (Section 11.1) de las transformadas
para encontrar la transformada de una seal ms complicada.

Example 12.1
Empezaremos con la siguiente seal:

z (t) = f1 (t) + f2 (t) (12.5)

Ahora, despus de tomar la transformada de Fourier, mostrada en la siguiente ecuacin, notemos


que la combinacin lineal de los trminos no es afectada por la transformada.

Z () = F1 () + F2 () (12.6)

12.2.2.2 Simetra

La simetra es una propiedad que nos puede hacer la vida ms fcil resolviendo problemas que involucran
la transformada de Fourier. Basicamente ;p que dice esta propiedad es que ya que la funcin rectangular
en el tiempo es una funcin sinc en la frecuancia, entonces una funcin sinc en el tiempo ser una funcin
rectangular en la frecuencia. Este es un resultado directo de las similaridades entre la CTFT y la inversa de
la CTFT. La nica diferencia es que es escalda por 2 y una revocacin de la frecuancia.

12.2.2.3 Escalamiento en el Tiempo

Esta propiedad trata con el efecto de la representacin del dominio de frecuancia de una seal si la variable
tiempo es alterada. El concepto ms importante par entender para la propiedad de escalammiento es que
las seales que son estrechas en el tiempo son amplias en la frecuancia y vice versa. El ejemplo ms sencillo
8
de esto es la funcin delta, un pulso unitario con una muy pequea duracin, en el tiempo que se convierte
en funcin constante de longitud-innita en frecuencia.
La tabla anterior muestra esta idea para una transformacin general del dominio-tiempo de la seal.
Usted debera de ser capaz de notar que esta ecuacin muestra la relacin mencionada anteriormente: si la
variable tiempo incrementa entonces el rango de la frecuencia sera decreciente.

12.2.2.4 Desplazamiento en el tiempo

El desplazamiento en el tiempo muestra que un desplazo en el tiempo es equivalente a un desplazo de fase


lineal en la frecuencia. Ya que el contenido de la frecuencia depende solamente de la forma de la seal, el
cual es invariabe en el desplazo en el tiempo, entonces solamente la fase del espectro ser alterada. Esta
propiedad ser provada facilmente usando la Transformada de Fourier, asi que mostraremos los pasos bsicos
a continuacin:

Example 12.2
Primero empezaremos dejando que z (t) = f (t ). Ahora tomemos la transformada de Fourier
con la expresin anterior sustituida para z (t).
Z
Z () = f (t ) e(jt) dt (12.7)

8 "Elemental Signals": Section Pulse <http://cnx.org/content/m0004/latest/#pulsedef>

Available for free at Connexions <http://cnx.org/content/col10373/1.2>


225

Ahora hagamos un pequeo cambio de variables, donde = t . A travs de la calculacin


antarior, podemos ver que solamente la variable en el exponencial es alterada solo cambiando la
fase en el dominio de la frecuencia.

R
Z () =
f () e(j(+ )t) d
(j )
R
= e
f () e(j) d (12.8)

(j )
= e F ()

12.2.2.5 Modulaci(Desplazo de la Frecuencia)

La modulacin es absolutamente imprescindible para las aplicaciones de comunicaiones. Siendo capaces de


desplazar una seal a diferentes frecuencias, nos que mas que tomar ventaja de diferentes partes de los
espectros del electromagnetismo es lo que nos permite transmitir la televisin, radio y otroas aplicaciones a
travs del mismo espacio sin interferencia signicativa.
La demostracin de la propiedad del desplzamiento de la frecuencia es muy similar a la de desplazamiento
en el tiempo (Section 12.2.2.4: Desplazamiento en el tiempo); Sin embargo, aqui usaremos la transformada
inversa de Fourier. Ya que vamos a travs de los pasos anteriores, la demostracin de desplazamiento ene l
tiempo, a continuacin solo mostrara los pasos iniciales y nales de esta demostracin:
Z
1
z (t) = F ( ) ejt d (12.9)
2

Ahora simplemente reducimos esta ecuacin por medio de un cambio de variable y simplicando los trminos.
Despus probaremos la propiedad expresada en la tabla anterior:

z (t) = f (t) ejt (12.10)

12.2.2.6 Convolucin

Convolucin es una de las grandes razones para convertir seales en dominios de frecuancia ya que la
convolucin en el tiempo se convierte en multiplicacin en frecuencia. Esta propiedad es tambin otro
buen ejemplo de la simetria entre el tiempo y la frecuencia. Tambin muestra que hay muy poca ganancia
cambiando el dominio de frecuancia cuando la multiplicacin en el tiempo esta involucrada.
Introduciremos la integral de convolucin, pero si no la ha visto anteriormente o necesita refrescar la
memoria vase el modulo de convolucin de tiempo-continuo (Section 3.2) para una explicacin mas profunda
y su derivacin.

y (t) = (f1 (t) , f2 (t))


R (12.11)
= 1
f ( ) f2 (t ) d

12.2.2.7 Time Diferenciacin

Ya que los sistemas LTI (Section 2.1) pueden ser representados en trminos de ecuaciones diferenciales, es
evidente que con esta propiedad que conviertiendo al dominio de frecuencia nos permitir convertir esta
complicada ecuacin diferencial a una ecuacin ms sencilla que involucre multiplicacin y adicin. Esto
9
tambin es visto con mas detalle durante el estudi de la Transformada de Laplace .

9 "The Laplace Transform" <http://cnx.org/content/m10110/latest/>

Available for free at Connexions <http://cnx.org/content/col10373/1.2>


CHAPTER 12. TRANSFORMADA DE FOURIER DE TIEMPO CONTINUO
226
(CTFT)

Solutions to Exercises in Chapter 12


Solution to Exercise 12.1.1 (p. 222)
Para poder calcular la transformada de Fourier, todo lo que necesitamos es usar los (12.1) (Transformada
10
de Fourier de Tiempo-Continuo), expnenciales complejos , y clculos bsicos.

R
F () =
f (t) e(jt) dt
R
= 0
e(t) e(jt) dt
R (t)(+j) (12.12)
= 0
e dt
1
= 0 +j

1
F () = (12.13)
+ j

Solution to Exercise 12.1.2 (p. 222)


Aqui usaremos la (12.2) (Inversa de la CTFT) para encontrar la inversa de la FT, dado eso t 6= 0.

1
R M j(,t)
x (t) = 2 M e d
1 j(,t)
= 2 e |,=ejw (12.14)

1
= t sin (M t)
 
M Mt
x (t) = sinc (12.15)

10 "Continuous Time Complex Exponential" <http://cnx.org/content/m10060/latest/>

Available for free at Connexions <http://cnx.org/content/col10373/1.2>


Chapter 13

Teorema de Muestreo

13.1 Muestreo1
13.1.1 Introduccin
Una computadora puede procesar seales de tiempo discreto usando un lagoritmo extremandamente
exible y poderoso. Mas sin embargo la mayoria de las seales de interes son de tiempo continuo, que es
como casi siempre aparecen al natural.
Este modulo introduce la idea de trasladar los problemas de tiempo continuo en unos de tiempo discreto,
y podra leer ms de los detalles de la importancia de el muestreo.
Preguntas clave

Cmo pasamos de una seal de tiempo continuo a una seal de tiempo discreto (muestreo, A/D)?
Cundo podemos reconstruir (Section 13.2) una seal CT exacta de sus muestras (reconstruccin,
D/A)?
Manipular la seal DT es lo que reconstruir la seal?

13.1.2 Muestreo
Muestreo (y reconstruccin) son los mejores entendimiento en dominio de frecuencia. Empezaremos viendo
algunos ejemplos:

Exercise 13.1.1 (Solution on p. 248.)


Qu seal CT f (t) tiene la CTFT (Section 12.1) mostrada anterirormente?

Z
1
f (t) = F (jw) ejwt dw
2
1 This content is available online at <http://cnx.org/content/m12964/1.3/>.

Available for free at Connexions <http://cnx.org/content/col10373/1.2>

227
228 CHAPTER 13. TEOREMA DE MUESTREO

Figure 13.1: La (Transformada de Fourier de Tiempo Continuo)CTFT de f (t).

Pista: F (jw) = F1 (jw) F2 (jw) donde las dos partes de F (jw) son:

(a) (b)

Figure 13.2

Exercise 13.1.2 (Solution on p. 248.)


Qu seal DT fs [n] tiene la DTFT (Section 11.4) mostrada anteriormente?

Z
1
fs ejw ejwn dw

fs [n] =
2

Figure 13.3: DTFT que es priodica (con period = 2 ) versin de F (jw) en la Figure 13.1.

Available for free at Connexions <http://cnx.org/content/col10373/1.2>


229

Figure 13.4: f (t) es la seal de tiempo-continuo anterior y fs [n] es la versin muestreada de tiempo-
discreto de f (t)

13.1.2.1 Generalizacin

Por supuesto, que los resultados de los ejemplos anteriores pueden ser generalizados a cualquier f (t) con
F (jw) = 0, |w| > , donde f (t) es limitado en banda a [, ].

(a) (b)

Figure 13.5: F (jw) es la CTFT de f (t).

Available for free at Connexions <http://cnx.org/content/col10373/1.2>


230 CHAPTER 13. TEOREMA DE MUESTREO

(a) (b)

Fs ejw es la DTFT de fs [n].


`
Figure 13.6:

 
Fs ejw es perodico (Section 6.2) (con perodo 2 ) versin de F (jw). Fs ejw es la DTFT de muestreo
de seal en los enteros. F (jw) es la CTFT de seal.

note: Si f (t) es limitado en banda para [, ] entonces la DTFT de la versin muestreada

fs [n] = f (n)

es solo peridica (con perodo 2 ) versin de F (jw).

13.1.3 Cambiando una Seal Discreta en una Seal Continua


Ahora veamos como cambiar una seal DT en una seal continua en el tiempo. Sea
 fs [n] una seal DT con
DTFT Fs ejw

(a) (b)

Fs ejw es la DTFT de fs [n].


`
Figure 13.7:

Ahora, sea

X
fimp (t) = fs [n] (t n)
n=

La seal CT, fimp (t), es no-cero solo en los enteros donde hay implulsos de altura fs [n].

Available for free at Connexions <http://cnx.org/content/col10373/1.2>


231

Figure 13.8

Exercise 13.1.3 (Solution on p. 248.)


Cal es la CTFT de fimp (t)?
Ahora, dadas las muestras fs [n] de un limitado en banda para la seal [, ], nuestro siguiente paso es
ver como podemos reconstruir (Section 13.2) f (t).

Figure 13.9: Diagrama de bloque mostrando cada paso bsico usado para reconstruir f (t). Podemos
hacer nuestro resultado igual a f (t) exactamente?

13.2 Reconstruccin2
13.2.1 Introduction
El proceso de reconstruccin empieza tomando una seal muestreada, que estar en tiempo discreto, y
haciendo unas operaciones para poder convertirla en tiempo-continuo y con algo de suerte en una copia de
la seal original. Un mtodo bsico usado para reconstruir una seal limitada en banda de [, ] de sus
muestras en los enteros es hacer los siguientes pasos:

cambiar muestra de la secuencia fs [n] en un tren de impulso fimp (t)



ltro pasa bajas fimp (t) para obtener la reconstruccin f (t) ( freq. = )
2 This content is available online at <http://cnx.org/content/m12969/1.2/>.

Available for free at Connexions <http://cnx.org/content/col10373/1.2>


232 CHAPTER 13. TEOREMA DE MUESTREO

Figure 13.10: Diagrama de bloque de reconstruccin con el ltro pasa bajas (lowpass lter (LPF)).

La respuesta al impulso del ltro pasa bajas es g (t). La siguiente ecuacin nos permite reconstruir nuestra

seal (Figure 13.11), f (t).

f (t) = g (t) fimp (t)
P
= g (t) n= fs [n] (t n)

= f (t) (13.1)
P
= n= fs [n] (g (t) (t n))
P
= n= fs [n] g (t n)

Figure 13.11

13.2.1.1 Ejemplos de Filtros g

Example 13.1: Filtros de Orden Cero


Este tipo de "ltro" es uno de los ms bsicos en los ltros de reconstruccin. Este simplemente
mantiene el valor que esta en el fs [n] para segundos. Esto crea un bloque o pasos como funcin
donde cada valor del pulso en fs [n] es simplemente arrastrado al siguiente pulso. La siguiente
ecuacin y la ilustracin (Figure 13.12) representan como el ltro de reconstruccin funciona con

Available for free at Connexions <http://cnx.org/content/col10373/1.2>


233

la siguiente g:
1 if 0<t<
g (t) =
0 otherwise


X
fs [n] = fs [n] g (t n) (13.2)
n=

(a) (b)

Figure 13.12: Mantiene el Orden Cero


note: Cmo es que f (t) reconstruida con orden cero se compara con la original f (t) en el
domionio de frecuencia?

Example 13.2: Orden N-esimo


Aqu veremos algunos ejemplos rpidos de la varianza del ltro de orden cero visto en el ejemplo
anterior.

Available for free at Connexions <http://cnx.org/content/col10373/1.2>


234 CHAPTER 13. TEOREMA DE MUESTREO

(a)

(b)

(c)

Figure 13.13: Ejemplo de N-esimo Orden (n-esimo orden es igual a un B-spline de n-esimo orden) (a)
Primer Orden (b) Segundo Orden (c) Orden

13.2.2 ltimo Filtro de Reconstruccin


note: Cal es el ltimo ltro de reconstruccin?

Recordando que ( Figure 13.14)

Available for free at Connexions <http://cnx.org/content/col10373/1.2>


235

Figure 13.14: Diagrama de bloque de nuestra reconstruccin. Notese que cada una de estas seales
tiene su propia CTFT o DTFT correspondiente.

Si G (j) tiene la siguiente forma (Figure 13.15):

Figure 13.15: Filtro pasa baja ideal

entonces

f (t) = f (t)
Por lo tanto,un ltro pasa baja ideal nos dara una reconstruccin perfecta!
En el dominio en el tiempo, la respuesta al impulso

sin (t)
g (t) = (13.3)
t
P
f (t) = n= fs [n] g (t n)
P
= n= fs [n] sin((tn))
(tn)
(13.4)

= f (t)

13.2.3 Conclusiones sorprendentes


Si f (t) es limitado en banda a [, ], puede ser reconstruido perfectamente de su muestra en lo enteros
fs [n] = f (t) |t=n

X sin ( (t n))
f (t) = fs [n] (13.5)
n=
(t n)

Available for free at Connexions <http://cnx.org/content/col10373/1.2>


236 CHAPTER 13. TEOREMA DE MUESTREO

La ecuacin anterior para una reconstruccin perfecta merece una mirada ms cercana (Section 13.3),
que se ver en la siguiente seccin para un mejor entendimiento de la recostruccin. Aqu estan algunas
cosas para empezar a pensar en ellas por ahorita:

sin((tn))
Que
(tn) iguala a los enteros diferentes a n?
sin((tn))
Cal es el soporte de
(tn) ?

13.3 Ms sobre Reconstruccin Perfecta3


13.3.1 Introduccin
En el modulo previo en reconstruccin (Section 13.2), dimos una introduccin de como trabaja la recon-
struccin y temporalemte derivamos una ecuacin usada para realizar una perfecta reconstruccin. Ahora
tomemos un vistazo ms cercano a la formula de la reconstruccin perfecta:


X sin ( (t n))
f (t) = fs (13.6)
n=
(t n)

Escribiremos f (t) en trminos de las funciones sinc desplazadas y escaladas

 
sin ( (t n))
(t n) nZ

es una base (Section 5.1.3: Bases) para el espacio de seales limitadas en bada [, ] . Pero espere . . . .

13.3.1.1 Formulas de la Derivada de Reconstruccin

Que es
sin ( (t n)) sin ( (t k))
< , >=? (13.7)
(t n) (t k)
Este producto interno (Section 7.3) puede ser difcil de calcular en el dominio del tiempo, asi que usaremos
el Teorema de Plancharel (Section 7.12)

Z
1
< , >= e(jn) ejk d (13.8)
2

3 This content is available online at <http://cnx.org/content/m12970/1.2/>.

Available for free at Connexions <http://cnx.org/content/col10373/1.2>


237

(a)

(b)

Figure 13.16

si n=k R
< sincn , sinck > = 1
2
e(jn) ejk d
(13.9)
= 1
si n 6= k
1
R (jn) jn
< sincn , sinck > = 2 e e d
1
R j(kn)
= 2 e d
(13.10)
1 sin((kn))
= 2 j(kn)
= 0

note: En la (13.10) usamos el echo de que la integral de la senosoidal en un intervalo completo


es 0 para simplicar nuestra ecuacin.

As,
sin ( (t n)) sin ( (t k)) 1 if n=k
< , >= (13.11)
(t n) (t k) 0 if n 6= k

Por lo tanto  
sin ( (t n))
(t n) nZ

es una base ortonormal (Section 7.7.3: Base Ortonormal) (ONB) para el espacio de funciones limitadas de
banda de [, ] .

note: Muestreo es lo mismo que calcular los coecientes de ONB, que es el producto interno con
sincs

13.3.1.2 Resumen

Una ltima vez para f (t) [, ] limitado en banda

Available for free at Connexions <http://cnx.org/content/col10373/1.2>


238 CHAPTER 13. TEOREMA DE MUESTREO

Sntesis

X sin ( (t n))
f (t) = fs [n] (13.12)
n=
(t n)

Anlisis
fs [n] = f (t) |t=n (13.13)

Para poder entender un poco ms sobre como podemos reconstruir una seal exacatamente, ser til
4
examinar la relacin entre las transformadas de Fourier (CTFT y DTFT) a ms profundidad.

13.4 Teorema de Nyquist5


13.4.1 Introduccin
Anteriormente habia estado expuesto a los conceptos detras del muestreo (Section 13.1) y el teorema de
muestreo. Mientras aprenda estas ideas, debio haber empezado a notar que si muestreamos a muy bajo valor,
hay una oportunidad que nuestra seal original no sea nicamente denida por nuestra seal muestreada.
Si esto sucede, entonces no es garantia de que recontruyamos (Section 13.2) correctamente la seal. Como
resultado de esto, el Teorema de Nyquist ha sido creado. A continuacin veremos exactamente lo que
este torema nos dice.

13.4.2 Teorema de Nyquist


Sea T igual a nuestro perodo de muestreo (distancia entre las muestras). Despus sea s = 2
T (frecuencia
de muestreo radianes/seg). Hemos visto que si
    f (t)
 es limitado en banda en [B , B ] y muestreamos con
2
perodo T < b s < B (s > 2B ) entonces podemos reconstruir f (t) de sus muestras.

Theorem 13.1: Teorema de Nyquist ("Teorema Fundamental de Procesamiento Digital de Seales


DSP")
Si f (t) es limitado en banda a [B , B ], podemos reconstruirlo perfectamente de sus muestras

fs [n] = f (nT )
2
para s = T > 2B
N = 2B es llamada la "frecuencia Nyquist " para f (t). Para la reconstruccin perfecta de ser
posible
s 2B
donde s es la frecuancia de muestreo y B es la frecuencia ms alta en la seal.

4 "Examing Reconstruction Relations" <http://cnx.org/content/m10799/latest/>


5 This content is available online at <http://cnx.org/content/m12971/1.2/>.

Available for free at Connexions <http://cnx.org/content/col10373/1.2>


239

Figure 13.17: Illustracin de la Frecuencia Nyquist

Example 13.3: Ejemplos:

El odo humano oye frecuencias hasta 20 kHz CD el valor de la muestra es 44.1 kHz.
La linea telefnica pasa frecuencias de hasta 4 kHz la muestra de la compaia de telefonos
es de 8 kHz.

13.4.2.1 Reconstruccin

La formula de la reconstruccin en el dominio del tiempo se ve como



X sin T (t nT )
f (t) = fs [n]
n= T (t nT )

Podemos concluir, desde antes que




sin T (t nT )
, nZ
T (t nT )

es una base (Section 5.1) para el espacio de [B , B ] funciones limitadas en banda, B = T . Los coecientes
2
calculados muestreando f (t) en el valor
de expansin para esta base son
T = 2B .

note: La base tambin es ortogonal. Para hacerla ortonormal (Section 7.8), necesitamos un factor

de normalizacin de T.

13.4.2.2 La gran Pregunta

Exercise 13.4.1 (Solution on p. 248.)


Que pasa si s < 2B ? Qu sucede cuando muestreamos abajo del valor de Nyquist?

Available for free at Connexions <http://cnx.org/content/col10373/1.2>


240 CHAPTER 13. TEOREMA DE MUESTREO

13.5 Aliasing6
13.5.1 Introduccin
Cuando consideramos la reconstruccin (Section 13.2) de una seal, usted ya debe de estar familiarizado con
la idea de el valor de Nyquist. (Section 13.4) Este concepto nos permite encontrar el valor de muestreo que
nos dara una reconstruccin perfecta de nuestra seal. Si nosotros muestreamos en un valor muy bajo (abajo
del valor de Nyquist), entonces surgirn problemas para hacer una reconstruccin perfecta imposible-este
problema es coocido como aliasing (algunos autores traducen este trmino como solapamiento). Aliasing
ocurre cuando hay un traslapo en el desplazamiento, copias priodicas en nuestra seal FT, es decir espectro.
En el dominio de frecuencia, notaremos que parte de la seal se trasladara con la seal siguiente a l. En
este solapamiento los valores de la frecuencia sern sumados juntos y la forma del espectro de la seal ser
indeseablemente alterado. Este solapamiento o aliasing hace posible determinar correctamente la fuerza de
la frecuencia. La Figure 13.18 nos da un ejemplo visual de este fenmeno:

Figure 13.18: El espectro de una seal limitada en banda (a W Hz) es mostrada arriba en la grca.
Si el intervalo muestreado Ts es elegida demasiado gande relativo con el ancho de banda W , el aliasing
ocurrira. En la grca de la parte de abajo, el intervalo muestreado es elegido sucientemente pequeo
para evitar el aliasing. Note que si la seal no fuera limitada en banda, el componente del espectro
siempre sera traslapado.

13.5.2 Aliasing y Muestreo


Si muestreamos demasiado lento, es decir,


s < 2B , T >
B
No podemos recuperar la seal de su muestra debido al aliasing.

6 This content is available online at <http://cnx.org/content/m12973/1.3/>.

Available for free at Connexions <http://cnx.org/content/col10373/1.2>


241

Example 13.4

Sea f1 (t) tiene CTFT.

Figure 13.19: En esta gura, note la siguiente ecuacin: B s


2
=a

Sea f2 (t) tiene CTFT.

Figure 13.20: Las porciones originales de la seal resultan del solapamiento con replicas desplazadas-
mostrando la demostracin visual del aliasing.

Trate de bosquejar y resolver las siguientes ecuaciones por su cuenta:

Qu es lo que hace la DTFT de f1,s [n] = f1 (nT ) ?


Qu es lo que hace la DTFT de f2,s [n] = f2 (nT ) ?
Alguna otra seal tiene la misma DTFT como f1,s [n] y f2,s [n]?

CONCLUSIN: Si muestreamos debajo de la frecuencia de Nyquist, hay muchas seales que pueden producir
la secuencia dada de la muestra.

Available for free at Connexions <http://cnx.org/content/col10373/1.2>


242 CHAPTER 13. TEOREMA DE MUESTREO

Figure 13.21: Todas son iguales

Por qu el trmino "aliasing"? Por que la misma muestra de secuencia puede ser representada por
diferentes seales CT (en comparacin a cuando muestreamos la frecuencia de Nyquist anterior, entonces la
secuencia muestra representa una seal CT nica).

Figure 13.22: Estas dos seales contienen las mismas cuatro muestras, con todo son seales muy
diferentes.

Example 13.5
f (t) = cos (2t)

Available for free at Connexions <http://cnx.org/content/col10373/1.2>


243

Figure 13.23: La gura muestra la funcin coseno, f (t) = cos (2t), y su CTFT.

Caso 1: Muestra s = (8) rad 1


seg T = 4 seg.

note: s > 2B
w 8
 rad
Caso 2: Muestra
s = 3 seg T = 34 seg.
note: s < 2B

Cuando corremos la DTFT del Caso #2 a travs de los pasos de reconstruccin, nos damos
cuenta que terminamos con el siguiente coseno:

 
f (t) = cos t
2
Esta es una versin "estrecha" de la versin original. Claramente nuestro valor de muestra no fue
lo suencitemente alto para asegurar la reconstruccin correcta de las muestras.

Probablementa ya habra visto algunos efectos del aliasing tal como: una rueda del carro que da vuelta al
7
revs en una pelcula occidental. Aqu hay algunas ligas que ilustran dicho efecto, de bajo muestreo y
8
aliasing. Este es un ejemplo de una imagen que tiene artefactos de Moire como resultado de una escaneada
a una frecuencia muy baja.

13.6 Filtros Anti-Aliasing9


13.6.1 Introduction
La idea de aliasing (Section 13.5) fue descrita como el problema que ocurre si una seal es no muestreada
(Section 13.1) en un valor sucientemente grande (por ejemplo, debajo de la Frecuencia de Nyquist (Sec-
tion 13.4)). Pero exactamente qu tipo de distorcin produce el aliasing?

7 http://owers.ofthenight.org/wagonWheel
8 http://www.dvp.co.il/lter/moire.html
9 This content is available online at <http://cnx.org/content/m12974/1.2/>.

Available for free at Connexions <http://cnx.org/content/col10373/1.2>


244 CHAPTER 13. TEOREMA DE MUESTREO

(a)

(b)

Figure 13.24

Frecuencias altas en la seal original "se dobla" en frecuencias bajas.


Frecuencias altas disfrazadas como frecuencias bajas producen artefactos altamente indeseables en la
seal reconstruida.

note: Debemos de evitar el aliasing de la forma que podamos.

13.6.2 Evitando el Aliasing


Cmo si esto es impracticamente/imposible para muestras como s > 2B ?
s
Filtre hacia afuera las frecuencias anteriores
2 antes de que muestree, la mejor forma de visualizarlo
es imaginar los siguientes pasos:

1. Tomar la CTFT (Transformada de Fourier de Tiempo-Continuo) de la seal, f (t).


s
2. Pasar la seal a travs de un ltro pasa bajas con las siguiente especicacin,
2 .c =
s
3. Ahora tenemos una grca de nuestra seal en el dominio de frecuencia con todos los valores de
2 || >
igual a cero. Ahora tomamos la inversa de la CTFT para regresar a nuestra seal de tiempo continuo,
fa (t).
4. Y nalmente estamos listos para muestrear nuestra seal.

Example 13.6
Valor de muestra para CD = 44.1KHz.
Muchos instrumentos musicales, contienen las frecuencias arriba de 22KHz (aun cucando no los
podemos escuchar).
Debido a esto, podemos ltrar la seal de salida del instrumento antes de que la muestremos
usando el siguiente ltro:

Available for free at Connexions <http://cnx.org/content/col10373/1.2>


245

Figure 13.25: Este ltro cortara la frecuencia alta inecesaria, donde |c | > 222kHz

Ahora la seal esta lista para ser muestreada.

Example 13.7: Otro Ejemplo


El discurso del ancho de banda es (20kHz), pero es perfectamente inteligible cuando el ltrado
pasa baja a un rango (4kHz) . Debido a esto podemos tomar una seal de discurso normal y
pasarla a travs del ltro como la mostrada en la Figure 13.25, donde ahora ponemos |c | > 24kHz.
La seal que recibimos de este ltro solamente contiene valores donde || > 8k .
Ahora podemos muestrear en 16k = 8kHz  rango de la telefona estandard.

13.7 Procesamiento de Tiempo Discreto de Seales de Tiempo


Continuo10

Figure 13.26

Cmo esta relacionada la CTFT de y(t) con la CTFT de F(t)?


Sea G (j) = respuesta de la frecuencia del ltro de reconstruccin

Y (j) = G (j) Yimp (j)

donde Yimp (j) es secuencia de impulso creada de ys [n]. As que,

ejT = G (j) H ejT Fs ejT


  
Y (j) = G (j) Ys

 !
jT
 1 X F 2r
Y (j) = G (j) H e F j
T r= T
 
1  X F 2r
Y (j) = G (j) H ejT F j
T r=
T
10 This content is available online at <http://cnx.org/content/m12976/1.2/>.

Available for free at Connexions <http://cnx.org/content/col10373/1.2>


246 CHAPTER 13. TEOREMA DE MUESTREO

   s s 
Ahora asumiremos que f(t) es limitado en banda a T , T = 2 , 2 y G (j) es un ltro perfecto de
recontruccin. Entonces
F (j) H ejT 
if || T
Y (j) =
0 otherwise

note: Y (j) tiene le mismo "limite en banda" como F (j).

Entonces, para seales limitadas en banda, y con un valor de muestra sucientemente alto y un ltro de
reconstruccin perfecto

Figure 13.27

es equivalente a usar un ltro anlogo LTI

Figure 13.28

donde
H ejT  if ||
T
Ha (j) =
0 otherwise

Siendo cuidadosos podemos implementar el sistema LTI para seales limitadas en banda en nuestra propia
computadora.

Figure 13.29

Nota importante:
Ha (j) = ltro inducido por nuestro sistema.

Available for free at Connexions <http://cnx.org/content/col10373/1.2>


247

Figure 13.30

Ha (j) es LTI si y solo si

h, es sistema DT es LTI
F (j), la entrada, es limitada en bada y el valor de la muestra es sucientemente grande.

Available for free at Connexions <http://cnx.org/content/col10373/1.2>


248 CHAPTER 13. TEOREMA DE MUESTREO

Solutions to Exercises in Chapter 13


Solution to Exercise 13.1.1 (p. 227)
Z
1
f (t) = F (jw) ejwt dw
2

Solution to Exercise 13.1.2 (p. 228)


Ya que F (jw) = 0 afuera de [2, 2]
Z 2
1
f (t) = F (jw) ejwt dw
2 2

Tambin , ya que solo utilizamos un intervalo para reconstruir fs [n] de su DTFT, tenemos

Z 2
1
fs ejw ejwn dw

fs [n] =
2 2

jw

Ya que F (jw) = Fs e en [2, 2]
fs [n] = f (t) |t=n
es decir fs [n] es una versin muestreada de f (t).
Solution to Exercise 13.1.3 (p. 231)


X
fimp (t) = fs [n] (t n)
n=
R
F imp (jw) = f
imp
(t) e(jwt) dt
R P (jwt)
= n= fs [n] (t n) e dt
P R (jwt)
n= fs [n] (t n) e
= dt (13.14)
P (jwn)
= n= fs [n] e

= Fs ejw
As que la CTFT de fimp (t) es igual a la DTFT de fs [n]
R
note: Usamos la propiedad de desplazamiento para mostrar

(t n) e(jwt) dt = e(jwn)

Solution to Exercise 13.4.1 (p. 239)


Vayase a travs de los pasos: (vase la Figure 13.31)

Available for free at Connexions <http://cnx.org/content/col10373/1.2>


249

Figure 13.31


Finalmente, Qu le pasara ahora a Fs ej ? Para contestar esta ltima pregunta, necesitamos ver el
concepto de aliasing (Section 13.5).

Available for free at Connexions <http://cnx.org/content/col10373/1.2>


250 CHAPTER 13. TEOREMA DE MUESTREO

Available for free at Connexions <http://cnx.org/content/col10373/1.2>


Chapter 14

Transformada de Laplace y Diseo de


Sistemas

14.1 La Transformada de Laplace1


La transformada de Laplace es una generalizacin de la Transformada de Fourier de Tiempo-Continuo
(Section 12.1). Sin embargo, en lugar de usar funciones senosoidales complejas (Section 8.2) de la forma
ejt , como lo hace la CTFT, la transformada de laplace utiliza una forma ms generalizada, est , donde
s = + j .
Aunque las transformadas de Laplace rara vez se resuelven mediante integracin (si no por medio de
tabla (Section 14.3) y uso de computadoras (por ejemplo Matlab) es ms comun), aqu veremos los pares
bilaterales de la transformada de Laplace . Esto dene la transformada de Laplace y su inversa. Notese
las similitudes entre la transformada de Laplace y su inversa. Esto nos dara como resultado muchas de las
simetrias encontradas en el anlisis de Fourier. (Section 8.1)

Transformada de Laplace Z
F (s) = f (t) e(st) dt (14.1)

Transformada Inversa de Laplace


Z c+j
1
f (t) = F (s) est ds (14.2)
2j cj

14.1.1 Encontrando la Transformada de Laplace y su Inversa


14.1.1.1 Resolviendo la Integral

Probablemente el mtodo ms difcil y menos usado para encontrar la Transformada de Laplace es resolviendo
la integral. Aunque es tcnicamente posible es extremadamente consumidor de tiempo. Dada la facilidad de
los siguientes dos mtodos para encontrarla, no se vera de otra manera. Las integrales estan primordialmente
para entender de donde se originan los siguientes mtodos.

1 This content is available online at <http://cnx.org/content/m12978/1.2/>.

Available for free at Connexions <http://cnx.org/content/col10373/1.2>

251
252 CHAPTER 14. TRANSFORMADA DE LAPLACE Y DISEO DE SISTEMAS

14.1.1.2 Usando una Computadora

El uso de una computadora para encontrar la transformada de Laplace es relativamente sencillo. Matlab
tiene dos funciones, laplace e ilaplace, las dos forman parte de las librerias simbolicas, y encontraremos la
transformada de Laplace y su inversa respectivamente. Este mtodo es preferido generalmente para funciones
ms complicadas. Funciones ms sencillas e ideales usualmente se encuetran ms facil mediante el uso de
tablas (Section 14.1.1.3: Usando Tablas).

14.1.1.3 Usando Tablas

Cuando se aprende por primera vez la transformada de Laplace, las tablas es la forma ms comun para
encontrarla. Con suciente prctica las tablas se hacen inecesarias. Para el proposito de sta seccin, nos
enfocaremos en la transformada inversa de Laplace, dado que la gran parte del diseo de aplicaciones empieza
en el dominio de Laplace y dan como resultado una solucin en el dominio del tiempo. El mtodo es el
siguiente:

1. Se escribe la funcin que se desea transformar


Pm H (s), como la suma de otras funciones H (s) =
i=1 Hi (s) donde cada una de las Hi se encuentra en la tabla (Section 14.3).
2. Invertir cada Hi (s) para obtener su hi (t).
Pm
3. Se suma cada hi (t) para obtener h (t) = i=1 hi (t)
Example 14.1
1
Calcule h (t) para H (s) = s+5 , Re (s) > 5
Esto puede ser resuelto directamente usando la tabla (Section 14.3) para ser h (t) = e(5t)
Example 14.2
25
Encontrar la representacin en el dominio del tiempo h (t), de H (s) =
s+10 , Re (s) > 10
1
Para resolver esto, primero notemos que H (s) tambin puede ser escrito como 25 s+10 .

Entonces podemos ir a la tabla (Section 14.3) para encontrar h (t) = 25e(10t)


Example 14.3
1 25
Podemos extender los dos ejemplos anteriores encontrando h (t) para H (s) = s+5 + s+10 , Re (s) >
5
Para realizar esto, tomamos ventaja de la propiedad de aditividad y linealidad y el mtodo de
los tres pasos descrito anteriormente para obtner como resultado h (t) = e(5t) + 25e(10t)
Para ejemplos ms complicados, seri ms difcil descomponer la funcin de transferencia en partes que se
2
encuentren en la tabla. En este caso, es necesario el uso de expansin en fracciones parciales para obtener
la funcin de transferencia en una forma ms til.

14.1.2 Visualizando la Transformada de Laplace


Con la transfromada de Fourier, tenemos una funcin de valores complejos de variables puramente
imaginarias F (j). Esto es algo que se podra visualizar mediante grcas en 2-dimensiones (parte real e
imaginaria o magnitud y fase). Sin emabrgo, con Laplace tenemos una funcin de valores complejos de
una variable compleja. Para examinar la magnitud y la fase o la parte real e imaginaria de esta funcin,
debemos examinar la grca de supercie en 3-dimensiones de cada componente.

2 "Partial Fraction Expansion" <http://cnx.org/content/m2111/latest/>

Available for free at Connexions <http://cnx.org/content/col10373/1.2>


253

Ejemplo de grcas real e imaginaria

(a) (b)

Figure 14.1: Parte real e imaginaria de H (s) son cada una supercies en 3-dimensiones. (a) La parte
real de H (s) (b) La parte Imaginaria de H (s)

Ejemplos de grcas de magnitud y fase

(a) (b)

Figure 14.2: Magnitud y Fase de H (s) son tambin supercies en 3-dimensiones. Esta representacin
es ms comun que las partes real e imaginaria. (a) La magnitud de H (s) (b) La fase de H (s)

Mientras que estas son maneras legitimas de ver una seal en el dominio de Laplace, es algo difcil
dibujarlas y analizarlas. Por esta razon, un mtodo ms sencillo ha sido desarrollado; que aqu no ser
discutido a detalle, el mtode de Polos y Ceros (Section 14.6) donde es mucho ms sencillo de entender la
Transformada de Laplace y su contraparte discreta en el tiempo la Transformada Z (Section 15.1) y son
representadas grcamente.

14.2 Propiedades de la Transformada de Laplace3

3 This content is available online at <http://cnx.org/content/m12979/1.2/>.

Available for free at Connexions <http://cnx.org/content/col10373/1.2>


254 CHAPTER 14. TRANSFORMADA DE LAPLACE Y DISEO DE SISTEMAS

Propiedad Seal Transformada de Regin de


Laplace Convergencia (ROC)

Linealidad x1 (t) + x2 (t) X1 (s) + X2 (s) Al menos una


ROC1 ROC2
(s )
Desplazamiento en el x (t ) e X (s) ROC
Tiempo

Desplazamiento de et x (t) X (s ) Desplazado una ROC (


Frecuencia s debe de estar en la
(modulacin) regin de convergencia)

Escalamiento en el x (t) (1 ||) X (s ) Escalado una ROC (


Tiempo s debe de estar en
la regin de
convergencia)

Conjugacin x (t) X (s ) ROC
Convolucin x1 (t) x2 (t) X1 (t) X2 (t) Al menos una
ROC1 ROC2
d
Diferenciacin en el
dt x (t) sX (s) Al menos una ROC
Tiempo
d
Diferenciacin de la (t) x (t) ds X (s) ROC
Frecuencia
Rt
Integracin en el

x ( ) d (1 s) X (s) Al menos una
Tiempo ROC (Re (s) > 0)

Table 14.1

Available for free at Connexions <http://cnx.org/content/col10373/1.2>


255

14.3 Tabla de Transformadas de Laplace Comunes4


Seal Transformada de Laplace Regin de Convergencia

(t) 1 All s
(sT )
(t T ) e All s
1
u (t) s Re (s) > 0
1
u (t) s Re (s) < 0
1
tu (t) s2 Re (s) > 0
n n!
t u (t) sn+1 Re (s) > 0
n n!
(t u (t)) sn+1 Re (s) < 0
(t) 1
e u (t) s+ Re (s) >
e(t) u (t) 1

s+ Re (s) <
(t) 1
te u (t) (s)2
Re (s) >
n (t) n!
t e u (t) (s+)n+1
Re (s) >
tn e(t) u (t) n!

(s+)n+1
Re (s) <
s
cos (bt) u (t) s2 +b2 Re (s) > 0
b
sin (bt) u (t) s2 +b2 Re (s) > 0
(at) s+a
e cos (bt) u (t) (s+a)2 +b2
Re (s) > a
e(at) sin (bt) u (t) b
(s+a)2 +b2
Re (s) > a
dn n
dtn (t) s All s

Table 14.2

14.4 Regin de Convergencia para la Transformada de Laplace5


6
Con la transformada de Laplace , el plano s representa un conjunto de seales (exponenciales complejos
(Section 1.5)). Para cualquier sistema LTI, alguna de estas seales puede causar que la salida del sistema
inversa, mientras otras hacen que la salida del sistema diverja (explote). El conjunto de las seales que
causa que la salida de los sistemas converja se encuentran en la regin de convergencia (ROC).este
modulo discutir como encontrar la regin de convergencia para cualquier sistema LTI continuo.
Recuerde la denicin de la transformada de Laplace,

Transformada de Laplace Z
H (s) = h (t) e(st) dt (14.3)

Si consideramos un exponencial complejo causal (Section 2.1), h (t) = e(at) u (t), obtenemos la siguiente
ecuacin,
Z Z
e(at) e(st) dt = e((a+s)t) dt (14.4)
0 0
4 This content is available online at <http://cnx.org/content/m12959/1.1/>.
5 This content is available online at <http://cnx.org/content/m12961/1.1/>.
6 "The Laplace Transform" <http://cnx.org/content/m10110/latest/>

Available for free at Connexions <http://cnx.org/content/col10373/1.2>


256 CHAPTER 14. TRANSFORMADA DE LAPLACE Y DISEO DE SISTEMAS

evaluando esto obtenemos,


1  
limit e((s+a)t) 1 (14.5)
s + a t
Nota que esta ecuacin ira innito cuando limit e((s+a)t) vaya al innito. Para entender por que pasa
t
esto, tomaremos un paso mas al usar s = + j para realizar ecuaciones como

limit e(jt) e((+a)t) (14.6)


t

Al reconocer que e(jt) es senosoidal, se vuelve aparente e((a)t) va determinar si la ecuacin explota o
no. lo que encontramos es que si +a es positivo, el exponencial va a tener una potencia negativa, lo que
va a causar que esto se vaya a cero cuando t vaya a innito. Pero si +a es negativa o cero, el exponencial
no tendr una potencia negativa, lo que prevendr que vaya a cero y el sistema no va a converger. Lo que
todo esto nos dice es que para una seal causar, tenemos convergencia cuando

Condicin para Convergencia


Re (s) > a (14.7)

Aunque no pasaremos por este proceso otra vez paras seales anticausales, podramos hacerlo. Al hacerlo,
nos daramos cuenta que la condicin necesaria para convergencia es cuando

Condicin Necesaria para Convergencia Anti-causal

Re (s) < a (14.8)

14.4.1 Entendiendo el ROC Grcamente


Talvez la mejor manera del ver la regin de convergencia es el ver el plano S lo que observamos es que para
un solo polo, la regin de convergencia se encuentra a la derecha de las seales causales y a la izquierda de
las seales anticausales.

(a) (b)

Figure 14.3: (a) La ROC para una seal causal. (b) La ROC para una seal anti-causal.

Despus de reconocer esto, la pregunta necesaria es esta: Que hacemos cuando tenemos polos mltiples?
La respuesta ms simple es que tenemos que tomar la interseccin de todas las regiones de convergencias
para cada respectivo polo.

Available for free at Connexions <http://cnx.org/content/col10373/1.2>


257

Example 14.4
Encuentre H (s) y diga la regin de convergencia para h (t) = e(at) u (t) + e(bt) u (t)
Al separa esto en dos trminos obtenemos que las funciones de transferencia y la respectivas
regiones de convergencia de
1
H1 (s) = , Re (s) > a (14.9)
s+a
y
1
H2 (s) = , Re (s) < b (14.10)
s+b
Combinando esto obtenemos la regin de convergencia de b > Re (s) > a. Si a > b, podemos
representar esto grcamente. Si no, no abra una regin de convergencia.

Figure 14.4: Regin de convergencia de h (t) si a > b.

14.5 La Transformada Inversa de Laplace7


14.5.1 To Come
8
En la Funcin de Transferencia estableceremos que la funcin para la transformada de Laplace es h es
Z
1
L1 (h) (t) = e(c+yj)t h ((c + yj) t) dy (14.11)
2

donde j 21 y el numero real cson escogidos para que todas las singularidades de h se encuentren
en el lado izquierdo del integral.

7 This content is available online at <http://cnx.org/content/m12962/1.1/>.


8 "Eigenvalue Problem: The Transfer Function" <http://cnx.org/content/m10490/latest/>

Available for free at Connexions <http://cnx.org/content/col10373/1.2>


258 CHAPTER 14. TRANSFORMADA DE LAPLACE Y DISEO DE SISTEMAS

14.5.2 Continuando con la Transformada Inversa de Laplace


Con la transformada inversa de Laplace podemos expresar la solucin de x0 = Bx + g , como

 
1
x (t) = L1 (sI B) (L (g) + x (0)) (14.12)

Por ejemplo, tomaremos el primer componente de L (x),nombrado



0.19 s2 + 1.5s + 0.27
Lx1 (s) = .
1 4

s+ 6 (s3 + 1.655s2 + 0.4078s + 0.0039)

Denimos:

Denition 14.1: polos


Tambin llamadas como singularidades estos son los punto s en los cuales Lx1 (s) explota.

Son las races del denominador,


273
1/100, 329/400 , and 1/6. (14.13)
16
Las cuatro son negativas, es suciente tomar c = 0 as la integracin de (14.11) continua en el eje imaginario.
No suponemos que el lector aya encontrado integraciones en el plano complejo pero esperamos que este
ejemplo provea la motivacin necesaria para examinarla. Sin embargo antes de esto hay que notar que
9
MATLAB tiene el clculo necesario para desarrollar este punto. Volviendo a b3.m observamos que el
comando ilaplace produce

t t
3 2
x1 (t)  =
(329t)
211.35e
 100  (0.0554t +4.5464t
 + 1.085t + 474.19) e 6 +
273t 273t
e 400 262.842cosh 16 + 262.836sinh 16
9 http://www.caam.rice.edu/caam335/cox/lectures/b3.m

Available for free at Connexions <http://cnx.org/content/col10373/1.2>


259

10
Figure 14.5: Los 3 potenciales asociados con el modelo del circuito RC .

Los otros potenciales, vistos en esta gura posen una expresin similar. Por favor note que cada uno de
los polos de L (x1 ) se muestra como exponencial x1 y que los coecientes del exponencial son polinomios con
grados que son determinados por el orden de su respectiva polos.

14.6 Polos y Ceros11


14.6.1 Introduccin
12
Es muy difcil analizar cualitativamente la transformada de Laplace y la transformada Z (Section 15.1),
ya que al gracar su magnitud y ngulo a su parte real e imaginaria da como resultado varias gracas de
supercies de dos dimensiones en espacios de tres dimensiones. Por esta razn, es comn el examinar la
13
graca de la funcin de transferencia con sus polos y ceros y tratar una vez mas una idea cualitativa de lo
que hace el sistema.
Dada a una funcin de transformacin continua, en el dominio de Laplace, H (s), o en el dominio discreto
de Z, H (z), un cero es cualquier valor de s o z para los cuales la funcin de transferencia es cero, un polo es
cualquier valor de s o z para la cual la funcin de trasferencia es innita. Lo siguiente da a una denicin
precisa:

Denition 14.2: Ceros


1. El valor(es) para z donde el numerador de la funcin de trasferencia es iguala cero
2. Las frecuencias complejas que hacen que la ganancia de la funcin de transferencia del ltro sea
cero.
10 "Nerve Fibers and the Dynamic Strang Quartet", Figure 1: An RC model of a nerve ber
<http://cnx.org/content/m10168/latest/#RC_model_g>
11 This content is available online at <http://cnx.org/content/m12963/1.2/>.
12 "The Laplace Transform" <http://cnx.org/content/m10110/latest/>
13 "Transfer Functions" <http://cnx.org/content/m0028/latest/>

Available for free at Connexions <http://cnx.org/content/col10373/1.2>


260 CHAPTER 14. TRANSFORMADA DE LAPLACE Y DISEO DE SISTEMAS

Denition 14.3: polos


1. El valor(es) para z donde el denominador de la funcin de transferencia es igual a cero
2. Las frecuencias complejas que hacen de la ganancia de la funcin de transferencia del ltro se
innita.

14.6.2 Gracas de los Polos y Ceros


Cuando gracamos estos en su plano s o z, representamos los ceros con o y los polos con x. Vea este
14
modulo para observa detalladamente como gracar los ceros y polos en la transformada-z en el plano-z.

Example 14.5
s2 +6s+8
Encuentre los polos y ceros de la funcin de trasferencia H (s) = s2 +2 y graque los resultados
en el plano-s.
Lo primero que tenemos que reconocer que la funcin de transferencia ser igual a cero cuando
lo de arriba, s2 + 6s + 8, sea igual a cero. Para encantar que esto iguala a cero factorizamos esto
para obtener, (s + 2) (s + 4). Esto da a ceros en s = 2 y s = 4. Si esta funcin hubiera sido
mas complicada, talvez tendramos que usar la formula cuadrtica.
Para los polos, tenemos que reconocer que la funcin de transferencia ser innita cuando la
2
 s +2 es cerro para encontrar esto, tenemos que factorizar
parte de abajo es cero. Esto sucede cuando

la funcin esto nos da s + j 2 s j 2 . Lo que signica que tenemos races imaginarias de j 2

y j 2
Al gracar esto nos da Figure 14.6 (Graca de Polos y Zeros)

Graca de Polos y Zeros

Figure 14.6: Mestra de la Graca

Ya que hemos encontrado y gracado los polos y cero, tenemos que preguntarnos que es lo que nos dice esta
graca. Lo que podemos deducir es que magnitud de la funcin de trasferencia ser mayor cuando se encuentre

14 "Understanding Pole/Zero Plots on the Z-Plane" <http://cnx.org/content/m10556/latest/>

Available for free at Connexions <http://cnx.org/content/col10373/1.2>


261

cerca de los polos y menos cuando se encuentre cerca de los ceros. Esto nos da un entendimiento cualitativo
de lo que el sistema hace en varias frecuencias y es crucial para la funcin de estabilidad (Section 3.4).

14.6.3 Repeticiones de Polos y Ceros


Es posible obtener mas de un polo lo cero en el mismo punto. Por ejemplo, la funcin de transferencia
1
discreta H (z) = z 2 tendr dos ceros en el origen y la funcin H (s) = s25 tender 25 polos en el origen.

14.6.4 La Cancelacin de Polos y Ceros


(s+3)(s1)
Un error comn es el pensar que la funcin es la misma que s + 3. En teora son equivalentes,
s1
ya que el polo y el cero que se encuentra en s = 1 se cancelan mutuamente lo que es conocido como la
cancelacin de polos y ceros. Sin embargo, piense lo que pasara si esto fuera una funcin de transferencia
de un sistema que fue creado fsicamente con un circuito. En este caso, no es comn que el polo y el cero
permanezca en un mismo lugar. Un cambio de temperatura, podra causar que ellos se movieran. Si esto
pasara se creara volatilidad en esa rea, ya que ocurri un cambio de innito en un polo a cero en el cero en
una regin de seales. Generalmente es una mala manera de eliminar un polo. Una mejor manera de mover
el polo a otro lugar es usando la teora de control.

Available for free at Connexions <http://cnx.org/content/col10373/1.2>


262 CHAPTER 14. TRANSFORMADA DE LAPLACE Y DISEO DE SISTEMAS

Available for free at Connexions <http://cnx.org/content/col10373/1.2>


Chapter 15

La Transformada Z y Filtros Digitales

15.1 La Transformada Z: Denicin1


15.1.1 Denicin Bsica de la Transformada-Z
La transformada-z de una secuencia es denida por


X
X (z) = x [n] z n (15.1)
n=

Algunas veces esta ecuacin es conocida como la transformada-z bilateral. En veces la transformada-z
es denida por

X
X (z) = x [n] z n (15.2)
n=0

la cual es conocida como la transformada-z unilateral.


Hay una relacin cercana entre la transformada-z y la transformada de Fourier de una seal discreta,
la cual es denida como

X
j
x [n] e(jn)

X e = (15.3)
n=

Note que cuando z n es remplazada con e(jn) la transformada-z se convierte en la transformada de


Fourier. Cuando la transformada de Fourier existe, z = ej , la cual debe de tener la magnitud unitaria
para z.

15.1.2 El Plano Complejo


Para entender la relacin entre la transformada de fourier y la transformada-z uno tiene que ver el plano
complejo o el plano-z. echemos un vistazo al plano complejo:
1 This content is available online at <http://cnx.org/content/m12951/1.1/>.

Available for free at Connexions <http://cnx.org/content/col10373/1.2>

263
264 CHAPTER 15. LA TRANSFORMADA Z Y FILTROS DIGITALES

Plano-Z

Figure 15.1

El plano-Z es un plano complejo con un eje imaginario y real que se reejen eso se reere a la variable
compleja z. . La posicin en el plano complejo es dada por rej , y el ngulo se toma del eje real positive

al rededor del plano y es dado por . X (z) es denida en todos los lados del plano. X ej es denida solo
donde |z| = 1, la cual se reere al circulo unitario. Por ejemplo, =1 en z=1 y = en z = 1. Esto
ayuda por que, al representar la transformada de fourier como una transformada-z en el crculo unitario, se
puede ver muy fcilmente la periodicidad de la transformada de Fourier.

15.1.3 Regin de Convergencia


La regin de convergencia, tambin conocida como ROC, es importante entender por que dene la regin
donde la transformada-z existe. La ROC para una x [n] , is dened as the range of z es denida como el
rango de z para la cual la transformada-z converge. Ya que la transformada z es una serie de potencia,
converge cuando x [n] z n es absolutamente sumable. En otras palabras,


X
|x [n] z n | < (15.4)
n=

Se tiene que satisface para la convergencia. Esto se explica mejor al ver las diferentes ROC de las
transformadas-z de n u [n] y n u [n 1].
Example 15.1
Para
x [n] = n u [n] (15.5)

Available for free at Connexions <http://cnx.org/content/col10373/1.2>


265

Figure 15.2: x [n] = n u [n] donde = 0.5.

P
X (z) = n= x [n] z n
P n n
= n= u [n] z
P (15.6)
n n
= n=0 z
1 n
P 
= n=0 z

Esta secuencia es un ejemplo de una exponencial del lado derecho por que tiene un valor de no
cero para n 0. Solo converge cuando |z 1 | < 1. Cuando converge,

1
X (z) = 1z 1
(15.7)
z
= z

P n
Si |z 1 | 1, entonces las series, n=0 z 1 no convergen. As que el ROC es el rango de
valores cuando
|z 1 | < 1 (15.8)

o, equivalentemente,
|z| > || (15.9)

Available for free at Connexions <http://cnx.org/content/col10373/1.2>


266 CHAPTER 15. LA TRANSFORMADA Z Y FILTROS DIGITALES

Figure 15.3: ROC para x [n] = n u [n] donde = 0.5

Example 15.2
Para
x [n] = (n ) u [(n) 1] (15.10)

Available for free at Connexions <http://cnx.org/content/col10373/1.2>


267

Figure 15.4: x [n] = (n ) u [(n) 1] donde = 0.5.

P
X (z) = n= x [n] z n
P
= n= (n ) u [n 1] z n
P1
= n z n
n=
P1 n (15.11)
= n= 1 z
P n
= n=1 1 z
P n
= 1 n=0 1 z
En este caso la ROC es en el rango de valores donde

|1 z| < 1 (15.12)

o, equivalentemente
|z| < || (15.13)

Si la ROC se satisface, entonces

1
X (z) = 1 11 z
(15.14)
z
= z

Available for free at Connexions <http://cnx.org/content/col10373/1.2>


268 CHAPTER 15. LA TRANSFORMADA Z Y FILTROS DIGITALES

Figure 15.5: ROC para x [n] = (n ) u [(n) 1]

15.2 Tabla de Transformadas-Z Comunes2


La siguiente tabla provee transformadas de z unilateral y bilateral (Section 15.1). Esta tabla tambin
muestra la regin de convergencia (Section 15.3).

note: la notacin usada para z en esta tabla puede ser diferente a la notacin encontrada en otras
tablas. Por ejemplo la transformada bsica de u [n] se puede describen de dos maneras diferentes,
que son equivalentes:

z 1
= (15.15)
z1 1 z 1

2 This content is available online at <http://cnx.org/content/m12950/1.2/>.

Available for free at Connexions <http://cnx.org/content/col10373/1.2>


269

Seal Transformada-Z ROC

[n k] z k All (z)
z
u [n] z1 |z| > 1
z
u [(n) 1] z1 |z| < 1
z
nu [n] (z1)2
|z| > 1
z(z+1)
n2 u [n] (z1)3
|z| > 1
3 z (z 2 +4z+1)
n u [n] (z1)4
|z| > 1
z
(n ) u [(n) 1] z |z| < ||
n z
u [n] z |z| > ||
n z
n u [n] (z)2
|z| > ||
2 n z(z+)
n u [n] (z)3
|z| > ||
Qm
k=1 nk+1 z
m m! n u [n] (z)m+1
z(zcos())
n cos (n) u [n] z 2 (2cos())z+ 2 |z| > ||
zsin()
n sin (n) u [n] z 2 (2cos())z+ 2 |z| > ||

Table 15.1

15.3 Regin de Convergencia para la Transformada- Z3


15.3.1 La Regin de Convergencia
La regin de convergencia, tambin conocida como ROC, es importante entender por que dene la regin
donde la transformada-z (Section 15.1) existe. La transformadaz se dene por


X
X (z) = x [n] z n (15.16)
n=

La ROC para una x [n] , es denida como el rango de z para la cual la transformada-z converge. Ya que la
transformadaz es una serie de potencia, converge cuando x [n] z n es absolutamente sumable. En otras
palabras,

X
|x [n] z n | < (15.17)
n=

tiene que ser satisfecha para la convergencia.

15.3.2 Propiedades de la Regin de Convergencia


La regin de convergencia tiene propiedades que dependen de la caractersticas de la seal, x [n].

La ROC no tiene que contener algn polo. Por denicin un polo es donde X (z) es innito. Ya
que X (z) tiene que ser nita para todas las z para tener convergencia, no puede existir ningn polo
para ROC.

3 This content is available online at <http://cnx.org/content/m12956/1.2/>.

Available for free at Connexions <http://cnx.org/content/col10373/1.2>


270 CHAPTER 15. LA TRANSFORMADA Z Y FILTROS DIGITALES

Si x [n] es una secuencia de duracin nita, entonces la ROC es todo el plano-z, excepto
en z = 0 o |z| = . Una secuencia de duracin nita es aquella que tienen valor de no cero en
un intervalo nito n1 n n2 . Mientras que cada valor de x [n] es nito entonces la secuencia ser
1
absolutamente sumable. Cuando n2 > 0 entonces existir z trminos por lo tanto la ROC no incluye
z = 0. Cuando n1 < 0 la suma ser innita por lo tanto la ROC no incluye |z| = . Pero si, n2 0
entonces la ROC incluir z = 0, y cuando n1 0 la ROC incluir |z| = . Con esta condiciones, la
nica seal que tiene una ROC que cubre todo el plano-z es x [n] = c [n].

Figure 15.6: Ejemplo de una secuencia de duracion nita.

Las siguientes propiedades se aplican a secuencias con duracin innita. Como se menciono anterior
mente la transformada-z converge cuando |X (z) | < . As que podemos escribir


n
X X X
n n
|X (z) | = | x [n] z | |x [n] z |= |x [n] |(|z|) (15.18)
n= n= n=

Podemos separar la suma innita en su tiempo positive y negativa. Por lo tanto

|X (z) | N (z) + P (z) (15.19)

donde
1
n
X
N (z) = |x [n] |(|z|) (15.20)
n=
y

n
X
P (z) = |x [n] |(|z|) (15.21)
n=0

Para que |X (z) | se innita, |x [n] | tiene que estar restringida entonces denamos

|x (n) | C1 r1 n (15.22)

Available for free at Connexions <http://cnx.org/content/col10373/1.2>


271

para
n<0
y
|x (n) | C2 r2 n (15.23)

para
n0
De estos tambin se pueden derivar ms propiedades:

Si x [n] es una secuencia del lado derecho entonces la ROC se extiende hacia fuera en
el ultimo polo desde X (z). Una secuencia del lado derecho es aquella donde x [n] = 0 para
n < n1 < . Si vemos la porcin del tiempo positive de la ultima derivacin, se puede deducir que

 n
X n
X r2
P (z) C2 r2 n (|z|) = C2 (15.24)
n=0 n=0
|z|

Por lo tanto para que la suma converja, |z| > r2 , as que la ROC de una secuencia del lado derecho
tiene la forma de |z| > r2 .

Figure 15.7: Secuencia de lado derecho

Available for free at Connexions <http://cnx.org/content/col10373/1.2>


272 CHAPTER 15. LA TRANSFORMADA Z Y FILTROS DIGITALES

Figure 15.8: LA ROC de una secuencia de lado derecho.

Si x [n] es una secuencia del lado izquierdo, entonces la ROC se extiende hacia dentro
desde el polo mas cercano en X (z). Una secuencia del lado izquierdo es aquella donde x [n] = 0
para n > n2 > . Si vemos la porcin del lado negativa de la ultima derivacin se puede deducir
que
1 1  n  k
X n
X r1 X |z|
N (z) C1 r1 n (|z|) = C1 = C1 (15.25)
n= n=
|z| r1
k=1

Por lo tanto para que la suma converja, |z| < r1 , as que la ROC de la secuencia del lado izquierdo
tiene la forma de |z| < r1 .

Figure 15.9: Secuencia de lado izquierdo.

Available for free at Connexions <http://cnx.org/content/col10373/1.2>


273

Figure 15.10: La ROC de una secuencia de lado izquierdo.

Si x [n] es una secuencia con dos lados, la ROC va ser un anillo en el plano-z que esta
restringida en su interior y exterior por un polo. Una secuencia de dos lados es aquella con
duracin innita en direcciones positivas y negativas. De la derivacin de las dos propiedades, podemos
deducir que si r2 < |z| < r2 converge, entonces el tiempo positivo y el tiempo negativo converge, y
X (z) converge tambin. Por eso la ROC de una secuencia de dos lados tiene la forma de r2 < |z| < r2 .

Figure 15.11: Una secuencia de dos lados.

Available for free at Connexions <http://cnx.org/content/col10373/1.2>


274 CHAPTER 15. LA TRANSFORMADA Z Y FILTROS DIGITALES

Figure 15.12: La ROC de una secuancia de dos lados.

15.3.3 Ejemplos
Para entender esto mejor veremos los siguientes ejemplos.

Example 15.3
Tomemos  n  n
1 1
x1 [n] = u [n] + u [n] (15.26)
2 4
1 n
u [n] es zz 1 con ROC en 1

La transformada de
2 |z| > 2.
2

La ROC de
` 1 n
Figure 15.13:
2
u [n]

Available for free at Connexions <http://cnx.org/content/col10373/1.2>


275

1 n 1
u [n] es z+z 1 con ROC en

La transformada de
4 |z| > 4 .
4

La ROC de
` 1 n
Figure 15.14:
4
u [n]

Usando linealidad,
z
X1 [z] = z 12
+ z+z 1
4
2z (z 81 ) (15.27)
=
( z 12 )( z+ 14 )
1 1
Al observar esto se vuelve claro que hay dos cero, uno en 0 y el otro en
8 , y dos polos, uno en 2 ,
1 1
y en
4 . Usando las propiedades, la ROC es |z| > 2.

Available for free at Connexions <http://cnx.org/content/col10373/1.2>


276 CHAPTER 15. LA TRANSFORMADA Z Y FILTROS DIGITALES

La ROC de x1 [n] =
` 1 n ` 1 n
Figure 15.15:
2
u [n] + 4
u [n]

Example 15.4
Ahora tomemos  n  n
1 1
x2 [n] = u [n] u [(n) 1] (15.28)
4 2
1 n

La transformada y ROC de
4 u [n] fueron mostradas en el ejemplo anterior (Example 15.3).
1 n
u [(n) 1] es zz 1 con una ROC en |z| > 21 .
 
La transformada de 2 2

Available for free at Connexions <http://cnx.org/content/col10373/1.2>


277

La ROC de
` ` 1 n
Figure 15.16:
2
u [(n) 1]

Usando linealidad,
z
X2 [z] = z+ 14
+ zz 1
2
z (2z 81 ) (15.29)
=
(z+ 14 )(z 12 )
1 1 1
Podemos observar que hay dos ceros, en 0 y
16 , y dos polos en 2 , y 4 . En este caso la ROC es
1
|z| < 2.

Available for free at Connexions <http://cnx.org/content/col10373/1.2>


278 CHAPTER 15. LA TRANSFORMADA Z Y FILTROS DIGITALES

La ROC de x2 [n] = u [(n) 1].


` 1 n ` 1 n
Figure 15.17:
4
u [n] 2

15.4 La Transformada Inversa de Z4


Cuando usamos la transformada-z (Section 15.1)


X
X (z) = x [n] z n (15.30)
n=

Es de gran ayuda el poder encontrar x [n] dado X (z). Existen al menos 4 mtodos diferentes para hacer
esto:

1. Inspeccin (Section 15.4.1: El Mtodo de Inspeccin)


2. Expansion de fracciones parciales (Section 15.4.2: Mtodo de Expansin de Fracciones Parciales)
3. Expansin de series de potencia (Section 15.4.3: Mtodo de Expansin de Series de Potencia)
4. Integracin del contorno (Section 15.4.4: Mtodo de Integracin del Contorno)

15.4.1 El Mtodo de Inspeccin


Este "mtodo" es el familiarizarse con la tabla para los pares de la transformada-z (Section 15.2)y el usar
ingeniera inversa.

Example 15.5
Dado
z
X (z) =
z
4 This content is available online at <http://cnx.org/content/m12958/1.2/>.

Available for free at Connexions <http://cnx.org/content/col10373/1.2>


279

Con un ROC (Section 15.3) de


|z| >
Al inspeccionar podemos determinar que

x [n] = n u [n]

15.4.2 Mtodo de Expansin de Fracciones Parciales


Cuando tratamos con sistemas lineares de tiempo invariante la transformada-z tiene la forma

B(z)
X (z) = A(z)
P M
bk z k (15.31)
= k=0
PN k
k=0 ak z

Que tambin se puede expresar como

QM
a0 k=1 1 ck z 1
X (z) = QN (15.32)
b0 k=1 1 dk z 1

donde ck representa los ceros con valor no ceros de X (z) y dk representa los polos con valores no cero
Si M < N entonces X (z) se puede representar por
N
X Ak
X (z) = (15.33)
1 dk z 1
k=1

Esta forma ayuda el invertir cada termino de la suma usando el mtodo de inspeccin (Section 15.4.1: El
Mtodo de Inspeccin) y la tabla de transformadas (Section 15.2). Por lo tanto el numerador es un polinomio
5
se vuelve necesario usar la expansin de fracciones parciales para poder poner X (z) en la forma descrita
arriba. Si M N entonces X (z) se puede expresar como

M N PN 1 ' k
X bk z
X (z) = Br z r + Pk=0
N
(15.34)
k
r=0 k=0 k z
a

Example 15.6
Encuentre la transformada inversa-z de

1 + 2z 1 + z 2
X (z) =
1 3z 1 + 2z 2
Donde el ROC es |z| > 2. En este caso M = N = 2, as que tenemos que la divisin larga para
obtener
1 7 1
1 2 + 2z
X (z) = +
2 1 3z 1 + 2z 2
Tenemos que factorizar el denominador.

1 + 5z 1
X (z) = 2 +
(1 2z 1 ) (1 z 1 )
5 "Partial Fraction Expansion" <http://cnx.org/content/m2111/latest/>

Available for free at Connexions <http://cnx.org/content/col10373/1.2>


280 CHAPTER 15. LA TRANSFORMADA Z Y FILTROS DIGITALES

Ahora utilizamos la expansin de fracciones parciales.

9
1 A1 A2 1 2 4
X (z) = + + = + +
2 1 2z 1 1 z 1 2 1 2z 1 1 z 1
Ahora cada trmino se puede invertir usando el mtodo de inspeccin y la tabla de transformada-z.
ya que la ROC es |z| > 2,
1 9
x [n] = [n] + 2n u [n] 4u [n]
2 2

15.4.3 Mtodo de Expansin de Series de Potencia


Cuando la transformada-z es denida con una serie de potencia en la forma de


X
X (z) = x [n] z n (15.35)
n=

entonces cada termino de la secuencia x [n] se puede terminar al ver los coecientes del respectivo poder
z n .
Example 15.7
Ve la siguiente transformada -z de la secuencia de tamao nito.

z 2 1 + 2z 1 1 21 z 1 1 + z 1
  
X (z) =
(15.36)
= z 2 + 52 z + 1
2 + z 1

En este caso, ya que no hay polos, multiplicamos los factores de X (z). Al inspeccinala, se vuele
claro que
5 1
x [n] = [n + 2] + [n + 1] + [n] + [n 1]
2 2
.

Una de las ventajas de las series de potencia es que muchas funciones usadas en funciones de ingeniera
tienen tabuladas sus series de potencia. Por lo tanto funciones como el Log, seno, exponencial, seno h, etc.,
se pueden invertir fcilmente.

Example 15.8
Suponga
X (z) = logn 1 + z 1


Al notar que

X 1n+1 xn
logn (1 + x) =
n=1
n
Entonces

X 1n+1 n z n
X (z) =
n=1
n
Por lo tanto
1n+1 n

n if n1
X (z) =
0 if n0

Available for free at Connexions <http://cnx.org/content/col10373/1.2>


281

15.4.4 Mtodo de Integracin del Contorno


Sin entrar en detalles I
1
x [n] = X (z) z n1 dz (15.37)
2j
r

donde r es el contorno del lado contrario del reloj en la ROC de X (z) circulando el origen del plano-z. Para
expandir este mtodo para encontrar la inversa se necesita conocimiento de teora de variables complejas y
esto no se vera en este modulo.

15.5 Funciones Racionales6


15.5.1 Introduccin
Cuando usamos operaciones con polinomios, el trmino de funciones racionales es una manera simple de
describir una relacin particular entre dos polinomios.

Denition 15.1: Funcin Racional


Para dos polinomios cualquiera, A y B, su fraccin es conocida cono una funcin racional.
Example
Aqu se muestra un ejemplo de una funcin racional, f (x). Note que el numerador y denomi-
nador pueden ser polinomios de cualquier orden, pero la funcin racional es indenida cuando el
denominador es cero.
x2 4
f (x) = (15.38)
2x2 + x 3

Si usted ha empezado a usar la transformada-z (Section 15.1), usted debi notar que todas las transfor-
madas son funciones racionales. En seguida veremos algunas de las propiedades de las funciones racionales y
como se pude usar para reveler caractersticas importantes de la transformada-z, y por lo tanto revelar una
seal o un sistema LTI.

15.5.2 Propiedades de las Funciones Racionales


Para poder entender lo que hace que las funciones racionales sean tan especiales, tenemos que observar
algunas de sus propiedades y caractersticas. Si usted esta familiarizado con las funciones racionales y que
las funciones bsicas de algebra, podr ir directamente a la siguiente seccion (Section 15.5.3: Las Funciones
Racionales y la Transformada-z) y ver como las funciones racionales nos ayudan a entender la transformada-z.

15.5.2.1 Races

Para entender muchas de las caractersticas de las funciones racionales, tenemos que empezar por encontrar
las races de la funcin racional. Para ser esto, factoricemos los dos polinomios para poder observar las races
fcilmente. Como todos los polinomios, las races nos darn informacin sobre muchas de sus propiedades.
Esta funcin nos muestra el resultado que da el factorizar la funcin racional anterior, (15.38).

(x + 2) (x 2)
f (x) = (15.39)
(2x + 3) (x 1)
As, las races para esta funcin son las siguientes:
Las races del numerador son: {2, 2}
Las races del denominador son: {3, 1}
6 This content is available online at <http://cnx.org/content/m12960/1.1/>.

Available for free at Connexions <http://cnx.org/content/col10373/1.2>


282 CHAPTER 15. LA TRANSFORMADA Z Y FILTROS DIGITALES

note: Para entender las funciones racionales, es esencial el saber y entender las races que forman
parte de esta funcin.

15.5.2.2 Discontinuidades

Ya que estamos viendo la divisin de dos polinomios, tenemos que ver los valores de la variable que darn que
el denominador de nuestra fraccin sea cero. Cuando esto pasa, la funcin racional se vuelve indenida, por lo
tanto, tenemos una discontinuidad en la funcin. Ya que sabemos nuestras races, es muy fcil saber cuando
ocurre esto. Cuando tenemos una variable en el denominador igual a cualquier raz en el denominador, la
funcin se vuelve indenida.

Example 15.9
Usando la funcin racional anterior, (15.38), podemos observar que funcin tendr discontinuidades
en los siguientes puntos: x = {3, 1}
Con respecto al plano cartesiano podemos decir que las discontinuidades ocurren con respecto al eje de las
x. Esta discontinuidades sern asintoticamente vertical en la graca y representara los valores donde la
funcin esta indenida.

15.5.2.3 Dominio

Usando las races anteriores, el dominio de la funcin racional se puede denir.


Denition 15.2: dominio
El grupo, o conjunto, de valores que son denidos por una funcin.
Example
Usando la funcin racional anterior, (15.38), el dominio se puede denir como cualquier numero real
x donde x no iguala a 1 o negativa 3. Expresando esto matemticamente, obtenemos lo siguiente:

{ x R | (x 6= 3) and (x 6= 1) } (15.40)

15.5.2.4 Intercepciones

La intercesin en la x es denida como el punto (s) donde f (x),en otras palabras la salida de la funcin
racional iguala a cero. Ya que hemos encontrado las races de la funcin este proceso es muy simple. Usando
algebra, sabemos que la salida ser cero cuando el numerador de la funcin igual a cero. Por lo tanto, la
funcin tendr una intercepciones en x cuando sea igual a una de las races del numerador.
La intercepcin y ocurre cuando x es igual a cero. Se puede encontrar al tener todos los valores de x
igual a cero y resolver la funcin racional.

15.5.3 Las Funciones Racionales y la Transformada-z


Como ya lo hemos mencionado todas las transformadas-z se pueden escribir como funcin racional, lo cual es
la manera mas comn de representar la transformada-z. Por esto, podemos usar las transformadas anteriores
especialmente las de la races, para reveler algunas de las caractersticas de la seal o sistema LTI descritos
por la transformada-z.
La siguiente ecuacin muestra la forma general de escribir la transformada-z como una funcin racional:

b0 + b1 z 1 + + bM z M
X (z) = (15.41)
a0 + a1 z 1 + + aN z N

Available for free at Connexions <http://cnx.org/content/col10373/1.2>


283

7
Si usted ya vio el modulo titulado el entender gracas de polos y ceros y la transformada-z , usted sabr
como las races de las funciones racionales forman un papel importante para entender la transformada-z.
La ecuacin anterior, (15.41), se puede expresar en una forma factorizada similar la funcin racional vista
anterior menta, vea (15.39). Por lo tanto podemos encontrar las races del numerador y denominador de la
transformada-z. Las siguientes dos relaciones se vuelven obvias:

Relacin de Races en Polos y Ceros

Las races del numerado en la funcin racional son lo ceros en la transformada-z


Las races en el nominador de la funcin racional son los polos de la transformada-z

15.5.4 Conclusin
Una vez que usemos nuestro conocimiento de funciones raznales para encontrar sus races, podemos ma-
nipular una transformada-z en varias maneras tiles. Podemos usar este conocimiento para representar un
8
sistema LTI grcamente usando una graca de polos y ceros , o para analizar o disear un ltro digital a
9
travs de la transformada-z .

15.6 La Ecuacin de Diferencia10


15.6.1 Introduccin
Uno de los conceptos ms importantes para el DSP es poder representar la relacin de entrada y salida
de cualquier sistema LTI. Una ecuacin de diferencia de coeciente linear constante (LCCDE) nos
sirve para expresar esta relacin en un sistema discreto. El escribir la secuencia de entradas y salidas, las
cuales representan las caractersticas del sistema LTI, como una ecuacin de diferencia nos ayuda entender
y manipular el sistema.

Denition 15.3: La Ecuacin de Diferencia


Es una ecuacin que muestra la relacin entra valores consecutivos de una secuencia y la diferencia
entre ellos. Usualmente se escribe en una ecuacin recurrente para que la salida del sistema se
pueda calcular de las entradas de la seal y sus valores anteriores.
Example

y [n] + 7y [n 1] + 2y [n 2] = x [n] 4x [n 1] (15.42)

15.6.2 Formulas Generales para la Ecuacin de Diferencia


La ecuacin de diferencia nos ayuda a describir la salida del sistema descrito por la formula para cualquiern.
La propiedad mas importante para esta ecuacin es la habilidad de poder encontrar la transformada, H (z),
del sistema. Las siguientes subsecciones veremos la forma genera de la ecuacin diferencial en la conversin
a la transformada-z directamente de su ecuacin de diferencia.
7 "Understanding Pole/Zero Plots on the Z-Plane" <http://cnx.org/content/m10556/latest/>
8 "Understanding Pole/Zero Plots on the Z-Plane" <http://cnx.org/content/m10556/latest/>
9 "Discrete Time Filter Design" <http://cnx.org/content/m10548/latest/>
10 This content is available online at <http://cnx.org/content/m12965/1.1/>.

Available for free at Connexions <http://cnx.org/content/col10373/1.2>


284 CHAPTER 15. LA TRANSFORMADA Z Y FILTROS DIGITALES

15.6.2.1 Ecuacin de Diferencia

La forma general de este tipo de ecuacin es la siguiente:

N
X M
X
ak y [n k] = bk x [n k] (15.43)
k=0 k=0

Tambin se puede expresar como una salida recurrente, la cual se ve as:

N
X M
X
y [n] = ak y [n k] + bk x [n k] (15.44)
k=1 k=0

De esta ecuacin, note que y [n k] representa las salidas y x [n k] epresenta las entradas. El valor de
N representa el orden de la ecuacin de diferencia que corresponde a la memoria del sistema representado.
Ya que la ecuacin depende de los valores pasados de la salida, para calcular una solucin numrica, algunos
valores pasados, conocidos como condiciones iniciales, se deben saber.

15.6.2.2 Conversin a la Transformada-z

Usando la formula, (15.43),podemos generalizar la funcin de transferencia, H (z), para cualquier funcin
de diferencia. Los siguientes pasos se deben de tomar para convertir cualquier funcin de diferencia en su
11
funcin de diferencia. Primero se tiene que tomar la transformada de Fourier de todos los trminos en
la (15.43). Despus usando la propiedad de linealidad para sacar la transformada fuera de la sumatoria
y usamos la propiedad de desplazamiento en el tiempo de la transformada z para cambiar los trminos
desplazados en el tiempo a exponenciales. Despus de hacer esto, llegamos a la siguiente ecuacin: a0 = 1.
N
X M
X
Y (z) = ak Y (z) z k + bk X (z) z k (15.45)
k=1 k=0

Y (z)
H (z) = X(z)
PM k (15.46)
k=0 bk z
= 1+ N
P
a z k
k=1 k

15.6.2.3 Conversin a la Respuesta de Frecuencia

Ya que tenemos la transformada- z, podemos tomar el siguiente paso y denir la respuesta a la frecuencia
del sistema, o ltro, que esta siendo representado por la ecuacin de diferencia.

note: Recuerde que la razn por la que usamos estas formulas es el poder disear un ltro. Un
LCCDE es una de las maneras ms fciles de representar los ltros FIL. Al encontrar la respuesta
de frecuencia, podemos ver las funciones bsicas de cualquier ltro representado por una simple
LCCDE. La formula general para la respuesta de frecuencia en la transformada-z es la siguiente.

La conversin es simplemente tomar la formula de la transformada-z, H (z), y remplazarla en cualquier


instante de z con ejw .
H (w) = H (z) |z,z=ejw
PM (jwk) (15.47)
k=0 bk e
= PN
a e (jwk)
k=0 k

Despus de que usted entienda la derivacin de esta formula vea el modulo titulado el ltro y la transformada-
12
z para que vea las ideas de la transformada-z (Section 15.1) y la ecuacin de diferencia, y las gracas de
13
polos y ceros se usan al disear un ltro.

11 "Derivation of the Fourier Transform" <http://cnx.org/content/m0046/latest/>


12 "Discrete Time Filter Design" <http://cnx.org/content/m10548/latest/>
13 "Understanding Pole/Zero Plots on the Z-Plane" <http://cnx.org/content/m10556/latest/>

Available for free at Connexions <http://cnx.org/content/col10373/1.2>


285

15.6.3 Ejemplo
Example 15.10: Encontrando la Funcin de Diferencia
Aqu se muestra un ejemplo de tomar los pasos opuestos a los descritos anteriormente: dada una
funcin de transferencia uno puede calcular fcilmente la ecuacin de diferencia del sistema.

2
(z + 1)
H (z) = (15.48)
z 12 z + 43
 

Dada una funcin de transferencia de un ltro, queremos encontrar la funcin de diferencia. Para
hacer esto, expanda los os polinomios y divdalos por el orden mas alto de z.
(z+1)(z+1)
H (z) =
(z 21 )(z+ 43 )
z 2 +2z+1 (15.49)
= z 2 +2z+1 38
1+2z 1 +z 2
= 1+ 14 z 1 38 z 2

De esta funcin de transferencia, los coecientes de los dos polinomios sern nuestros valores de
ak y bk que se encuentran el la forma general de la funcin de diferencias, (15.43). Usando estos
coecientes y la forma anterior de la funcin de transferencia, podemos escribir la ecuacin de
diferencia as:
1 3
x [n] + 2x [n 1] + x [n 2] = y [n] + y [n 1] y [n 2] (15.50)
4 8
En el ltimo paso re-escribimos la ecuacin de diferencia en una forma ms comn, mostrando su
naturaleza recurrente del sistema.

1 3
y [n] = x [n] + 2x [n 1] + x [n 2] + y [n 1] + y [n 2] (15.51)
4 8

15.6.4 Resolviendo un LCCDE


Para resolver este tipo de ecuaciones y poderlas usar en el anlisis de sistemas LTI, tenemos que encontrar
las salidas del sistema que estn basadas en una entrada del sistema, x (n) y basadas en un conjunto de
condiciones iniciales. Existen dos mtodos para resolver un LCCDE: el mtodo directo, y el mtodo
indirecto, este ltimo basado en la transformada-z. En las siguientes subsecciones se explica brevemente
las formulas para resolver los LCCDE as como el uso de estos dos mtodos.

15.6.4.1 Mtodo Directo

La solucin nal de la salida, utilizando este mtodo, es una suma dividida en dos partes expresadas de la
siguiente manera:
y (n) = yh (n) + yp (n) (15.52)

La primera parte, yh (n), se conoce como la solucin homognea y la segunda parte, yh (n),se le llama
las solucin particular. El siguiente mtodo es similar al usado para resolver las ecuaciones diferenciales,
as que si ya tomo un curso de ecuaciones diferenciales, este mtodo le ser familiar.

Available for free at Connexions <http://cnx.org/content/col10373/1.2>


286 CHAPTER 15. LA TRANSFORMADA Z Y FILTROS DIGITALES

15.6.4.1.1 Solucin Homognea

Se empieza al asumir que la entrada es igual a cero, x (n) = 0. Despus, tenemos que solucionar la ecuacin
de diferencia homognea:
N
X
ak y [n k] = 0 (15.53)
k=0
Para resolver esto, asumimos que la solucin tiene la forma de un exponencial. Usaremos lambda, , para
representar los trminos exponenciales. Ahora tenemos que resolver la siguiente ecuacin:

N
X
ak nk = 0 (15.54)
k=0

Expandiremos la ecuacin y factorizaremos todos los trminos de lambda. Esto nos dar un polinomio
dentro del parntesis, lo cual se conoce como polinomio caracterstico. Las races de este polinomio son
la clave para resolver esto ecuacin. Si se tienen races diferentes, la solucin general es la siguiente:

n n n
yh (n) = C1 (1 ) + C2 (2 ) + + CN (N ) (15.55)

Sin embargo, si la ecuacin caracterstica contiene mltiples races la solucin mostrada anterior mente sufre
algunos cambios. La versin modicada de la ecuacin se muestra abajo, donde 1 tiene K races:

n n n n n n
yh (n) = C1 (1 ) + C1 n(1 ) + C1 n2 (1 ) + + C1 nK1 (1 ) + C2 (2 ) + + CN (N ) (15.56)

15.6.4.1.2 Solucin Particular

Esta solucin, yp (n), ser aquella que resuelva la ecuacin de diferencia general:

N
X M
X
ak yp (n k) = bk x (n k) (15.57)
k=0 k=0

Para resolverla, nuestra estimacin de la solucin para yp (n) tomara la forma de nuestra entrada, x (n).
Despus de esto, uno nada mas tiene que sustituir la respuesta y resolver la ecuacin.

15.6.4.2 Mtodo Indirecto

Este mtodo utiliza la relacin entre la ecuacin de diferencia y la transformada-z, para encontrar la solu-
cin. La idea es convertir la ecuacin de diferencia a su transformada-z, para obtener una salida, fue vista
anteriormente (Section 15.6.2: Formulas Generales para la Ecuacin de Diferencia),. Y (z). Al usar la
transformada inversa (Section 15.6.2.2: Conversin a la Transformada-z) y usando la expansin parcial de
fracciones, podemos encontrar la solucin.

15.7 Entendiendo las Gracas de Polos y Ceros en el Plano-Z14


15.7.1 Introduccin a los Polos y Ceros de la Trasformada-z
Despus de encontrar la transformada-z del sistema, uno puede usar la informacin del polinomio para
representar la funcin grcamente y as observar sus caractersticas. La transformada-z tendr la siguiente
estructura, basada en las funciones racionales (Section 15.5):

P (z)
X (z) = (15.58)
Q (z)
14 This content is available online at <http://cnx.org/content/m12966/1.1/>.

Available for free at Connexions <http://cnx.org/content/col10373/1.2>


287

Los dos polinomios, P (z) y Q (z), nos dejan encontrar los polos y ceros (Section 14.6) de la transformada-
z.

Denition 15.4: ceros


1. valor(es) dez donde P (z) = 0.
2. Las frecuencias complejas que hacen que la ganancia de la funcin de transferencia del ltro sea
cero.

Denition 15.5: polos


1. valor(es) de z donde Q (z) = 0.
2. Las frecuencias complejas que hacen que la ganancia de la funcin de transferencia del ltro sea
innita.

Example 15.11
Esta es la funcin de transferencia con polos y ceros.

z+1
H (z) =
z 12 z + 43
 

Los ceros son: {1}


1 3
Los polos son:
2, 4

15.7.2 El Plano-z
Despus de encontrar los polos y ceros de una trasformada-z, se pueden gracar en el plano-z. El plano-z
es un plano complejo con ejes reales e imaginarios para la variable compleja de z. La posicin del plano
complejo es dada por rej y el ngulo se da del lado positive del eje real del plano y se escribe . Al gracar
los polos y ceros, los polos son mostrados con "x" y los ceros con "o". La siguiente gura muestra el plano-z,
as como algunos ejemplos de como gracar polos y ceros en algn lugar particular en el plano.

Plano-Z

Figure 15.18

Available for free at Connexions <http://cnx.org/content/col10373/1.2>


288 CHAPTER 15. LA TRANSFORMADA Z Y FILTROS DIGITALES

15.7.3 Ejemplos de Gracas de Polos y Ceros


Esta seccin contiene ejemplos de como encontrar polos y ceros de una funcin de transferencia y el como
gracarlos en el plano-z.

Example 15.12: Graca Simple de Polos y Ceros

z
H (z) = 1
 3

z 2 z+ 4

Los ceros son: {0}


1 3
Los polos son:
2, 4

Gracas de Polos y Ceros

Figure 15.19: Usando los ceros y polos de la funcion de transferencia, un cero es gracado a el valor
cero y los dos polos se colocan en 21 y 43

Example 15.13: Graca Compleja de Polos y Ceros

(z j) (z + j)
H (z) = 1
21 j z 12 + 12 j
 
z 2

Los ceros son: {j,


 j}
1, 12 + 12 j, 12 12 j

Los polos son:

Available for free at Connexions <http://cnx.org/content/col10373/1.2>


289

Gracas de Polos y Ceros

Figure 15.20: Usando los ceros y polos de la funcion de transferencia, los ceros son gracados en (j),
los polos son colocados en 1, 21 + 12 j y 12 12 j

MATLAB- si se usa este programa, entonces usted podr usar funciones que crean este tipo de gracas
rpidamente. Abajo se muestra un programa que graca polos y ceros del ejemplo anterior.

% Set up vector for zeros


z = [j ; -j];

% Set up vector for poles


p = [-1 ; .5+.5j ; .5-.5j];

figure(1);
zplane(z,p);
title('Pole/Zero Plot for Complex Pole/Zero Plot Example');

15.7.4 Gracas de Polos y Ceros y la Regin de Convergencia


La regin de convergencia (ROC) para X (z) en el plano-z de puede determinar de la graca de polos y
ceros. Aunque varias ROC pueden existir, donde cada una corresponde a una respuesta al impulse diferente,
existen opciones que son mas practicas. Un ROC se puede escoger para hacer la funcin de transferencia
causal y/o estable dependiendo de la graca de polos y ceros.

Propiedades de Filtro Sacadas de la ROC

Si la ROC se extiende hacia afuera desde su ultimo polo, entonces el sistema es causal.
Si la ROC incluye el crculo unitario, entonces el sistema es estable.

Available for free at Connexions <http://cnx.org/content/col10373/1.2>


290 CHAPTER 15. LA TRANSFORMADA Z Y FILTROS DIGITALES

La siguiente graca es un posible ROC para la transformada-z del ejemplo graca simple de polos y ceros
(Example 15.12: Graca Simple de Polos y Ceros) la regin mostrada indica la ROC elegida para el ltro.
Podemos inferior que el ltro ser causal y estable ya que tiene las propiedades mencionadas anteriormente.

Example 15.14

z
H (z) = 1
 3

z 2 z+ 4

La regin de convergencia para la graca de polos y ceros

Figure 15.21: La area representa la ROC para la funcion de transferencia.

15.7.5 Respuesta de Frecuencia y el Plano-Z


La razn por lo cual es importante el entender y crear las gracas de polos y ceros es su habilidad de ayudar
en el diseo de ltros. Basado en la locacin de los polos y ceros, la respuesta de la magnitud del ltro se
puede comprender. Al empezar con este tipo de graca, uno puede disear un ltro y obtener su funcin de
15
transferencia fcilmente. Vea esta seccin para obtener informacin sobre la relacin de la graca de polos
y ceros con la respuesta de frecuencia.

15.8 Diseo de Filtros usando la Graca de Polos y Ceros de la


Transformada-Z16
15.8.1 Estimando la Respuesta de Frecuencia de un Plano-Z
Un factor de motivacin para el anlisis de la graca de polos y ceros es la relacin de la respuesta de
frecuencia del sistema. Basado en la posicin de los polos y ceros, uno puede determinar la respuesta de la

15 "Discrete Time Filter Design" <http://cnx.org/content/m10548/latest/>


16 This content is available online at <http://cnx.org/content/m12967/1.3/>.

Available for free at Connexions <http://cnx.org/content/col10373/1.2>


291

frecuencia. Este es un resultado de la correlacin entre la repuesta de frecuencia y la funcin de transferencia


evaluada en el crculo unitario de la graca de polos y ceros. La respuesta de frecuencia, o DTFT, del sistema
es denida por:

H (w) = H (z) |z,z=ejw


PM (jwk) (15.59)
k=0 bk e
= PN
a e (jwk)
k=0 k

Al factorizar la funcin de transferencia a sus polos y ceros y el multiplicar el numerador y denominador


por ejw se obtiene lo siguiente:
QM jw
b0 k=1 |e ck |
H (w) = | | QN (15.60)
a0 k=1 |e
jw dk |
De la (15.60) obtenemos la respuesta de frecuencia en una forma que se puede usar para interpretar las
caractersticas fsicas de las respuesta de frecuencia del ltro. El numerador y denominador contienen un
producto en trminos de la forma |ejw h|, donde h es cero, marcado por ck o un polo, descrito por dk .
Los vectores son usados para representar el trmino y sus partes en el plano complejo. El polo o cero, h, es
un vector que viene del origen a la su lugar en cualquier parte del plano complejo y ejw es un vector que
viene del origen hasta una posicin en el circulo unitario. El vector que conecta estos dos puntos, |ejw h|,
conecta el polo o cero a un lugar en el circulo unitario que depende del valor de w. De esto, podemos empezar
entender como la magnitud de la respuesta de frecuencia es un radio de distancias de polos y ceros presente
en el plano-z y w va de cero a pi. Estas caractersticas no ayudan a entender.|H (w) | de la siguiente manera:
Q
b0 distancias de los ceros
|H (w) | = | | Q (15.61)
a0 distancias de los polos
En conclusin, usando las distancias del crculo unitario a los polos y ceros, podemos gracar la respuesta de
la frecuencia del sistema. Cuando w va de 0 a 2 , las siguientes dos propiedades, tomadas de las ecuaciones
anteriores, especican como se debe gracar |H (w) |.
Mientras se mueva alrededor del crculo unitario. . .

1. si esta cercas de un cero, la magnitud es chica. Si el cero esta sobre el circulo unitario, entonces la
respuesta de frecuencia es cero en eso punto.
2. si esta cercas de un polo, la magnitud es grande. Si el polo esta sobre el circulo unitario, entonces la
respuesta de frecuencia es innita en ese punto.

15.8.2 Gracando la Respuesta de Frecuencia de una Graca de Polos y Ceros


Veremos varios ejemplos donde se determina la magnitud de la respuesta de frecuencia de aun graca de
polos y ceros de una transformada-z. Si se le olvido o no sabe que es una graca de polos y ceros, vea el
17
modulo de gracas de polos y ceros .

Example 15.15
En este ejemplo, tomaremos una transformada-z sencilla mostrada aqu:

H (z) = z + 1 = 1 + z 1

H (w) = 1 + e(jw)
Para este ejemplo, algunos de los vectores representados por |ejw h|, para valores aleatorios de
w, son gracados especcamente en el plano complejo, visto en la siguenta gura. Estos vectores
muestran como la amplitud de la respuesta de frecuencia cambia cuando w va de 0 a 2 , tambin
muestra el signicado fsico en trminos de (15.60). Se puede observar que cuando w = 0, el vector
17 "Understanding Pole/Zero Plots on the Z-Plane" <http://cnx.org/content/m10556/latest/>

Available for free at Connexions <http://cnx.org/content/col10373/1.2>


292 CHAPTER 15. LA TRANSFORMADA Z Y FILTROS DIGITALES

es mayor as que la respuesta de frecuencia tendr la mayor amplitud. Cuando w se acerca a , el


tamao del vector disminuye junto con la amplitud de |H (w) |. Ya que la transformada no contiene
polos, nada mas tenemos un vector en vez de un radio como en la (15.60).

(a) Graca de polos y ceros (b) Respuesta de Frecuancia: |H(w)|

Figure 15.22: La primera gura reprecenta la graca de polos y ceros con unos cuantos vectores, la
segunda muestra la respuesta de frecuancia que es maxima en +2 y es gracada entre positivo y negativo
.

Example 15.16
En este ejemplo, una funcin de transferencia ms compleja es analizada para poder representar
la respuesta de la frecuencia del sistema

z 1
H (z) = 1 =
z 2 1 12 z 1

1
H (w) =
1 12 e(jw)
Pedemos observar dos gures que describes las ecuaciones anteriores. La cnxn
target="eg2_g1"/> representa la graca sencilla de polos y ceros de la transformada-z, H (w).
Figure 15.23(b) (Respuesta de Frecuencia: |H(w)|) La secunda gura muestra la magnitud de la
respuesta de frecuencia. Usando las formulas y lo dicho en la seccin anterior, podemos ver que
cuando w = 0 la frecuencia ser mxima ya que en este valor de, w el polo esta mas cerca del
circulo unitario. El radio de la (15.60) nos ayuda a ver la conduccin matemticamente y ver la
relacin entre las distancias del circulo unitario y los polos y los ceros. Cuando w se mueve de 0
a , observamos como el cero empieza a cubrir los efectos del polo y as hacer que la respuesta de
frecuencia se vuelve casi igual a 0.

Available for free at Connexions <http://cnx.org/content/col10373/1.2>


293

(a) Graca de Polos y Ceros (b) Respuesta de Frecuencia: |H(w)|

Figure 15.23: La primera r=gura representa la graca de polos y ceros, la segunda muestra la
respuesta de frecuencia que es maxima en +2 y es gracada entre positivo y negativo .

Available for free at Connexions <http://cnx.org/content/col10373/1.2>


294 CHAPTER 15. LA TRANSFORMADA Z Y FILTROS DIGITALES

Available for free at Connexions <http://cnx.org/content/col10373/1.2>


Chapter 16

Tareas

Available for free at Connexions <http://cnx.org/content/col10373/1.2>

295
296 CHAPTER 16. TAREAS

16.1 Homework #11


note: Noon, Thursday, September 5, 2002

16.1.1 Assignment 1
Homework, tests, and solutions from previous oerings of this course are o limits, under the honor code.

16.1.1.1 Problem 1

Form a study group of 3-4 members. With your group, discuss and synthesize the major themes of this
week of lectures. Turn in a one page summary of your discussion. You need turn in only one summary per
group, but include the names of all group members. Please do not write up just a "table of contents."

16.1.1.2 Problem 2

Construct a WWW page (with your picture) and email Mike Wakin (wakin@rice.edu) your name (as you
want it to appear on the class web page) and the URL. If you need assistance setting up your page or
taking/scanning a picture (both are easy!), ask your classmates.

16.1.1.3 Problem 3: Learning Styles


2 3
Follow this learning styles link (also found on the Elec 301 web page ) and learn about the basics of
learning styles. Write a short summary of what you learned. Also, complete the "Index of learning styles"
self-scoring test on the web and bring your results to class.

16.1.1.4 Problem 4

Make sure you know the material in Lathi, Chapter B, Sections 1-4, 6.1, 6.2, 7. Specically, be sure to review
topics such as:

complex arithmetic (adding, multiplying, powers)


nding (complex) roots of polynomials
4
complex plane and plotting roots
vectors (adding, inner products)

16.1.1.5 Problem 5: Complex Number Applet


5 6
Reacquaint yourself with complex numbers by going to the course applets web page and clicking on the
Complex Numbers applet (may take a few seconds to load).
(a) Change the default add function to exponential (exp). Click on the complex plane to get a blue arrow,
which is your complex number z. Click again anywhere on the complex plane to get a yellow arrow, which
is equal to ez . Now drag the tip of the blue arrow along the unit circle on with |z| = 1 (smaller circle). For
which values of z on the unit circle does ez also lie on the unit circle? Why?
(b) Experiment with the functions absolute (abs), real part (re), and imaginary part (im) and report
your ndings.

1 This content is available online at <http://cnx.org/content/m10826/2.9/>.


2 http://www2.ncsu.edu/unity/lockers/users/f/felder/public/Learning_Styles.html
3 http://www-dsp.rice.edu/courses/elec301/
4 "The Complex Plane" <http://cnx.org/content/m10596/latest/>
5 "Complex Numbers" <http://cnx.org/content/m0081/latest/>
6 http://www.dsp.rice.edu/courses/elec301/applets.shtml

Available for free at Connexions <http://cnx.org/content/col10373/1.2>


297

16.1.1.6 Problem 6: Complex Arithmetic

Reduce the following to the Cartesian form, a + jb. Do not use your calculator!
 20
1j

(a)
2
1+2j
(b)
3+4j

1+ 3j
(c)
3j
(d) j
(e) jj

16.1.1.7 Problem 7: Roots of Polynomials

Find the roots of each of the following polynomials (show your work). Use MATLAB to check your answer
with the roots command and to plot the roots in the complex plane. Mark the root locations with an 'o'.
Put all of the roots on the same plot and identify the corresponding polynomial (a, b, etc...).
(a) z 2 4z
2
(b) z 4z + 4
2
(c) z 4z + 8
2
(d) z + 8
2
(e) z + 4z + 8
2
(f ) 2z + 4z + 8

16.1.1.8 Problem 8: Nth Roots of Unity


j2
e N is called an Nth Root of Unity.
(a) Why?
j2  2
(b) Let z=e 7 . Draw z, z , . . . , z 7 in the complex plane.
j4  2
(c) Let z=e 7 . Draw z, z , . . . , z 7 in the complex plane.

16.1.1.9 Problem 9: Writing Vectors in Terms of Other Vectors

A pair of vectors u C2 and v C2 are called linearly independent if

u + v = 0 if and only if ==0

It is a fact that we can write any vector in C2 as a weighted sum (or linear combination) of any two
linearly independent vectors, where the weights
are and
complex-valued.

3 + 4j 1 5
(a) Write as a linear combination of and . That is, nd and such that
6 + 2j 2 3

3 + 4j 1 5
= +
6 + 2j 2 3

x1 1 5
(b) More generally, write x= as a linear combination of and . We will denote
x2 2 3
the answer for a given x as (x) and (x).
(c) Write the answer to (a) in matrix form, i.e. nd a 22 matrix A such that

x1 (x)
A =
x2 (x)

Available for free at Connexions <http://cnx.org/content/col10373/1.2>


298 CHAPTER 16. TAREAS

(d) Repeat (b) and (c) for a general set of linearly independent vectors u and v.

16.1.1.10 Problem 10: Fun with Fractals

A Julia set J is obtained by characterizing points in the complex plane. Specically, let f (x) = x2 + with
complex, and dene
g0 (x) = x

g1 (x) = f (g0 (x)) = f (x)

g2 (x) = f (g1 (x)) = f (f (x))

.
.
.

gn (x) = f (gn1 (x))


Then for each x in the complex plane, we say xJ if the sequence

{|g0 (x) |, |g1 (x) |, |g2 (x) |, . . . }

does not tend to innity. Notice that if x J, then each element of the sequence {g0 (x) , g1 (x) , g2 (x) , . . . }
also belongs to J.
For most values of , the boundary of a Julia set is a fractal curve - it contains "jagged" detail no
matter how far you zoom in on it. The well-known Mandelbrot set contains all values of for which the
corresponding Julia set is connected.
(a) Let = 1. Is x = 1 in J ?
(b) Let = 0. What conditions on x ensure that x belongs to J?
(c) Create an approximate picture of a Julia set in MATLAB. The easiest way is to create a matrix of
complex numbers, decide for each number whether it belongs to J, and plot the results using the imagesc
command. To determine whether a number belongs to J, it is helpful to dene a limit N on the number of
iterations of g . For a given x, if the magnitude |gn (x) | remains below some threshold M for all 0 n N ,
we say that x belongs to J . The code below will help you get started:

N = 100; % Max # of iterations


M = 2; % Magnitude threshold
mu = -0.75; % Julia parameter
realVals = [-1.6:0.01:1.6];
imagVals = [-1.2:0.01:1.2];

xVals = ones(length(imagVals),1) * realVals + ...


j*imagVals'*ones(1,length(realVals));

Jmap = ones(size(xVals));
g = xVals; % Start with g0

% Insert code here to fill in elements of Jmap. Leave a '1'


% in locations where x belongs to J, insert '0' in the
% locations otherwise. It is not necessary to store all 100
% iterations of g!

Available for free at Connexions <http://cnx.org/content/col10373/1.2>


299

imagesc(realVals, imagVals, Jmap);


colormap gray;
xlabel('Re(x)');
ylabel('Imag(x)');

This creates the following picture for = 0.75, N = 100, and M = 2.

Figure 16.1: Example image where the x-axis is Re (x) and the y-axis is Im (x).

Using the same values for N, M, and x, create a picture of the Julia set for = 0.391 0.587j . Print
out this picture and hand it in with your MATLAB code.

note: Try assigning dierent color values to Jmap. For example, let Jmap indicate the rst
iteration when the magnitude exceeds M. Tip: try imagesc(log(Jmap)) and colormap jet for a
neat picture.

Available for free at Connexions <http://cnx.org/content/col10373/1.2>


300 GLOSSARY

Glossary

1 1 Linealmente Independiente
Un conjunto dado de vectores {x1 , x2 , . . . , xn }, es linealmente independiente si

c1 x1 + c2 x2 + + cn xn = 0

solo cuando c1 = c2 = = cn = 0
Example: Dados los siguientes dos vectores:

3
x1 =
2

6
x2 =
4

Estos son no linealmente independientes por el siguiente argumento, el cual por inspeccin,
se puede ver que no se apega a la denicin anterior de independencia lineal:

(x2 = 2x1 ) (2x1 + x2 = 0)

. Otro mtodo para ver la independencia de los vectores es gracando los vectores. Observando
estos dos vectores geomtricamente (como en la siguiente Figure 5.1), uno puede otra vez probar
que estos vectores son no linealmente independientes.

B Base Normada
una base (Section 5.1.3: Bases) {bi } donde cada bi tiene una norma unitaria

k bi k= 1 , i Z (7.19)

Base Ortogonal
una base {bi } en donde los elementos son mutuamente ortogonales

< bi , bj >= 0 , i 6= j

Base Ortonormal
Una base que es normalizada y ortogonal

k bi k= 1 , i Z

< bi , bj > , i 6= j
Base
Una base para Cn es un conjunto de vectores que: (1) generan Cn y (2) es linealmente
independiente.

Available for free at Connexions <http://cnx.org/content/col10373/1.2>


GLOSSARY 301

C Ceros
1. El valor(es) para z donde el numerador de la funcin de trasferencia es iguala cero
2. Las frecuencias complejas que hacen que la ganancia de la funcin de transferencia del ltro sea
cero.

ceros
1. valor(es) dez donde P (z) = 0.
2. Las frecuencias complejas que hacen que la ganancia de la funcin de transferencia del ltro sea
cero.

Convergencia Uniforme
La secuencia (Section 10.1) {gn } |
n=1 converge uniformemente a la funci g si para cada >0
existe un entero N tal que n N implica que
|gn (t) g (t) |  (10.7)

para todo t R.

D Desigualdad de Cauchy-Schwarz
Para x, y en un espacio de producto interno

| < x, y > | k x kk y k
que mantiene la igualdad si y solo si x y y son linealmente dependientes (Section 5.1.1:
Independencia Lineal), es decir x = y para un escalar .
dominio
El grupo, o conjunto, de valores que son denidos por una funcin.

Example: Usando la funcin racional anterior, (15.38), el dominio se puede denir como
cualquier numero real x donde x no iguala a 1 o negativa 3. Expresando esto matemticamente,
obtenemos lo siguiente:

{ x R | (x 6= 3) and (x 6= 1) } (15.40)

E Eigenvector
Un eigenvector de A es un vector v Cn tal que

Av = v (5.2)

donde es llamado el eigenvalor correspondiente. A solo cambia la longitud de v, no su


direccin.

Espacio Vectorial
Un espacio vectorial S es una coleccin de vectores tal que (1) si f1 S f1 S para todo
escalar (donde R C) y (2) si f1 S , f2 S , entonces (f1 + f2 ) S

F Funcin Racional
Para dos polinomios cualquiera, A y B, su fraccin es conocida cono una funcin racional.

Example: Aqu se muestra un ejemplo de una funcin racional, f (x). Note que el numerador y
denominador pueden ser polinomios de cualquier orden, pero la funcin racional es indenida
cuando el denominador es cero.
x2 4
f (x) = (15.38)
2x2+x3

Available for free at Connexions <http://cnx.org/content/col10373/1.2>


302 GLOSSARY

L La Ecuacin de Diferencia
Es una ecuacin que muestra la relacin entra valores consecutivos de una secuencia y la
diferencia entre ellos. Usualmente se escribe en una ecuacin recurrente para que la salida del
sistema se pueda calcular de las entradas de la seal y sus valores anteriores.

Example:
y [n] + 7y [n 1] + 2y [n 2] = x [n] 4x [n 1] (15.42)

lmite
Una secuencia {gn } |
n=1 converge a un lmite gR si para todo >0 existe un entero N tal que

|gi g| <  , i N
Usualmente denotamos un lmite escribiendo

limit gi = g
i


gi g

O Ortogonal
Decimos que x y y son ortogonales si:
< x, y >= 0

P polos
1. El valor(es) para z donde el denominador de la funcin de transferencia es igual a cero
2. Las frecuencias complejas que hacen de la ganancia de la funcin de transferencia del ltro se
innita.

polos
Tambin llamadas como singularidades estos son los punto s en los cuales Lx1 (s) explota.

polos
1. valor(es) de z donde Q (z) = 0.
2. Las frecuencias complejas que hacen que la ganancia de la funcin de transferencia del ltro sea
innita.

Producto Interno
El producto interno esta denido matemticamente de la siguiente manera:

< x, y > = yT x

x0

  x1
= y0 y1 ... yn1 (7.1)

.
.
.


xn1
Pn1
= i=0 xi yi

S secuencia

Available for free at Connexions <http://cnx.org/content/col10373/1.2>


GLOSSARY 303

Una secuencia es una funcin gn denida en los enteros positivos 'n'. Tambin denotamos una
secuencia por: {gn } |
n=1
Example: Una secuencia de nmeros reales:

1
gn =
n
Example: Una secuencia vector:

n
sin 2
gn =
n

cos 2

Example: Una secuencia de una funcin:



1
1 if 0t< n
gn (t) =
0 otherwise

note: Una funcin puede ser pensada como un vector de dimensin innito donde por
cada valor de 't' tenemos una dimensin.

Subespacio Generado
7
El subespacio generado o span del conjuto de vectores {x1 , x2 , . . . , xk } es el conjunto de
vectores que pueden ser escritos como una combinacin lineal de {x1 , x2 , . . . , xk }

subespacio generado ({x1 , . . . , xk }) = {1 x1 + 2 x2 + + k xk , i Cn }

Example: Dado el vector



3
x1 =
2

el subespacio generado de x1 es una linea.


Example: Dado los vectores

3
x1 =
2

1
x2 =
2

El subespacio generado por estos vectores es C2 .

7 "Subspaces", Denition 2: "Span" <http://cnx.org/content/m10297/latest/#defn2>

Available for free at Connexions <http://cnx.org/content/col10373/1.2>


304 INDEX

Index of Keywords and Terms


Keywords are listed by the section with that keyword (page numbers are in parentheses). Keywords
do not necessarily appear in the text of the page. They are merely associated with that section. Ex.
apples, 1.1 (1) Terms are referenced by the page they appear on. Ex. apples, 1

1 1 Linealmente Independiente, 75 15.8(290)


Ceros, 259, 283, 287
A alfabeto, 1.6(23), 25
circular, 4.3(68), 6.7(110)
alias, 13.5(240), 13.6(243)
coecientes, 6.2(96)
aliasing, 8.2(168), 13.5(240), 240,
coecientes de fourier, 6.2(96), 97, 6.3(99),
13.6(243)
6.9(114)
amplitud compleja, 20
complejidad, 191, 9.3(193)
Anti-Aliasing, 13.6(243)
complejo, 1.6(23), 8.2(168), 15.7(286)
anticausal, 1.1(1)
complex vector spaces, 7.1(125)
anticausales, 3
compuesto, 196
anlisis de fourier, 8.2(168)
Condiciones de Dirichlet, 6.10(116)
anlogo, 1.1(1), 2
Condiciones de Dirichlet debles, 6.10(116)
anlogos, 23
Condiciones de Dirichlet fuertes, 6.10(116)
aperiodica, 96
condiciones iniciales, 60, 284
aperidico, 1.1(1)
conjugados, 173
approximacin, 7.13(161)
conmutativa, 42, 49, 63
armnico, 8.2(168)
continua, 116
asintoticamente vertical, 282
continuo, 1

B base, 5.1(75), 78, 78, 78, 7.7(142),


continuous time, 3.2(42), 3.4(55),
11.1(207)
7.8(146), 146, 7.9(150), 7.10(151),
converge, 10.3(204)
7.11(158), 173, 236
convergencia, 6.9(114), 116, 10.1(199),
base cannica, 78, 7.8(146)
10.2(201), 10.3(204)
base normada, 7.7(142), 142, 143
convergencia de punto por punto, 6.9(114)
base ortogonal, 7.7(142), 143
convergencia no-uniforme, 6.11(118), 119
base ortonormal, 7.7(142), 144, 7.8(146),
convergencia uniforme, 10.3(204), 204
8.2(168), 174
convergente, 10.3(204)
bases, 5.1(75), 8.2(168), 8.3(175)
convoluciones, 4.3(68)
bases estandard, 8.3(175)
Convolucin, 4.2(63), 63, 4.3(68),
BIBO, 3.4(55)
6.7(110), 12.2(223)
bounded input bounded output, 3.4(55)
convolucin circular, 4.3(68), 68, 6.7(110)
buttery, 9.3(193)
convolucin linear, 68

C cada, 97 convolution, 3.2(42), 3.3(48)

Cartesian, 297 convolver, 4.3(68)

cascade, 2.2(33) Cooley-Tukey, 9.1(191), 9.3(193)

casi en todas partes, 6.11(118) cpnvergencia puntual, 10.2(201)

cauch-schwarz, 7.5(132) csi, 7.5(132)

cauchy, 7.5(132) CTFT, 12.1(221), 13.1(227)

causal, 1.1(1), 2.1(29), 31, 2.2(33), 289 cuachy, 7.5(132)

causales, 3
cero, 14.4(255), 14.6(259), 15.7(286),
D decompose, 7.8(146)
delta de Dirac, 14

Available for free at Connexions <http://cnx.org/content/col10373/1.2>


INDEX 305

descomponer, 1.6(23), 147 energa, 115


desigualdad de cauchy-schwarz, 7.5(132), 132 entrada acotada-salida acotada (BIBO), 32
desplazamiento circular, 8.5(184), 187 Escala en el eje del Tiempo, 1.2(9)
Desplazamiento en el eje del Tiempo, 1.2(9) escalamiento en el tiempo, 12.2(223)
desplazamiento en el tiempo, 12.2(223) escaln unitario, 1.3(12), 14
desplazamientos circulares, 8.5(184), 184 espacio de funciones, 79
Detector de Filtro Acoplado, 133 espacio de Hilbert, 131, 7.6(139), 7.8(146),
determinant, 5.3(79) 7.10(151), 7.13(161)
determinstica, 7 espacio de las funciones, 7.9(150)
dft, 4.3(68), 8.5(184), 9.2(192), espacio de producto interno, 131
11.2(208), 11.3(210) espacio lineal de funciones., 125
diferenciacin en el tiempo, 12.2(223) espacio lineal normado, 127, 131
diferenciales, 3.1(39) espacio normado, 7.6(139)
digital, 1.1(1), 2, 11.6(214), 11.7(215) Espacio Vectorial, 125, 7.6(139), 7.7(142),
Dirichlet, 116 7.9(150)
discontinuidad, 6.9(114), 116 espacio vectorial complejo, 125
discrete time, 3.4(55) espacio vectorial normado, 127
discreto, 1, 8.2(168), 13.7(245) espacio vectorial real, 125, 138
dominio, 282, 282 espacios de hilbert, 7.4(131), 7.6(139),
Dominio de la Frecuencia, 11.5(214) 7.7(142)
Dominio de la secuencia, 11.5(214) espacios de las funciones, 7.9(150)
dominio del tiempo, 4.1(59) espacios vectoriales, 7.6(139), 7.7(142)
dot product, 7.3(129) estable, 32, 289
DSP, 4.1(59), 11.7(215), 13.4(238), Euclidean norm, 7.2(127)
15.2(268) existencia, 116
DT, 4.2(63) expansin, 7.9(150), 7.10(151)
dtfs, 8.2(168), 8.3(175), 8.4(176) exponencial, 1.3(12), 1.6(23), 6.3(99)
DTFT, 11.2(208), 11.4(212), 13.1(227) exponencial complejo, 13, 1.5(19), 19
duracin nita, 270 Exponencial que Crece, 13
Exponencial que decae, 13
E ecuaciones diferenciales, 3.1(39)
ecuacin caracterstica, 41 F F, 62
ecuacin de diferencia de coeciente linear fasor, 20
constante, 283 fenmeno de Gibb, 121
Ecuacin de matriz, 8.3(175) fenmeno de Gibbs, 6.11(118), 119
ecuacin diferencial, 4.1(59), 59 FFT, 9.1(191), 9.2(192), 9.3(193),
eigen, 5.5(88) 11.2(208)
eigenfuncion, 6.4(100) ltro, 13.6(243), 15.8(290)
eigenfunciones, 6.2(96), 6.4(100) ltro de boxcar, 62
eigenfuncin, 5.6(89), 90 FIR, 62
eigenfunction, 5.3(79), 5.5(88) foltro acoplado, 7.5(132)
eigenfunctions, 5.5(88) forma, 193
eigenseal, 90 fourier, 5.3(79), 6.2(96), 6.3(99),
eigenvalor, 301, 81, 5.6(89) 6.4(100), 6.5(103), 6.6(106), 6.7(110),
eigenvalue, 5.3(79), 5.5(88) 6.8(111), 6.9(114), 6.10(116), 6.12(121),
eigenvalues, 5.3(79), 5.5(88) 7.10(151), 8.2(168), 8.3(175), 8.4(176),
eigenvector, 5.3(79), 80, 5.5(88), 5.6(89) 8.5(184), 9.2(192)
eigenvectors, 5.5(88) fourier series, 5.3(79)
ejemplo, 11.7(215) fourier transform, 11.1(207)
ejemplos, 11.7(215) fourier transform pairs, 5.7(92)
elec 301, 16.1(296) frecuencia continua, 11.4(212), 12.1(221)
elec301, 16.1(296) frecuencia Nyquist, 11.7(215), 13.4(238),

Available for free at Connexions <http://cnx.org/content/col10373/1.2>


306 INDEX

238 limitado en banda, 229


frequencia, 11.6(214) lineal, 2.1(29), 29
frequency shift keying, 7.5(132) Lineal de tiempo invariante, 5.6(89)
fsk, 7.5(132) linealidad, 12.2(223)
fuertes de Dirichlet, 118 linealmente independiente, 75
funcion, 15.5(281) linear, 2.2(33)
funcion racional, 15.5(281) linear combination, 297
funciones descontinas, 119 linear function spaces, 7.1(125)
funciones periodicas, 6.1(95) linear system, 5.3(79)
funciones racionales, 15.5(281), 281 linear time invariant, 3.2(42)
funcin de Delta de Dirac, 1.4(16) lineares de tiempo invariante, 279
funcin de sinc discreta, 218 linearly independent, 297
funcin de transferencia, 284 LTI, 3.2(42), 63, 5.5(88), 5.6(89),
funcin de valores complejos, 252, 252 6.2(96), 6.8(111)
funcin delta, 7.7(142) lmite, 199
funcin Delta de Dirac, 1.3(12)
funcin peridica, 95
M mariposa, 195
matched lter detector, 7.5(132)
Funcin Racional, 281
matched lters, 7.5(132)
G genera, 150 matriz de la base, 7.8(146), 148
matriz identidad, 148
H haar, 7.10(151)
mejor se aproxime, 163
hermitiana, 7.11(158)
metodo grco, 45
hilbert, 7.4(131), 7.5(132), 7.6(139),
modulacin, 12.2(223)
7.8(146), 7.10(151)
muestra, 13.5(240)
homework 1, 16.1(296)
muestreando, 13.4(238)
homework one, 16.1(296)
muestrear, 210

I I, 62
muestreo, 227, 13.3(236), 13.4(238),
13.5(240), 13.6(243)
identidad de Euler, 20
muestro unitario, 1.6(23)
impulso, 1.3(12), 1.4(16)
mutuamente ortogonales, 300
impulso unitario, 14
mximo, 131
independencia, 5.1(75)
mximos, 118
independencia lineal, 5.1(75)
mtodo directo, 285
inestable, 32
mtodo indirecto, 285
inner product, 7.3(129)
mnimos, 118
inner products, 7.3(129)
integral de convolucin, 42
N nivel de referencia, 135
intercepcin y, 282
no causal, 1.1(1)
intercesin en la x, 282
no-anticipativo, 31
interno, 7.4(131)
no-causal, 31
interpolacin, 13.2(231)
no-lineal, 29
invariante en el tiempo, 2.1(29), 30
nocausal, 2.1(29)
invariantes linear del tiempo, 63
nocausales, 3
IRR, 61
nolineal, 2.1(29)

K kronecker, 7.7(142)
norm, 7.2(127), 7.3(129)
norma, 127, 10.1(199), 10.2(201)

L La Ecuacin de Diferencia, 283 norma de convergencia, 10.1(199),

La Transformada Inversa de Laplace, 10.2(201)

14.5(257) normada, 7.7(142)

lado izquierdo, 272 normalizada, 300

laplace transform, 3.4(55) normalization, 7.2(127)

Available for free at Connexions <http://cnx.org/content/col10373/1.2>


INDEX 307

normed linear space, 7.2(127) Propiedades de la Transformada de Fourier


normed vector space, 7.2(127) Discreta en el Tiempo, 11.5(214)
norms, 7.2(127) propiedades simetricas, 6.6(106)
Nota importante:, 246 proporcional, 191
Nth Root of Unity, 297 proyeccin, 131, 7.13(161)
Nyquist, 11.6(214), 13.4(238) pulso cuadrado, 119
punto a punto, 121
O ondoleta, 7.10(151)
punto por punto, 6.9(114)
ondoleta de haar, 7.10(151)
puntual, 10.2(201)
ondoletas, 7.10(151), 151
puntualmente, 201
orden, 9.3(193), 14.5(257), 259, 284
orthogonal, 7.3(129) R R, 62
Ortogonal, 131, 7.7(142), 300 racional, 15.5(281)
ortogonales, 131, 162 real vector spaces, 7.1(125)
ortonormal, 7.7(142), 7.8(146), 8.2(168) reconstruccin, 13.2(231), 13.3(236),
13.4(238), 13.5(240)
P parallel, 2.2(33)
reconstruir, 13.3(236)
pares bilaterales de la transformada de
Reexin en el eje del Tiempo, 1.2(9)
Laplace, 251
region de convergerncia, 14.4(255)
pares de transformacion, 15.2(268)
regin de convergencia (ROC), 255
Parseval, 7.12(160)
respuesta al impulso, 4.2(63)
perfecta, 13.3(236)
respuesta de estado-cero, 39
periodicidad, 6.1(95)
respuesta de impulso, 19
periodico, 6.1(95), 8.4(176)
respuesta de salida-cero, 39
periodo, 2, 6.1(95), 95, 8.4(176)
ROC, 14.4(255), 15.1(263), 264, 269
periodo fundamental, 2
peridico, 1.1(1) S schwarz, 7.5(132)
Plancharel, 7.12(160) secuencia, 199
plano complejo, 1.5(19) secuencia de dos lados, 273
plano-s, 22 secuencia de funciones, 115, 10.3(204)
plano-z, 263, 15.7(286) secuencia del exponencial complejo, 24
pole, 3.4(55) secuencia del lado derecho, 271
polinomio, 15.5(281) secuencias, 1.6(23), 10.1(199), 10.3(204)
polinomio caracterstico, 286 seno, 1.6(23)
polo, 14.4(255), 14.6(259), 15.7(286), Senosoidal, 1.6(23), 6.3(99)
15.8(290) senosoidal compleja, 8.2(168)
polos, 14.5(257), 258, 260, 283, 287 senosoidales armnicos, 8.2(168), 169
procesamiento de seales digitales, 4.1(59), serie de potencia, 264, 269
11.7(215) series de fourier, 6.2(96), 6.3(99),
procesamiento de tiempo discreto, 13.7(245) 6.4(100), 6.6(106), 6.7(110), 6.9(114),
processing, 13.7(245) 6.10(116), 6.11(118), 6.12(121),
producto interno, 129, 130, 7.4(131), 7.7(142), 7.10(151), 8.1(167), 8.2(168),
7.5(132), 7.11(158) 8.3(175), 8.4(176)
producto punto, 129 series de fourier discretas en el tiempo,
productos internos, 7.5(132) 8.2(168), 168, 8.4(176)
projection, 7.3(129) series geomtricas, 215
promedio cuadrado, 121 seaes y sistemas, 4.2(63)
property, 3.3(48) seal, 4.2(63), 6.1(95), 6.12(121)
propiedad de desplazamiento, 1.3(12), seal aleatoria, 7
1.4(16), 18 seal de tamao nito, 8
propiedad de simetria, 6.6(106) seal exponencial compleja de
propiedades, 6.6(106) tiempo-continuo, 20

Available for free at Connexions <http://cnx.org/content/col10373/1.2>


308 INDEX

seal exponencial compleja en tiempo-discreto, 11.6(214), 11.7(215), 13.1(227),


21 14.6(259), 15.2(268)
seal impar, 4, 6.6(106) Tiempo-continuo, 12.2(223)
seal par, 1.1(1), 4, 6.6(106) tiempo-discreto, 1.6(23)
seales, 1.2(9), 1.3(12), 1.4(16), 1.5(19), time-invariant, 2.2(33)
1.6(23), 2.1(29), 6.2(96), 6.3(99), transforma, 86
8.1(167), 14.4(255), 14.6(259) transformacion lineal, 6.5(103)
seales de tiempo discreto, 227 transformacin linear, 104
Seales de Valores Simblicos, 1.6(23) transformada, 6.2(96)
seales ruidosas, 102 transformada de fourier, 4.3(68), 6.10(116),
seales y sistemas, 1.1(1) 8.1(167), 8.5(184), 9.2(192), 11.3(210),
signals, 3.2(42), 3.3(48), 3.4(55), 11.4(212), 11.5(214), 11.6(214),
11.1(207) 11.7(215), 12.1(221), 12.2(223),
simetria, 6.5(103), 6.6(106), 12.2(223) 15.1(263), 263
sinc, 13.3(236) Transformada de Fourier de Tiempo-Continuo,
Sinc Dirichlet, 11.2(208) 221
singularidades, 14.5(257), 257 transformada de fourier discreta, 4.3(68)
sistema, 4.3(68), 6.4(100), 6.12(121) Transformada de Fourier Discreta en el
sistema continuo, 29 Tiempo, 11.7(215)
sistema discreto, 29 transformada de laplace, 8.1(167),
sistema lineal, 3.1(39) 14.1(251), 14.2(253), 14.3(255),
sistema LTI, 6.2(96), 6.4(100) 14.4(255), 14.6(259)
sistemas, 1.6(23), 8.1(167), 14.4(255), transformada de z, 8.1(167)
14.6(259) Transformada Discreta de Fourier, 11.3(210),
sistemas de desplazamiento invariante, 210
4.1(59) transformada inversa, 6.2(96), 98
sistemas discretos, 4.1(59) Transformada Rpida de Fourier, 9.1(191),
solucin homognea, 285 9.2(192), 9.3(193)
solucin particular, 285 transformada z, 14.6(259), 15.2(268)
span, 5.5(88) transformada-z, 15.1(263), 263, 15.2(268)
stability, 3.4(55) transformada-z bilateral, 263
suaves, 102 transformada-z unilateral, 263
subespacio generado, 5.1(75), 77 transformadas de z unilateral y bilateral, 268
suma de convolucin, 63 transformadaz, 269
superposicin, 4.1(59) transforms, 149
superposition, 2.2(33) transpuesta, 7.11(158)
system, 5.5(88) trasformar, 98
systems, 3.2(42), 3.3(48), 3.4(55),
11.1(207)
U una seal de tamao innito, 8
uniforme, 10.3(204)
sntesis, 98
unilateral, 15.2(268)
T t-periodico, 6.1(95) unitario, 16
tamao nito, 280
Teorema de Nyquist, 13.4(238), 238
V valor complejo, 1.6(23)
valor real, 1.6(23)
Teorema de Parseval, 11.7(215)
variante en el tiempo, 2.1(29), 31
teora de control, 261
vector, 7.1(125), 10.2(201)
tiempo continuo, 1.1(1), 1.3(12), 3.1(39),
vector de coecientes, 7.8(146), 148
8.1(167), 12.1(221), 13.1(227), 227,
vector space, 7.1(125)
13.7(245), 14.1(251), 14.2(253),
vector spaces, 7.1(125)
14.3(255), 14.4(255), 14.6(259)
vectores, 10.2(201)
tiempo discreto, 1.1(1), 4.2(63), 8.1(167),
ventaja computacional, 194
8.3(175), 8.5(184), 11.4(212), 11.5(214),

Available for free at Connexions <http://cnx.org/content/col10373/1.2>


INDEX 309

W weighted sum, 297 lgebra lineal, 5.1(75)


nalogo, 11.6(214), 11.7(215)
Z z transform, 3.4(55)
z-transform, 15.5(281) nica, 147
zero, 3.4(55)

Available for free at Connexions <http://cnx.org/content/col10373/1.2>


310 ATTRIBUTIONS

Attributions
Collection: Seales y Sistemas
Edited by: Richard Baraniuk
URL: http://cnx.org/content/col10373/1.2/
License: http://creativecommons.org/licenses/by/2.0/

Module: "Clasicacin y Propiedades de las Seales"


By: Melissa Selik, Richard Baraniuk, Michael Haag, Ricardo von Borries
URL: http://cnx.org/content/m12818/1.8/
Pages: 1-9
Copyright: Ricardo von Borries, Erika Jackson, Fara Meza
License: http://creativecommons.org/licenses/by/2.0/
Based on: Signal Classications and Properties
By: Melissa Selik, Richard Baraniuk, Michael Haag
URL: http://cnx.org/content/m10057/2.16/

Module: "Operaciones para Seales"


By: Richard Baraniuk
URL: http://cnx.org/content/m12823/1.7/
Pages: 9-12
Copyright: Erika Jackson, Fara Meza
License: http://creativecommons.org/licenses/by/2.0/
Based on: Signal Operations
By: Richard Baraniuk
URL: http://cnx.org/content/m10125/2.5/

Module: "Seales tiles"


By: Melissa Selik, Richard Baraniuk
URL: http://cnx.org/content/m12819/1.11/
Pages: 12-16
Copyright: Erika Jackson, Fara Meza
License: http://creativecommons.org/licenses/by/2.0/
Based on: Useful Signals
By: Melissa Selik, Richard Baraniuk
URL: http://cnx.org/content/m10058/2.10/

Module: "Funcin de Impulso"


By: Melissa Selik, Richard Baraniuk
URL: http://cnx.org/content/m12824/1.9/
Pages: 16-19
Copyright: Erika Jackson, Fara Meza
License: http://creativecommons.org/licenses/by/2.0/
Based on: The Impulse Function
By: Melissa Selik, Richard Baraniuk
URL: http://cnx.org/content/m10059/2.16/

Available for free at Connexions <http://cnx.org/content/col10373/1.2>


ATTRIBUTIONS 311

Module: "El Exponencial Complejo"


By: Richard Baraniuk
URL: http://cnx.org/content/m12825/1.6/
Pages: 19-23
Copyright: Erika Jackson, Fara Meza
License: http://creativecommons.org/licenses/by/2.0/
Based on: The Complex Exponential
By: Richard Baraniuk
URL: http://cnx.org/content/m10060/2.18/

Module: "Seales en Tiempo-Discreto"


By: Don Johnson
URL: http://cnx.org/content/m12820/1.9/
Pages: 23-26
Copyright: Erika Jackson, Fara Meza
License: http://creativecommons.org/licenses/by/2.0/
Based on: Discrete-Time Signals
By: Don Johnson
URL: http://cnx.org/content/m0009/2.20/

Module: "Clasicacin y Propiedades de los Sistemas"


By: Melissa Selik, Richard Baraniuk
URL: http://cnx.org/content/m12822/1.4/
Pages: 29-32
Copyright: Fara Meza, Erika Jackson
License: http://creativecommons.org/licenses/by/2.0/
Based on: System Classications and Properties
By: Melissa Selik, Richard Baraniuk
URL: http://cnx.org/content/m10084/2.18/

Module: "Propiedades de los Sistemas"


By: Thanos Antoulas, JP Slavinsky
URL: http://cnx.org/content/m12826/1.2/
Pages: 33-38
Copyright: Erika Jackson, Fara Meza
License: http://creativecommons.org/licenses/by/2.0/
Based on: Properties of Systems
By: Thanos Antoulas, JP Slavinsky
URL: http://cnx.org/content/m2102/2.16/

Module: "Sistemas Lineales CT y Ecuaciones Diferenciales"


By: Michael Haag
URL: http://cnx.org/content/m12985/1.2/
Pages: 39-41
Copyright: Fara Meza, Erika Jackson
License: http://creativecommons.org/licenses/by/2.0/
Based on: CT Linear Systems and Dierential Equations
By: Michael Haag
URL: http://cnx.org/content/m10855/2.4/

Available for free at Connexions <http://cnx.org/content/col10373/1.2>


312 ATTRIBUTIONS

Module: "Convolucin de Tiempo-Continuo"


By: Melissa Selik, Richard Baraniuk
URL: http://cnx.org/content/m12828/1.4/
Pages: 42-48
Copyright: Fara Meza
License: http://creativecommons.org/licenses/by/2.0/
Based on: Continuous-Time Convolution
By: Melissa Selik, Richard Baraniuk
URL: http://cnx.org/content/m10085/2.25/

Module: "Propiedades de la Convolucin"


By: Melissa Selik, Richard Baraniuk
URL: http://cnx.org/content/m12829/1.2/
Pages: 48-55
Copyright: Erika Jackson, Fara Meza
License: http://creativecommons.org/licenses/by/2.0/
Based on: Properties of Convolution
By: Melissa Selik, Richard Baraniuk
URL: http://cnx.org/content/m10088/2.12/

Module: "Estabilidad BIBO"


By: Richard Baraniuk
URL: http://cnx.org/content/m12834/1.3/
Pages: 55-57
Copyright: Erika Jackson, Fara Meza
License: http://creativecommons.org/licenses/by/2.0/
Based on: BIBO Stability
By: Richard Baraniuk
URL: http://cnx.org/content/m10113/2.8/

Module: "Anlisis en el Dominio del Tiempo para Sistemas Discretos"


By: Don Johnson
URL: http://cnx.org/content/m12830/1.6/
Pages: 59-62
Copyright: Erika Jackson, Fara Meza
License: http://creativecommons.org/licenses/by/2.0/
Based on: Discrete-Time Systems in the Time-Domain
By: Don Johnson
URL: http://cnx.org/content/m10251/2.20/

Module: "Convolucin Discreta"


By: Ricardo Radaelli-Sanchez, Richard Baraniuk
URL: http://cnx.org/content/m12833/1.4/
Pages: 63-68
Copyright: Erika Jackson, Fara Meza
License: http://creativecommons.org/licenses/by/2.0/
Based on: Discrete-Time Convolution
By: Ricardo Radaelli-Sanchez, Richard Baraniuk
URL: http://cnx.org/content/m10087/2.17/

Available for free at Connexions <http://cnx.org/content/col10373/1.2>


ATTRIBUTIONS 313

Module: "Convolucin Circular y el DFT"


By: Justin Romberg
URL: http://cnx.org/content/m12831/1.3/
Pages: 68-72
Copyright: Erika Jackson, Fara Meza
License: http://creativecommons.org/licenses/by/2.0/
Based on: Circular Convolution and the DFT
By: Justin Romberg
URL: http://cnx.org/content/m10786/2.6/

Module: "Algebra Lineal: Conceptos Bsicos"


By: Michael Haag, Justin Romberg
URL: http://cnx.org/content/m12862/1.3/
Pages: 75-79
Copyright: Fara Meza, Erika Jackson
License: http://creativecommons.org/licenses/by/2.0/
Based on: Linear Algebra: The Basics
By: Michael Haag, Justin Romberg
URL: http://cnx.org/content/m10734/2.3/

Module: "Conceptos Bsicos de Vectores"


By: Michael Haag
URL: http://cnx.org/content/m12863/1.2/
Page: 79
Copyright: Fara Meza, Erika Jackson
License: http://creativecommons.org/licenses/by/2.0/
Based on: Vector Basics
By: Michael Haag
URL: http://cnx.org/content/m10844/2.0/

Module: "Eigenvectores y Eigenvalores"


By: Michael Haag, Justin Romberg
URL: http://cnx.org/content/m12870/1.2/
Pages: 79-85
Copyright: Fara Meza, Erika Jackson
License: http://creativecommons.org/licenses/by/2.0/
Based on: Eigenvectors and Eigenvalues
By: Michael Haag, Justin Romberg
URL: http://cnx.org/content/m10736/2.6/

Module: "Diagonalizacin de Matrices"


By: Michael Haag
URL: http://cnx.org/content/m12871/1.2/
Pages: 85-88
Copyright: Fara Meza, Erika Jackson
License: http://creativecommons.org/licenses/by/2.0/
Based on: Matrix Diagonalization
By: Michael Haag
URL: http://cnx.org/content/m10738/2.4/

Available for free at Connexions <http://cnx.org/content/col10373/1.2>


314 ATTRIBUTIONS

Module: "Generalidades de Eigenvectores y Eigenvalores"


By: Michael Haag, Justin Romberg
URL: http://cnx.org/content/m12872/1.3/
Pages: 88-89
Copyright: Fara Meza, Erika Jackson
License: http://creativecommons.org/licenses/by/2.0/
Based on: Eigen-stu in a Nutshell
By: Michael Haag, Justin Romberg
URL: http://cnx.org/content/m10742/2.4/

Module: "Eigenfunciones de los Sistemas LTI"


By: Justin Romberg
URL: http://cnx.org/content/m12874/1.3/
Pages: 89-92
Copyright: Fara Meza, Erika Jackson
License: http://creativecommons.org/licenses/by/2.0/
Based on: Eigenfunctions of LTI Systems
By: Justin Romberg
URL: http://cnx.org/content/m10500/2.7/

Module: "Propiedades de la Transformada de Fourier"


By: Don Johnson
URL: http://cnx.org/content/m12875/1.2/
Page: 92
Copyright: Fara Meza, Erika Jackson
License: http://creativecommons.org/licenses/by/2.0/
Based on: Fourier Transform Properties
By: Don Johnson
URL: http://cnx.org/content/m0045/2.8/

Module: "Seales Peridicas"


By: Michael Haag, Justin Romberg
URL: http://cnx.org/content/m12933/1.3/
Pages: 95-96
Copyright: Erika Jackson, Fara Meza
License: http://creativecommons.org/licenses/by/2.0/
Based on: Periodic Signals
By: Michael Haag, Justin Romberg
URL: http://cnx.org/content/m10744/2.5/

Module: "Series de Fourier: El Mtodo de Eigenfunciones"


By: Justin Romberg
URL: http://cnx.org/content/m12893/1.3/
Pages: 96-99
Copyright: Erika Jackson, Fara Meza
License: http://creativecommons.org/licenses/by/2.0/
Based on: Fourier Series: Eigenfunction Approach
By: Justin Romberg
URL: http://cnx.org/content/m10496/2.20/

Available for free at Connexions <http://cnx.org/content/col10373/1.2>


ATTRIBUTIONS 315

Module: "Derivacin de la Ecuacin de Coecientes de Fourier"


By: Michael Haag
URL: http://cnx.org/content/m12894/1.1/
Pages: 99-100
Copyright: Erika Jackson, Fara Meza
License: http://creativecommons.org/licenses/by/2.0/
Based on: Derivation of Fourier Coecients Equation
By: Michael Haag
URL: http://cnx.org/content/m10733/2.6/

Module: "Generalidades de las Series de Fourier"


By: Michael Haag, Justin Romberg
URL: http://cnx.org/content/m12896/1.2/
Pages: 100-103
Copyright: Erika Jackson, Fara Meza
License: http://creativecommons.org/licenses/by/2.0/
Based on: Fourier Series in a Nutshell
By: Michael Haag, Justin Romberg
URL: http://cnx.org/content/m10751/2.3/

Module: "Propiedades de la Serie de Fourier"


By: Justin Romberg, Benjamin Fite
URL: http://cnx.org/content/m12897/1.2/
Pages: 103-106
Copyright: Erika Jackson, Fara Meza
License: http://creativecommons.org/licenses/by/2.0/
Based on: Fourier Series Properties
By: Justin Romberg, Benjamin Fite
URL: http://cnx.org/content/m10740/2.7/

Module: "Propiedades de Simetra de las Series de Fourier"


By: Justin Romberg
URL: http://cnx.org/content/m12898/1.1/
Pages: 106-110
Copyright: Erika Jackson, Fara Meza
License: http://creativecommons.org/licenses/by/2.0/
Based on: Symmetry Properties of the Fourier Series
By: Justin Romberg
URL: http://cnx.org/content/m10838/2.4/

Module: "Propiedad de Convolucin Circular de las Series de Fourier"


By: Justin Romberg
URL: http://cnx.org/content/m12899/1.1/
Pages: 110-111
Copyright: Erika Jackson, Fara Meza
License: http://creativecommons.org/licenses/by/2.0/
Based on: Circular Convolution Property of Fourier Series
By: Justin Romberg
URL: http://cnx.org/content/m10839/2.4/

Available for free at Connexions <http://cnx.org/content/col10373/1.2>


316 ATTRIBUTIONS

Module: "Series de Fourier y los Sistemas LTI"


By: Justin Romberg
URL: http://cnx.org/content/m12900/1.2/
Pages: 111-114
Copyright: Erika Jackson, Fara Meza
License: http://creativecommons.org/licenses/by/2.0/
Based on: Fourier Series and LTI Systems
By: Justin Romberg
URL: http://cnx.org/content/m10752/2.7/

Module: "Convergencia de las Series de Fourier"


By: Michael Haag, Justin Romberg
URL: http://cnx.org/content/m12934/1.2/
Pages: 114-116
Copyright: Erika Jackson, Fara Meza
License: http://creativecommons.org/licenses/by/2.0/
Based on: Convergence of Fourier Series
By: Michael Haag, Justin Romberg
URL: http://cnx.org/content/m10745/2.3/

Module: "Condiciones de Dirichlet"


By: Ricardo Radaelli-Sanchez
URL: http://cnx.org/content/m12927/1.2/
Pages: 116-118
Copyright: Erika Jackson, Fara Meza
License: http://creativecommons.org/licenses/by/2.0/
Based on: Dirichlet Conditions
By: Ricardo Radaelli-Sanchez
URL: http://cnx.org/content/m10089/2.9/

Module: "El Fenmeno de Gibbs"


By: Ricardo Radaelli-Sanchez, Richard Baraniuk
URL: http://cnx.org/content/m12929/1.1/
Pages: 118-121
Copyright: Erika Jackson, Fara Meza
License: http://creativecommons.org/licenses/by/2.0/
Based on: Gibbs's Phenomena
By: Ricardo Radaelli-Sanchez, Richard Baraniuk
URL: http://cnx.org/content/m10092/2.8/

Module: "Resumen de las Series de Fourier"


By: Michael Haag, Justin Romberg
URL: http://cnx.org/content/m12932/1.3/
Pages: 121-122
Copyright: Erika Jackson, Fara Meza
License: http://creativecommons.org/licenses/by/2.0/
Based on: Fourier Series Wrap-Up
By: Michael Haag, Justin Romberg
URL: http://cnx.org/content/m10749/2.3/

Available for free at Connexions <http://cnx.org/content/col10373/1.2>


ATTRIBUTIONS 317

Module: "Espacios Vectoriales"


By: Michael Haag, Steven J. Cox, Justin Romberg
URL: http://cnx.org/content/m12878/1.2/
Pages: 125-126
Copyright: Fara Meza, Erika Jackson
License: http://creativecommons.org/licenses/by/2.0/
Based on: Vector Spaces
By: Michael Haag, Steven J. Cox, Justin Romberg
URL: http://cnx.org/content/m10767/2.4/

Module: "Normas"
By: Michael Haag, Justin Romberg
URL: http://cnx.org/content/m12877/1.2/
Pages: 127-129
Copyright: Fara Meza, Erika Jackson
License: http://creativecommons.org/licenses/by/2.0/
Based on: Norms
By: Michael Haag, Justin Romberg
URL: http://cnx.org/content/m10768/2.3/

Module: "Producto Interno"


By: Michael Haag, Justin Romberg
URL: http://cnx.org/content/m12876/1.2/
Pages: 129-131
Copyright: Fara Meza, Erika Jackson
License: http://creativecommons.org/licenses/by/2.0/
Based on: Inner Products
By: Michael Haag, Justin Romberg
URL: http://cnx.org/content/m10755/2.5/

Module: "Espacios de Hilbert"


By: Justin Romberg
URL: http://cnx.org/content/m12879/1.2/
Pages: 131-132
Copyright: Fara Meza, Erika Jackson
License: http://creativecommons.org/licenses/by/2.0/
Based on: Hilbert Spaces
By: Justin Romberg
URL: http://cnx.org/content/m10840/2.4/

Module: "Desigualdad de Cauchy-Schwarz"


By: Michael Haag, Justin Romberg
URL: http://cnx.org/content/m12880/1.2/
Pages: 132-139
Copyright: Fara Meza, Erika Jackson
License: http://creativecommons.org/licenses/by/2.0/
Based on: Cauchy-Schwarz Inequality
By: Michael Haag, Justin Romberg
URL: http://cnx.org/content/m10757/2.5/

Available for free at Connexions <http://cnx.org/content/col10373/1.2>


318 ATTRIBUTIONS

Module: "Espacios de Hilbert comunes"


By: Roy Ha, Justin Romberg
URL: http://cnx.org/content/m12881/1.2/
Pages: 139-142
Copyright: Fara Meza, Erika Jackson
License: http://creativecommons.org/licenses/by/2.0/
Based on: Common Hilbert Spaces
By: Roy Ha, Justin Romberg
URL: http://cnx.org/content/m10759/2.5/

Module: "Tipos de Bases"


By: Michael Haag, Justin Romberg
URL: http://cnx.org/content/m12902/1.2/
Pages: 142-146
Copyright: Fara Meza, Erika Jackson
License: http://creativecommons.org/licenses/by/2.0/
Based on: Types of Basis
By: Michael Haag, Justin Romberg
URL: http://cnx.org/content/m10772/2.4/

Module: "Expansin de Bases Ortonormales"


By: Michael Haag, Justin Romberg
URL: http://cnx.org/content/m12928/1.1/
Pages: 146-150
Copyright: Fara Meza, Erika Jackson
License: http://creativecommons.org/licenses/by/2.0/
Based on: Orthonormal Basis Expansions
By: Michael Haag, Justin Romberg
URL: http://cnx.org/content/m10760/2.4/

Module: "Espacio de Funciones"


By: Justin Romberg
URL: http://cnx.org/content/m12913/1.1/
Pages: 150-151
Copyright: Fara Meza, Erika Jackson
License: http://creativecommons.org/licenses/by/2.0/
Based on: Function Space
By: Justin Romberg
URL: http://cnx.org/content/m10770/2.4/

Module: "Base de la Ondoleta de Haar"


By: Roy Ha, Justin Romberg
URL: http://cnx.org/content/m12919/1.2/
Pages: 151-158
Copyright: Fara Meza, Erika Jackson
License: http://creativecommons.org/licenses/by/2.0/
Based on: Haar Wavelet Basis
By: Roy Ha, Justin Romberg
URL: http://cnx.org/content/m10764/2.6/

Available for free at Connexions <http://cnx.org/content/col10373/1.2>


ATTRIBUTIONS 319

Module: "Bases Ortonormales en Espacios Reales y Complejos"


By: Justin Romberg
URL: http://cnx.org/content/m12920/1.2/
Pages: 158-160
Copyright: Fara Meza, Erika Jackson
License: http://creativecommons.org/licenses/by/2.0/
Based on: Orthonormal Bases in Real and Complex Spaces
By: Justin Romberg
URL: http://cnx.org/content/m10765/2.7/

Module: "Teoremas de Plancharel y Parseval"


By: Justin Romberg
URL: http://cnx.org/content/m12931/1.1/
Pages: 160-161
Copyright: Fara Meza, Erika Jackson
License: http://creativecommons.org/licenses/by/2.0/
Based on: Plancharel and Parseval's Theorems
By: Justin Romberg
URL: http://cnx.org/content/m10769/2.4/

Module: "Approximacin y Proyeccin en el Espacio de Hilbert"


By: Justin Romberg
URL: http://cnx.org/content/m12935/1.2/
Pages: 161-164
Copyright: Fara Meza, Erika Jackson
License: http://creativecommons.org/licenses/by/2.0/
Based on: Approximation and Projections in Hilbert Space
By: Justin Romberg
URL: http://cnx.org/content/m10766/2.6/

Module: "Anlisis de Fourier"


By: Richard Baraniuk
URL: http://cnx.org/content/m12865/1.4/
Pages: 167-168
Copyright: Erika Jackson, Fara Meza
License: http://creativecommons.org/licenses/by/2.0/
Based on: Fourier Analysis
By: Richard Baraniuk
URL: http://cnx.org/content/m10096/2.10/

Module: "Anlisis de Fourier en Espacios Complejos"


By: Michael Haag, Justin Romberg
URL: http://cnx.org/content/m12848/1.5/
Pages: 168-175
Copyright: Erika Jackson, Fara Meza
License: http://creativecommons.org/licenses/by/2.0/
Based on: Fourier Analysis in Complex Spaces
By: Michael Haag, Justin Romberg
URL: http://cnx.org/content/m10784/2.5/

Available for free at Connexions <http://cnx.org/content/col10373/1.2>


320 ATTRIBUTIONS

Module: "Ecuacin de Matriz para la DTFS"


By: Roy Ha
URL: http://cnx.org/content/m12859/1.1/
Pages: 175-176
Copyright: Erika Jackson, Fara Meza
License: http://creativecommons.org/licenses/by/2.0/
Based on: Matrix Equation for the DTFS
By: Roy Ha
URL: http://cnx.org/content/m10771/2.6/

Module: "Extensin Peridica de las DTFS"


By: Roy Ha
URL: http://cnx.org/content/m12860/1.2/
Pages: 176-184
Copyright: Erika Jackson, Fara Meza
License: http://creativecommons.org/licenses/by/2.0/
Based on: Periodic Extension to DTFS
By: Roy Ha
URL: http://cnx.org/content/m10778/2.5/

Module: "Desplazamientos Circulares"


By: Justin Romberg
URL: http://cnx.org/content/m12864/1.1/
Pages: 184-189
Copyright: Erika Jackson, Fara Meza
License: http://creativecommons.org/licenses/by/2.0/
Based on: Circular Shifts
By: Justin Romberg
URL: http://cnx.org/content/m10780/2.6/

Module: "DFT: Transformada Rpida de Fourier"


By: Don Johnson
URL: http://cnx.org/content/m12937/1.1/
Pages: 191-192
Copyright: Erika Jackson, Fara Meza
License: http://creativecommons.org/licenses/by/2.0/
Based on: DFT: Fast Fourier Transform
By: Don Johnson
URL: http://cnx.org/content/m0504/2.8/

Module: "La Transformada Rpida de Fourier (FFT)"


By: Justin Romberg
URL: http://cnx.org/content/m12942/1.1/
Pages: 192-193
Copyright: Fara Meza, Erika Jackson
License: http://creativecommons.org/licenses/by/2.0/
Based on: The Fast Fourier Transform (FFT)
By: Justin Romberg
URL: http://cnx.org/content/m10783/2.5/

Available for free at Connexions <http://cnx.org/content/col10373/1.2>


ATTRIBUTIONS 321

Module: "Derivando la Transformada Rpida de Fourier"


By: Don Johnson
URL: http://cnx.org/content/m12941/1.1/
Pages: 193-196
Copyright: Erika Jackson, Fara Meza
License: http://creativecommons.org/licenses/by/2.0/
Based on: Deriving the Fast Fourier Transform
By: Don Johnson
URL: http://cnx.org/content/m0528/2.7/

Module: "Convergencia de Secuencias"


By: Richard Baraniuk
URL: http://cnx.org/content/m12943/1.2/
Pages: 199-200
Copyright: Fara Meza, Erika Jackson
License: http://creativecommons.org/licenses/by/2.0/
Based on: Convergence of Sequences
By: Richard Baraniuk
URL: http://cnx.org/content/m10883/2.4/

Module: "Convergencia de Vectores"


By: Michael Haag
URL: http://cnx.org/content/m12944/1.2/
Pages: 201-204
Copyright: Fara Meza, Erika Jackson
License: http://creativecommons.org/licenses/by/2.0/
Based on: Convergence of Vectors
By: Michael Haag
URL: http://cnx.org/content/m10894/2.2/

Module: "Convergencia Uniforme de Secuencias de Funciones."


By: Michael Haag, Richard Baraniuk
URL: http://cnx.org/content/m12945/1.1/
Pages: 204-205
Copyright: Fara Meza, Erika Jackson
License: http://creativecommons.org/licenses/by/2.0/
Based on: Uniform Convergence of Function Sequences
By: Michael Haag, Richard Baraniuk
URL: http://cnx.org/content/m10895/2.5/

Module: "Common Fourier Transforms"


Used here as: "Tabla de Transformadas de Fourier Comunes"
By: Melissa Selik, Richard Baraniuk
URL: http://cnx.org/content/m10099/2.12/
Pages: 207-208
Copyright: Melissa Selik, Richard Baraniuk
License: http://creativecommons.org/licenses/by/3.0/

Available for free at Connexions <http://cnx.org/content/col10373/1.2>


322 ATTRIBUTIONS

Module: "Transformacin Discreta de Fourier"


By: Phil Schniter
URL: http://cnx.org/content/m12843/1.2/
Pages: 208-210
Copyright: Fara Meza, Erika Jackson
License: http://creativecommons.org/licenses/by/2.0/
Based on: Discrete Fourier Transformation
By: Phil Schniter
URL: http://cnx.org/content/m10421/2.10/

Module: "Transformada Discreta de Fourier (DFT)"


By: Don Johnson
URL: http://cnx.org/content/m12844/1.1/
Pages: 210-212
Copyright: Fara Meza, Erika Jackson
License: http://creativecommons.org/licenses/by/2.0/
Based on: Discrete Fourier Transform (DFT)
By: Don Johnson
URL: http://cnx.org/content/m10249/2.23/

Module: "Transformada de Fourier Discreta en el Tiempo (DTFT)"


By: Richard Baraniuk
URL: http://cnx.org/content/m12861/1.1/
Pages: 212-213
Copyright: Erika Jackson, Fara Meza
License: http://creativecommons.org/licenses/by/2.0/
Based on: Discrete-Time Fourier Transform (DTFT)
By: Richard Baraniuk
URL: http://cnx.org/content/m10108/2.11/

Module: "Propiedades de la Transformada de Fourier Discreta en el Tiempo"


By: Don Johnson
URL: http://cnx.org/content/m12846/1.1/
Page: 214
Copyright: Fara Meza, Erika Jackson
License: http://creativecommons.org/licenses/by/2.0/
Based on: Discrete-Time Fourier Transform Properties
By: Don Johnson
URL: http://cnx.org/content/m0506/2.6/

Module: "Par de la Transformada de Fourier Discreta en el Tiempo"


By: Don Johnson
URL: http://cnx.org/content/m12845/1.1/
Pages: 214-215
Copyright: Fara Meza, Erika Jackson
License: http://creativecommons.org/licenses/by/2.0/
Based on: Discrete-Time Fourier Transform Pair
By: Don Johnson
URL: http://cnx.org/content/m0525/2.6/

Available for free at Connexions <http://cnx.org/content/col10373/1.2>


ATTRIBUTIONS 323

Module: "Ejemplos de DTFT"


By: Don Johnson
URL: http://cnx.org/content/m12847/1.1/
Pages: 215-218
Copyright: Fara Meza, Erika Jackson
License: http://creativecommons.org/licenses/by/2.0/
Based on: DTFT Examples
By: Don Johnson
URL: http://cnx.org/content/m0524/2.10/

Module: "Transformada de Fourier de Tiempo Continuo (CTFT)"


By: Richard Baraniuk, Melissa Selik
URL: http://cnx.org/content/m12955/1.1/
Pages: 221-222
Copyright: Fara Meza, Erika Jackson
License: http://creativecommons.org/licenses/by/2.0/
Based on: Continuous-Time Fourier Transform (CTFT)
By: Richard Baraniuk, Melissa Selik
URL: http://cnx.org/content/m10098/2.9/

Module: "Propiedades de la Transformada de Fourier de Tiempo-Continuo"


By: Melissa Selik, Richard Baraniuk
URL: http://cnx.org/content/m12957/1.2/
Pages: 223-225
Copyright: Fara Meza, Erika Jackson
License: http://creativecommons.org/licenses/by/2.0/
Based on: Properties of the Continuous-Time Fourier Transform
By: Melissa Selik, Richard Baraniuk
URL: http://cnx.org/content/m10100/2.13/

Module: "Muestreo"
By: Justin Romberg
URL: http://cnx.org/content/m12964/1.3/
Pages: 227-231
Copyright: Fara Meza, Erika Jackson
License: http://creativecommons.org/licenses/by/2.0/
Based on: Sampling
By: Justin Romberg
URL: http://cnx.org/content/m10798/2.6/

Module: "Reconstruccin"
By: Justin Romberg
URL: http://cnx.org/content/m12969/1.2/
Pages: 231-236
Copyright: Fara Meza, Erika Jackson
License: http://creativecommons.org/licenses/by/2.0/
Based on: Reconstruction
By: Justin Romberg
URL: http://cnx.org/content/m10788/2.5/

Available for free at Connexions <http://cnx.org/content/col10373/1.2>


324 ATTRIBUTIONS

Module: "Ms sobre Reconstruccin Perfecta"


By: Roy Ha, Justin Romberg
URL: http://cnx.org/content/m12970/1.2/
Pages: 236-238
Copyright: Fara Meza, Erika Jackson
License: http://creativecommons.org/licenses/by/2.0/
Based on: More on Perfect Reconstruction
By: Roy Ha, Justin Romberg
URL: http://cnx.org/content/m10790/2.4/

Module: "Teorema de Nyquist"


By: Justin Romberg
URL: http://cnx.org/content/m12971/1.2/
Pages: 238-239
Copyright: Fara Meza, Erika Jackson
License: http://creativecommons.org/licenses/by/2.0/
Based on: Nyquist Theorem
By: Justin Romberg
URL: http://cnx.org/content/m10791/2.4/

Module: "Aliasing"
By: Justin Romberg, Don Johnson
URL: http://cnx.org/content/m12973/1.3/
Pages: 240-243
Copyright: Fara Meza, Erika Jackson
License: http://creativecommons.org/licenses/by/2.0/
Based on: Aliasing
By: Justin Romberg, Don Johnson
URL: http://cnx.org/content/m10793/2.5/

Module: "Filtros Anti-Aliasing"


By: Justin Romberg
URL: http://cnx.org/content/m12974/1.2/
Pages: 243-245
Copyright: Fara Meza, Erika Jackson
License: http://creativecommons.org/licenses/by/2.0/
Based on: Anti-Aliasing Filters
By: Justin Romberg
URL: http://cnx.org/content/m10794/2.4/

Module: "Procesamiento de Tiempo Discreto de Seales de Tiempo Continuo"


By: Justin Romberg
URL: http://cnx.org/content/m12976/1.2/
Pages: 245-247
Copyright: Fara Meza, Erika Jackson
License: http://creativecommons.org/licenses/by/2.0/
Based on: Discrete Time Processing of Continuous Time Signals
By: Justin Romberg
URL: http://cnx.org/content/m10797/2.4/

Available for free at Connexions <http://cnx.org/content/col10373/1.2>


ATTRIBUTIONS 325

Module: "La Transformada de Laplace"


By: Richard Baraniuk
URL: http://cnx.org/content/m12978/1.2/
Pages: 251-253
Copyright: Fara Meza, Erika Jackson
License: http://creativecommons.org/licenses/by/2.0/
Based on: The Laplace Transforms
By: Richard Baraniuk
URL: http://cnx.org/content/m10110/2.12/

Module: "Propiedades de la Transformada de Laplace"


By: Melissa Selik, Richard Baraniuk
URL: http://cnx.org/content/m12979/1.2/
Pages: 253-254
Copyright: Fara Meza, Erika Jackson
License: http://creativecommons.org/licenses/by/2.0/
Based on: Properties of the Laplace Transform
By: Melissa Selik, Richard Baraniuk
URL: http://cnx.org/content/m10117/2.9/

Module: "Tabla de Transformadas de Laplace Comunes"


By: Melissa Selik, Richard Baraniuk
URL: http://cnx.org/content/m12959/1.1/
Page: 255
Copyright: Erika Jackson, Fara Meza
License: http://creativecommons.org/licenses/by/2.0/
Based on: Table of Common Laplace Transforms
By: Melissa Selik, Richard Baraniuk
URL: http://cnx.org/content/m10111/2.10/

Module: "Regin de Convergencia para la Transformada de Laplace"


By: Richard Baraniuk
URL: http://cnx.org/content/m12961/1.1/
Pages: 255-257
Copyright: Erika Jackson, Fara Meza
License: http://creativecommons.org/licenses/by/2.0/
Based on: Region of Convergence for the Laplace Transform
By: Richard Baraniuk
URL: http://cnx.org/content/m10114/2.8/

Module: "La Transformada Inversa de Laplace"


By: Steven J. Cox
URL: http://cnx.org/content/m12962/1.1/
Pages: 257-259
Copyright: Erika Jackson, Fara Meza
License: http://creativecommons.org/licenses/by/2.0/
Based on: The Inverse Laplace Transform
By: Steven J. Cox
URL: http://cnx.org/content/m10170/2.8/

Available for free at Connexions <http://cnx.org/content/col10373/1.2>


326 ATTRIBUTIONS

Module: "Polos y Ceros"


By: Richard Baraniuk
URL: http://cnx.org/content/m12963/1.2/
Pages: 259-261
Copyright: Erika Jackson, Fara Meza
License: http://creativecommons.org/licenses/by/2.0/
Based on: Poles and Zeros
By: Richard Baraniuk
URL: http://cnx.org/content/m10112/2.11/

Module: "La Transformada Z: Denicin"


By: Benjamin Fite
URL: http://cnx.org/content/m12951/1.1/
Pages: 263-268
Copyright: Erika Jackson, Fara Meza
License: http://creativecommons.org/licenses/by/2.0/
Based on: The Z Transform: Denition
By: Benjamin Fite
URL: http://cnx.org/content/m10549/2.9/

Module: "Tabla de Transformadas-Z Comunes"


By: Melissa Selik, Richard Baraniuk
URL: http://cnx.org/content/m12950/1.2/
Pages: 268-269
Copyright: Erika Jackson, Fara Meza
License: http://creativecommons.org/licenses/by/2.0/
Based on: Table of Common z-Transforms
By: Melissa Selik, Richard Baraniuk
URL: http://cnx.org/content/m10119/2.12/

Module: "Regin de Convergencia para la Transformada- Z"


By: Benjamin Fite
URL: http://cnx.org/content/m12956/1.2/
Pages: 269-278
Copyright: Erika Jackson, Fara Meza
License: http://creativecommons.org/licenses/by/2.0/
Based on: Region of Convergence for the Z-transform
By: Benjamin Fite
URL: http://cnx.org/content/m10622/2.5/

Module: "La Transformada Inversa de Z"


By: Benjamin Fite
URL: http://cnx.org/content/m12958/1.2/
Pages: 278-281
Copyright: Erika Jackson, Fara Meza
License: http://creativecommons.org/licenses/by/2.0/
Based on: Inverse Z-Transform
By: Benjamin Fite
URL: http://cnx.org/content/m10651/2.4/

Available for free at Connexions <http://cnx.org/content/col10373/1.2>


ATTRIBUTIONS 327

Module: "Funciones Racionales"


By: Michael Haag
URL: http://cnx.org/content/m12960/1.1/
Pages: 281-283
Copyright: Erika Jackson, Fara Meza
License: http://creativecommons.org/licenses/by/2.0/
Based on: Rational Functions
By: Michael Haag
URL: http://cnx.org/content/m10593/2.7/

Module: "La Ecuacin de Diferencia"


By: Michael Haag
URL: http://cnx.org/content/m12965/1.1/
Pages: 283-286
Copyright: Erika Jackson, Fara Meza
License: http://creativecommons.org/licenses/by/2.0/
Based on: Dierence Equation
By: Michael Haag
URL: http://cnx.org/content/m10595/2.5/

Module: "Entendiendo las Gracas de Polos y Ceros en el Plano-Z"


By: Michael Haag
URL: http://cnx.org/content/m12966/1.1/
Pages: 286-290
Copyright: Erika Jackson, Fara Meza
License: http://creativecommons.org/licenses/by/2.0/
Based on: Understanding Pole/Zero Plots on the Z-Plane
By: Michael Haag
URL: http://cnx.org/content/m10556/2.8/

Module: "Diseo de Filtros usando la Graca de Polos y Ceros de la Transformada-Z"


By: Michael Haag
URL: http://cnx.org/content/m12967/1.3/
Pages: 290-293
Copyright: Erika Jackson, Fara Meza
License: http://creativecommons.org/licenses/by/2.0/
Based on: Filter Design using the Pole/Zero Plot of a Z-Transform
By: Michael Haag
URL: http://cnx.org/content/m10548/2.9/

Module: "Homework 1"


By: Richard Baraniuk, Justin Romberg
URL: http://cnx.org/content/m10826/2.9/
Pages: 296-299
Copyright: Richard Baraniuk, Justin Romberg
License: http://creativecommons.org/licenses/by/1.0

Available for free at Connexions <http://cnx.org/content/col10373/1.2>


Seales y Sistemas
Este curso trata acerca de seales, sistemas, y transformadas a partir de las bases matmaticas y teorticas
hasta las implementaciones prcticas en circuitos y algoritmos. Al terminar ELEC 301, tendr un amplio
entendimiento de las matmaticas y temas prcticos relacionados con las seales en tiempo continuo y tiempo
discreto, sistemas lineales invariantes en el tiempo, la convulocin, y la trasformada de Fourier.

About Connexions
Since 1999, Connexions has been pioneering a global system where anyone can create course materials and
make them fully accessible and easily reusable free of charge. We are a Web-based authoring, teaching and
learning environment open to anyone interested in education, including students, teachers, professors and
lifelong learners. We connect ideas and facilitate educational communities.

Connexions's modular, interactive courses are in use worldwide by universities, community colleges, K-12
schools, distance learners, and lifelong learners. Connexions materials are in many languages, including
English, Spanish, Chinese, Japanese, Italian, Vietnamese, French, Portuguese, and Thai. Connexions is part
of an exciting new information distribution system that allows for Print on Demand Books. Connexions
has partnered with innovative on-demand publisher QOOP to accelerate the delivery of printed course
materials and textbooks into classrooms worldwide at lower prices than traditional academic publishers.

S-ar putea să vă placă și